You are on page 1of 417

Solutions Set for 4

th

Edition of CA. Parag Gupta




Cost Accounting & Management:
1. CVP Analysis 1 12
2. Activity-based costing management 13 33
3. Target Costing, Value Chain analysis & Life
Cycle Costing
34 42

4. Service Sector 43 59
5. Standard Costing & Variance Analysis 60 133
6. Budget & Budgetary Control 134 175
7. Transfer Pricing 176 218
8. Decision Making 219 298
9. Miscellaneous Theory Chapters 299 308

Operations Research:
10. Linear Programming Problems 309 330
11. The Transportation Problem 331 359
12. The Assignment Problem 360 373
13. Network Analysis-PERT/CPM 374 397
14. Simulation 398 410
15. Learning Curve Theory 411 416
Selling Price per unit = 187500/7500 =Rs. 25
Profit Rs.
Sales 10000 x 25

2,50,000
Less: Total Cost 1,93,75

Profit 56,250
P/V Ratio Profit/Margin of Safety
56250/187500

30%
BEP Sales 2500 x25

Rs. 62,500
Fixed Cost 62500 x 30%=

Rs.

Al ternative Answer 2

Ans. 13 (Pg. 11): Margin of Safety(%) = MoS Units/Actual Sales Units
CVP

= 7500/(7500+2500) = 75%

Total Sales = 187500/0.75 = Rs.2,50,000/-
Profit = Total sales Total Cost

= 250000 193750 = Rs.56250

P/V Ratio = Profit/MoS (Rs.) x 100

= 56250/187500 x 100 = 30% BEP Sales =
Total Sales / (100 MS)
= 2,50,000 x 0.25 = Rs.62,500

Fixed Cost = Sales x P/V Ratio

= 250000 x 0.30-56250 = 18750

Al ternate Answer 1

Margin of Safety = Selling Price per unit x ( 7500 units) Rs. 187500 =
Selling Price per unit x ( 7500 units)
Therefore ,




Selling price = Rs 187500/ 7500 = Rs.25

Total Cost at Break Even point=Rs.25 x 2500 = 62500 = Break Even Sales

(Total Cost Total Cost of BE)/(Total Units Break Even Units) = Variable Cost per Unit

(93,750 62,500)/(10,000 2,500) = 1,31,250/7,500 = Rs.17.50 per unit

Selling Price = 25.00

Variable Cost =

17.50

Contribution = 7.50

P/V Ratio = 7.50/25 = 30%
Fixed Cost = 7.50 x 2500 units = Rs.18750.

Profit = 7.50 x 7500 = Rs. 56,250


Ans. 12 (Pg. 11)

(1)

P/V Ratio

1
In year 2, additional NP which means additional contribution

8,000
Additional sales

40,000
P/V Ratio

20%




Fixed cost = Contribution NP

= (2,40,000 * 20%) 18,000 48,000 18000 30,000
BEP = FC/PV Ratio 30,000/0.20 1,50,000


(3)

Margin of Safety


Year 1

2,40,000 1,50,000

90,000
Year 2 2,80,000 1,50,000 1,30,000


( ) fi


(Contribution*PV Ratio) Fixed Cost (2,00,000 * 20%) 30,000 10,000



OR

Cap Sales

(-) BEP

Margin of Safety

(-) PV Ratio

NP
2,00,000

1,50,000
50,000

20%
10,000




5) Sales Required







BEP

Margin of Safety Req (100/20*40,000)
Sales Required

100/20 ( 30,000(FC) + 40,000(NP))



OR

3,50,000







1,50,000

2,00,000
3,50,000



2
(6)
a) 20% decrease in sale Qty

Reduction in Contribution & in net profit



20% *(2,80,000*20%)



20%
(56,000)
Reduction in Contribution & in net profit

Rs.11,200
(b) Revenue Sales ( 2,80,000*80%) *110% 2,46,400
(-) Revenue Cost (2,80,000*80%) * 80% 1,79,200
Revenue contribution

67,200
(-) Revenue Fixed Cost

(26,500)
Revenue NP

40,700

(-) Given NP

Increase in NP
(26,000)
OR

14,700
3

(b) Revenue Sales (2,80,000*80%) *110% 2,46,400
P/V Ratio (now)

100-80 = 20
(new) 3/11 11080 =30

(Reconciliation of NP change)

Change Effection NP
1) Reduction in Sales Qty (as per (a))

2) Increase in Sales Price (2,80,000*80%*10%)

3) Reduction in Fixed Cost

Increase in NP
(11,200)

22,400

3500
14,700

Ans. 3 (Pg. 14) (1) Evaluation of proposal to replace product Z with product S. a: net profit if we
continue with product Z.

X (5,00,000*40%/20)*(20-10) 1,00,000
Y (5,00,000*35%/25)*(25-25) 70,000
Z (5,00,000*25%/30)*(30-18) 50,000
Total contribution

2,20,000
(-) Fixed Cost

1,50,000
Net Profit

70,000



b) Net profit if we replace with S
X (4,50,000*50%/20)*10 1,12,500
Y (4,50,000*30%)/25*10 54,000
Z (4,50,000*20%)/28*14 45,000
Total contribution

2,11,500
(-) Fixed Cost

1,60,000
Net Profit

51,500


The company should continue with product Z because the replacement of Z with S would
result in reduction net profit.




2) Statement showing the overall breakeven point of the 2 alternatives.


XYZ XYZ
4
Contribution 2,20,000 2,11,500

Sales 5,00,000 4,50,000

Fixed cost 1,50,000 1,60,000
BEP 50/22*1,50,000 3,40,909 3,40,426


The above calculation are based on the presumption, in addition to the usual presumptions that
the sales of products X, Y & Z would always be in the ratio of Rs.40:35:25 and that of X, Y
& Z would be in the ratio of 50:30:20

Ans. 6 (Pg. 15)
a) Statement showing the budgeted net income for 2003
Fees collected (4,000 *50)
Less: Budgeted cost
G.B 4000* 10

Evaluation 4000*20

Hall rev.

Hon. To Chief Adm.
Super changer (50*4 * 4000/100)
Total
2,00,000



80,000

40,000

8,000

6,000

8,000
1,48,000
Budgeted N.I 52,000


b) (i) Calculation of supervision cost
Fees per student


Less: Variable cost + semi variable cost
Evaluation

QB



Semi- variable

(supervision)

Gross contribution
Gross Fixed Cost
Gross BEP
Therefore, no. of Supervisory required.
Therefore, Supervision Cost
Net fixed cost





20

10
50












32

18

20,000

1111.11

12

2,400

22,400
30

2







20,000/18



12*200

20,000+2,400
5



(ii)

BEP
Fixed Cost 22,400

Net CTR per student

Fees 50

(-) Variable Cost (30)

20
BEP 1,120


(C) (i)
Calculation of total contribution required
Gross contribution per student

Gross Fixed Cost

Net Profit Required

Gross Total Contribution Required
Gross no. of students (40,000/18)
No. of Supervision required
Supervision cost (23*200)
Net Fixed Cost (20,000+4,600)

Net Contribution Required (20,000+24,600)
Net Total Contribution required
Net Contribution per Student

Fees 50
(-) Variable Cost (30)
No. of Students required
18

20,000

20,000

40,000

2,222.22

23

4,600

24,600

44,600

44,600





20
2230


Ans. 7 (Pg.15): (i) Statement of profitability of Special Health Care
Department
(for the years 2001 and 2002)
Year Year
2001 2002
Rs. Rs.
Total contribution : (A)
8,225 bed days Rs. 260 21,38,500
6
8,225 bed days Rs. 243.50 20,02,788
(Refer to working notes 1, 2, & 3)

Fixed costs :
Department fixed costs 6,22,500 6,84,750
Apportioned fixed costs 10,00,000 12,50,000
(Refer to working note 4)
Nursing staff 6 2,88,000 3,24,000
(6 Nurse (6 Nurse

Total fixed costs : (B) 19,10,500 22,58,750
Rs. 48,000 Rs. 54,000)

Profit (Loss) : { (A) (B)} 2,28,000 (2,55,962)



Working notes :

1. Total number of bed days of occupancy
= Total fees collected Fee per bed days = Rs. 34,95,625 Rs. 425 = 8,225

2. Variable cost per bed day
Variable cost per bed das (Rs.) 165.00
(Rs. 13,57,125 / 8,225)
Variable cost per bed day (Rs.) 181.50
in the year 2002
(Rs. 165 + 10% Rs. 165)

3. Contribution per bed day
Contribution per bed days 260.00
in the year 2001 (Rs.)
(Rs. 425 -- Rs. 165)
Contribution per bed days 243.50
In the year 2002 (Rs.)
(Rs. 425 -- Rs. 181.50)

4. Departmental fixed costs
Departmental fixed costs (Rs.) 6,22,500
for the year 2001
Department fixed cost (Rs.) 6,84,750
for the year 2002
(Rs. 6,22,500 + 10% Rs. 6,22,500)

(ii) Break even bed capacity for the year 2002

= Total fixed costs Contribution per bed day
= Rs. 22,58,750 Rs. 243.50 = 9,276 bed days (approx.)
(this is not a valid answer because for 9,276 bed days 8 nurses service will be
required)
7


Nursing staff required; 8
Remuneration of 8 nursing staff (Rs.) 4,32,000
8 nurses Rs. 54,000
Department fixed costs (Rs.) 6,84,750
Apportioned fixed costs (Rs.)
Total fixed costs 23,66,750
12,50,000

Break even point = Rs. 23,66,750 Rs. 243.50 = 9,720 bed days

Increase in fee per day required to justify continuance of the Special Health Care
department

Desired contribution (Rs.) 22,58,750
Bed days of occupancy 8,225

Contribution per bed days (Rs. ) ; (a) 274.62
(Rs. 22,58,750 / 8,225)
Variable costs (Rs.) ; (B)
Required fee per bed day; {(A) + (B) }
181.50
456.12
Increase in fee per bed day (Rs.) 31.12

(Rs. 456.12 425)

Ans. 9 (Pg. 16): (i ) Profit Statement of M/s Satish Enterprises for first and second year on monthly and
yearly basis.
First year Second Year
Monthly
Rs.
Yearly
Rs.
Monthly
Rs.
Yearly
Rs.
Sales revenue: (A) 600 7,200 600 7,200
(3,000 units Rs.200)
Material cost 180 2,160 180 2,160
(3,000 units Rs.60)
Labour cost 75 900 75 900
(3,000 units Rs.25)
Variable overheads 60 720 60 720
(3,000 units Rs.20)
Primary packing cost 45 540 45 540
(3,000 units Rs.15
Boxes cost 24 288 24 288

months 12
units Rs.3,000
400

Total fixed overhead 108 1,296 110 1,320
8
(Refer to working
note 1)

months 12
Rs.1,296

month 12
Rs.1,320


Total cost : (B) 492 5,904 494 5,928
Profit : C = [(A)-(B)] 108 1,296 106 1,272

Worki ng Note :
1. (i )
Fixed overhead First year : (Rs.) Second year (Rs.)
Depreciation 8,96,000 8,96,000
years 3
duty 0 Rs.2,88,00 24,00,000 Rs, +


Other fixed overheads 4,00,000 4,24,000
Total Fixed overheads 12,96,000 13,20,000
(i i ) Statement of monthly break even units of the first year.
Levels No. of units (Refer to
working note)
1351
1400
1401
1450
1451
1500
1501
1500
Rs. Rs. Rs. Rs.
Fixed costs (A)
Total fixed overheads per month
(Refer to working note)
1,08,000 1,08,000 1,08,000 1,08,000
Semi variable costs (Special boxes
cost) (B)
11,200 11,600 12,000 12,400
(28 boxes
Rs.400)
(29 boxes
Rs.400)
(30 boxes
Rs.400)
(31boxes
Rs.400
Total fixed and semi variable costs :
(A+B)
1,19,200 1,19,600 1,20,000 1,20,000
Break-even level of units: 1490 1495 1500 1505

unit per on Contributi


costs variable - semi and fised Total

(Rs.
1,19,200 /
Rs.80)
(Rs.
1,19,600 /
Rs.80)
(Rs.
1,20,000 /
Rs.80)
(Rs.
1,20,000
/ Rs.80)
The first and second break-even level of unit viz. 1490 and 1495 units falls outside the range of 1351
1400 and 1401 1450 units respectively. Here a monthly break-even level of units is 1,500 units
which lies in the range of 1451 1500 units.
Statement of yearly break-even points of the first year
Levels No. of units 17851-17900 17901-17950 17951-18000 18001-18050
Rs. Rs. Rs. Rs.
Fixed Costs (A) 12,96,000 12,96,000 12,96,000 12,96,000
Semi-variable costs
(Special boxes costs): (B)
1,43,200 1,43,600 1,44,000 1,44,000
9
(358 boxes
Rs.400)
(359 boxes
Rs.400)
(360 boxes
Rs.400)
(361 boxes
Rs.400)
Total fixed and semi-
variable costs (A + B)
14,39,200 14,39,600 14,40,000 14,40,400
Break-even level units 17,990 17,995 18,000 18,005
(Rs.14,39,200
/ Rs.80)
(Rs.14,32,600
/ Rs.80)
(Rs.14,40,000
/ Rs.80)
(Rs.14,40,400
/ Rs.80)
Have a break-even level of units (on yearly basis) is 18,000 units which lies in the range of 17,951
18,000 units as well. The other first two figures do not lie in the respective ranges, so they are
rejected.
Worki ng note:
Rs.
1. Fixed overhead in the first year 12,06,000
Fixed overhead per month 1,08,000
Contribution per unit (S.P. per unit VC per unit) 80
Hence the break-even number of units will be above 1,350 units

Rs.80
0 Rs.1,08,00

(i i i ) If the number of toys goes beyond the level of 1,500 numbers, one more box will be required to
accommodate each 50 additional units of toys. In that case the additional cost of a box will be
Rs.400/- this amount can be recovered by the additional contribution of 5 toys. Hence, the second
break-even point in such a contingency is 1,505 toys. (Refer to 1(b) (ii) last column of first
statement).
(i v) Comments: Yearly break-even point of 18,000 units of toys in the first instance is equal to 12 times
the monthly break-even point of 1,500 units, because the monthly and yearly figures of break-even
point fell on the upper limit of the respective range.
In the second instance, it is not so because the monthly and early break-even point fell within the
range of 50 toys.

Ans. 10 (Pg. 16):
(a) Statement showi ng total costs i ndi cati ng each i tem of cost
No. of students 60 120 180 240 300
Rs. Rs. Rs. Rs. Rs.
Variable costs:
Break fast 420 840 1,260 1,680 2,100
Lunch 1,800 3,600 5,400 7,200 9,000
Tea 180 360 540 720 900
Entrance fee for Zoo &
Aquarium
300 600 900 1,200 1,500
10
Total (A) 2,700 5,400 8,100 10,800 13,500
Rent of buses 1,400 2,100 2,800 3,500 4,200
(Refer to working note 1)
Special permit fee 100 150 200 250 300
(Refer to working note 2)
Daily allowance paid to
teacher (Refer to working
table)
400 600 800 1,000 1,200
Block entrance fee 200 300 300 450 450
(Refer to given table)
Cost of prizes 1,050 1,050 1,300 1,400 1,500
(Refer to given table)
Total (B) 3,150 4,200 5,400 6,600 7,650
Grand Total (A) + (B) 5,850 9,600 13,500 17,400 21,150
(b) Average cost per student at each of the above l evel s
No. of students: (A) 60 120 180 240 300
Total Costs (Rs.) : (B) 5,850 9,600 13,500 17,400 21,150
[Refer to (a) Part]
Average cost (Rs.): (B)/(A) 97.50 80 75 72.50 70.50

(c) Statement showi ng number of students to break-even
No. of students in
the trip
51-100 101-125 126-150 151-200 201-250 251-300
No. of buses 2 3 3 4 5 6
Semi variable
costs

Bus Rent (Rs.) 1,400 2,100 2,100 2,800 3,500 4,200
Permit fee (Rs.) 100 150 150 200 250 300
Block entrance
fee (Rs.)
200 300 300 300 450 450
Daily allowance
paid to teachers
(Rs.)
400 600 600 800 1,000 1,200
Cost of prizes 1,050 1,050 1,200 1,300 1,400 1,500
Total cost (Rs.) 3,150 4,200 4,350 5,400 6,600 7,650
No. of students to
break even:
105 140 145 180 220 255
(Total semi (Rs.3,150 Rs.4,200/ Rs.4,350/ Rs.5,400/ Rs.6,600/ Rs.7,650
11
variable
cost/contribution
per student)
/ Rs.30 Rs.30 Rs.30 Rs.30 Rs.30 / Rs.30
As the figure of 105 and 140 student fall outside the limits (No. of students in the trip), therefore there are four
break-even points in this case viz., 145,180, 220 and 255 students.
The college authorities should keep these figures in mind while hiring 3, 4, 5 and 6 buses respectively to avoid
losses.
The college incurred loss during the previous year s they hired 5 buses and 72% of total students (216 out of
300 students) joined the trip. The break-even in case college authorities hire 5 buses for the trip comes to 220
students.
Working Notes:
1. No. of buses required and Rent of buses @ Rs.700/- per bus
No. of students 60 120 180 240 300
Bo. of buses 2 3 4 5 6
Rent of buses (Rs.) 1,400 2,100 2,800 3,500 4,200
(No. of buses Rs.700)
2. Special permit fee:
No. of buses Rs.50) 100 150 200 250 300
3. Allowance paid to Teachers (Rs.)
No. of buses Rs.200) 400 600 800 1,000 1,200
4. Contribution per student towards semi-variable costs
Rs.
Collection from each student 65
Subsidy provided by the college
75
10
Less: Variable cost per student
Contribution per student
45

30
12
Activity Based Costing
Ans. 9

The total production overheads are `26,00,000:
Product A: 10,000 `30 = `3,00,000

Product B: 20,000 `40 = `8,00,000

Product C: 30,000 ` 50 = `15,00,000

On the basis of ABC analysis this amount will be apportioned as follows:

Statement of Activity Based Production Cost

Activity Cost
Pool
Cost Driver Ratio Total
Amount
(`)
A
(`)
B
(`)
C
Stores
Receiving
Inspection

Dispatch

Machine Setups

Total Activity
Cost


Quantity Sold

Unit Cost

Add: Conversion
Cost
Total
Purchase
requisition
Production Runs

Orders
Executed
Set ups
6:9:10

5:7:8

6:9:10

12:13:15
2,96,000

8,94,000

2,10,000

12,00,000
71,040

2,23,500

50,400

3,60,000

7,04,940



10,000

70.49

80

150.49
1,06,560

3,12,900

75,600

3,90,000

8,85,060



20,000

44.25

80

124.25
1,18,400

3,57,600

84,000

4,50,000

10,10,000



30,000

33.67

90

123.67

(i) Traditional Method
Ans 10:


`, Cost per Unit

P Q R
Direct Method 90 80 120
Direct Labour 80 240 160
Overhead @ `6/Hr on Machine Hour 60
(10 x6)
108
(18 x 6)
84
(14 x 6)

230 428 364


Workings under ABC
Product No. of Units M Hrs/Unit M Hrs Batches Inspection Purchase Order
P 3000 10 30000 20
(3000/150)
100
(20 x 5)
60
(20 x 3)
Q 5000 18 90000 10
(5000/500)
40
(10 x 4)
100
(10 x 10)
R 20000 14 280000 20
(20000/1000)
60
(20 x 3)
160
(20 x 8)

400000 50 200 320
Overhead @ `6/Hr =4L x 6`24L


[`In ooos]
(ii) Activities % Cost Pool C Driver CDQ CDQ Rate(`)
13
MC Setup 20 480 Batches 50 Batches 9600
M
c
Operation 30 720 M Hrs 4L 1.80
Inspection 40 960 Inspection 200 4800
Mat 10 240 Purchase Order 320 750

100 2400



(iii) Link of overheads
Product Set up cost Machine
Operation Cost
Inspection Cost Purchase
Order Cost
Total Rate
P 192000
(20 x 9600)
54000
[30000x1.8]
480000
[100x4800]
45000
[60x750]
771000 257
Q 96000
[10x9600]
162000
[90000x1.8]
192000
[40x4800]
75000
[100x750]
525000 105
R 192000
[20x9600]
504000
[280000x1.8]
288000
[60x4800]
120000
[160x750]
1104000 55.2

Cost sheet under ABC


P Q R
Direct Material 90 80 120.00
Direct Labour 80 240 160.00
Overhead 257 105 55.20

427 425 335.20
Ans. 11:

(i ) Computation of the activity based overheads
Step 1: Compute cost per unit of cost driver = Cost pool / cost driver volume

Activity Cost Driver Cost Pool
(a)
Cost driver
volume/yr (b)
Cost/Unit of cost
driver (a)/(b)
Purchasing
Setting
Materials
handling
Inspection
Machining
Purchase orders
Batches produced
Material movements

Batches produced
Machine hours
`75,000
`112,000


`96,000
`70,000
`150 000
1,500
2,800


8,000
2,800
50,000
`50/pruchse order
`40/batch


`12/movement
`25/batch
`3/machine hour


Step 2: Compute the volume of cost drivers consumed by Product Nova Shaft

Purchase orders (given) = 25
Batches = 15,000/100 = 150


Materials movement = 150 batches 6 = 900
Machine hours = 15,000 units 0.1 = 1,500
Step 3: Compute the Activity Based Overheads Cost for Product Nova Shaft

Activity Cost Driver Costing
Rate /
Cost
Driver
Unit `

14
Purchasing

Setting

Material handling
Inspection
Machining
Purchase orders
Batches produced
Material movements
Batches produced
Machine hours
50

40

12

25

3
25 order `50

150 batches `40

900 movement `12

150 batches `25

1,500 hours `3
`1,250

`6,000

`10,800

`3,750


`4,500
`26,300
(i i ) Computation of budgeted overheads costs for Product Nova Shaft using absorption
costi ng

Budgeted overheads = (`75,000 + `96,000 + `112,000 + `70,000 + `150,000)
= `503,000

Budgeted absorption cost/machine hour = `503,000 / 50,000 = `10.06

Budgeted machining hours for Product Nova Shaft = 1,500

Budgeted absorbed overhead = 1,500 `10.06 = `15,090

(i i i ) Ways i n which the company can reduce the ABC for product Nova Shaft:

Reduce the number of batches by increasing the batch size which will then reduce the setting up
overhead, materials handling and inspection costs.

Reduce the number of purchase orders

Innovate ways of speeding up production so that the machining hours are reduced.


(a)
Ans. 12:

Sales A B C Total
(i) Units ` 25,000 56,000 27,000 1,08,000
Selling price/unit 18 14 12
(ii) Sales Value (`) 4,50,000 7,84,000 3,24,000 15,58,000
(iii) Prime Cost Overhead 12 9 8
(iv) No. of units/run 2,520 2,810 3,010
(v) Prime Cost ` 3,02,400 5,05,800 2,16,720
(vi) Gross Margin (ii v) 1,47,600 2,78,200 1,07,280 5,33,080
15

Worki ngs:


A B C Total
Gross Production/unit /run (1) 2,520 2,810 3,010
Defectives/run (2) 20 10 10
Good units / run (3) 2,500 2,800 3,000
Sales (Goods units)(4) 25,000 56,000 27,000
No. of runs (5) 10 20 9
Gross Production (6) = (1) (5) 25,200 56,200 27,090
Prime Cost / unit (7) 12 9 8
Prime Cost (8) ` 3,02,400 5,05,800 2,16,720 10,24,920
Inspection hours/run (9) 3 4 4
Inspection hours (10) = (9) (5) 30 80 36 146
M/c hours / run (11) 20 12 30
M/c hours (12) = (1) (5) 200 240 270 710
Dye Cost/run (13) 200 300 250
Dye cost (14) (13) (5) 2,000 6,000 2,250 10,250
Conventi onal Accounti ng System
Total A B C

Sales units / Production (good units) 1,08,000 25,000 56,000 27,000
Gross Margin (`) 5,33,080 1,47,600 2,78,200 1,07,280
Production overheads (`) 2,25,250 52,141 1,16,797 56,313
Selling Overhead (`) 1,62,000 37,500 84,000 40,500
Sub-Total Overhead (`) 3,87,250 89,641 2,00,797 96,813
Net profit (`) 1,45,830 57,959 77,403 10,467
16
Ranking II I III
Acti vi ty Based System

Sub-Total Overhead (`) 62,787 2,16,963 1,07,500
Net profit (`) 84,813 61,237 (220)
Ranking I II III

Ans. 13:
Total Machine hours = Volume Machine hour required for each period
(i) Factory overhead applicable to machine oriented activity = `37424
= (500 ) + (5000 ) + (6001) + (7000 3/2) = 12475 hours
Machine overhead charges = `37424/12475 hours = `3 per hour
Setup Costs = `4355/17 i.e., total number of setups = `256.18
Material ordering cost = `1920/10 operations = `192
Material handling cost = `7580/27 operations = `280.74
Spare parts = `8600/12 parts = `716.67

Products
Overheads Items A B C D
Machine
Overhead
1/4`3 = 0.75 1/4`3 = 0.75 1 `3 = 3.00 3/2`3 = 4.50
Setup cost 1256.18/500 = .51 6256.18/5000=.31 2256.18/600=.85 8256.18/7000=.29
Material ordering
cost
1192/500=.38 4192/5000=.15 1192/600=.32 4192/7000=.29
Material handling
cost
2280.74/500=1.12 10280.74/5000=.56 3280.74/600=1.40 12280.74/7000=.48
Spare parts cost 2716.67/500=2.87 5716.67/5000=.72 12716.67/600=1/19 4716.67/7000=.41

(ii) Competition of overhead per unit based on two system and their difference
Products Machine
overhead
`
Setup
`
Material
ordering
`
Material
handling
`
Spare
parts
`
Total
(ABC
system)
Old
system
`
Difference
A 0.75 0.51 0.38 1.12 2.87 5.63 1.20 +4.43
B 0.75 0.31 0.15 0.56 0.72 2.49 1.20 +1.29
C 3.00 0.85 0.32 1.40 1.19 6.76 4.80 +1.96
D 4.50 0.29 0.11 0.48 0.41 5.79 7.20 -1.41
The traditional system does not make correct assumptions that all overheads are related to volume and
machine time. Under traditional system products A and C are under costed because it misallocates costs for
small volume products. The activity based costing system recognizes the amount of input to each cost unit.
Product B previously avoided its full share of overheads because of its low machine time and may still do so if
part of `37425 of machine oriented overhead should be apportioned on some other basis. Product D is
overcosted because the additional system loaded it with overhead attributable to activities concerned with
products A, B & C as a result of using a volume-based and machine oriented rate which failed to pay proper
attention to activity costing.

Ans.: 14
(i) Statement of Operating income and Operating income as a
percentage of revenues for each product line
17
(When support costs are allocated to product lines on the basis of cost of goods sold of each product)
Soft
Drinks
Rs.
Fresh
Produce
Rs.
Packaged
Foods Rs.
Total Rs.
Revenues: (A) 7,93,500 21,00,600 12,09,900 41,04,000
Cost of Goods sold (COGS):
(B)
6,00,000 15,00,000 9,00,000 30,00,000
Support cost (30% of COGS):
(C)
1,80,000 4,50,000 2,70,000 9,00,000
Total cost: (D) = {(B) + (C)} 7,80,000 19,50,000 11,70,000 39,00,000
Operating income: E= {(A)-
(D)}
13,500 1,50,600 39,900 2,04,000
Operating income as a
percentage of revenues: (E/A)
x 100)
1.70% 7.17% 3.30% 4.97%
Working notes:
1. Total support cost:
Rs.
Bottles returns 12,000
Ordering 1,56,000
Delivery 2,52,000
Shelf stocking 1,72,800
Customer support
Total support cost
3,07,200
2. Percentage of support cost to cost of goods sold (COGS):
9,00,000

= 100
sold goods of cost Total
cost support Total


100
000 , 00 , 30 . Rs
000 , 00 , 9 . Rs
= = 30%

3. Cost for each activity cost driver:
Activity
(1)
Total cost Rs.
(2)
Cost allocation
base
(3)
Cost driver rate
(4)=[(2)(3)]
Ordering 1,56,000 1,560 purchase
orders
100 per purchase
order
Delivery 2,52,000 3,150 deliveries 80 per delivery
Shelf-stocking 1,72,800 8,640 hours 20 per stocking
hour
Customer support 3,07,200 15,36,000 items
sold
0.20 per item sold
(ii) Statement of Operating income and Operating income as a percentage of revenues for each
product line
18
(When support costs are allocated to product lines using an activity-based costing system)
Soft drinks

Rs.
Fresh
Produce
Rs.
Packaged
Food
Rs.
Total

Rs.
Revenues: (A) 7,93,500 21,00,600 12,09,900 41,04,000
Cost & Goods sold 6,00,000 15,00,000 9,00,000 30,00,000
Bottle return costs 12,000 0 0 12,000
Ordering cost*
(360:840:360)
36,000 84,000 36,000 1,56,000
Delivery cost*
(300:2,190:660)
24,000 1,75,200 52,800 2,52,000
Shelf stocking cost*
(540:5,400:2,700)
10,800 1,08,000 54,000 1,72,800
Customer Support cost*
(1,26,000:11,04,000:3,06,000)
25,200 2,20,800 61,200
Total cost: (B)
3,07,200
7,08,000 20,88,000 11,04,000 39,00,000
Operating income C:{(A)-
(B)}
85,500 12,600 1,05,900
Operating income as a % of
revenues
2,04,000
10.78% 0.60% 8.75% 4.97%
* Refer to working note 3
(iii) Comment: Managers believe that activity based costing (ABC) system is more credible
than the traditional costing system. The ABC system distinguishes with different type of
activities at family store more precisely. It also tracks more precisely how individual product
lines use resources.
Soft drinks consume less resources than either fresh produce or packaged food. Soft drinks have
fewer deliveries and require less shelf stocking time.
Family store managers can use ABC information to guide their decisions, such as how to
allocate a planned increase in floor space.
Pricing decision can also be made in a more informed way with ABC information.

Ans. 15

(a) Statement showing total cost of different products, assuming absorption of overhead on a
machine hour basis
Product A Product B Product C Product D
Direct material 40 50 30 60
Direct labour* 28 21 14 21
Overhead 80 60 40 60
Cost of production
per unit
148 131 84 141
Output in units 120 100 80 120
Total Costs (`) 17760 13100 6720 16920
* Rate per machine hour = `26000/1300 hours = `20
Machine Hours = 480 + 300 + 160 + 360 = 1300 hours
(b)
Cost ` Drivers No. Cost/unit of driver
Setups 5250 Production runs 21* `250
Stores/receiving 3600 Requisitions 80@ 45
Inspection/quality 2100 Production runs 21 100
19
Handling/dispatch 4620 Orders 42 110
* Production runs = (120/20) + (100/20) + (80/20) + (120/20)
@ Requisitions = 20 for each product or 80 in total.
It may be pointed out that machine department cost of `10430 will continue to be absorbed on a machine
hour basis as before. The relevant absorption rate will be = `10430/1300 = `8.02 per machine hour.
Total cost (`)
A B C S
Direct material 4800 5000 2400 7200
Direct labour 3360 2100 1120 2520
Set-ups 1500 1250 1000 1500
Stores/receiving 900 900 900 900
Inspection/quality 600 500 400 600
Handling/dispatch 1320 1100 880 1320
Machine dept.
costs
3851 2407 1284 2888
16331 13257 7984 16928
Cost per unit 136.09 132.57 99.80 141.07
(c)
A B C D
Cost per unit (a) 148 131 84 141
Cost per unit (b) 136.09 132.57 99.80 141.07
Difference (11.91) 1.57 15.80 0.07
The total overheads which are spread over the four products have been apportioned on different bases,
causing the product cost to differ substantially in respect of products A and C. A change from traditional
machine hour rate to an activity based system may have effect on:
(a) pricing and profits tot the extent that pricing is based on a cost plus approach.
(b) Reported profits to the extent that stock levels fluctuate between reporting periods.

(a) Total cost of di fferent products (overhead absorpti on on Machi ne hour basi s)
Ans. 16
A
`
B
`
C
`
D
`
Direct material 42 45 40 48
Direct labour 10 09 07 08
Overhead 72 54 36 18
Cost of production per unit 124 108 83 74
Out put in unit 720 600 480 504
Total cost 89,280 64,800 39,840 37,296

Machine hours (720 4 + 600 3 + 480 2 + 504 1) = 6,144 hours.
Rate per hour =
hours 6,144
1,10,592 Rs
= `18 per hour.
(b) Acti vi ty based costi ng system
Set up Store
receiving
Inspection
Machine operation and maintenance cost of
` 63,000 to be distributed in the ratio of 4: 3: 2.
28,000 21,000 14,000

20
Cost ` Drivers No Cost per unit
of driver (`)
Set up 48,000 Production runs 96 500
Store receiving 36,000 Requisitions raised 200 180
Inspection 24,000 Production runs 96 250
Material handling and
disp

2,592
Orders 192
13.50

Production Run for A (720/24) = 30 ; B (600/24) = 25 ; C (480/24) = 20 ; D (504/24) = 21.
A (`) B(`) C(`) D(`)
Direct material 30,240 27,000 19,200 24,192
Direct labour 7,200 5,400 3,360 4,032
Setup 15,000 12,500 10,000 10,500
Store receiving 9,000 9,000 9,000 9,000
Inspection 7,500 6,250 5,000 5,250
Material handling and dispatch 810 675 540 567
Total cost 69,750 60,825 47,100 53,541
Per unit cost 96.875 101.375 98.125 106.23
(c)
A B C D
Cost per unit (a) 124 108 83 74
Cost per unit (b) 96.88 101.38 98.13 106.23
Difference (27.12) (6.62) 15.13 32.23

The total overheads which are spread over the four products have been apportioned on different bases,
causing the product cost to differ substantially: in respect of product A and D a change from traditional machine
hour rate to an activity system may have effect on price and profits to the extent that pricing is based on cost
plus approach.

(a) Budget Cost Statement
Ans. 17:

Activity Activity Cost
(`)
(Budgeted)
Activity Driver No. of Units of
Activity Driver
(Budget)
Activity
Rate
(`)
Deposits Loans Credit
Cards
1.ATM Services

2. Computer
Processing
3. Issuing
Statements
4. Customer
Inquiries
Budgeted Cost
8,00,000

10,00,000

20,00,000

3,60,000

41,60,000
ATM
Transaction
Computer
Transaction
No. of
Statements
Telephone
Minutes
2,00,000

20,00,000

5,00,000

7,20,000
4

0.50

4.00

0.50
6,00,000

7,50,000

14,00,000

1,80,000
-

1,00,000

2,00,000

90,000
2,00,000

1,50,000

4,00,000

90,000
29,30,000 3,90,000 8,40,000
21
Units of product as estimated in the budget period
Budgeted Cost per unit of the product
58,600
50
13,000
30
14,000
60
Worki ng Notes:

(i) ATM 4,00,000 + 2,00,000 + 2 1,00,000 = 8,00,000
(ii) Computer 5,00,000 (Fixed = 2,50,000) Variable= 10,00,000
2,50,000 increase to 3 times = 7,50,000

(iii) Issuing Statements 2,00,000 + 80% 2,00,000 = 2 + 1.6 = 3,60,000

(a) Worki ng:
Ans. 18:
Cal cul ati on of Di rect Labour hours:
`
Total Indirect Costs (`)* 23,85,000
Total Direct labour hours (30,000 +
9,750)
39,750
Overhead absorption rate
hour per 60 Rs.
hours 39,750
23,85,000 Rs.
=
(i ) Statement showi ng total manufacturi ng costs and profi ts

Product A
(60,000 units)
Product B
(15,000 units)
Total (`)
Per unit
Amount (`)
Per unit
Amount (`)

Direct materials 18.75 11,25,000 45.00 6,75,000 18,00,000
Direct labour 10.00 6,00,000 13.00 1,95,000 7,95,000
Prime cost 28.75 17,25,000 58.00 8,70,000 25,95,000
Indirect costs
(absorbed on
the basis of
direct labour
hours)
30.00
(18,00,000/
60,000
units)
18,00,000
(30,000 hours
@ `60 per
hour)
39.00
(5,85,000/
15,000
units)
5,85,000
(9,750 hours
@ `60 per
hour)
23,85,000
Total cost 58.75 35,25,000 97.00 14,55,000 49,80,000
Sales 63.00 37,80,000 137.00 20,55,000 58,35,000
Profit
(Sales Total
cost)
4.25 2,55,000 40.00 6,00,000 8,55,000
* Calculation of total Indirect Cost:
`
Cleaning and maintenance wages 2,70,000
Designing costs 4,50,000
Set-up costs 3,00,000
Manufacturing operations cost 6,37,500
Shipment costs 81,000
Distribution costs 3,91,500
Factory Administration Costs 2,55,000
23,85,000
Indirect cost allocation to products A and B:
22
Product A Product B
Direct labour hours 30,000 9,750
Direct labour hour rate: `
60
60


Indirect costs `18,00,000 5,85,000
Output (units) 60,000 15,000
Cost per unit of output `
30
39
Statement showing the total manufacturing costs and profits using direct labour hour basis of absorption
and treating cleaning and maintenance cost as indirect cost:
Product A Product B Total
`/unit Amount `/unit Amount
Output (units) 60,000 15,000
` ` `
Sales 63.00 37,80,000 137.00 20,55,000 58,35,000
Direct
Materials
18.75 11,25,000 45.00 6,75,000 18,00,000
Direct Labour 10.00 6,00,000 13.00 1,95,000 7,95,000
Prime Cost 28.75 17,25,000 58.00 8,70,000 25,95,000
Indirect costs 30.00 18,00,000 39.00 5,85,000 23,85,000
Total costs 58.75 35,25,000 97.00 14,55,000 49,80,000
Profit 4.25 2,55,000 40.00 6,00,000 8,55,000
(i i ) Cal cul ati on of Setup hours
Product A Product B
Total Output (in units) 60,000 15,000
No. of quantity produced
per batch
240 50
Setup time per batch 2 hours 5 hours
Setup hours (Total)
(No. of batches set up
time per batch)

2
240
60,000
= 500

5
50
15,000
= 1,500
Calculation of Cost Driver, Rates and summary of indirect cost relating to Product A & B:
Activity and Cost Drivers Amount
(`)
Cost Drivers for Product Activity Cost Rates Indirect Costs

A
B (Amount / total of
cost driver)
Product A
Product B
Cleaning & Maintenance
(Direct Labour hours)
2,70,000 30,000 9,750 39,750 6.7925 per Direct
labour hour
2,03,775 66,227
Designing costs (square feet) 4,50,000 30 sq. feet 70 sq. feet 100 4,500 per sq. feet 1,35,000 3,15,000
23
Setup costs (setup hours) 3,00,000 500 hours 1,500 hours 2,000 150 per setup hour 75,000 2,25,000
Manufacturing operations
costs (molding machine
hours)
6,37,500 9,000 3,750 12,750 50 per molding hours 4,50,000 1,87,500
Shipment costs (No. of
shipments)
81,000 100 100 200 405 per shipment 40,500 40,500
Distribution costs (area in
cubic feet)
3,91,500
45,000
cubic feet
22,
500 cubic feet
67,500 5.80 per cubic feet 2,61,000 1,30,500
Factory administration costs
(direct labour hours)
2,55,000 30,000 9,750 39,750 6.4151 per labour
hour
1,92,453 62,547
Production (units) 13,57,728 10,27,274
60,000 15,000
22.63 68.48
Cost Sheet based on acti vi ty based costi ng system:
Description Product A Product B
Total cost Per unit Total cost Per unit
` ` ` `
Sales 37,80,000 63.00 20,55,000 137.00
Direct Cost
Direct
Materials
11,25,000 18.75 6,75,000 45.00
Direct
Labour
6,00,000 10.00 1,95,000 13.00
Total 17,25,000 28.75 8,70,000 58.00
Indirect costs 13,57,728 22.63 10,27,274 68.48
Total costs 30,82,728 51.38 18,97,274 126.48
Profit 6,97,272 11.62 1,57,726 10.52

(i i i ) Compari son of resul ts:
Description Product A Product B
Traditional
Costing
System
Activity
Based
System
Traditional
Costing
System
Activity
Based
System
` ` ` `
Selling Price 63.00 63.00 137.00 137.00
Direct costs 28.75 28.75 58.00 58.00
Indirect costs 30.00 22.63 39.00 68.48
Total cost
per unit
58.75 51.38 97.00 126.48
Profit per
unit
4.25 11.62 40.00 10.52
Opi ni on:
In the traditional costing system, Product B appears to be more profitable than Product A whereas under
the activity based costing system, Product A appears to be more profitable than product B. The activities
24
like designing, set up, manufacturing operation cost, shipment and distribution are support service
activities and the consumption of resources relating to these activities are not dependent on direct labour
hours. The quantum of consumption of resource of each support service activity is different in respect of
the two products manufactured and hence activity based costing presents a true view of cost of
production. Moreover, the suggestion to treat cleaning and maintenance activity as a direct cost pool is
commendable because costs should be charged direct wherever possible. The results reveal that the
company should concentrate upon product B.
Alternative Solution:
Cleaning and maintenance activity will not find a place in the statement of calculation of cost driver rates.
However, other cost driver rates will be unchanged.
Statement showi ng total cost and profi ts on the basi s of Acti vi ty Based Costi ng
Product A Product B Total (`)
Per
unit
Amount (`) Per unit Amount
(`)

Direct materials 18.75 11,25,000 45.00 6,75,000 18,00,000
Direct labour 10.00 6,00,000 13.00 1,95,000 7,95,000
Cleaning &
maintenance
expenses
2.00 1,20,000* 10.00 1,50,000* 2,70,000
Prime cost 30.75 18,45,000 68.00 10,20,000 28,65,000
Indirect costs:
Designing 2.25 1,35,000 21.00 3,15,000 4,50,000
Setup 1.25 75,000 15.00 2,25,000 3,00,000
Manufacturing
operation
7.50 4,50,000 12.50 1,87,500 6,37,500
Shipments 0.67 40,500 2.70 40,500 81,000
Distribution 4.35 2,61,000 8.70 1,30,500 3,91,500
Factory
administration
3.21 1,92,453 4.17 62,547 2,55,000
Total indirect costs 19.23 11,53,953 64.07 9,61,047 21,15,000
Total costs 49.98 29,98,953 132.07 19,81,047 49,80,000
Sales 63.00 37,80,000 137.00 20,55,000 58,35,000
Profits
(Sales total costs)

13.23

7,81,047

4.93

74,953

8,55,000

* The Cost Accountant identified `1,20,000 for Product A and balance
`1,50,000 of cleaning and maintenance wages for Product B.
(i i i ) Compari son of resul ts:
Product A Product B
Allocation basis Direct
Labour
Activity
Based
Direct
Labour
Activity
Based
25
Hour Costing Hour Costing
Selling Price 63 63 137.00 137.00
Prime cost 28.75 30.75 58.00 68.00
Total Indirect costs 30.00 19.23 39.00 64.07
Total costs
(Prime cost + Total
indirect costs)


58.75


49.98


97.00


132.07
Profit per unit 4.25 13.02 40.00 4.93
Comments:
It is evident from the comparison of results that under single cost pool system the product A
is overcost and product B is undercost. This is due to allocation of indirect cost on the basis
of blanket rate based on direct labour hour and considering one of the significant cost as an
indirect one. Cost Accountants decision for allocation of indirect costs on the basis of
ABC methods and identifying be cleaning and maintenance cost as direct element of cost
appears to be a good decision. Result show that the firm enjoys competitive advantage with
regards to product A.

Ans. 19
(1) Single factory direct labour hour overhead rate =
2,000
3,10,000 Rs
= ` 155 per direct labour hour
Computation of unit cost ( existing system)
R (`) S(`) T(`)
Direct labour cost @ ` 12 per hour 300 5,760 600
Direct material 1,200 2,900 1,800
Overheads(direct labour hours ` 155 per hour 3,875 74,400 7,750
5,375 83,060 10,150
Quantity Produced (No) 560 12,800 2,400
Cost per unit 9.60 6.49 4.23
(2) ABC system involves the following stages,
1. Identifying the major activities that take place in an organisation.
2. Creating a cost pool /cost centre for each activity
3. Determining the cost driver for each activity
4. Assigning the cost of activities to cost objects (e.g. products, components, customers etc)
The most significant activities have been identified e.g. receiving components consignments from
suppliers, setting up equipment for production runs, quality inspections, and despatching orders to
customers. The following shows the assignment of the costs to these activities,
(` ,000)
Receiving
supplies
Set ups Quality
inspection
Despatch Total
Equipment operation expenses 18.75 87.50 18.75 125.00
Maintenance 3.75 17.50 3.75 25.00
Technicians wages initially 3.83 17.85 3.82 25.50
26
allocated to Maintenance(30% of
` 85,000= ` 25,500 and then
reallocated on same basis on
maintenance)
Balance of technicians wages
allocated to set ups and quality
inspections
34.00 25.50 59.50
Stores wages - Receiving 35.00 35.00
Despatch wages - Despatch 40.00 40.00
61.33 156.85 25.50 66.32 310.00

Note : Equipment operation expenses and Maintenance allocated on the basis 15%,70% and 15% as
specified in the question.
The next stage is to identify the cost drivers for each activity and establish cost driver rates by dividing the
activity costs by a measure of cost driver usage for the period. The calculations are as follows :-
Receiving supplies (
980
61,330 Rs
) = ` 62.58 per component.
Performing set ups (
1,020
1,56,850
) = ` 153.77 per set up
Despatching goods (
420
320 , 66
) = ` 157.93 per despatch
Quality inspection (
640
500 , 25
) = ` 39.84 per quality inspection
Finally, costs are assigned to components based on their cost driver usage. The assignments are as
follows,
R (`) S(`) T(`)
Direct labour 300 5,760 600
Direct materials 1,200 2,900 1,800
Receiving supplies 2,628.36 1,501.92 1,752.24
Performing set ups 2,460.32 2,767.86 1,845.24
Quality inspections 398.40 318.72 717.12
Despatching goods 3,474.46 13,424.05 7,264.78
Total costs 10,461.54 26,672.55 13,979.38
No of units produced 560 12,800 2,400
Cost per unit 18.682 2.08 5.82


For components, the overhead costs have been assigned as follows,
(Component R)
Receiving supplies (42 receipts at ` 62.58)
Performing set ups (16 production runs at ` 153.77)
Quality inspections (10 at ` 39.84)
27
Despatching goods ( 22 at ` 157.93).

Ans 20:
Overhead rate per labour hour

= Overhead incurred in first half year = `21,00,000 = `52.50 per labour hour
Direct labour hours worked 40,000 hours

Apportionment of technical staff salaries
Machine maintenance = 6,37,500 X 31/100 = ` 1,91,250
Set up = 6,37,500 X 40/100 = ` 2,55,000
Quality Inspection = 6,37,500 X 30/100 = ` 1,91,250

Statement showing apportionment of Machine operation and Machine maintenance between stares and
production activity (set up) in ratio 20:80
Particulars Total
Expenses
Stores /
Receiving
Set up/
Production run
Machine operation
Machine maintenance
(`1,87,500 + `1,91,250)
10,12,500
3,78,750
2,02,500
75,750
8,10,000
3,03,000

Particulars Total
Expenses
Stores /
Receiving
Set up /
Production run
Wages and salaries of stores staff
Component of set- up cost

Total
2,62,500
2,55,000
2,62,500
-
-
2,55,000
19,08,750 5,40,750 13,68,000

Rate per activity cost driver
Particulars Stores /
Receiving
Set up/
Production run
Quality
inspection
Total overheads (`)
Units of activities carries out
Rate per activity cost driver (`)
5,40,750
1,960
275.89
13,68,000
2,040
670.59
1,91,250
1,280
149.41

Statement showing computation of cost of products P and Q (Based on the existing system of single
overhead recovery rate)
Particulars Product
P Q
Direct Labour hours
Unit made
Direct materials cost
Direct labour cost (@ `6 per D.L.H.)
Overheads ( @ `52.50 per D.L.H.)
Total cost of products

Cost per unit
960
15,000
6,000
5,760
50,400
62,160
100
5,000
4,000
600
5,250
9,850

4.144

1.97

Statement showing computation af cost of products P and Q (Using activity based costing system)
28
Particulars Product
P Q
Units
Direct materials cost
Receiving/ Stores cost
Receiving Stores cost (48 X 275.89)
(52 X 275.89)
Production runs / Set ups cost (36 X 670.59)
(24 X 670.59)
Inspection cost (30 X 149.41)
(10 X 149.41)
Total Cost products
Coat per unit

15,000
6,000
5,760
13,243

24,141

4,482


5,000
4,000
600
14,346

16,094

1,494
53,626 36,534
3.58 7.31

Computation of sales value per quarter of component K (Using activity based costing system)
Units of component K
To be delivered per quarter
3,000

Component of initial design cost per quarter ( `60,0000/8 quarters)
Direct material costs
Direct labour cost (600 hours X `6)
Receiving cost (50 X `275.89)
Production runs cost (6 X `670.59)
Inspection cost (24 X `149.41)
Total cost
Add: Mark up (25% of cost)
Sales value
Selling price per unit of K (`43,035/3,000 units)
`
7,500
12,000
1,800
5,518
4,024
3,586
34,428
8,607
43,035
16.34

Ans 21
(i) Job cost sheet for Host Restaurant and Pizza Hut (using a simplified costing system)
Host
Restaurant
(`)
Pizza Hut


(`)
Professional labour cost:

25 hours @ `60 per hour
1,500

40 hours @ `60 per hour
2,400
(Refer to working note 1)

Professional Support staff

25 hours @ `120per hour
3,000

40 hours @ `120 per hour
4,800
(Refer to working note 2)

Total

4,500 7,200
(ii) Job cost sheet using an Activity based costing
Host Restaurant Pizza Hut
(`) (`)
Professional labour cost

500

29
5 hours @ `100 per hour
3,000
30 hours @ `100 per hour
(Refer to working note 3)

Associate labour cost

800

20 hours @ `40
400
10 hours @ `40
(Refer to working note 4)

Design support

1,690

`1.30 `1,300
4,420
`1.30 `3,400
(Refer to working note 5)

Staff support

1,056

25 hours @ `42.22
1,689
40 hours @ `42.22
(Refer to working note 6)


4,046 9,509
(iii) Determining the amount by which each job was under or overcosted using a simplified costing
system.

Host Restaurant


(`)
Pizza
Hut
(`)
Cost using simplified system

4,500 7,200
Cost using Activity Based system

4,046 9,509
Difference

454 (2,309)
The simplified costing system overcosted Host Restaurant job by `454 and undercosted
Pizza Hut job by `2,309.

30




31
Ans. 22:
(i ) Compari son of manufacturi ng cost per uni t.
Audio Player Model
AB 100 AB 200
` `
Direct material cost 1,000.00 800.00
Direct manufacturing labour cost 200.00 180.00
Machining costs 200.00 160.00
Testing costs 250.00 200.00
Rework costs 150.00 75.00
Ordering costs 2.00 1.25
Engineering costs 198.00 198.00
Total manufacturing cost per unit 2,000.00 1,614.25
Working notes for audio player model AB 200
(i) Machining hours and cost: Machining hours = (1 hour0.20 hours) or 0.80 hours)
Machining cost is 0.80 hours `200 or `160
(ii) Testing hours and cost: Testing hours = 2 hours (1 hour 0.20) or 1.60 hours.
Testing cost is 1.60 hours `125 or `200
(iii) Rework cost per unit:
Rework units = 5% 10,000 units or 500 units. Rework cost = 500 units `1,500 or `7,50,000.
Rework cost per unit `7,50,000 / 10,000 units or `75 per unit.
(iv) Ordering cost:
No. of orders per month 50 components 2 orders = 100
Ordering cost per month 100 orders `125 per order = `12,500
Ordering cost per unit = `12,500 / 10,000 units = `1.25 per unit.
(v) It is assumed that total available engineering hours will be used for manufacturing AB 200 model of audio
player.
(i i ) Effect of desi gn change and pri ci ng deci si on on operati ng i ncome of ABC.
(`Lakhs)
Revenue loss on 10,000 units (40)
(`10,000 units `400)
Saving in cost:
Direct material costs 20.00
(`200 10,000 units)
Direct manufacturing labour costs 2.00
(`20 10,000 units)
Rework costs 7.50 29.50
32
(5% 10,000 units `1,500)
Net effect on operating income (10.50)
Conclusion:
Operating income per month will be reduced by `10.50 Lakhs.
Effects of reduction in components, machining time, and testing time will not have any immediate effect,
because it is difficult to adjust the available facilities in ordering department, machining department and testing
department.
33
Target Costing, Value Chain Analysis
Maximum capacity 80,000 units
Ans. 7:
Presented sales 20,000 units @ `100 p.u.
Selling price/unit Demand
100 20,000
90 40,000
80 80,000 = Full capacity
Target cost/unit = 80 25% of sales
= 80- 20 = 60 p.u.
(b) At present
Variable cost/unit = 40% of cost i.e. 75 = `30
Fixed cost/unit = 100 25% = 75
COS 75
Less: Variable cost/unit
Fixed cost 45 p.u. Total fixed
cost 45

80,000 = 36 lakhs
30
Add full capacity target cost = `60/unit 80,000 units

= `48 lakhs
Total estimate cost
Fixed cost 36 lakhs
Variable cost (80,000

40)
60 lakhs
24 lakhs
Required. Cost reduction following value engineering is `12 lakhs.

(e) Rate of return 15% Profit p.u. 25% of 80 = 20/unit
Profit before tax = 20

80,000 = 16 lakhs
ROCE = (PBI/Investment)
Investment = (PBI/ROCE) = 16 lakhs/15% = `10666667.

Ans. 8

:
Target profit 25,000
Add: Fixed cost 1,40,000
Add: Additional Advertisement
(a) Total contribution
28,500
1,93,500
(b) Required. Sales volume 12,000
contribution/unit (ab) 16.125
Target Selling price/unit 32
Less: Contribution/unit
Target variable cost p.u.
16.125
15.875
Less: material cost p.u.

Labour + Variable overhead
8.000

7.875
34
Labour: x hr. @ 4

Variable overhead x hr. @ 0.5

4.5x = 7.875
x (hr.) 1.75

Time/unit 1.75

Present _

2.00
Time reduced 0.25 hr.


(i) Cost of product as per Target Costing
Ans. 9
Coco Stawberry Vanilla
Selling Price per unit 23.00 18.00 13.00
Less: Markup (25% of cost or 20% of
selling Price)
4.60 3.60 2.60
Target Cost per unit (`) 18.40 14.40 10.40

(ii) Cost of product as per Traditional Costing
Coco Stawberry Vanilla
Maximum Volume (units) 60,500 24,200 72,600
` ` `
Material 8.00 6.00 5.00
Labour 5.00 4.00 3.00
Prime Cost 13.00 10.00 8.00
Store Support (30% of Prime Cost) 3.90 3.00 2.40
Total Cost per unit 16.90 13.00 10.40
Total Cost 10,22,450 3,14,600 7,55,040

(iii) Cost of product as per Activity Based Costing
Coco Stawberry Vanilla
Maximum Volume (units) 60,500 24,200 72,600
` ` `
Material 8.00 6.00 5.00
Labour 5.00 4.00 3.00
Prime Cost 13.00 10.00 8.00
Overheads (Working Note-2) 3.29 5.23 2.17
Total Cost per unit 16.29 15.23 10.17
Total Cost 9,85,320 3,68,670 7,38,100

(iv) Comparision in Cost of each product under each method
Coco Stawberry Vanilla
As per Target Costing 18.40 14.40 10.40
As per Traditional Costing 16.90 13.00 10.40
As per Activity based Costing 16.29 15.23 10.17

Comment: Since cost of Strawberry is high in ABC costing in comparison to target costing and traditional
methods, it is indicating that actual profit under target costing is less than targeted.

Working Note-1 :

Coco Stawberry Vanilla
Current Selling Price per unit (`) 25.00 20.00 15.00
Current Sales (units) 50,000 20,000 60,000
Selling Price (`) 24.00 19.00 14.00
Revised Sales (units) 55,000 22,000 66,000
Selling Price (`) 23.00 18.00 13.00
Revised Sales (units) (upto production capacity) 60,500 24,200 72,600

35

Working Note-2 :

Coco Stawberry Vanilla
Ordering Cost (35/30/15 @ 800) 28,000 24,000 12,000
Delivery Cost (112/66/48 @ 700) 78,400 46,200 33,600
Shelf Stocking (130/150/160 @ 199) 25,870 29,850 31,840
Customer Support
(60,500/24,200/72,600 @ 1.1)
66,550 26,620 79,860
TOTAL COST 1,98,820 1,26,670 1,57,300
No. of units 60,500 24,200 72,600
Cost per unit 3.29 5.23 2.17
Note: On calculation of total overhead costs under traditional & ABC system, costs are same i.e. `4,82,790,
hence we will ignore the line In ABC these costs are coming under customer support and assistance. written
in question.

(a) (i ) The target cost of each product after reduction is computed as follows:
Ans. 10:

Product Present Price
(`)
Proposed Price
(`)
Target Cost (`)
(with 25% Margin)
A
B
C
D
180
175
130
180
175
170
125
175
140
136
100
140

(i i ) Statement showi ng cost/uni t of Dri ver as per ABC

Cost Amount Driver No. Cost/unit of
Driver
Set-ups 26,250 Production runs 105* `250.00
Stores receiving 18,000 Requisition 400** `45.00
Inspection/Quality 10,500 Production runs 105 `100.00
Handling/Dispatch 23,100 Orders 210 `110.00
Machine Department 52,130 Machine Hrs. 6,500 `8.02

* Production runs = (600/20) + (500/20) + (400/20) + (600/20) = 105

** Requisitions = 100 for each product or 400 total

Machine hours = 2,400 + 1,500 + 800 + 1,800 = 6,500 hours.

Statement showing Total Cost and Cost Per Unit as per ABC

Item A B C D
` ` ` `
Direct Material 24,000 25,000 12,000 36,000
Direct Labour 16,800 10,500 5,600 12,600
Set-up 7,500 6,250 5,000 7,500
Stores receiving 4,500 4,500 4,500 4,500
Inspection/Quality 3,000 2,500 2,000 3,000
Handling/Dispatch 6,600 5,500 4,400 6,600
36
Machine Dept. Cost 19,248 12,030 6,416 14,436
Total Cost 81,648 66,280 39,916 84,636
Output (Units) 600 500 400 600
Cost per unit 136.08 132.56 99.79 141.06
(iii) Comparison of Actual Cost and Target Cost

Cost A B C D
` ` ` `
Actual 136.08 132.56 99.79 141.06
Target 140.00 136.00 100.00 140.00
Difference (-) 3.92 (-) 3.44 (-) 0.21 (+) 1.06

Comment:

The total actual cost of A, B and C product is less than the target cost so there is no problem in
reducing the cost of these product by `5 from the present price. It will increase the profitability of
the company but the cost of D is slightly more than the target cost, it is therefore, suggested
that the company should either control it or redesign it.

Ans. 11

: Working Notes:
Part icul ars P Q
(a) Production/Sales Quantity (units) 1,00,000 50,000
(b) Batch Size (units) 1000 500
(c) No. of batches 100 100
(d) Set up time per batch (hours) 30 36
(e) Total set up hours (c d) (hours) 3,000 3,600
(f) Machine set up cost (`)
4,62,000
(g) Cost driver per machine set up hour

4,62,000
= `70

6,600

(h) Testing time per unit 5 hours 9 hours
(i) Total testing time (a h) (hours) 5,00,000 4,50,000
(j) Testing cost

`23,75,000

(k) Cost driver per testing hour
23,75,000
= `2.50

9,50,000

(a) Computati on of ful l cost per uni t usi ng Acti vi t y Based Costi ng:

Part icul ars Basi s P Q
Direct material Direct 42,00,000 30,00,000
Direct labour Direct 15,00,000 10,00,000
Direct machine cost Direct 7,00,000 5,50,000
Machine set up cost 3,000 hours @ `70 2,10,000

3,600 hours @ `70
2,52,000
37
Testing cost 5,00,000 hours @ `2.50 12,50,000

4,50,000 hours @ `2.50
11,25,000
Engineering cost Allocated 8,40,000 14,10,000
Total cost (`)
87,00,000 73,37,000
Cost per unit (`) 87.00 146.74
(b) Mark up on full cost basi s for Product P:


Part icul ars Per uni t
Selling price 100.05
Less: Full cost 87.00
Mark up 13.05
Percentage of mark up on full cost = 13.05 /87 = 15 %
(c) Target cost of Product P after new desi gn i s i mpl emented

Target price (given) 86.25

Mark-up
86.25 15 11.25

115
Target cost per unit (`) 75.00


(d) Statement of cost for new design of P

Parti cul ars Basi s Cost P.U. Total Cost

Direct Material
Decreased by `5 p.u.
37.00 37,00,000
Direct Labour
Decreased by `2 p.u.
13.00 13,00,000
Direct Machining cost No change as machine is 7.00 7,00,000

dedicated

Machine set up cost
100 set up 28 hours `70
1.96 1,96,000
Testing cost 1,00,000 units
`2.5
4 hours 10.00 10,00,000
Engineering cost No change 8.40 8,40,000
Total cost 77.36 77,36,000


The target cost is `75 p.u. and estimated cost of new design is `77.36 p.u.
The new design does not achieve the target cost set by Computo Ltd. Hence the target mark
up shall not be achieved.
(e) Possi bl e Management Acti on:
Value engineering and value analysis to reduce the direct material costs.

Time and motion study in order to redefine the direct labour time and related costs.
Exploring possibility of cost reduction in direct machining cost by using appropriate
techniques.

Identification of non-value added activities and eliminating them in order to reduce
overheads.

The expected selling price based on estimated cost of `77.36 per unit is (`77.36 + 15%)
`88.96. Introduce sensitivity analysis after implementation of new design to study the sales
quantity changes in the price range of `
86.25 to `88.96.


Ans. 12:
38
P1 P2
`/unit `/unit
Material 407.5 292.1
Overhead-Material handling 851.2 = 102 461.2 = 55.2
Assembly Management 403.2 = 128 401.9 = 76
Machine insertion 480.7 = 33.6 310.7 = 21.7
Manual insertion 362.1 = 75.6 252.1= 31.5
Quality testing 1.42 1.125 5 = 35
Present cost
= 27.5
781.70 504.00
Target cost 680.00 390.00


Revised P1

Revised P2
`/unit `/unit
Direct material 381.20 263.10
Overhead:
Material handling (711.2) = 85.2 (391.2) = 46.8
Assembly hour (2140) = 84.0 (1.640) = 64.0
Machine inspection (590.7) = 41.3 (290.7) = 20.30
Manual inspection (122.10) = 25.2 (102.10) = 21.00
Electronics (1.225) = 30.00 (0.925) = 22.50
Estimated cost 646.90 437.70
Target cost 680.00 390.00
Achieved not achieved

Ans. 2

4: Machine X-Life 12 years
Year Cost Discount Discounted
` Factor Cost `
Purchase price 0 19,000 1.00 19,000
Overhead cost 8 4,000 0.47 1,880
Trade-in-value 12 (3,000) 0.32 (960)
Annual repair cost 1-12 2,000 6.81 13,620
33,540
Annualized equivalent =`33,540 / 6.81=`4,925

Machine W-Life 6 years
Year Cost Discount Discounted
` Factor Cost `
Purchase price 0 13,000 1.00 13,000
Overhead cost 4 2,000 0.68 1,360
Trade-in value 6 (3,000) 0.56 (1,680)
Annual repair cost 1-6 2,600 4.36 11,336
24,016
Annualized equivalent `24,601 / 4.36=`5,508
Recommendation : Purchase machine X
Assumptions:
a. Same performance, capacity and speed.
b. No. inflation.
c. 12 year-estimates are as accurate as 6 year estimates.
39
d. Cash flow at the year end.

Ans. 25
Product design = `250 per design hour (`2m/8000 hours)
: The cost driver rates are as follows:
Purchasing = `50 per purchase order (`200000/4000 orders)
Production (excluding depreciation) = `100 per machine hour ((`1 500000-`300000)/ 12000 hours)
Packing =`20 per cubic meter (`400000/ 20000)
Distribution =`5 per kg (`600000/ 120000)
The activity based overhead cost per unit is as follows:
Product design (400 design hours at `250 per hour=`100000
(`)
Divided by life cycle output of 5000 units) 20.00
Purchasing (5 purchase orders at 50 units per order costing
A total of `250 per output of 250 units) 1.00
Production (0.75 machine hours at `100 per machine hour) 75.00
Depreciation (Asset cost over life cycle of 4 years= 16 quarters
Depreciation at `8000 per quarter divided by life cycle
Output of 5000 units) 25.60
Packing (0.4 cubic meters at `20) 8.00
Distribution (3 kg at `5)
Total costs
15.00

144.60
Ans. 26
(i) So total cost for Electric immersion heater =`160 + 200X5 =`1160
: The total cost consists of the installation cost plus electrical charges for 5 years.
(ii) Total cost for a gas boiler =`760 + `80X5 =`1160
Hence, on the total cost basis, both the equipments have equal preference, and the housewife can choose
any one. Let us now calculate the present value of money for each of the two possibilities.
Year PV factor @
9% p.a
Electric Immersion heater Gas Boiler
Operating Cost ` Discounted Cost ` Operating Cost
`
Discounted
Cost `
0 1.0000 160 160.00 760 760.00
1 0.9174 200 183.48 80 73.39
2 0.8417 200 168.34 80 67.33
3 0.7722 200 154.44 80 61.78
4 0.7084 200 141.68 80 56.67
5 0.6499 200 129.98 80 51.99
Total Cost=937.92
(`938,say)
Total Cost
=1071.16
(`1071 say)

On the basis of present value @ 9% p.a over a period of five years, the total cost of Electric immersion heater
is `938 and that of a Gas boiler is `1071. Hence, the housewife is advised to purchase an electric immersion
heater.
If the equipment are to be considered for a period of 8 years, then
Total cost for electrical immersion heater =`160+200X8 =`1760
Total cost for gas boiler =`760+`80X8 =`1400
Hence, the housewife will be advised to purchase a gas boiler.
Year PV factor @
9% p.a
Electric Immersion heater Gas Boiler
Operating Cost ` Discounted Cost ` Operating Cost
`
Discounted
Cost `
6 0.5963 200 119.26 80 47.70
7 0.5470 200 109.40 80 43.76
8 0.5019 200 100.38 80 40.15
329.04 (329,say) 131.61 (`132
say)

40
Present value in case of electric immersion heater
= P.V. over five years + P.V. over next three years =`938+`329 =`1267
Present value in case of gas boiler =`1071+`132 =`1203
Hence, over a 8 years period, the present value of a gas boiler is less.
On the basis of total cost as well as present value of money, gas boiler is cheaper over 8 years period, hence
the housewife is advised to purchase a gas boiler.

Ans. 2
Purchase Cost (Cash outflow) (a)
7: Relevant Operating Cash outflow p.a. if part X 248 is outsourced
50000
Relevant Cash inflow from outsourcing:
Direct materials 22000
Direct Labour 11000
Variable Overhead 7000
Product and Process engineering 4000
Rent
Total Cash Savings (b)
1000
Net Cash Outflow (a) - (b)
45000
(5000)
Net Present Value of cash inflow if part is outsourced
Particulars Year Amount ` P.V. factor @ 12% P.V `
Disposal value of machine 0 15000 1000 15000
Cash Outflow due to outsourcing 1 5000 0.893 (4465)
2 5000 0.797 (3985)
3 5000 0.712 (3560)
4 5000 0.636 (3180)
5 5000 0.567
NPV
(2835)

Analysis : Since the NPV is negative , it is desirable to manufacture the part internally.
(3025)
Notes:
(1) Equipment depreciation is a non- cash cost item. Therefore, it is not relevant.
(2) Product and process engineering cost being avoidable hence relevant for the entire period of
outsourcing i.e. for 5 years.
(3) Allocated rent is irrelevant but rent saved (i.e, `1000) is relevant.
(4) Allocated general plant overhead is irrelevant.

(ii) Sensitivity analysis with respect to quantity is desirable:
If demand for the part decreases vendor is willing to supply a lower quantity at the same price (`
50/-).
If the part is continued to be made internally, the costs would not decrease quite fast with lower
quantities because of fixed costs.
Net cash outflows of outsourcing will be smaller if lower quantities of the part are demanded. But if
the demand increase, it would be preferable to make the part in house.

Non financial factors:
Will the units of part required be delivered on schedule?
Will quality be maintained?
Can suggested modifications be really accommodated?
Will the subcontractor remain in business for next five years?

(iii) As the outsourcing of part X 248 will start from July 1998, the bonus of Gemini enterprises based on
the accounting income, which Mr. Sen wishes to maximise will remain unchanged for the year 1997 - 98


Ans. 28
Alternative I :Repairs to existing machine: (`)
: Evaluation of Alternative proposals
Cost of Repairs 19000 X 50 / 100 =`9500
Equivalent annual cost for 5 years (9500 / 3.791) 2506
Add: Running and Maintenance cost p.a net of tax (20000 X 50 / 100)
Present value of cash outflows p.a
10000
12506

41
Alternative II : Replace the old machine
Purchase cost of new machine 49000
Less: sale proceeds of old machine
Net: Cash Outflow
5000
Equivalent annual cost for 10 years (44000 / 6.145)
44000
7160
Add: Running and maintenance cost p.a. net of tax (14000 X 50 / 100)

7000
14160
Less : Tax Saving on depreciation (49000 / 10 ) X 50 / 100
Present value of cash outflow p.a.
2450
Analysis : From the above analysis it is observed that alternative II i.e., replacement of old machine with a
new machine is more profitable, since the cash outflow p.a. will decrease by `796 (i.e. `12506 `11710 ) if
old machine is replaced with new machine.
11710

42
Costing in Service Sector
Ans. 8.
Total Room days = No of rooms x Days in a year = 300 rooms x 365 days
= 10,95,000 Rooms days
= Rs. 50-Rs.10 = Rs.40

Dally contribution required per room (Rs.)
Desired profit after tax
Add Income Tax (Rs.6,00,000X40/60)
600000
400000
Desired profit before tax
Add: Fixed cost
10,00,000
7,50,000
Total Revenue to be earned 17,50,000
No. of room days to be rented = Rs.17,50,000 / Rs.40 = 43.750 Room days
No. of rooms to be rented to attain break- even = Rs.7,50,000 / Rs.40 = 18.750 Room days

Ans 9:
Room Occupancy days per annum
Single rooms (180 rooms X 365days X85/100) 55845
Double rooms (60 rooms X 365 days X 85/100) 18615

Variable and Fixed cost p.a.
Particulars Single rooms Double rooms Total
Room occupancy days 55845 18615
Variable cost per day 300 500
Total Variable cost 16753500 9307500 26061000
Fixed cost per room day 500 780
Total Fixed cost 27922500 14519700 42442200

Margin of Safety desired at 20% of total revenue. Therefore, Break even should be at 80% of total revenue.
Revenue at break even level = Variable cost + Fixed cost = 26061000+42442200 = Rs. 68503200
Desired total revenue to be = Rs. 68503200 X 100/80 = Rs. 85629000

(i) Computation of tariff per room day
Single room days occupancy 55845
Double room days occupancy equivalent to single room day (18615X160/100) 29784
Total single room days 85629
Rent per single room day = Rs. 85620000/85629 room days = Rs. 1000
Rent per double room day = Rs. 1000 X 160/100 = Rs. 1600
Tariff per room for single room = Rs. 1000 X 100/80 = Rs. 1250
Tariff per room for double room = Rs. 1600 X 100/80 = Rs. 2000

(ii) Computation of increase in occupancy of the remaining single rooms days required to compensate the
loss arising from the discount.
Number of single rooms intends to reserve for corporate customers = 12
Occupancy days for reserved rooms = 12 rooms X 365 days X 85/100 = 3723
Discount given on room rent per day = Rs. 1000 X 10/100 = Rs. 100
Amount of revenue lost due to discounting = 3723 room days X 100 = Rs.372300
Contribution per day on a single room = Rs. 1000- Rs. 300 = Rs. 700
Increase in occupancy days required in single rooms = Rs. 372300/Rs. 700 = 532 days

Ans. 10
Working Name:
Calculation of occupancy
(a) Single room occupancy p. a. ( 100 rooms X 365 days X 75/100) 27,375
43
(b) Double room occupancy p. a. ( 20 rooms X 365 X 75/1000) = 5475
Conversion of double room to single room occupancy ( 5,475 X 1.20)

6,570
Total 33,945

Statement of Rent chargeable to single room and double room per day
Particulars Single Room Double room
No. of occupancy days (a) 27,375 5,475
Costs per day
Variable cost
Fixed cost
Rs.
400
200
Rs.
500
250
(b) 600 750
Total (Rs.) (a) X (b) 1,64,25,000 41,06,250

(Rs.)
Total Cost (Rs.1,64,25,000 + Rs.41,06,250)
Add: 20% Margin safety on hire of room (Rs.25% 0n cost)
2,05,31,250
51,32,812
Total rental charges to be received 2,56,64,062

Room rent per day to be collected
(Rs.)
(a) Single Room (Rs.2,56,64,062 / 33,945) 756
(b) Double Room (Rs.756 X 1.20) 907

Profitability statement of restaurant (Rs.)
Sales Revenue (Rs.1,00,000 X 365 days) 3,65,00,000
Contribution (30% of Rs.365 Lakhs)
Less: Fixed cost p. a.
1,09,50,000
10,00,000
Profit 99,50,000

Profitability statement of sports centre
(Rs.)
Contribution p. a. (50 persons X Rs.50X 365 days)
Less: Fixed Cost p. a.
9,12,500
5,00,000
Profit 4,12,500

Profitability statement of shopping arcade
(Rs.)
Contribution p. a. (Rs.50,000 X 12 months)
Less: Fixed Cost p. a.
6,00,000
6,00,000
Profit Nil

Ans. 11
(i) Income Statement of Kangan Resort for the next year

Rs.

Sales Revenue
Lodging house room receipts (40 Rooms 200 days Rs.
200 85%)

13,60,000

Shopping Arcade (40 Rooms 2 persons 200 days Rs.
50 85%)

6,80,000
Restaurant (40 Rooms 2 persons 200 days) Rs. 80
85%)
10,88,000
44
Total Sales Revenue 31,28,000
Less: Variable Cost
Lodging house rooms (40 Rooms 200 days Rs. 30
85%)
2,04,000
Shopping Arcade (50% of Rs. 6,80,000) 3,40,000
Restaurant (60% of Rs. 10,88 ,000) 6,52,800
Total Variable Cost 11,96,800
Contribution (Total Sales Revenue Total Variable Cost) 19,31,200
Less: Fixed Cost 10,00,000
Profit (Estimated) 9,31,200


(ii) Income Statement on the basis of reduced room rent

Rs.

Sales Revenue
Lodging house room receipts (40 Rooms 200 days Rs.
150 95%)

11,40,000

Shopping Arcade (40 Rooms 2 persons 200 days Rs.
50 95%)

7,60,000
Restaurant (40 Rooms 2 persons 200 days) Rs. 80
95%)
12,16,000
Total Sales Revenue 31,16,000
Less: Variable Cost
Lodging house rooms (40 Rooms 200 days Rs. 30
85%)
2,28,000
Shopping Arcade (50% of Rs. 7,60,000) 3,80,000
Restaurant (60% of Rs. 12,16 ,000) 7,29,600
Total Variable Cost 13,37,600
Contribution (Total Sales Revenue Total Variable Cost) 17,78,400
Less: Fixed Cost 10,00,000
Profit 7,78,400
The profitability decreases by 9,31,200 7,78,400 = Rs. 1,52,800. Hence reducing room rent
proposal may not be accepted.

Ans. 12
Estimated Income Statement for the coming year (Rs.)
Revenue
Hotel Room Rent (100 rooms X 250 days X Rs.150 X 75/100)
Receipts from shop (100 rooms X 2 persons X 250 days X Rs.30 X 75/100)
Receipts from Restaurant (100 rooms X 2 persons X 250 days X Rs.60 X 75/100)

28,12,500
11,25,000
22,50,000
(a) 61,87,500
Variable Cost
Hotel Rooms (100 rooms X 250 days X Rs.25 X 75/100)
Shops (Rs.11,25,000 X 50/100)
Restaurant (Rs.22,50,000 X 55/100)

4,68,750
5,62,500
12,37,500
(b) 22,68,750
45
Contribution (a) - (b)
Less: Fixed Costs
39,18,750
19,50,000
Estimated Profit 19,68,750

(a) Revised estimated income statement or the coming year ( if room rent reduced to Rs.125 per day to
enhance occupancy to 90%) (Rs.)
Revenue
Hotel Room Rent (100 rooms X 250 days X Rs.125 X 90/100)
Receipts from shop (100 rooms X 2 persons X 250 days X Rs.30 X 90/100)
Receipts from Restaurant (100 rooms X 2 persons X 250 days X Rs.60 X 90/100)

28,12,500
13,50,000
27,00,000
(a) 68,62,500
Variable Cost
Hotel Rooms (100 rooms X 250 days X Rs.25 X 90/100)
Shops (Rs.13,50,000 X 50/100)
Restaurant (Rs.27,00,000 X 55/100)

5,62,500
6,75,000
14,85,000
(b) 27,22,500
Contribution (a) - (b)
Less: Fixed Costs
41,40,000
19,50,000
Estimated Profit 21,90,000
(b) Analysis: With the reduction in room rent from Rs.150 per day to Rs.125 the occupancy will increase to
90% which will result in increase of profit by Rs.2,21,250 (i.e, Rs.21,90,000- Rs.19,68,750).

Ans. 13

(i) Occupancy:
Single rooms 100 X 365 X 80/100=29,200
Double rooms 20 X 365 X 80/100 = 5,840

Variable costs:
Single rooms (29,200 X 220)
Double rooms (5,840 X 350)

64,24,000
20,44,000


84,68,000
Fixed Costs:
Single rooms (29,200 X 120)
Double rooms (5,840 X 250)

35,04,000
14,60,000


49,64,000
Total costs: 1,34,32,000
Margin of safety 20%, Break- even point 80%
Sales at BEP = Total Costs =Rs.1,34,32,000
Total revenue = 1,34,32,000 X 100
80 =Rs.1,67,90,000
(Rs.)
Single rooms (29,200 X 1)
Double rooms (5,840 X 1.25)
29,200
7,300
National single rooms/days 36,500

Rent per day per Single room = 1,67,90,000
36,500 = Rs.460

Rent per day per Double room =Rs.460 X 1.25 = Rs.575

(ii) Restaurant
(a) Sales /day Rs.25,000 Contribution 30%
Total contribution 25,000 X 30/100 = Rs.7,500 per day
(Rs.)
Contribution p. a. (7,500 X 365)
Fixed cost p. a.
27,37,500
8,00,000
46
Profit 19,37,500

(b) Sports centre (Rs.)
No. of persons /days
Average contribution per person / day
50
15
Total contribution/day (50X 15) 750

(Rs.)
Total contribution/p. a. (750X 365)
Fixed Overheads
2,73,750
4,00,000
Loss 1,26,250
(c) Shopping arcade
Average contribution p.m. Rs.35,000 (Rs.)
Average contribution p. a. (Rs.35,000 X 12)
Fixed expenses
4,20,000
4,00,000
Profit 20,000
Profit Statement (Rs.)
Hotel accommodation Rentals 1,67,90,000
Less: Costs 1,34,32,00
Restaurant
Sports centre
Shopping arcade

33,58,000
19,37,500
(1,26,250)
20,000
Total 51,89,250
(III) Reservation = 10 rooms X 365 X 80 /100 = 2,920
Rent = 2,920 X 460 =Rs.13,43,200
Discount 10% =Rs. 1,34,320

Total contribution of remaining rooms (Rs.)
Single 90 X 365 X 80/100 X (460-220)
Double 20 X 365 X 80/100 X (575-350)
63,07,200
13,14,000
Total 76,21,200
Increase in contribution required
76,21,200 + 1,34,320 = Rs.77,55,520

% occupancy =
7755520
80 81.41
7621200
(i.e. Current Occupancy level) =
= Say 81.5%
Alternatively,
% Increase in contribution required =
134320
100 1.76%
7621200
=
Current occupancy level = 80
Revised occupancy level = 101.76% of 80 = 81.41% = Say 81.5%

(IV) Total profit per annum = Rs.51, 89,250

Capital recovery factor 3.79
Discounted income for 5 years Rs.1, 96, 67,257
Lease rent Rs.1, 75, 00,000
Hence lease not acceptable

Ans 14: Calculation of variable cost
Distance Distance
X Y
47
One side distance 24 km 16 km
Round trip 48 km 32 km
Variable cost @ 0.80 per km Rs. 38.40 Rs. 25.60

Calculation of fixed cost
Distance Distance
X Y
Actual running time for round trip distance at the
Speed of 24 km per hour 120 Min 80 Min
Filling time 40 Min 30 Min
Empty time 40 Min 40 Min
Total time 200 Min 150 Min

Fixed cost @ Rs. 7.50 per hour Rs. 25 Rs. 18.75
Calculation of ton km
Capacity 8 tones 8 tones
Full load 24 km 16 km
Tons km 192 128

Cost per ton km
192
25 40 . 38 +
= Rs. 0.33
128
75 . 18 60 . 25 +
= Rs. 0.347

Ans.15
Working notes:
(1) Total distance travelled (in 25 days)
= 60 km.(two sides ) X 6 trips per day X 25 days = 9,000 km.
(2) Total passenger km.
= 9,000 km. X 20 seats = 1,80,000 passenger km.
(3) Depreciation p.a.
= Purchase price Scrap value = (Rs.4,00,000-Rs.10,000) =Rs.78,000
5 Years 5 Years
Statement suggesting fare per passenger km (Rs.)
Fixed Expenses Cost per annum Cost per annum
Insurance
Garage rent
Road Tax
Administrative charges
Depreciation
Interest on Loan

Running Expenses
Repair and maintenance
Replacement of tyre-tube
Diesel and oil cost (9,000 km. X Rs.5
Driver and conductors salary
Total cost (per month)
Add: Profit (20% of total revenue or 25% of total cost
15,000
9,000
3,000
5,000
78,000
10,000
1,20,000






10,000

1,250
300
45,000
5,000
61,550.00
15,387.50

Total Revenue 76,937.50
Rate per passenger km =Rs.76,937.50/1,80,000 passenger km. =0.4274305 or 0.43 Paise

48
Ans.16
(i) Comparative cost sheet
Particulars 10 Tonne Capacity
Trucks
8 Tonne Capacity
Trucks
Total trips per day
No. of days per month
Total trips per month
Tonnes carried per truck
Capacity to be handed p.m. tones
No .of trucks required
No. of drivers (including relievers)
Total km. run per truck per month (120 X 12)
Total km. run by all trucks per month
Km. per litre of diesel
Diesel required ( Litres)

5
24
120
1,200
24,000
20
22
1,440
28,800
3
9,600
5
24
120
960
24,000
25
27
1,440
36,000
4
9,000
Monthly Sheet
No. of Trucks 10 Tonne Capacity 20 10 Tonne Capacity
25
(a) Variable with km run Diesel
@ Rs.10 per litre
Oil and sundries Rs.10 per 100 km.
Total

(b) Variable with No. of trucks run
Repairs & Maintenance
Road Tax
Drivers Salary
Depreciation
Total

Fixed
Supervisor
Mechanic
Fitter
Miscellaneous Expenses
Total

Grand Total
Tonnage hauled
Cost / Tonne

96,000
2,880
98,880


78,500
4,000
35,200
1,16,000
2,33,700


3,200
2,000
1,600
3,000
9,800
3,42,380
24,000
14.27

90,000
3,600
93,600


80,000
5,000
43,200
1,50,000
2,78,200


3,200
2,000
1,600
3,000
9,800
3,81,600
24,000
15.90
Cost/Tonne: 10 Tonne Trucks Rs.14.27
8 Tonne Trucks Rs.15.90
Hire charges Rs.18.00
Hence buy 10 tonne trucks.
(iii) Before taking final decision on purchase of trucks, on factor that may have to be given weight age
is that we have assumed consistent operation of all the 20 trucks for 24 days in a month,
transporting 24,000 tonnes without default for a period of five years. This aspect must be
considered on the basis of past recorded of hiring trucks on day to day basis over a three y3ear
period so that optimum calculations on saving get properly weighed down.
Second issue that an immediate investment of Rs.86 lakhs in purchase of 20 tracks has to be
made. This could be totally from own resources or totally out to borrowings or could be partly
either way. For own investment technique of discounted cash flow is to be applied while is case of
borrowings, recurrent interest cost as also initial cost of procuring the same has to be provided
out of saving from year to year apart from meeting normal schedule of loan repayment. Net
49
saving works out to Rs.10.74 lakhs per annum on hauling of 24,000 tonnes for 12 months in
comparison to hiring of trucks.
Third issue is to compare return on investment of own funds made for procurement of trucks
either fully or in part vis--vis return in alternate outlets. This is opportunity cost of capital will
have to be given consideration. Decision will be made after considering all the above factors.

Ans. 17:
Costs specific to booking operations:



Direct persons salary 20,000
Mobile expenses 3,000
Conveyance 4,000 27,000
Share of other overheads:



Office space 4,000
General Telephone 2,400
Security/Maintenance 1,600
Miscellaneous Expenses 1,000 9,000
Total Cost allocated to the service 36,000

Average demand per month=
25003+10002+7007
12
=1200

Total cost per booking=
Total cost per month
average booking per month
=
36000
1200
=`30

Revenue per ticket = Rs. 30

Total revenue less total cost = 30 - 30 = 0

Assuming that other overheads will anyway exist even of the service is not provided, the
manager can hope to achieve a profit of Rs. 30x 1,200 - 27,000 is Rs. 9,000 for the full year.
Minimum average volume to set up the service will be the amount needed to recover the specific costs
of this service, is 27,000 per month.

Minimum average bookings =
27,000
= 900 bookings

30

Ans.18
Working Notes:
(1) Calculation of requirement of trucks:
No. of Trips X No. of working days in a month X No. of tones
10 tonne = 5 X 24 X 10 = 1,200 tonnes
8 tonne = 5 X 24 x 8 = 960 tonnes

No. of trucks required to handle 24,000 tonnes

10 tonne trucks =24,000 tonnes/1200 tonnes = 20 trucks
8 tonne trucks =24,000 tonnes/960 tonnes =25 trucks

(2) No. of drivers required:
10 tonne =20 trucks X 2 drivers =40 Drivers
50
8 tonne =25 trucks X 2drivers =50 Drivers

(3) Total monthly depreciation:

10 tonne = 20 trucks X Rs.10,00,000 X 1 =Rs.3,33,333
5 years 12

8 tonne = 25 trucks X Rs.8,50,000 X 1 =Rs.3,54,167
5 years 12
(4) Diesel Required:
(No. of km. X No. of trips X No. of days in month X No. of trucks)
Diesel required =(6 km. X 10 trips X 24 days x 20 trucks )/No. km. per litre of diesel
10 tonne =(6 km. X 10 trips X 24 days x 20 trucks )/3. km. per litre =9,600 litres
8 tonne =(6 km. X 10 trips X 24 days x 25 trucks )/4. km. per litre =9,000 litres

Comparative Cost Sheet (Rs.)
Particulars 10 tonne 8 tonne
Fixed charges (p.m.)
Drivers salary(@ Rs.3,000 p.m)
Staff Expenses
Other fixed expenses
(i)
Operating and Maintenance charges
Depreciation
Diesel Cost
Lubricants & Sundries
Repairs & Maintenance
(ii) Total
Operating Cost (i) + (ii)
Tonnage carried (tonnes)
Cost per tonne

1,20,000
9,000
5,000
1,34,000

3,33,333
1,44,000
5,760
1,00,000
5,83,093
7,17,093
24,000
Rs.29.88

1,50,000
9,000
3,000
1,62,000

3,54,167
1,35,000
7,200
1,00,000
5,96,367
7,58,367
24,000
Rs.31.60
Analysis : From the above analysis it is observed that cost per tonne is lowest if 10 tonne trucks are used, and
the cost of Rs.50 per tonne presently incurring is highest and it can be reduced to Rs.29.88 by using 10 tonne
trucks.

Ans.19
(a) Statement of operating income of Modern Airways
operating between EXETOWN and WYETOWN (on each one way flight)
Rs.
Fare received (per flight): (A) 10,00,000
200 passenger Rs. 5,000

Variable costs (per flight)
Commission paid 80,000
Rs. 10,00,000 8%
Food services
200 passengers Rs. 200 40,000
Fuel costs 1,40,000
Total variable costs: (B) 2,60,000
Contribution (per flight): (C): {(A) (B)} 7,40,000

Fixed costs (per flight):
Fixed annual lease costs 5,30,000
Baggage handling (Fixed ground services) costs 70,000
Fixed salaries of flight crew ___40,000
51
Total fixed costs: (D) 6,40,000
Operating income (per flight): {(C) (D)} 1,00,000

Rs.
(b) Fare received (per flight): (X) 10,17,600
212 passenger Rs. 4,800

Variable costs:
Commission paid 81,408
Rs. 10,17,600 8%
Food services
212 passenger Rs. 200 42,400
Fuel costs 1,40,000
Total variable cost: (Y) 2,63,808
Contribution per flight: (Z): {(X) (Y)} 7,53,792
Excess contribution due to lowering of fare: {(Z) (C)} 13,792
[Refer to (a) part] (Rs. 7,53,792 Rs. 7,40,000)

Modern Airways should lower its fare as it would increase it contribution towards profit
by Rs. 13,792 per flight.

(C) Financial consideration of Modern Airways to Charter its plane to Zed Tours and Travel should use
option (b) and not (a).
Rs.
Under option (b) Modern Airways
Receives contribution (per flight): 7,53,792
Modern Airways would get (per flight) 7,50,000
If it charters the plane

A comparison of the above data clearly shows that the Modern Airways would be financially better off
by not chartering the plane.

Other consideration with regard to chartering a plane to Zed Tours and Travels

1. The loss of contribution involved in chartering a plane is Rs. 3,792 (per flight). This loss is on a
lower side as compared with uncertainties about the number of passengers on scheduled
fights.

2. modern Airways passengers may be inconvenienced when a plane is chartered to zed Tour
and Travel. They may go other airlines.

3. The relationship between the two parties is important. If it is not a long term arrangement.
Modern Airways may lose.

Ans.20
Working Notes:
Calculation operating capacity of a single aircraft =160 seats X 60/100 =96 passengers per flight
(i) Calculation of net operating income per flight (Rs.)
Fare collection (96 X 7000)
Variable costs:
Fuel
Food (96 X 130)
Commission @ 5%
Total Variable costs
Contribution per flight
Fixed Costs: Lease 3,50,000
6,72,000

95,000
12,480
33,600
1,41,080
5,30,920
52
Crew 72,000
Net Income per flight

4,22,000
1,08,920

(ii) Evaluation of proposal if Occupancy increases to 108 passengers per flight and the fare reduced to
Rs.6,720 (Rs.)

Fare collection (108 X 6720)
Variable costs:
Fuel
Food (108 X 130)
Commission @ 5%

Contribution
7,25,760

95,000
14,040
36,288
1,45,328
5,80,432
Analysis: The contribution will increase by Rs.49,512 (i.e Rs.5,80,432-Rs.5,30,920). Hence, it is suggested to
accept the proposal
(iii) Evaluation of proposal to charter the aircraft
Current contribution
Less: Fixed charge
Loss:
5,30,920
5,00,000
30,920

Analysis: if the aircraft is given on charter, it will cause loss of contribution by Rs.30,920. Hence the proposal
is not suggested.

Ans. 21:
(i) With respect to the passenger, the only variable costs are :

10% Commission on fare Rs. 500

Food Rs. 300

Total variable cost/passenger Rs. 800
Revenue per passenger = gross fare = 5000

Contribution = 5000 800 = Rs. 4200

Total Contribution 4200 x 240 10,08,000
Less: Costs/flight
Fuel 90,000
Lease 2,00,000
Baggage 40,000
Flight Crew 48,000 3,78,000
Profit per flight 6,30,000
(ii) Cost per flight Rs. 3,78,000 are fixed in relation to the number of passengers.
B.E.=
378000
4200
=90 passengers

Effect of Mid Airs offer Rs
A to D Fare 2000
Less: Comm. 200

1800
53

Less:Snacks 300

Contribution per passenger 1500


Additional
Cost ( Rs)
Additional
Revenue
(Rs)
50 seats x 2500 ( D to B ) 1,25,000
Fuel 45,000
Baggage 19,000
Snacks @ Rs 200 for passenger ( 240 -25+ 50):
200 x [ 240 25 + 50 ] 53,000
Additional Contribution (A to D) 60 x 1500 90,000



Contribution lost (A to B) : 25 x 4200 ( opportunity cost) 1,05,000
2,22,000 2,15,000
Aero will loose Rs. 7,000 per flight if it accepts Mid Airs offer.
Decision : Reject Mid Airs offer.

Ans.22

Calculation of variable cost per student of last year (Rs.lakhs)
Revenue
1. Students tuition-75% (Rs.3,600 X 12,000 students)
2. Endowment & contribution-25% (Rs.432 lakhs X 25/75)
Total revenue
Less: Fixed cost
Variable cost

432
144
576
300
276

Variable cost per student =Rs.2,76,00,000
12,000 students =Rs.2,300 per student
(i) Calculation of amount available in the first year for capital improvements and building (Rs.lalhs)

Revenue
1. Tuition Fee (Rs.4,200 X 11,200 students)
2. Endowment & contribution
3. Grant
Total revenue
Less: Variable cost (2,300 X 1.10 X 11200 students)
Contribution
Less: Fixed cost (Rs.300 lakhs + Rs.30 lakhs)
Balance available for capital improvements and building

470.40
144.00
50.00
664.40
283.36
381.04
330.00
51.04
Calculation of break-even if the grant is received and costs increases as predicted for the coming year
(Rs.lakhs)
Variable cost (Rs.2,300 X 1.10 X 12,000 students)
Fixed Cost (Rs.300 lakhs + Rs. 30 lakhs)
Capital improvement
Total cost
Less: Endowment and contribution 144.00
303.60
330.00
40.40
674.00

54
Grant 50.00
Balance amount to be collected as tuition fee
194.00
480.00
Tuition fee to be collected per student = Rs.4,80,00,000
12,000 students =Rs.4,000 per student

Ans.23
Working Notes:
(i) Expected Variable cost this year (Re.per ride)
Variable cost last year
Add: Expected increase this year (25% of Re.0.80)
Expected variable cost this year
0.80
0.20
1.00

(ii) Expected fixed costs this year (Rs.)
Fixed cost last year
Add: Expected increase this year (10% of Rs.32,00,000)
Expected variable cost this year
32,00,000
3,20,000
35,20,000

(1) Rides which DD Amusement park sell last year

(No. of rides DD sell last year) = Total Sales of rides last year =Rs.48,00,000 =12,00,000 rides
Charges per ride last year Rs.4

(2) Expected net income for the year if price increase if not implemented (Rs.)
Charges per ride
Less: Expected Variable cost per ride
Contribution per ride
No. of rides
Total expected contribution
Less: Expected fixed costs
Expected net income
4
1
3
12,00,000
36,00,000
35,20,000
80,000

(3)Price indifference point for the new ride
Price indifference point is a point at which the expected profits remains the same irrespective of sales
price and costs. (Rs.)

New ride price
Less: Variable cost
Contribution per ride
Fixed Costs of this year
Net Income of last year
Contribution require

5.00
1.00
4.00
35,20,000
6,40,000
41,60,000

Price- Indifference point = Rs.41,60,000 =10,40,000 rides
Rs.4
(4) Break even point for this year using the old price and the new price

Break-even point = Fixed costs
Contribution per ride


At old price = Rs.35,20,000 =11,73,334 rides
Rs.4-Re.1

At New price = Rs.35,20,000 =8,80,000 rides
55
Rs.5-Re.1

(5) Expected net income if the price increase will reduce ride volume by 10% from the last years levels
(Rs.)
Charges per ride
Less: Variable cost
Contribution per ride: (a)
No. of rides (12,00,000-1,20,000): (b)
Total contribution for all rides: (a) X (b)
Less: fixed costs
Expected net income
5.00
1.00
4.00
10,80,000
43,20,000
35,20,000
80,000

Justification: Since the increase in price of a ride will increase the net income by Rs.1,60,000(Rs.8,00,000-
Rs.6,40,000) the management should raise the price of a ride.

Ans 24: (1) Total number of patients attended
Number of patients attended per day by a
physician: 20
Number of physicians employed 6
Number of days in week 6
Number of weeks in a year 52
Total number of patients attended = 206 652 = 37,440.

(2) Patient Mix:
Adults (50%) 37,440 50/100 = 18,720
Children (40%) 37,440 40/100 = 14,976
Senior Citizens (10%) 37,440 10/100 = __3,744
37,440

(3) Patient Appointments:
No treatment required (70%) 37,440 70/100 = 26,208
Minor treatment (20%) 37,44020/100 = 7,488
Major treatment (10%) 37,440 10/100 = ___3,744
37,440
(4) Income from Insurance Companies:
Number of Rs. Rs.
patients
(A) (B) (AB)
No treatment patients 26,208 60 15,72,480
Minor treatment patients 7,488 250 18,72,000
Major treatment patients 3,744 500 18,72,000
Total 53,16,480

(5) Co-payment from adult patients:
Number of Payment Total
Patients Rs. Payment
(Rs.)
Total number of adult patients 18,720
No treatment patients (70%) 13,104 60 7,86,240
Minor treatment (20%) 3,744 250 9,36,000
Major treatment (10%) 1,872 500 9,36,000
Total 26,58,240
56

(6) Net income:
Rs. Rs.
Payment from Insurance companies 53,16,480
Co-payment from adult patients 26,58,240
Total 79,74,720
Other Income (fixed) 2,25,280
Total Income (A) 82,00,000
Less: Expenditure
Variable expenses:
Material and consumables 22,32,000
Fixed expenses:
Physicians salary (6 4,50,000) 27,00,000
Assistants salary (7 1,50,000) 10,50,000

Administrative staffs salary (2 90,000) 1,80,000
Establishment and other operating costs 16,00,000 55,30,000
Total Expenditure (B) 77,62,000
Net Income (A B) __4,38,000

(ii) 1. Contribution Analysis:
(Rs.)
Total Fees from Insurance Companies and adult patients 79,74,720
Less: Variable costs 22,32,000
Contribution 57,42,720
Average contribution per patient (57,42,72037,440) 153.38

2. Break-even patients:
(Rs.)
Fixed costs 55,30,000
Less: Fixed income 2,25,280
Net Fixed costs 53,04,720

Break-even patients = (Net fixed costs Contribution per patient)
= (53,04,720 34,585) 153.38


3. Percentage of maximum capacity required to be utilized in order to break-even
Present utilization =
patients
patients
24
20
= 83.33% = 37,440

100% patient capacity is 37,440 0.8333 =44,930 patients

Percentage of maximum capacity required to be utilized in order to break-even
Break Even patients 100% patients capacity 100
= {(34,585 44,930)100 } = 76.98% say 77%.

Assumption: Patient mix and mix of patient appointments will be same in the next year.


57
Ans 25
(a) Statement of Total Cost
Total cost Amount
(Rs)

Salary of Supervisor , Nurses, Ward
boys
4,25,000
Repairs and Maintenance 90,000
Salary of doctors 13,50,000
Food supplied to patients 40,000
Laundry charges for their bed linens 80,500
Medicines supplied 74,000
Cost of oxygen, X ray etc, other
than directly borne for treatment of
patients
49,500
General administration charges 63,000 Rs 21,72,000
Building rent (10
12,000)
Rs 1,20,000
Additional building rent on takings 5% on Total Taking
Hire charges extra beds Rs 12,000
Fees to heart specialists (3 15,000) Rs 45,000
Total cost Rs 23,49,000 + 5% on
Total Taking
Profit 20% on Total Taking
Total takings Rs 23,49,000 + 25% of
Total Taking
Total taking(assuming X to be the
rent per day)
1,05,000 X
Rent to be charged
1,05,000 X = 23,49,000 +25% (1,05,000 X) = 78750 X = 23,49,000 or X =
29.83(Rounded Off)
No of beds with Equivalent Rent
Nature of wards Occupancy Weight of
rent
Ward
Days
General ward 100 360
100%
36,000 1 36,000
Additional general ward
20
000 , 12

600 1 600
Cottage ward 50 360 80% 14,400 2.5 36,000
Deluxe ward 50 360 60% 6,480 5 32,400
Total 1,05,000
Rent to be charged
Particulars Basic Service tax Total
General ward 29.83 2.39 32.22
58
Cottage ward 74.58 5.97 80.55
Deluxe ward 149.15 11.93 161.08
Note : You may assume Total Taking to include Service Tax also.
Rent = 23,49,000 + 25% (1,05,000 X 1.08) + 0.08 (1,05,000X ) = 1,05,000X 1.08
= 23,49,000 + 28350X + 8400X = 1,13,400X
Therefore X = Rs 30.65
Rent to be charged
Particulars Basic Service tax Total
General ward 30.65 2.45 33.10
Cottage ward 76.63 6.13 82.76
Deluxe ward 153.25 12.26 165.51

59
Standard Costing
Working Notes
Ans.2
(1) For actual (standard) output of 85 kgs. Std. Input is 100 kgs.
For actual output of 1,700 kgs. the Std. input =
100 1700
85
kgs kgs
kgs

=2,000 kgs.
(2) 2,000 kgs of standard input for an actual output of 1,700 kgs. Contains the Materials A and B in the proportion of
(40:60) i.e., 800 kgs. of A and 1,200 kgs. of Material B.

(3) Actual Material consumption for 1,700 kgs. of actual output (Kgs.)
Particulars Materials
A B
Stock on 1-9-2004
Add: Purchase during Sept. 2004

Less: Stock on 30-09-2004
Material consumed during Sept.2004
35
800
835
5
830
40
1,200
1,240
50
1,190

(4) Calculation actual purchase price per kg. of material
A =
.3400
.4.25
800
Rs
Rs
kgs
= B =
.3000
.2.50
1200
Rs
Rs
kgs
=
Statement shoeing Standard and Actual Cost of Actual output
Material Standard Actual
Quantity
Kg.
Rate
Rs.
Amount
Rs.
Quantity
Kg.
Rate
Rs.
Amount
Rs.
A

B




Loss
Output
800

1,200



2,000
300
1,700
4

3
3,200

3,600





6,800
35
830
795
40
1190
1150

`
)

`
)

2,020
320
1,700
4.00
4.25
3.00
2.50

`
)

`
)

140.00
3,378.75

120.00
2,875.00



6,513.75

Calculation of Material Variances
(a) Material price variance
Actual quantity (Std. price Actual Price)
A = [35 (4 4)] + [ 795 ( 4 4.25)] =Rs.198.75 (A)
B = [40 ( 3 3)] +[1,150 ( 3 2.50 )] =Rs. 575 (F)

=Rs.376.25 (F)
(b) Material Usage variance
Std. rate (Std. quantity Actual Quantity)
A = 4 (800 830) =Rs.120 (A)
B = 3 (1,200 1,190) =Rs. 30 (F) =Rs.90 (A)

(c) Material Yield Variance
Std. rate of output (Actual yield Std. Yield)
=[Rs.6,800
1,700
x ( 1,700 kg. 1,717 kg.)] =Rs.68 (A)

* Std. Yield =
Actual std Output
Actual input
Std. Input
=
85
2020 1717
100
kgs
kgs kgs
kgs
=

(d) Material Mix Variance
Actual Quantity ( Std. cost of Std. mix per kg. Std. cost of actual mix per kg. )
60
=
Rs. 6800 Rs. 6890*
2020 kgs
2000 kgs 2020 kgs
| |

|
\ .
=Rs.22(A)
*[(830 kgs. x Rs.4)] + [(1,190 kgs x Rs.3 )] =Rs.6,890

(e) Total Materials Cost Variance
Std. Cost Actual Cost
=Rs.6,800 Rs.6,513.75 =Rs.286.25 (F)


Summary of Material variance (Rs.)
Price variance
Usage variance
1. Yield variance 68 (A)
2. Mix variance
Total Material cost variance
22 (A)
376.25 (F)


90 (A)
286.25 (A)


Ans. 3:

Working Note:
Standard cost Actual cost Revised std.quantity
Component Qty. Rate Amount
Kg. Rs. Rs.
Qty. Rate Amount
Kg. Rs. Rs.
Oty.
Kg.
A

B
48 10 480


112 30 224
72 12 864
(B.F.)

108 8 864
54


126
Total Input
(-) Loss
160 704
16(10%)
180 1728
36
180
Total output 144 144 5,360

Solution
(i) Mix variance = Std. price (Revised Std. quantity Actual quantity)
A: 10 (54-72) = 180 (A)
B: 2 (126-108) =

36 (F)
144 (A)
(ii) Yield variance = Std. price of yield (Actual yield Std. yield for actual
mix)

= Rs.
880
180
(144 18090%) = Rs. 88 (A)
(iii) Price variance =Actual qty. (Std. price Actual price.)
A: 72 (10-12) = 144 (A)
B: 108 (2-8) =

648 (A)
792 (A)
(iv) Total usage variance = Std. price (Std. qty. Actual qty.)

A: 10 (48-72) = 240 (A)
B: 2 (112-108) =

8 (F)

232 (A)
Ans. 4: Take the good output of 182 kgs. The standard quantity of material required for 182 kg. of output
61
is
182
100 202.22
90
=
Statement showing the standard and actual costs and standard cost of actual mix


Standard cost Actual cost Revised std.quantity
Component Qty. Rate Amount
Kg. Rs. Rs.
Qty. Rate Amount
Kg. Rs. Rs.
Oty.
Kg.
A (40% of
202.22 kg.)
B (60% of
202.22 kg.)
80.89 60 4,853.40


121.33 30 3,639.90
90 18 1,620


110 34 3,740
80


120
Total Input
(-) Loss
202.22 8,493.30
20.22
200 5,360
18
200
Total output 182.00 182 5,360
Standard yield in actual input is 90 % of 200 kg. i.e. 180 kg.

Variances :

(i) Price variance =Actual qty. (Std. price Actual price.)
A: 90 (60-18) = 3780 (F)
B: 110 (30-34) =

440 (A)
3340 (F)
(ii) Total usage variance = Std. price (Std. qty. Actual qty.)

A: 60 (80.89-90) = 546.60 (A)
B: 30 (121.33-110) =

339.90 (A)
206.70 (A)
(iii) Mix variance = Std. price (Revised Std. quantity Actual quantity)

A: 60 (80-90) = 600 (A)
B: 30 (120-110) =

300 (F)
300 (A)
(iv) Yield variance = Std. price of yield (Actual yield Std. yield for actual
mix)

= Rs.
8493 30
182
.
(182
182
200
202 22 .
) = Rs. 93.30 (F)
(v) Total variance = Std. cost Actual cost
= Rs. 8,493.30 Rs. 5,360 = Rs. 3133.30 (F)

Note : (iii) and (iv) above are subparts of total usage variance
Proof : Price variance + Mix variance + Yield variance = Total variance
Rs. 3340 (F) + Rs.300 (A) + Rs. 93. 30 (F) = Rs. 3133.30 (F)

Ans. 5:
(i) Since the actual output is 1,000 units, the standard quantity of materials required for the actual
output is 1,000 units 4 kgs. = 4,000 kgs.
Working Notes :
(ii) Statement showing computation of standard cost, standard cost of actual quantity and actual
cost.

62
Material Std. cost
per Kg.


Rs.
Actual
cost per
Kg.

Rs.
Std. qty
in Kgs.
Actual qty
in Kgs.
Std. cost
(Std. qty
Std.
price)
Rs.
Std. cost of
actual qty.
(Actual
qty. Std.
price) Rs.
Actual
cost (Actual
qty. Actual
price)
Rs.
a b c d e = ac f = ad g = bd
A
B
C
D
1.25
1.50
3.50
3.00
1.30
1.80
3.40
3.00
1,200
1,600
800
400
1,180
1,580
830
440
1,500
2,400
2,800
1,200
1,475
2,370
2,905
1,320
1,534
2,844
2,822
1,320
4,000 4,030 7,900 8,070 8,520

(iii) Standard cost per unit of the standard mix
Rs. 7,900
=
4,000 Kgs.
= Rs.1.975


(iv) Standard cost per unit of the actual mix =
.8070
.2.002
4030
Rs
Rs
kgs
=
Variances:

(i) Price variance = Actual qty. (Std. price Actual price)
= Rs.8,070 Rs. 8,520 = Rs. 450 (A)
(ii) Mix variance = Total actual qty. (Std. cost per unit of
std.mix Std. cost per unit of actual mix)
= 4,030 Kgs. (Rs. 1.975 Rs. 2.002) = Rs. 110 (A)
(iii) Sub usage variance = Std. price per unit of std. mix (Total std. qty
Total actual qty.)
= Rs. 1.975 (4,000 4,030) = Rs. 60.00 (A)
(iv) Total material cost variance = Std. cost Actual cost
= Rs. 7,900 Rs.8,520 = Rs. 620 (A)

Proof : Price variance + Mix variance + Sub-usage variance = Total variance
Rs. 450 (A) + Rs. 110 (A) + Rs. 60 (A) = Rs. 620 (A)

Note : Mix variance and sub usage variance are sub-part of total usage variance which may
be calculated as below:

Usage variance = Std. price (Std. qty. Actual qty.)
= Standard cost Standard cost of actual quantity
= Rs. 7,900 Rs. 8,070 = Rs. 170 (A)

Basic data for calculation of Labour variances
Ans.6
Category of Workmen Standard Actual
Weeks Rate
Rs.
Amount
Rs.
Weeks Rate
Rs.
Amount
Rs.
Skilled
Semi Skilled
Unskilled
3,000
1,200
1,800
60
36
24
1,80,000
43,200
43,200
2,560
1,600
2,240
65
40
20
1,66,400
64,000
44,800
Total 6,000 2,66,400 6,400 2,75,200

63
Calculation of Labour variances
(1) Direct Labour Cost Variance
Std. cost for actual output Actual Cost
=2,75,200 2,66,400 =Rs.8,800 (A)


(2) Direct Labour Rate Variance
Actual time (Std. rate Actual rate)
Skilled = 2,560 (60 65) =Rs.12,800 (A)
Semi Skilled =1,600 (36 40) =Rs. 6,400 (A)
Unskilled =2,240 (24 20) =Rs. 8,960 (F)

=Rs.10,240(A)



(3) Direct Labour Efficiency Variance
Std. rate ( Std. time for actual output Actual time)
Skilled =60(3,000 -2,560 ) =Rs.26,400 (F)
Semi Skilled =36 (1,200 -1,600) =Rs.14,400 (A)
Unskilled =24 (1,800 2,240) =Rs.10,560(A)
Direct Material efficiency Variance can be further analysed into:
=Rs.1,440(F)

(a) Direct Labour Mix Variance
Std. rate ( Revised Std. time Actual time)
Skilled =60(3,200 -2,560 ) =Rs.38,400 (F)
Semi Skilled =36 (1,280 -1,600) =Rs.11,520 (A)
Unskilled =24 (1,920 2,240) =Rs. 7,680 (A)
* Revised Std. time
=Rs.19,200 (F)

Skilled =6,400
6,000
x 3,000 =3,200


Semi- skilled =6,400
6,000
x 1,200 =1,280


Unskilled =6,400
6,000
x 1,800 =1,920
(b) Direct Labour Revised Efficiency variance
Std. rate ( Std. time for actual output Revised Std. time)
Skilled =60(3,000 -3,200 ) =Rs.12,000 (A)
Semi Skilled =36 (1,200 -1,280) =Rs. 2,880 (A)
Unskilled =24 (1,800 1,920) =Rs. 2,880 (A)
Summary of Labour variances (Rs.)
=Rs.17,760(A)
Rate variance
Efficiency variance
(a) Mix variance 19,200 (F)
(b) Revised efficiency variance 17,760 (A)
Direct Material cost variance
10,240 (A)


1,440 (F)
8,800 (A)

Ans. 7:
Gang: -
In a 40 hour week, the standard gang should have produced 1,000 std. hours as
shown below:

Skilled 16 No. of workers 40 hrs. 640
Semi - skilled 6 No. of workers 40 hrs. 240
Unskilled 3 No. of workers 40 hrs. 120
64

1,000 hours

However, the actual output is 900 standard hours. Hence to find out the total labour cost
variance, the standard cost (or cost charged to production) is to be computed with reference to 900
standard hours. This is done in the following statement:

Statement showing the Standard cost, Actual cost and
Standard cost of Actual time for Actual output, i.e. 900
Standard hours.

Gang Standard cost

Hours Rate Amount
Rs. Rs.
Actual cost

Hours Rate Amount
Rs. Rs.
Standard cost of
Actual time
Hours Rate Amount
Rs Rs.
Skilled

600
900
1000

`
) 576 3 1,728


Semi-skilled

240
900
1000

`
) 216 2 432

Unskilled
120
900
1000

`
) 108 1 108



1440 = 560 4 2,240




9 40 = 360 3 1,080


2 40 = 80 2 160


560 3 1,680




360 2 720


80 1 80
900 2.52 2,268 1,000 3.48 3,480 1,000 2.48 2,480

Variances:

(i) Rate variance = Actual time (Std. rate Actual rate)

= (Standard cost of actual time Actual cost)

= Rs. 2,480 Rs.3,480 = Rs. 1,000 (A)
(ii) Gang variance = Total actual time ( Std. rate of std. gang
Std. rate of actual gang)


= 1,000 (Rs. 2.52 Rs. 2.48) = Rs. 40(F)
(iii) Sub-efficiency variance = Std. rate (Total std. time Total actual time)

= Rs. 2.52 (900 hours 1,000) = Rs. 252 (A)
(iv) Total labour cost variance = Std. labour cost Actual labour cost

= Rs. 2,268 Rs. 3,480 = Rs. 1,212 (A)
The gang composition variance may also be known as labour mix variance and is part of efficiency
variance which may be computed as under:

Efficiency variance = Std. rate (Std. time Actual time)
= Standard cost Std. cost of actual time
= Rs. 2,268 Rs. 2,480 = Rs.212 (A)


Ans. 8:
(1,000 units 2.5 hours Rs.2)
Standard cost charged to production Rs. 5,000
Actual wages paid Rs. 4,500
Actual wage rate per hour (Rs. 45002000) Rs. 2.25
Std. wage rate per hour Rs. 2.00
Abnormal idle time (25% of 2,000 hours) 500 hrs.
Variances :

(i) Wage rate variance = Actual time (Std.rate Actual rate)
65

= 2,000 hours (Rs.2 Rs.2.25) = Rs.500 (A)
(ii) Efficiency variance = Std. rate (Std.time Actual time*)

Rs.2 (2,500 hrs. 1500 hrs.) = Rs. 2,000 (F)
(iii) Idle time variance = Idle time Std.rate

= 500 hrs. Rs. 2 = Rs. 1,000 (A)
(iv) Total variance = Std.labour cost Actual labour cost

Rs. 5,000 Rs. 4,500 = Rs. 500 (F)
*Actual time less idle time.

Basic data for Standard and actual labour cost of producing 1,000 articles of A and standard cost of actual
labour hours
Ans.9
Standard Cost Actual Cost
Labour Hours Rate
Rs.
Amount
Rs.
Hours Rate
Rs.
Amount
Rs.
Std. cost of
actual
labour
hours (
Actual
hours x
Std. rate)Rs
Skilled
Semi Skilled
Unskilled
10,000
8,000
16,000
3.00
1.50
1.00
30,000
12,000
16,000
9,000
8,400
20,000
4.00
1.50
0.90
36,000
12,600
18,000
27,000
12,600
20,000
Total 34,000 58,000 37,400 66,600 59,600

Calculation of Labour variances
(1) Labour Cost Variance
Std. cost Actual Cost
=Rs.58,000 Rs.66,600 =Rs.8,600 (A)


(2) Labour Rate Variance
Actual Hours (Standard rate Actual rate)
OR

Std. cost of actual hours Actual Cost
=Rs.59,600 Rs.66,600 =Rs.7,000 (A)


(3) Labour Efficiency Variance
Std. rate of Std. mix (Total Std. hours for actual output Total Actual hours)

= ( )
Rs. 58000
34000 37400
34000
=Rs.5,800(A)

(4) Labour Mix Variance
Total actual hours ( Std. rate of standard mix Std. rate of actual mix)

=
58000 59600
34000
34000 37400
| |

|
\ .
=Rs.4,200(F)
Summary of Labour variances (Rs.)
Rate variance
Efficiency variance
Mix variance
Labour Cost variance
7,000 (A)
5,800 (A)
4,200 (F)
8,600 (A)
66

Ans. 10:
= (Standard variable overhead Actual variable overhead)
(i) Variable overhead variance:
= (Rs. 2,40,000 Rs. 2,00,000) = Rs. 40,000 (Favourable)
(Refer to Working note 1)

(ii) Variable overhead budget variance:
= (Budgeted variable overhead for actual hours Actual variable overhead)
= Rs. 2,24,000 Rs. 2,00,000 = Rs. 24,000 (Favourable)
(Refer to Working note 2)

(iii) Variable overhead efficiency variance:

= Standard variable overhead rate per hour [Std. hours for actual output Actual hours]
= Rs. 2 [1,20,000 hours 1,12,000 hours]
= Rs.2 8,000 hours = Rs. 16,000 (Favourable)

Working notes:
(1) Standard variable overhead
= Standard cost of actual output = 20,000 units 6 hours Rs. 2
= Rs. 2,40,000
(2) Budgeted variable overhead (for actual hours)
= 1,12,000 hours Rs.2 = Rs.2,24,000

Ans. 11:

Actual output = 9,000 units
Idle time = 5,000 hours

Production time (Actual) = 1,05,000 hours
Standard hours for actual production = 10 hours / unit 9,000 units = 90,000 hours.
Labour efficiency variance = 3,75,000 (A)
i.e. Standard rate (Standard Production time Actual production time) = 3,75,000(A).

SR (90,000 1,05,000) = 3,75,000

SR =
3,75,000
= Rs. 25

15,000

(i) Idle time variance = 5,000 hours 25 Rs. / hour = 1,25,000. (A)
(ii) Standard Variable Overhead = Rs. 150 / unit
Standard hours = 10 hours / unit

Standard Variable Overhead rate / hour = 150 / 10 = Rs. 15 / hour

Total Variable Overhead variance = Standard Variable Overhead Actual Variable Overhead
= Standard Rate Standard hours Actual rate Actual hours
= (15) (10 9,000) 16,00,000

= 13,50,000 16,00,000

Total Variable Overhead Variance = 2,50,000 (A)

(iii) Variable Overhead Expenditure Variance = (Standard Rate Actual Hours) (Actual Rate Actual Hours)
= (15 1,05,000) 16,00,000

= 15,75,000 16,00,000

67
= 25,000 (A)
(iv) Variable Overhead Efficiency Variance = Standard Rate (Standard Hours for actual output Actual hours for
Actual output)
= 15 (90,000 1,05,000)
= 15 (15,000)
= 2,25,000 (A)

(b) Al ternati ve Sol uti on

Actual Output = 9,000 Units
Idle time = 5,000 hrs
Direct Wages Paid = 1,10,000 hours @ Rs. 22 out of which 5,000 hours being idle, were not recorded in production.
Standard hours = 10 per unit.
Labour efficiency variance = Rs. 3,75,000 (A)
or
Standard Rate (Standard Time Actual Time) = 3,75,000
Or Standard Rate = Rs 25/-
(i) Idle time variance = Standard Rate Idle time
25 5,000 = Rs 1,25,000 (A)

(ii) Standard Variable Overhead / unit = 150

Standard Rate =
150
= Rs.15/hour

10
Standard Quantity = 10 hours
Actual Variable Overhead = 16,00,000
Standard Variable Overhead = 150 9,000 = 13,50,000

Actual Variable Overhead = 16,00,000
Total Variable Overhead Variance = 2,50,000 (A)
(iii) Variable Overhead expenditure

= Standard Variable Overhead for

actual hours Actual Variable

Overhead
= (150 1,05,000) 16,00,000
= 15,75,000 16,00,000

= 25,000 (A)

(iv) Variable overhead efficiency variance = Standard Variable Overhead for actual output
Standard Variable Overhead for Actual hours)
= 15 (10 hours 90,000 units
1,05,000)

= 15 (90,000 1,05,000)

= 15 (15,000)
= 2,25,000 (A)
Standard Cost
Ans.12: Computation of standard cost and actual cost
Direct Materials (6,000 x Rs.12)
Direct Labour (6,000 x Rs.4.40)
Variable Overheads (6,000 x Rs.3)
Total standard Costs (a)
Actual Costs
Direct Materials (12,670meters x Rs.5.70)

72,000
26,400
18,000
1,16,400

68
Direct Wages
Variable Overheads
Total Actual Costs (b)
Total Variance (a)-(b)

72,219
27,950
20,475
1,20,644
4,244,(A)

Computation of Missing figures
(1) Actual Labour hours
Standard variable overhead rate hour (Standard hours Actual hours) = Rs.1,500 (A)
Rs.1,500 A =Rs.3 (6,000 x 1 hour Actual hours)
Rs.1,500 A =Rs.18,000 (Rs.3 x actual hours)
(Rs.3 x Actual hours) =Rs.18,000 + Rs.1,500
Actual hours =Rs.19,500 / 3 = 6,500 hours

(2) Actual Wage rate hour = Actual wages paid =Rs.27,950
Total Actual hours 6,500 hours
=Rs.4.3
Computation of Material Labour and Variable Overhead Variances
1. Material variances
(1) Material Cost Variance
Standard Cost- Actual Cost
=(Rs.72,000 Rs.72,219) =Rs.219 (A)
(2) Material Price Variance
Actual Quantity of Material consumed (Std, price- Actual Price)
=12,670 meters (Rs.6- Rs.5.70) =Rs.3,801 (F)
(3) Material Usage Variance
Standard price (Standard Quantity Actual Quantity)
=Rs.6 (12,000 metres -12,670 metres) =Rs.4,020 (A)
2. Labour Variances

(1) Labour Cost Variance
Standard Cost- Actual Cost
=(Rs.26,400 Rs.27,950) =Rs.1,550 (A)
(3) Labour Rate Variance
Actual hours (Std. wage rate per hour- Actual wage rate per hour)
=6,500 hours (Rs.4.40- Rs.4.30) =Rs.650 (F)
(3) Labour Efficiency Variance
Standard rate per hour (Standard hours Actual hours)
=Rs.4.40 (6,000 hours- 6,500 hours) =Rs.2,200 (A)
3. Variable Overhead Variances

(1) Total Variable overhead Variance
Standard Variable Overhead- Actual Variable Overhead
=Rs.18,000 Rs.20,475 =Rs.2,475 (A)
(4) Variable overhead Efficiency Variance
Standard Variable overhead rate per hour (Std. hours for actual output-Actual hours)
=Rs.3 ( 6,000 6,500) =Rs.1,500 (A)
(3) Variable overhead Budget Variance
Budgeted variable overhead Actual variable overhead)
=(Actual hours worked x Std. variable overhead per hour) Actual variable overhead
=(6,500 x Rs.3 ) Rs.20,475 =Rs.975 (A)
Note: (F) denoted Favourable Variance; (A) denoted Adverse Variance

Working Notes :
Ans 13:

1. Standard cost of raw-material consumed : Rs. Rs.
Total standard cost of ZED (1,000 units Rs.21) 21,000
Less: Standard cost : Labour 8,000
69
Over heads 1,600 9,600
Standard cost of raw materials used

11,400
2. Standard cost of rawmaterial per finished unit.

3. Standard quantity of raw - material per finished unit and total quantity of raw material
required:

Total quantity 3.8 kg. 1,000 units = 3,800 kgs.
4. Total material cost variance :
Actual cost of raw material Rs.10,000
Standard cost of raw material Rs.11,400
Total material cost variance Rs. 1,400 (F)
5. Actual quantity (A Q) of rawmaterial (in kgs):
Material usage variance = Standard rate (Standard quantity Actual quantity). or, Rs. 600
(A) = Rs. 3 (3,800 Kgs. AQ)
or, 3AQ = 12,000 kgs. or, AQ = 4,000 kgs.
(Material usage variance is as given in the question and standard quantity is as per (3)
above )
6. Actual rate of raw material per kg

7. Standard direct labour rate
Standard direct labour hours = 1,600 (given) Standard direct labour cost = Rs. 8,000 (given)

8. Actual labour cost and actual labour rate per hour:
Actual total cost of 1,000 units Rs. 21,070
1,000 units (Rs. 21 + Re. 0.07)
Less : Actual cost of material Rs. 10,000
Actual variable overheads Rs. 1 Rs. 1,62
Actual direct labour cost
1,620
Rs. 9,450


9. Standard labour hours to produce one unit:

10. Standard labour cost per unit:

Standard labour cost per unit = 1.6 hours Rs. 5 = Rs.8

11. Actual hourly rate of variable overheads
:
(a) Standard qu antity of raw material per unit of ZED : 3.8 kg. (Refer to working note 3).
(b) Standard direct labour rate per hour Rs. 5 (Refer to working note 7).
70
(c) Standard direct material cost per unit of ZED : Rs. 11.40 (Refer to working note 2 ) .
(d) Standard direct labour cost per unit of ZED: Rs. 8 (Refer to working note 10).
(e) Standard total material cost for the output: Rs. 11,400 (Refer to working note 1). (f) Actual
total direct labour cost for the output: Rs. 9,450 (Refer to working note 8). (g) Material price
variance = Total material cost variance Material usage variance.
= Rs. 1,400 (favourable)* Rs. 600 (Adverse)
(*Refer to working note 4)
= Rs. 2000 (Favourable)
Alternatively,
= Actual quantity (Standard rate Actual rate)

= 4,000 units (Rs. 3 Rs. 2.50)* (* Refer to working note 6)

= Rs. 2,000 (Favourable)

(h) Labour rate variance:
= Actual hours (Standard rate Actual rate)
= 1,800 hours (Rs. 5 Rs. 5.25)
= Rs. 450 (Adverse)
(i) Labour efficiency variance:
Standard rate (Standard hours Actual hours)
= Rs. 5 per hour (1,600 hours 1,800 hours) = Rs. 1,000 (Adverse)
(j) Variable overhead expenditure variance :
= Actual hours (Standard rate Actual rate)
= 1,800 hours (Re. 1 Re. 0.90)* = Rs. 180 (Favourable) (*Refer to working note)
(k) Variable overhead efficiency variance
= Standard rate (Standard hours Actual hours)
= Re. 1 per hour (1,600 hours 1,800 hours) = Rs. 200 (Adverse)


Ans. 14:
12000
500 /
24
hrs
hrs day
days
= Budgeted daily hours per day of June =
Actual available hours for June = 500 hours 25 days = 12,500 hours

Calendar Variance = Std. fixed overhead rate per hr
(No. of hrs. in actual period No. of hrs. in budgeted period)
= Re.0.50 (12,500 hours 12,000 hours) = Rs. 250 (F)
Alternatively, this variance can be calculated by using number of days instead of hours. In that case,
overhead rate will be on per day basis.

Ans. 15:
Standard output per man hour: 1
Actual output : 8,400 hours 22days 1.2 units per hour = 2,21,760 units.
Standard hours produced or std. hrs. for actual production :2,21,760 units1 hr. = 2,21,760 hrs.
Budgeted hrs. in budgeted days: 8,000 hours 20 days = 1,60,000 hours
Budgeted hours (capacity) in actual working days: 8,000 hrs. 22 days = 1,76,000 hours
Actual hours worked: 8,400 hours 22 days = 1,84,800 hours
Overheads as per budget: 8,000 hours 20 days Rs. 2 per hour = Rs.3,20,000
71
Rs.

(a) Standard cost charged to production : 2,21,760 hours Rs.2 4,43,520
(b) Actual hours worked Standard rate : 1,84,800 hours Rs.2 3,69,600
(c) Budgeted hours in actual days Std. rate: 1,76,000 Rs.2 3,52,000
(d) Overheads as per budget 3,20,000
(e) Actual overheads 3,25,000
Efficiency variance = Std.fixed overhead rate per hour (Std. hrs. for
production Actual hrs.)
= Rs.2 (2,21,760 hours 1,84,800 hours) = Rs.73,920 (F)
Capacity variance = Standard fixed overhead rate per hour (Actual capacity
Budgeted capacity)
= Rs.2 (1,84,800 hours 1,76,000 hours) = Rs.17,600 (F)
Calendar variance = Standard fixed overhead rate per hour (Budgeted hrs. in
actual days Budgeted hrs. in budgeted days)
= Rs.2 (1,76,000 hours 1,60,000 hours) = Rs.32,000 (F)
Volume variance = Standard fixed overhead rate per hour
(Actual volume in hrs. Budgeted volume in hrs.)
= Rs.2 (2,21,760 hours 1,60,000 hours) = Rs. 1,23,520(F)
Expenses variance = Budgeted expenses Actual expenses
= Rs.3,20,000 Rs.3,25,000 = Rs.5,000 (A)
Total variance = Overheads charged to production Actual overheads
= Rs. 4,43,520 Rs.3,25,000 = Rs. 1,18,520 (F)
OR
Rs.

Efficiency variance : (a b) 73,920 (F)
Capacity variance : (b c) 17,600 (F)
Calendar variance : (c d) 32,000 (F)
Volume variance : (a d) 1,23,520 (F)
Expense variance : (d e) 5,000 (A)
Total variance : (a e) 1,18,520 (F)

Ans. 16:

(a)Total fixed overhead variance = Absorbed fixed overheads Actual fixed overheads
= (5,200units Rs. 2) Rs. 10,200 = Rs.200 (F)
(b) Expenditure variance = Budgeted overheadsActual overheads
= Rs. 10,000 Rs. 10,200 = Rs. 200(A)
(c) Volume variance = Standard rate of absorption per unit
(Actual production Budgeted production
= Rs.2 (5,200 units 5,000 units)=Rs. 400 (F)
This can be divided into capacity variance and efficiency variance as shown below :

Capacity variance = Standard rate of absorption per hour (Actual hours capacity Budgeted
hours capacity)
= Re. 0.50 (20,100 hours 20,000 hours) = Rs 50(F)
Efficiency variance = Standard rate of absorption per hour (Standard hours required Actual hours)

= Re.0.50 (20,800 hours 20,100 hours) = Rs.350 (F)
72
Working Notes :
Std. fixed overhead rate of absorption per unit =
.10000
.2
5000
Rs
Rs
units
=
Std. fixed overhead rate of absorption per hour:
.10000
Re.0.50
5000 4 .
Rs
units hrs
=


Std. hours required for actual production: 5,200 units 4 hours = 20,800 hours


Ans. 17:
Working Notes:

1) Budgeted output in units
40,000 man hours X 3.2 units per man hours = 1,28,000 units.

2) Standards variable overhead rate per unit
Rs, 1,02,400/1,28,000 units = Rs. 0.80 per unit

3) Standard variable overhead rate per man hour
Rs. 1,02,400/40,000 man hours = Rs. 2.56 per man hour

4) Standard fixed overhead rate per unit
Rs 32,000/1,28,000 units = Rs. 0.25 per unit

5) Actual Production units
43,000 man hours X 3 units per man hour = 1,29,000 units

Computation of variable Overhead variances:

i) Total Variable Overhead Variances
= Variable overhead recovered on actual output Actual variable overhead
= (1,29,000 units X 0.80 P Rs. 1,14,000) = Rs. 11,200 (A)

ii) Variable Overhead Expenditure Variance
= Budgeted variable overhead for actual hours Actual Variable overhead
= (43,000 X 2.56 Rs. 1,14,400) = Rs. 4,320 (A)

iii) Variable Overhead Efficiency Variance
= Standard variable overhead rate per hour (Standard hours for actual output- Actual hours)
= Rs. 2.56 (40,312.5 hours 43,000 Hours) = Rs. 6,880 (A)


Computation on Fixed Overhead Variances:
i) Total Fixed Overhead Cost Variance
= Fixed overhead recovered on actual output Actual fixed overhead
= (1,29,000 units 0.25 P Rs. 31,500) = Rs. 750 (F)

ii) Fixed Overhead Expenditure Variance
= Budgeted fixed overhead Actual fixed overhead
= (Rs. 32,000 Rs. 31,500) = Rs. 500 (F)

iii) Fixed Overhead Volume Variance
= Standard fixed overhead per unit (Actual output units Budgeted output units)
= 0.25 P (1,29,000 1,28,000) = Rs. 250 (F)

iv) Fixed Overhead Efficiency Variance
= Standard fixed overhead rate per unit (Actual Quantity Standard Quantity)
73
= 0.25 P (43,000 hours X 3.2 units 1,29,000 units) = Rs. 2,150 (A)

v) Fixed Overhead Capacity Variance
= Standard fixed overhead rate per hour (Actual capacity hours Budgeted capacity hours in actual days)
= (Rs. 32,000/40,000 hours) (43,000 21 days X 2,000 hours) = Rs. 800 (F)

vi) Calendar Variance
= (Budgeted Days Actual Days) Standard fixed overhead per day
= (20 days 21 days) (Rs. 32,000/20 days) = Rs. 1,600 (F)

Computation of Total Overhead Variances
= Total variable overhead variances + Total fixed overhead variances
= Rs. 11,200 (A) + Rs. 750 (F) = Rs. 10,450


Ans. 18:

Basic calculation:
Product Budgeted
price
Actual
price
Budgeted
quantity
Actual
quantity
Budgeted
sales
Actual
quantity at
budgeted
sales
Price
Actual
sales

a b c d (e)=a c f=(a d) g=(b d)

A
B
C
D
Rs.
2.50
5.00
7.50
10.00
Rs.
3.00
4.50
7.00
10.50

2,000
1,500
1,000
500

2,400
1,400
1,200
400
Rs.
5,000
7,500
7,500
5,000
Rs.
6,000
7,000
9,000
4,000
Rs.
7,200
6,300
8,400
4,200
5,000 5,400 25,000 26,000 26,100

Computation of Variances :

Sales price variance = Actual quantity (Actual price Budgeted price)
= Actual sales Standard sales
= Rs.26,100 Rs. 26,000 = Rs.100(F)
Sales volume variance = Budgeted price (Actual quantity Budgeted quantity)
= Std. sales Budgeted sales
= Rs.26,000 Rs.25,000 = Rs.1,000 (F)
Total variance = Actual sales Budgeted sales
= Rs.26,100 Rs.25,000 = Rs.1,100 (F)
Average budgeted price per unit of budgeted mix:
Average budgeted price per unit of actual mix:
Hence, Sales mix variance = Actual total qty. (Budgeted price per unit of actual mix
Budgeted price per unit of budgeted mix)





74

=
=
5,400 units (Rs.4.815Rs.5.00)
Rs. 1,000 (A)
Sales quantity variance = Budgeted price per unit of budgeted mix

= (Actual total qty. Budgeted total qty.)

= Rs.5 (5,400 5,000) = Rs. 2,000 (F)
Note: Instead of computing average price, one may use total figures to do away with the effect of
rounding off.
For example, in case of sales mix variance figures may be as under:

= Rs. 26,000 Rs 27,000 = Rs.1,000 (A)

A. (a) Analysis of variances to show the effects on turnover :
Ans. 19:
B. Working Notes :
( 1 ) Budgeted sales :
Budgeted sales units at budgeted (or standard) prices.
Units Price Amount
Rs. Rs.
Bravo 5,000 100 5,00,000
Champion 4,000 200 8,00,000
Super 6,000 180 10,80,000
15,000 23,80,000
(2) Actual sales :
Actual sales units at actual prices




(3) Standard sales:
Actual sales units at Budgeted (or Standard) prices.

Units Price Amount
Rs. Rs.
Bravo 5,750 100 5,75,000
Champion 4,850 200 9,70,000
Super 5,000 180 9,00,000
15,600 24,45,000
Computation of Variances :
(i) Sales price variance = Actual quantity (Actual price Budgeted price)
or
Actual sales Standard sales
Units Price
Rs.
Amount
Rs.
Bravo 5,750 120 6,90,000
Champion 4,850 180 8,73,000
Super 5,000 165 8,25,000

15,600 23,88,000
75
= Rs.23,88,000 Rs.24,45,000 = Rs.57,000 (A)
(ii) Sal es mi x vari ance = Total actual quantity (Budgeted price of actual mix Budgeted price of
budgeted mix

.2445000 .2380000
15600
15600 15000
Rs Rs
units
| |
=
|
\ .

=Rs. 2475200 Rs. 2380000 = Rs. 95200F
= Rs. 24,45,000 Rs. 24,75,200 = Rs. 30,200 (A)
(iii) Sales quantity variance = Budgeted price of budgeted mix
(Total actual quantity Total budgeted quantity)

Rs. 23,80,000

=
15,000 units
( 15,600 units 15,000 units)


= Rs. 24,75,200 Rs. 23,80,000 = Rs. 95,200 (F)
(iv) Total sales value variance = Actual sales Budgeted sales
= Rs.23,88,000 Rs.23,80,000 = Rs. 8,000 (F)
(b) Analysis of variances to show the effects on profit : Working
Notes :
(1) Budgeted margin per unit


Sales price
Rs.
Cost
Rs.
Margin
Rs.
Bravo 100 90 10
Champion 200 170 30
Super 180 130 50
(2) Actual margin per unit

76


Computation

of variances:

(i) Sale margin price variance
Actual quantity (Actual margin Budgeted margin)
or
Actual profit Standard profit
Rs. 3,96,000 Rs. 4,53,000 = Rs. 57,000 (A)
(ii) Sales margin mix variance = Total actual quantity (Budgeted margin on actual mix
Budgeted Margin on budgeted mix

= Rs. 4,53,000 Rs. 4,88,800 = Rs. 35,800 (A)
(iii) Sales quantity variance
= Budgeted margin on budgeted mix (Total actual qty. Total budgeted qty.)


= Rs. 4,88,800 Rs. 4,70,000 = Rs. 18,800 (F)
(iv) Total sales margin variance = Actual profit Budgeted profit
= Rs. 3,96,000 Rs. 4,70,000 = Rs. 74,000 (A)

Ans. 20:
1. Statement of budgeted average contribution margin per unit for the year 1995:
Worki ng Notes:
Product different PC
models
Budgeted contribution
margin per unit of each
product
Budgeted sales
volume
Total budgeted
contribution
margin
(Rs.) (Units) (Rs.)
PC 10,000 7,000 7,00,00,000
Portable PC 6,000 1,000 60,00,000
Super PC 40,000 2,000 8,00,00,000
10,000 15,60,00,000
Budgeted average contribution margin per unit =
units 10,000
0,000 Rs.15,60,0

= Rs.15,600
77
2. Actual market share percentage =
sales industry Actual
models PC 3 - of sales Actual
100
=
units 68,750
units 11,000
100
= 16
3. Actual sales mix percentage of product =
models PC 3 of sale Actual Total
Product of sales Actual
100
Actual sales mix percentage of product PC =
units 11,000
units 8,250
100 = 75
Actual sales mix %age of product Portable PC =
units 11,000
units 1,650
100 = 15
Actual sales mix %age of product Super PC =
units 11,000
units 1,100
100 = 10
(i ) Computation of individual product and total sales volume variance
Sales =
unit
per margin
on contributi
Budgeted
units in
Volume
Sales
Budgeted
units in
Volume
Sales
Actual

(
(
(
(


Individual product sales volume variance:
PC = (8,250 units 7,000 units) Rs.10,000 = Rs.1,25,00,000 (Fav.)
Portable PC = (1,650 units 1,000 units) Rs.6,000 = Rs.39,00,000 (Fav.)
Super PC = (1,100 units 2,000 units) Rs.40,000 = Rs.2,60,00,000 (Adv.)
Total Sales Volume Variance = Rs.1,96,00,000 (Adv.)
(i i ) Computation of total sales quantity variance:
Total sales quantity variance =
unit per
margin on contributi
average Budgeted
units Sales
Budgeted Total
Unit sales
actual Total

(
(


= (11,000 units 10,000 units) Rs.15,600 = Rs.1,56,00,000 (Fav.)
(i i i ) Computation of the market size and market share variance
1. Market size variance:
Budgeted market Share %age =
unit per
margin on contributi
average Budgeted
units in Sales
Industry Budgeted
units in Sales
Industry Actual

(


= 0.20 (68,750 units 50,000 units) Rs.15,600 = Rs.5,85,00,000 (Fav.)
2. Market share variance:
78
=
(
(
(
(

(
(
(
(

(
(


unit per
margin on Contributi
average Budgeted
units in
Volume Sales
Total Actual
percentage share
market Budgeted
percentage share
market Actual

= (0.16 0.20) 68,750 units Rs.15,600 = Rs.4,29,00,000 (Adv.)
(i v) Computation of individual product and total sales mix variances
1. Individual product and total sales mix variance:
Sales mix variance:

(
(
(
(

(
(
(
(

(
(
(
(


margin
on contributi
average Budgeted
margin
on Contributi
Individual
Budgeted
units in
Volume Sales
Total Actual
product
of %age mix
sales Budgeted
product
of %age mix
sales Actual

PC*** = (0.75 0.70) 11,000 units (Rs.10,000 Rs.15,600)
= Rs.30,80,000 (Adv.)
Super PC****= (0.10 0.20) 11,000 units (Rs.40,000 Rs.15,600)
= Rs.2,68,40,000 (Adv.)
2. Total sales mix variance = rs.3,52,00,000 (Adv.)
* Refer to working note 1.
**Refer to working note 2.
***Refer to working note 3.
Note: Sales variances can also be calculated by using sales value approach.
(v) Comment on above results:
1. Favourable sales quantity variance of Rs.1.56 crores was because of growth in industry as a whole.
However the firm could not retain the budgeted market share of 20%. As a result the benefit of
increased market size i.e. Rs.5.85 crores is partly offset by loss due to fall in market share i.e.
Rs.4.29 crores.
2. Increase in the percentage sale of computers below-average budgeted margins and a decrease in
the percentage sale of computers above-average budgeted margins had resulted in the reduction of
operating profit by Rs.3.52 crores.
3. As a result of above, the operating profit of Super Computers had been adversely affected by
Rs.1.96 crores due to sales variances.

Ans 21:
1. Material data
Working Notes

Standard data for actual output
Actual output 6,400 units
Actual data for actual output
Quantity
Kgs.
Price
Per Kg.
Amount
Rs.
Quantity
Kgs.
Price
Per Kg.
Amount
Rs.
32,000 8 2,56,000 36,000 7.50 2,70,000

2. Labour data

Standard data for actual output
Actual output 6,400 units
Actual data for actual output
Labour Rate/hour Amount Labour Rate/hour Amount
79
hours Rs. Rs. hours Rs. Rs.
32,000 8 2,56,000 36,000 7.50 2,70,000

3. Variable overheads data

Standard/Budgeted data Actual data
Budgeted variable overheads for
actual hours
6,50,000 Actual variable overheads
(Rs.)
6,48,000
Actual Units 6,400
Actual Hours 65,000
Standard variable overhead
Rate/hour
Rs. 10
Standard variable overhead rate/
unit
Rs. 100

4. Sales data

Budgeted data Actual data
Sales
Units
Budgeted
Margin p.u.
Rs.
Amount
Rs.
Sales
Units
Actual
Margin p.u.
Rs.
Amount
Rs.
6,000
50
(Rs. 250 Rs. 200)
3,00,000 6,400
65
(Rs. 265 Rs. 200)
4,16,000

1. Market Size Variance
= Budgeted market share percentage [Actual industry sales in units Budgeted industry sales
in units]
Budgeted contribution margin per unit

= 0.12 [60,000 units 6,000 units/12%] Rs. 50
= 0.12 [60,000 units 50,000 units] Rs. 50 = Rs. 60,000 (F)

2. Market Share Variance
= [

=[0.106666 0.12] 60,000 units X 50
= (6,400 units 7,200 units) Rs. 50 = Rs. 40,000 (A)

3. Gross Margin Sales Volume Variance
= (Actual quantity Budgeted quantity) Budgeted margin per unit
= (6,400 units 6,000 units) Rs. 50 = Rs. 20,000 (F)

4. Gross Margin Sales Price Variance
= (Actual margin per unit Budgeted margin per unit) Actual quantity of units sold
= [(Rs. 65 Rs. 50) 6,400] 6,400 units = Rs. 96,000 (F)

5. Direct Material Usage Variance
= (Standard quantity Actual Quantity) Standard Price per kg.
= (32,000 kgs 36,000 kgs.) Rs. 8 = Rs. 32,000 (A)

Direct Material Price Variance
= (Standard price/kg. Actual price/kg.) Actual quantity of material used
= (Rs. 8 Rs. 7.50) 3,600 kgs. = Rs. 18,000 (F)

6. Direct Labour Efficiency Variance
= (Standard labour hours Actual labour hours) Standard rate per hour
= (64,000 hours 65,000 hours) Rs. 6 = Rs. 6,000 (A)

Direct Labour Rate Variance
= (Standard labour rate per hour Actual labour rate per hour) Actual time taken in hours
80
= (Rs. 6 Rs. 6.40) 65,000 hours = Rs. 26,000 (A)

7. Variable Overhead Efficiency Variance
= (Standard hours for actual output Actual Hours) Standard variable overhead per hour
= (64,000 hours 65,000 hours) Rs. 10 = Rs. 10,000 (A)

Variable Overhead Expense Variance
= Budgeted Variable Overhead Actual Variable Overhead
= Rs. 6,50,000 Rs. 6,48,000 = Rs. 2,000 (F)
Operating Statement
(Reconciling the budgeted contribution with actual contribution

Rs. Rs. Rs.
Budgeted Contribution 3,00,000
Gross margin sales volume variance 20,000 -
Gross margin sales price variance 96,000 - 1,16,000
4,16,000
Cost Variances
Material usage - 32,000
Material price 18,000 -
Labour efficiency - 6,000
Labour rate - 26,000
Variable overhead efficiency - 10,000
Variable overhead expense 2,000 -
20,000 74,000 54,000
Total Actual Contribution 3,62,000

Verification:
Actual Contribution
= Actual sales revenue Actual variable costs
= Rs. 16,96,000 [ RS. 2,70,000 (actual material cost) + Rs. 4,16,000 (actual labour cost) + Rs. 6,48,000 (actual variable
overheads)]
= Rs. 16,96,000 Rs. 13,34,000 = Rs. 3,62,000

Ans.22:
(i) Normal / Budgeted hours =60,000 Direct Labour hours.
Working
(ii) Budgeted output =60,000/ 12 =5,000 units
(iii) Budgeted fixed overhead rate =9,00,000 / 60,000
=Rs.15 per hour or 9,00,000 / 5,000 =Rs.180 per unit
(iv) standard cost and profit per unit (Rs.)
Direct materials (20kg X 10)
Direct labour (12 hrs. X 5.50)
Variable overheads (12 hrs. X 10)
Fixed Overheads (12 hrs. X 15)
Total
Selling price
Standard profit

200
66
120
180
566
600
34


(v) Actual profit (Rs.)
Sales
Less: cost of sales;
Direct Material 10,50,000
Direct wages 3,10,000
Overheads
Actual profit
15,26,000
28,32,000



28,86,000
(54,000)
Direct Material variances
81
DMCV = Standard Cost for actual output Actual cost
=(4,800 X 200 )-10,50,000 =9,60,000-10,50,000 =Rs.90,000 (A)
DMPV = Actual qty. X ( standard rate Actual rate)
=1,00,000 X (10-10.5) =Rs.50,000 (A)
DMUV = Std. rate X (std. qty. for actual output- actual qty.)
=10 x ( 4,800 X 20)-1,00,000 ) =10 X (96,000-1,00,000) =Rs.40,000 (A)


Direct Labour variances
DLCV = Standard Cost of actual output Actual cost
=(4,800 X 12 X 5.50 )-3,10,000 =3,16,800-3,10,000 =Rs.6,800 (F)
DLRV = Actual Time X ( Standard rate Actual rate)
=62,000 X (5.50-5) =Rs.31,000 (F)
DLEV = Std. Rate X (Std. Time. for actual output- actual Time)
=5.50 x ( 4,800 X 12)-62,000 ) =5.50 x (57,600-62,000) =Rs.24,200 (A)

Overhead variances
VOCV = Recovered variable Overheads Actual variable Overheads
=(4,800 X 120 ) 5,86,000 = 5,76,000 5,86,000 =Rs.10,000 (A)
FOCV = Recovered fixed overheads Actual fixed overheads
=(4,800 X 180 ) 9,40,000 =8,64,000 9,40,000 =Rs.76,000 (A)
FOEXPV = Budgeted fixed overheads Actual fixed overheads
=9,00,000 9,40,000 =Rs.40,000 (A)
FOVV = Recovered fixed overheads Budgeted fixed overheads
=8,64,000 9,00,000 =Rs.36,000 (A)
FOCAPV = Std. rate per hour (Actual time budgeted time)
=15 X (62,000 60,000 ) =Rs.30,000 (F)
FOEFEV =Std. Rate per hour X (Std. time for actual output Actual time)
=15 X (4,800 X 12) 62,0000 =15 X (57,600 62,0000=15 X 4400 =Rs.66,000(A)

Sales Variances
Sales Value = Budgeted Sales Actual Sales
Variance =( 5,000 X 600 ) -28,32,000 = Rs.30,00,000 Rs.28,32,000 =1,68,000(A)
Sales Price = Actual qty. (Std. Price Actual price)
Variance = 4,800 X ( 600 590) =Rs.48,000 (A)
Sales Volume = Std. Price X (Budgeted qty. Actual qty.)
Variance =600 X ( 5,000 4,800) =Rs.1,20,000(A)
Loss of profit due to loss of sales volume = 200 X 34 =Rs.6,800 (A)

Ans. 23:
Rs.
Working Notes :
( a ) Actual sales 2,22,750

Less : Price variance (Favourable) 6,750
Standard sales

2,16,000
Units sold

4,800
82

( d ) Standard direct wage rate is Rs.4.50 per hour.
Hence standard time per unit: Rs. 9 4.50 hour = 2 hours
(e) Variable overheads :
Standard rate Rs.7.50 per hour
Variable overhead per unit: 2 hrs. Rs.7.50 = Rs. 15
(Note : Alternatively, this may be calculated by adjusting variances as in other cases).


(f) Fixed overhead spent Rs.39,000
Less : Fixed overhead expense
variance (Adverse) Rs.1,500
Budgeted overheads Rs. 37,500

(g) Fixed overhead recovered: 4,800 units Rs.7.50 = Rs.36,000
(h) Fixed overhead volume variance

Rs.36,000 Rs.37,500 = Rs.1,500 (Adverse)
(i) Budgeted sales: 5,000 units Rs.45 = Rs.2,25,000
(j) Standard sales: 4,800 units Rs.45 = Rs.2,16,000
(k) Actual sales = Rs.2,22,750
(1) Sales volume variance: = Rs.9,000 (Adverse)
Rs. 2,16,000 Rs.2,25,000
(m) Sales price variance:
Rs.2,22,750 Rs.2,16,000 = Rs. 6,750 (Favourable)

(i) Original budget:

Rs.
Budgeted sales : (A) (5,000 units Rs.45) 2,25,000
83
Budgeted costs
Direct material (5,000 units Rs.6) 30,000
Direct wages (5,000 units Rs.9) 45,000
Variable overheads (5,000 units Rs.15) 75,000
Fixed overheads (5,000 units Rs.7.50) 37,500
Total budgeted costs : (B)

1,87,500
Profit : (A) (B) 37,500
(ii) Standard product cost sheet per unit


Rs.
Direct materials 6.00
Direct wages 9.00
Prime cost 15.00
Variable overheads 15.00
Fixed overheads 7.50
Total cost 37.50
Profit 7.50
Selling price 45.00
(iii) Statement showing Reconciliation of the original Budgeted Profit and the Actual
Profit.
Rs. Rs.
Budgeted profit 37,500
Less: Sales margin volume variance (Adverse)*
or loss of profit on sales volume variance
= Rs. 9000 16
2
%
3
**
Standard profit 36,000
1500
*Sales margin volume variance (Adverse)
(200 units Rs.7.50 = Rs.1,500)

**Profit as % of selling price : Rs. 7.50 %

Add: Sales price variance (Favourable)

6,750
42,750
Add: Favourable cost variances:
Wage rate 750
Variable overhead expenses 3,000 3,750
Less : Adverse cost variances 46,500
Material price 300
Material usage 600
Labour efficiency 2,250
Variable overhead efficiency 3,750
Fixed overhead expense 1,500 8,400
38,100
Less: Fixed overhead volume variance (Adverse) 1,500
[See working note (h)]
36,600
84

Ans. 24

:Detai l s of ori gi nal and revi sed standards and actual achi eved
Original standards Revised standards Actual
Fruit 400 Kgs Rs16 Rs6,400 400 Kgs Rs 19 Rs7,600 428 Kgs Rs 18 Rs7,704
Glucose 700 Kgs Rs10 Rs7,000 700 Kgs Rs12 Rs 8,400 742 Kgs Rs 12 Rs 8,904
Pectin 99 Kgs Rs 33.2 Rs 3286.8 99 Kgs Rs 33.2 Rs 3286.8 125Kgs Rs 32.8 Rs 4,100
Citric acid 1 Kg Rs 200 Rs 200 1 Kg Rs 200 Rs 200 1 Kg Rs 95 Rs 95
1,200 kgs Rs16,886.8 1,200 kgs Rs19,486.8 1,296 kgs Rs20,803
Labour Rs 585.0 Rs 585.0 Rs 600
1,200 kgs 17,471.8 1,200 kgs 20,071.8 1,296 kgs 21,403
Loss 36 kgs 36kgs 132
1,164kgs Rs 17,471.8 1,164kgs Rs 20,071.8 1,164 Kgs Rs 21,403
(i ) Pl anni ng vari ances
*
Fruit extract (6,400 less 7,600)
Rs 1,200(Adverse)
Glucose syrup (7,000 less 8,400) Rs1,400(Adverse)
Total Rs 2,600(Adverse)
* (Std qty Std price less Std qty Revised Std price)
(i i ) Ingredi ents operati ng vari ances
Total (19,486.8 less 20,803) = Rs 1,316.2(Adverse)
Ingredients Price variance
(Revised Material Price Actual Material Price)( Actual Qty Consumed)
Variance in Rs
Fruit extract (19 18) 428 428(F)
Glucose syrup Nil
Pectin (33.2 32.8) 125 50(F)
Citric acid (200 95) 1 105(F)
583(F)
Usage variance
(Std Qty on Actual Production less Actual Qty on Actual Production) Revised Std Price/Unit
Rs Variance in Rs
Fruit extract (400 428) 19 532(A)
Glucose syrup (700 742) 12 504(A)
Pectin (99 125) 33.2 863.2(A)
Citric acid Nil
1,899.2(A)
(i i i ) Mi x Vari ance
(Actual usage in std mix less Actual usage in actual mix ) std price
Variance in Rs
Fruit extract (432 428) 19 76(F)
85
Glucose syrup (756 742) 12 168 (F)
Pectin (106.92 125) 33.2 600.3(A)
Citric acid (1.08 1) 200 16(F)
340.3 (A)
Yield variance
(Actual yield Std yield from actual output) Std cost per unit of output
= (1,164 1,296 0.97)
1164
8 . 19486
= 1,558.9(A)
Labour operating variance
585 600 = 15(A)
(i v) Total variance = Planning variance + Usage Variance + Price Variance + labour operating Variance.
Or Total Variance = (2,600) + ( 1,899.2 ) + 583 + (15) = 3931.2 (A).

Ans.26:
Product X Product Y Total
Standard hours produced
Out put (units) 1,200 800
Hours per unit 8 12
Standard hours 9,600 9,600 19,200
Actual hours worked
100 workers 8 hours 22 days = 17,600
Budgeted hours per month
1,86,000/12 = 15,500

Capacity Ratio =
500 , 15
600 , 17
100
hours Budgeted
hours actual
= = 113.55 %

Efficiency Ratio = 100
600 , 17
200 , 19
100
hours Actual
Produced Hours Standard
= 109.09%

Activity Ratio = 100
500 , 15
200 , 19
100
hours Budget
Produced Hours Standard
= 123.87%
Relationship : Activity Ratio = Efficiency Ratio Capacity Ratio

or 123.87 =
100
55 . 113 09 . 109


Ans: 27:

(1) Capacity Ratio
=
Budgeted working hours x 100
Actual working Hours
86

= 25 days x 8 hours x 50 workers
8,500 hours (i.e.,1,02,000/12)
x 100 =117.65%

(2) Activity Ratio

=Actual production in standard hours
Budgeted hours
x 100

=(1,000 units x 5 hours) + (600 units x 10 hours)
8,5000 hours
x 100 =129.41%

(3) Efficiency Ratio
=Standard hours for actual production
Actual hours
x 100

=(1,000 units x 5 hours ) + (600 units x 10 hours)
10,000 hours
x 100 =110%
Inter relationship
Capacity Ratio x Efficiency Ratio =Activity Ratio
117.65% x 110% =129.41%

Ans. 28:
(a)
Report to the Departmental Manager showing the cost ratios:
Standard hours produced 2112
Efficiency Ratio = 100 110%
Actual hours worked 1920
= =
(b)
Standard hours produced 2112
Activity Ratio = 100 82.50%
Budgeted Std. Hours 2560
= =
(c)
Budgeted Std. Hours 2560
Standard Capacity usage Ratio = 100 80%
Maximum Possible Hours 3200
= =
(d)
Actual hours worked 1920
Actual Capacity utilisation Ratio = 100 75%
Budgeted hours 2560
= =
(e)
24
Calendar Ratio = 100 96%
25
=

(ii) Report to the Departmental Manager Setting out the analysis of variances

Standard fixed overhead rate per hour =
15360
.6
2560
Rs =

A. Fixed Overheads Rs.
(a) Charged to production (2112 6) 12672
(b) Actual hours Std. rate (1920 6) 11520
(c) Revised budgeted hours Std. rate (24816
80
100
6) 14746
(d) Original budgeted overheads 15360
(e) Actual overheads 16500

Variances:
Efficiency variance (a-b) 1152(F)
Capacity variance (b-c) 3226(A)
87
Calendar variance (c-d) 614(A)
Volume variance (a-d) 2688(A)
Expenditure variance (d-e) 1140(A)
Total variance 3828(A)

B. Variable overheads:
Standard variable overhead rate per hour =
20840
2560
=Rs.8
(a) Charged to production (2112 8) 16896
(b) Actual hours Std. rate (1920 8) 15360
(c) Actual overheads 14500

Variances:
Efficiency variance (a-b) 1536(F)
Expenditure variance (b-c) 860(F)
Total variance (a-c) 2396(F)

Working note:
Maximum possible hours (25816) 3200
Budgeted hours: 3200 less 20% downtime 2560
Actual hours 1920
Budgeted standard hours 2560
Standard hours produced 5112
Budgeted working days 25
Actual working days 24

Ans. 29:
Maximum capacity in a budget period

= 50 employees 8 hrs.5 days4 weeks
= 8,000 hrs.
Budgeted hours
40 employees 8 hrs.5 days4 weeks
= 6,400 hrs.
Actual hrs. = 6,000 hrs. (from the sum) Standard hrs.
for actual output = 7,000 hrs.
Budget no. of days = 20 days = 20 days (4 weeks 5 days) Actual no.
of days = 20-1 = 19 days

100 {(7000 6000) 100} 116.67%
Standard Hrs
1. Efficiency ratio =
Actual Hrs
= =

2. Activity ratio = {(7,0006,400)100} = 109.375%
3. Calendar Ratio = (Available working days budgeted working days) 100

88
= {(1920)100} = 95%

4. Standard Capacity Usage Ratio =
(Budgeted hours Max. possible hours in the budgeted period) 100

= {(6,4008,000)100} = 80%

5. Actual Capacity Usage Ratio =
(Actual hours worked Maximum possible working hours in a period) 100

= {(6,0008,000)100} = 75%

6. Actual Usage of Budgeted Capacity Ratio = (Actual working
hours Budgeted hours) 100
= {(6,0006,400)100} = 93.75%


(i )
Ans.30:
Dr. Material Control A/c
Dr. or Cr. Material Price Variance A/c
Cr. Creditors A/c
(Being price variance during purchase of
materials)

(i i ) Dr. WIP Control A/c
Dr. or Cr. Material Usage Variance A/c
Cr. Material Control A/c
(Being recording of usage variance at
Standard cost of excess/under utilized
quantity)

(i i i ) Dr. Wages Control A/c
Dr. or Cr. Labour Rate Variance A/c
Cr. Cash
(Being entry to record wages at standard rate)

Ans. 31:

(A) The cost sheet f or 900 uni t s wi l l appear as under :
Cost Std. qty. Std. rate Std.cost
Rs.
Direct material 9,000 1.00 9,000
Direct labour 2,250 3.00 6,750
Overheads 2,250 6.00 13,500
29,250
(B) Calculation of variances:
Material price variance = 9,500 Pcs. (Re. 1.00 Rs.1.10) = Rs. 950 (A)
Material usage variance = Re. 1.00 (9,000 pcs. 9,500 pcs.) = Rs. 500 (A)
Labour rate variance = 2,475 hrs. (Rs. 3.00 Rs. 3.50)
= Rs. 1,237.50 (A)
Labour efficiency variance = Rs. 3.00 (2,250 hrs. 2,475 hrs.) = Rs. 675(A)
Overhead variances :
(a) Charged to production as per cost sheet Rs. 13,500 (b) Actual
89
Work-in-Progress A/c Dr. 17,000
To Overhead Expense Control A/c 17,000
(Being the actual overhead expenses incurred)
Finished Stock Control A/c Dr. 29,250
To Work-in-Progess A/c 29,250

hours Std. rate: 2,475 hrs. Rs. 6 Rs. 14,850 (c)
Overheads as per budget Rs. 16,500
(d) Actual overheads Rs. 17,000

Efficiency variance : (a b) Rs. 1,350 (A)
Capacity variance : (b c) Rs.1,650 (A) (idle time)
Expense variance : (c d) Rs. 500 (A)
Total variance : (a d) Rs. 3,500 (A)
(C) The. j ournal ent ri es f or recordi ng t hese t ransact i ons are as under
Dr. Cr.
Rs. Rs.
(i) Material Control A/c
To General Ledger Adjustment A/c
(Being the purchase value of 10,000
pieces of materials at Rs. 1.10 each)
Dr. 11,000


11,000
(ii) Work-in-Progress A/c Dr. 10,450
To Material Control A/c 10,450
(Being the cost of 9,500 pieces of
materials actually issued to production
at the actual price of Rs. 1.10 each)
(iii) Work-in-Progress A/c Dr. 8,662.50
To Wages Control A/c 8,662.50
(Being the actual amount of direct
wages paid for 2,475 hours at Rs. 3.50 per hour
(iv)




(v)


(Being the standard cost of production
transferred to finished goods account)
(vi) Cost of Sales A/c Dr. 29,250
To Finished Stock Control A/c 29,250
(Being the standard cost of goods
sold transferred to Cost of Sales A/c)

After the basic transactions are posted, the materials control account will show the actual value of
stock of material in hand and the work-in-progress account will show a balance representing the
cumulative variances on all the accounts and closing balance of work-in- progress at standard cost.
The variances have already been analysed in Para (B) above and they will be carried to the respective
accounts pending investigation before being finally disposed off. In this problem we have assumed that
there is no closing balance of work-in- progress.

(D) The journal entries for transferring the variances to their respective
90
accounts are as under

Rs. Rs. Material
price variance A/c Dr. 950.00
Material usage variance A/c Dr. 500.00
Labour rate variance A/c Dr. 1,237.50
Labour efficiency variance A/c Dr. 675.00

Overhead efficiency variance A/c Dr. 1,350.00

Overhead capacity variance A/c Dr. 1,650.00

Overhead expense variance A/c Dr. 500.00

To work-in-progress A/c 6,862.5
(E) The ledger accounts will appear as under:

Dr. Material Control A/c Cr.

Rs. Rs.

To Opening balance - By Work-in-Progress A/c 10,450
To General Ledger

By Balance c/d 550
Adjustment A/c 11,000

11,000
11,000
Work-in-Progress Control A/c

Rs. Rs.
To Opening balance By Finished stock control A/c 29,250.00
To Material control A/c 10,450.00 By material price variance A/c 950.00
To Wages control A/c 8,662.50 By material usage variance A/c 500.00
To Overheads control A/c 17,000.00 By labour rate variance A/c 1,237.50
By labour efficiency variance A/c 675.00
By overhead efficiency A/c
Variance A/c 1,350.00
By overhead capacity
Variance A/c . 1,650.00
By overhead expense
Variance A/c 500.00
36,112.50 36,112.50

Ans. 32:
(i) Material price variance: 8,600 pcs. (Rs. 2.15 Rs. 2.50) = Rs. 3,010 (A) (ii) Material
usage variance: Rs. 2.15 (8,400 Pcs. 8,600 Pcs.) = Rs. 430 (A)
(A)Computati on of vari ance:
[Standard requirement of materials = 2,800 units produced 3 pcs. per unit = 8,400 pcs.]
(iii) Labour efficiency variance:
Dept. A: Standard time required = 2,800 pcs. 2 hrs. = 5,600 hours. Dept. B: Standard
time required = 2,800 pcs. 4 hrs. = 11,200 hours.

Variances :
Dept. A: 1.75 (5,600 5,200) = Rs. 700 (F) Dept. B: 1.50 (11,200 12,000) = Rs. 1,200 (A)
(iv) Overheads variances:
91



(i) Material Control A/c Dr. 18,490

Material price variance A/c Dr. 3,010
To Creditors A/c 21,500
(ii) Work-in-Progress Dept. A. A/c Dr. 18,060
Material usage variance A/c Dr. 430
To Material Control A/c 18,490
(iii) Work-in-progress Dept. A. A/c Dr. 9,800
To wages control A/c 9,800
(iv) Wages Control A/c Dr. 700
To Labour Efficiency Variance Dept A A/c 700
(v) Work-in-Progress Dept. B A/c. Dr. 16,800
Labour Efficiency Variance Dept. B A/c Dr. 1,200
To Wages Control A/c 18,000
(vi) Work-in-Progress Dept. A A/c Dr. 2,800
Overhead Capacity Variance Dept. A. A/c Dr. 400
To Overhead Efficiency Variance Dept. A. A/c 200
To Overhead Expense Control Dept. A A/c 3,000
(vii) Work-in-Progress Dept. B A/c Dr. 11,200
Overhead Efficiency Variance A/c Dr. 800
Overhead Expenses Variance A/c Dr. 500
To Overhead Control Dept. B A/c 12,500
(viii) Work-in-Progress Dept. B A/c Dr. 30,660
To Work-in-Progress Dept. A A/c 30,660
(Being the transfer at standard cost of finished
Production of Department A to Department B
for processing in Department B)
(ix) Finished Stock control A/c Dr. 58,660
92
To Work-in-Progress Dept. B A/c 58,660


93


94


Ans.33:

All figures of Ans. 31are 5 times of Ans. 32
Ans. 34:
= Rs24, 000 Rs21, 600 = Rs2, 400 (F)
Material 1 Rate Variance = Standard cost of material purchased Actual cost
Material 2 Quantity Variance = SR SQ SR AQ
= Rs900 80 units Rs75, 600
= Rs3, 600 (A)
Labour Spending Variance = SR AH AR AH
= Rs24/per hour 2300 hours Rs51, 750
= Rs3, 450 (A)
Labour Efficiency Variance = SR (SH AH)
7200 = 24 (SH 2300)
SH = 2000 Hrs.
Rs
Total Cost of material purchased 1,27,200
Less Purchase Value of Material 2 1,05,600
Cost of material 1 21,600
Worki ng Notes:
(1) Standard Cost of Material 2 actually consumed in production = Rs72, 000 (Given)
Standard cost of Material 2 per unit: 5 litres Rs180 = Rs900
No of units produced = Rs72, 000 / Rs900 = 80 units
Total material 1 used in production = Rs18, 000 (Given)
Add Closing Inventory = Rs6, 000 (Given)
Less Opening Inventory = 0
Hence Standard Cost of Material 1 purchased = Rs24, 000
95
(2) Standard Rate of Material -1 = Rs24, 000 / 1,000kg
= Rs24 per kg
Standard Cost of Material 1 = Rs18, 000
Add favourable Quantity Variance = Rs1, 200
Material 1 allowed = Rs19, 200
Standard quantity of Material 1 allowed = Rs19, 200/Rs24= 800 Kg.
Standard quantity per unit = 800kg/80units = 10 kg
Standard purchase price for Material 2 = (550liters Rs180)= Rs99, 000
Add unfavourable Rate Variance = Rs6, 600
Actual cost Price of Material 2 = Rs1, 05, 600
(3) Opening balance of Material 2 = Rs18, 000
Add Standard Cost of Purchase (550 litres Rs180) = Rs99, 000
Less Closing Balance = Rs41, 400
Material-2 Consumed at Standard cost = Rs75, 600

Ans. 35:
We know that:
(i) Budgeted Machine Hours:
Volume variance =
|
|
.
|

\
|

output actual for
hours machined Budgeted
output actual for
hours machine Std.
hour per rate
overhead fixed Std.

or Rs.80,000 (Fav.) = Rs.100 (11,300 Y)
or 800 = 11,300 Y
or Y = (11,300 800) hours
or Y = 10,500 hours
Hence budgeted machine hours for actual output are 10,500 hours.
(ii) Actual machine Hours: We know that:
Efficiency variance =
|
|
.
|

\
|

output actual for
hours Actual
output actual
for hours Std.
hour per rate
overhead variable Std.

or Rs.36,000 (Fav.) = Rs.60 (11,300 hours X)
or 600 = 11,300 hours X
or X = 10,700 hours.
Hence Actual machine hours are 10,700 hours.
(iii) Applied Manufacturing Overhead:
Applied Manufacturing overhead
Actual overhead incurred + Total Variance
= Rs.16,50,000 + Rs.30,000
(Refer to working note)
= Rs.16,80,000
Hence total applied manufacturing overhead are Rs.16,80,000.
96
(iv) Total Amount of Fixed Overhead Cost:
We know that:
Spending variance = (Flexible budget for actual hours Actual factory overhead incurred)
Rs.86,000 (Adv.) = 10,700 hours Rs.60 + total amount of fixed overhead) Rs.16,50,000)
Rs.86,000 (Adv.) = (Rs.6,42,000 + Total amount of fixed overhead cost (budgeted) Rs.16,50,000)
Total amount of fixed overhead cost = Rs.10,08,000 Rs.86,000 = Rs.9,22,000
Total amount of fixed overhead cost = Rs.9,22,000
Worki ng note:
Given that:
Spending variance (Rs.) 86,000 (Adv.)
Efficiency variance (Rs.) 36,000 (Fav.)
Volume variance (Rs.) 80,000 (Fav.)
Therefore,
Total variance = Spending variance + Efficiency variance + Volume variance
= Rs.86,000 (Adv.) + Rs.36,000 (Fav.) + Rs.80,000 (Fav.)
= Rs.30,000 (Fav.)
Alternative approach:
Total factory overhead variance = {factory overhead applied - actual factory overhead incurred}
= (Std. hours for actual output Budget rate per hour Actual cost incurred)
= (11,300 hours Rs.160 Rs.16,50,000)
= Rs.1,58,000 (Fav.)
Under alternative approach, Applied Manufacturing Overhead and Total Amount of Fixed Overhead Cost would
come to Rs.18,08,000 and Rs.10,50,000. Budgeted and actual machine hours would come to 10,500 and 10,700.
Spending, Efficiency and Volume Variances would come to Rs.42,000 (Fav.), Rs.36,000 (Fav.) and Rs.80,000 (Fav.)
respectively.


Ans. 36:
Material cost variance = Standard cost of material of actual output Actual material cost incurred
(1) Actual material cost incurred
Or Actual material cost incurred =
| |
|
. \

Standard variable of material Material cost
variance of actual output

= (10,000 units 2 units Rs.15 + Rs.50,000)
= Rs.3,00,000 + Rs.50,000
(2) Standard cost of materials actually consumed
Material price variance = (Standard cost Actual cost) Actual quantity consumed
Or Standard cost of materials actually consumed =
| |
|
\ .
+
Actual material Material price
cost incurred variance

= Rs.3,50,000 Rs.70,000 = Rs.2,80,000
(3) Labour efficiency variance (Refer to working note 1)
97
=
hour per
rate Standard
worked
hours Actual
output actual
for hours Standard
|
|
.
|

\
|

= (10,000 units 3 hours 35,000 hours) Rs.20
= (Rs.6,00,000 Rs.7,00,000) = Rs.1,00,000 (Adv.)
(4) Variable OH efficiency variance (Refer to working note 2)
=
|
|
.
|

\
|

hours
Actual
hours
Standard
hour per rate
overhead variable Standard

= Rs.5 (30,000 hours 35,000 hours) Rs.25,000 (Adv.)
(5) Variable OH expenditure variance (Refer to working note 1)
=
|
|
.
|

\
|

overhead
variable Actual
hours actual for
overhead variable Budgeted

= (Rs.5 35,000 hours Rs.2,00,000) Rs.25,000 (Adv.)
(6) Fixed OH efficiency variance (Refer to working notes 1 & 2)
=
(


hours
Actual
ouput actual
for hour Standard
hour per rate
overhead fixed Standard

= Rs.5 (30,000 hours 35,000 hours) = Rs.25,000 (Adv.)
Fixed OH capacity variance (Refer to working notes 1 & 2)
=
(


hours capacity
Budgeted
hours
capacity Actual
hour per rate
overhead variable Standard

= Rs.5 (35,000 hours 50,000 hours) = Rs.75,000 (Adv.)
(7) Fixed OH volume variance (Refer to working note 3)
=
|
|
.
|

\
|

output
Budgeted
output
Actual
hour per rate
overhead variable Standard

= Rs.15
|
.
|

\
|

hours 3
hours 50,000
units 10,000
= Rs.1,50,000 Rs.2,50,000 = Rs.1,00,000 (Adv.)
Worki ng notes:
1. Labour rate variance:
= (Standard rate per hour Actual rate per hour) Actual hours (x)
Or Rs.50,000 = 20x Rs.6,50,000
Or x = 35,000 hours
2. Standard hours = 10,000 units 2 hours = 30,000 hours
Budgeted hours =
|
.
|

\
|
60
100 hours 30,000
= 50,000 hours
Budgeted fixed overhead = Actual fixed overhead + Expenditure variance
= Rs.3,00,000 Rs.50,000 = Rs.2,50,000
98

(

hour per rate recovery


overhead fixed Standard
=
hours 50,000
0 Rs.2,50,00
= Rs.5 per hour
Total overhead rate per hour = Rs.10
Variable overhead rate per hour = Rs.5 (Rs.10 Rs.5)
3. Standard fixed overhead per unit = Rs.15 (3 hours Rs.5/-)

Ans. 37:
1. (a) Budgeted fixed overhead per unit:
Working notes:
= (Budgeted fixed overheads p.a / Budgeted output for the year)
= Rs.4,80,000 p.a. / 1,20,000 units = Rs.4 per unit.
(b) Budgeted fixed overhead hour:
= Budgeted fixed overhead per unit / Standard labour hours per unit
= Rs.4 / 2 hours = Rs.2 per hour
2. (a) Standard cost per unit:
Rs.
Direct material 20
(5 kg Rs.4/- per kg)
Direct labour 6
(2 hours Rs.3/- per hour)
Fixed overhead 4
(2 hours Rs.2)
Total standard cost (per unit) 30
(b) Budgeted selling price per unit
Standard cost per unit 30
Standard profit per unit 10
(25% on slaes or 33 1/3% of standard cost)
Budgeted selling price per unit 40
3 (a) Actual output units for April, 2001:
Fixed overhead volume Variance
= Efficiency variance + Capacity variance
or (Budgeted output units Actual output units) Budgeted fixed overhead p.u.
Rs.2,400 (Favourable) + Rs.4,000 (Adverse) = Rs.1,600 (Adverse)
or (10,000 units x units) Rs.4 Rs.1,600 (Adverse)
or (10,000 units 400 units) = x (Actual output units)
or Actual output units = 9,600 units
(b) Actual fixed overhead expenses:
(budgeted fixed overhead Actual fixed overhead) = Fixed overhead expenses variance
or (Rs.40,000 x) = Rs.1,400 (Favourable)
99
or x = Rs.40,000 Rs.1,400
= Rs.38,600
4. (a) Actual sales quantity units: Sales volume variance
= Budgeted margin per unit
|
|
.
|

\
|

units quantity
Budgeted
units quantity
sales Actual

= Rs.4,000 (Adverse) = Rs.10 (x 10,000 units)
or 400 units = x 10,000 units
or x (Actual sales quantity) = 9,600 units
(b) Actual selling price per units
Sales price variance =
units Sales
Actual
unit per price
selling Budgeted
unit per price
Selling Actual
|
|
.
|

\
|

or Rs.9,600 (Fav.) = (x Rs.40) 9,600 units
or Actual selling price per unit = Rs.41/-
5. (a) Actual quantity of material consumed:
Material usage variance =
unit per
price Standard
quantity
Actual
quantity
Standard
|
|
.
|

\
|

or 6,400 (Adv.) = (9,600 units 5 kgs.) Rs.4
or x kgs. = 49,600 kgs.
(actual quantity of material consumed)
(b) Actual price per kg:
Actual price per kg.:
Material price variance = (Standard price per kg Actual price per kg) Actual quantity of material
consumed
-Rs.4,960 = (Rs.4 Rs. y per kg.) 49,600 kg.
-0.1 = (Rs.4 Rs. y per kg)
or y = Rs.4.10 per kg.
6. (a) Actual direct labour hour used:
Labour efficiency variance = (Standard hours Actual hours) Standard rate per hour
Rs.3,600 (Favourable) = (9,600 units 2 hours p hours) Rs.3
Rs.3,600 (Favourable) = (19,200 hours p hours) Rs.3
P hours = (19,200 hours 1,200 hours) 18,000 hours
(Actual direct labour hours)
(b) Actual direct labour hour rate:
Labour rate variance =
hours labour
Direct Actual
hour per
rate Actual
hour per rate
Standard
|
|
.
|

\
|

Rs.3,600 (Adverse) = (Rs.3 per hour t per hour) 18,000 hours
or t = Rs.3 + Rs.0.20 Rs.3.20 per hour
100
(actual direct labour hour rate)
7. Actual fixed overheads:
Fixed overhead expense variance = Budgeted fixed overhead Actual fixed overhead
or Rs.1,400 (Favourable) = 10,000 units Rs.4 p.u. Actual fixed overhead
or Actual fixed overhead = Rs.40,000 Rs.1,400
or Actual fixed overhead = Rs.38,600
Annual financial Profit /Loss Statement
(for April, 2001)
Account Qty./ Hours Rate/Price Actual/ Value
(a) (b) (c) (d)=(b)(c)
Sales: (A) 9,600 units 41 3,93,600
(Refer to working note 4)
Direct Materials 49,600 kgs. 4.10 per kg. 2,03,360
(Refer to working note 5)
Direct labour 18,000 hours 3,20 per hour 57,600
(Refer to working note 6)
Fixed Overheads 18,000 hours 2.14444 per hour 38,600
(Refer to working note 6 (a) and 7)
(Rs.38,600/18,000 hours)
(absorbed on direct labour hour basis)
Total costs: (B) 2,99,560
Profit : [(A) (B)] 94,040

Working notes:
Ans: 38.
1. Direct material units in actual output (Units)
Output of units produced
Add: Closing WIP units (200 units x 50% complete)
Less: Opening WIP units (300 units x 100% complete)
Total direct material units in actual output (work done)(i.e. units introduced)
7,620
100
(300)
7,420

2. Basic data of direct materials

Standard Data
Actual output units 7,420
Actual Data
Standard quantity of
material

11.130
(7,420 units x 1.5 kgs.)
S.P./ KG.
Rs.

24
Amount
Rs.

2,67,120
Actual qty. of
material kgs.

11,224
A.P./KG.
Rs.

23.75
Amount
Rs.

2,66,570


3. Direct wages and overhead units in actual output (Units)
Output of units produced
Add: Closing WIP units (200 units x 40% complete)
Less: Opening WIP units (300 units x 60% complete)
Total direct wages and overhead units in actual output (work done)(i.e. units introduced)
7,620
80
(180)
7,520
4. Basic data of direct wages

Standard Data
Actual output units 7,520
Actual Data
101
Standard Labour
hours

22,560
(7,520 units x 3 hours)
S.W./ hour
Rs.

400
Amount
Rs.

90,240
Actual Labour hours


22,400
A.W./ hour
Rs.

4.30
Amount
Rs.

96,320


5. Budgeted variable overhead per unit =
Difference in output
Difference in factory overhead

=
(7,500 units 6,000 units)
Rs.92,400 Rs.81,600

=Rs.7.20 per unit

6 Budgeted fixed overheads (Rs.)
Total overhead on 8,000 units (8,000 units x 12 0
Less: Variable overhead of 8,000 units @ Rs.7.20 per unit
Budgeted fixed overheads
96,000
(57,600)
38,400

7. Basic data for variable overhead
Budgeted data Actual data
Budgeted variable overhead
For actual hours (22,400 hours x Rs.2.40
Standard hours required per unit
Standard variable overhead rate p.u

Standard variable overhead rate p.u.
Rs.53,760

3
Rs.7.20

Rs.240
Actual variable overhead
Actual output units
Actual hours
Actual variable overhead
Recovery rate per hour
Rs.58,240
7,520
22,400
Rs.2.60


8. Basic data for fixed overhead
Standard / Budgeted data Actual data
Budgeted fixed overhead

Budgeted output
Budgeted hours
Standard fixed overhead rate per hour
Standard fixed overhead p.u
Standard hours required p.u.
Rs.38,400

8,000 units
24,000
Rs.1.60
Rs.4.80
3
Actual fixed overhead
(Rs.96,440 Rs.58,240)
Actual output
Actual hours


Rs.38,200

7,520 units
22,400

Computation of Variances:
Material variances
1. Material usage variance = (S.Q.-A.Q.) S.P.
= (11,130 kgs.-11,224 kgs.) Rs.24 =Rs.2,256 (A)

2. Material price variance = (S.P.-A.P.) A.Q
= (Rs.24-Rs.23.75) 11,224 kgs. =Rs.2,806 (F)

3. Material cost variance = (S.C.-A.C.)
= (Rs.2,67,120-Rs.2,66,570) =Rs.550 (F)

Labour variances
1. Labour efficiency variance = (S.H.-A.H.) S.R.
= (22,500 hours- 22,400 hours) Rs.4 =Rs.640 (F)

2. Labour rate variance = (S.R.-A.R.) A.H
= (Rs.4-Rs.4.3) 22,400 hours =Rs.6,720 (A)

3. Labour cost variance = (S.C.-A.C.)
= (Rs.90,240 - Rs.96,320) =Rs.6,080 (A)

Variable Overhead variances
102
1. Variable overhead ={ Budgeted variable overhead Actual variable overhead}
Expenditure variance
= (Rs.53,760 Rs.58,240) =Rs.4,480 (A)
2. Variable overhead =Standard variable {Standard hrs. Actual hrs}
Efficiency variance overhead rate per hour
=Rs.2,40 (22,560 hrs 22,400 hrs) =Rs.384 (F)

3. Total variable overhead cost variance = { Standard variable overhead Actual variable overhead}

= (7,520 units x Rs.7.20 Rs.58,240) =Rs.4,096 (A)

Fixed Overhead variances
1. Expenditure variance ={ Budgeted fixed overhead Actual fixed overhead}

= (Rs.38,400 Rs.38,200) =Rs.200 (F)
2. Volume variance = { Budgeted volume Actual volume} Standard fixed overhead rare per
unit

=(8,000 units 7,520 units) Rs.4.80 =Rs.2,304 (A)

3. Efficiency variance = { Standard hours for actual production Actual hours} Standard fixed
overhead rate per hour

= 22,560 hours 22,400 hours) Rs.1.60 =Rs.256 (F)
4.Capacity variance ={Budgeted hours Actual Hours } standard fixed overhead rate per hour
= (24,000 hours 22,400 hours ) Rs.1.60 =Rs.2,560 (A)

5.Total fixed overhead cost variance ={Fixed overhead recovered Actual overhead}
={7,520 units x Rs.4.80 Rs.38,200} =Rs.2,104 (A)

Statement of Equi valent Production in Uni ts
Ans. 39:

Particulars

Materials
Wages & Overhead
% age Units %age Units
Units Completed 100%

9000 100% 9000
Closing W.I.P. 100% 900 50% 900
Equivalent Units 9900 9900

Material Variances
Standard qty for actual output **
x std price
Actual qty
X actual price
Material A
Material B
19,800 @ 3
9,900 @ 4
= 59,400
= 39,600
22,275@2.8*
10,889 @4.1*
= 62,370
= 44,649
29,700 99,000 33,165 1,07,019


*Actual Cost / Actual Quantity

** Standard Quantity for actual output = ( std qty/ budgeted prod) x actual output

MCV = TSC TAC
= 99,000 1,07,019 = 8,019 (A) MPV = AQ (SP AP)

A = 22,275 (3 2.80) = 4,455 (F)
B = 10,890 (4 4.10) = 1,089 (A)
103

3,366 (F)
MUV = SP (SQ AQ)

A = 3 (19,800 22,275) = 7,425 (A)
B = 4 (9,900 10,890) = 3,960 (A)

11,385 (A)
MMV = SP (RSQ AQ)

A = 3 {19,800 29,700 33,165 22,275} = 495 (A)
B = 4 {9,900 29,700 3,165 10,890} = 660 (F)
165 (F)
MYV = S. C Per Unit (S. O. For Actual Mix A. O.)
= 99,000 9,900 {9,900 29,700 33,165 9,900}

= 10 (11.055 9,900) = 11,550 (A)

Labour Variances:

LCV = TSC TAC

= 2,40,000 12,000 9,450 1,91,250 = 2,250 (A)
LRV = AH (SR AR)
= 48,000 {4 (1,91,250 48,000)} = 750 (F)
LITV = No. of Idle hours SR
= 48,000 (47,500 4) = 1,200 (A)
LEV = SR (SH AH)
= 4 {(60,000 12,000) 9,450 47,700} = 1,800 (A)

(i i ) Variable Overhead Variances

VOC = Recovered Overheads Actual Overheads

= 9,450 5 45,000 = 2,250 (F)

V.O (Exp.) V = Standard V.O. Actual V.O.

= 47,700 1 45,000 = 2,700 (F)

V.O. (Eff.) V = Recovered Overheads Standard Overheads

= 9,450 5 47,700 = 450 (A)

Fixed Overheads Variances

FOCV = Recovered Overheads Actual Overheads

= (1,20,000 12,000) 9,450 1,20,900 = 94,500 1,20,900

= 26,400 (A)

F.O.(Exp.) V = Budgeted Overheads Actual Overheads

= 1,20,000 1,20,900 = 900 (A)

FOVV = Recovered Overheads Budgeted Overheads

= 95,500 1,20,000 = 25,500 (A)

Sales Variances

Sales Price Variance = Actual Unit Sold (SP AP)

104
= 9,000 {50 (4,57,500 9,000)} = 7,500 (F)
Sales Volume Variance (Contribution Loss)
= S. R. of Profit (Budgeted Qty. Actual Qty.)

= (60,000 12,000) (12,000 9,000) = 15,000 (A)

(a) sales Variance
Ans 40:.
Present Market size =60,000 units.

At 16% the share should have been = 60,000 x 16
100 =9,600 units.


Standard Gross Margin : SP Rs.53 ( DM Rs.9 + DL Rs.24 + VO Rs.4 + FO Rs.12) = Rs.4
Budgeted Qty. Revised Budgeted
Qty
Actual Qty. Booked Actual Qty.
Supplied
Std. Gross Margin
(Rs.)
8,000 9,600 8,200 7,500 4

(Rs.)
Budgeted Qty. x
Std. G.M.
Revised
Budgeted Qty x
Std. G.M.
Actual Qty.
Booked x Std.
G.M.
Actual Qty.
Supplied x
Std.G.M.
Actual G.M. Actual Qty.
supplied x
Actual G.M.
32,000 38,400 32,800 30,000 5 37,500

Market size variance 32,000 - 38,400 =6,400 F
Market share variance 38,400 32,800 =5,600 A
Sales volume variance 32,800 30,000 =2,800 A
Sales price variance 30,000 37,500 =7,500 F
Sales Margin Production Quantity Variance = (7500-8200)X4 = 2800 A

[Note: Since actual order received actual sales quantity, Market share variance will be on the basis of actual order
received and we will also calculate one further variance regarding inefficiency of production department about fulfilling
order quantity, Sales Margin Production Quantity Variance = (Actual sales quantity Sales order quantity) Std. margin
p.u. While calculating all other variance sales order quantity shall be ignored.]

(b) Direct Material Variances (Units)
Std. requirement 7,200 units @ 1.5 kg. =10,800 kg.

Std. Qty.


Actual Qty. S.P.

Rs.
Std. Qty. x
SP
Rs.
Actual Qty. x
SP
Rs.
AP

Rs.
Actual Qty. x
AP
Rs.
10,800 12,000 6 64,800 72,000 6/50 78,000

Usage Variance Rs.64,800 Rs.72,000 =Rs.7,200 A
Price Variance Rs.72,000 Rs.78,000 =Rs.6,000 A
Total Variance Rs.64,800 Rs.78,000 =Rs.13,200 A

(c ) Direct Labour Variance
Std. hours produced
7,230 x 4 = 28,920
Production
- Op. Stock
+ Cl. Stock
Introduced
7,500
600
300
7200
Production

Less: Op. Stock 600 x 100
75


Add: Cl. Stock 300 x 100
60

7,500

450


180
7,230
105


Std. Hours


Actual Hours S.R.

Rs.
Std. Hrs. x
SR
Rs.
Actual Hrs. x
SR
Rs.
AR

Rs.
Actual Hrs. x
AR
Rs.
28,920 29,000 6 1,73,520 1,74,000 6 / 25 1,81,250

Efficiency Variance (1,73,520 1,74,000) =480 A
Rate Variance (1,74,000 1,81,250) =7,250 A
Total variance =7,730 A

(d) Variable Overheads Variance
Rs.
A =Charged to Production 28,920 Efficiency variance =Rs.80A
28,920 x 1

B =Std. Cost of Actual Hours 29,000 Expenditure variance =Rs.7,000 A
29,000 x 1

C =Actual Overheads 36,000

A C Total V =Rs.7,080 A


(e) Fixed Overhead Variance

(Rs.)
A = Charged to Production 28,920 x 3 86,760
B = Std. Cost of Act. Hrs. 29,000 x 3 87,000
C = Budget 96,000
D = Actual 94,000
Efficiency Variance (86,760 87,000) =Rs.240 (A)
Volume Variance (86,760 94,000) =Rs.9,240 (A)




Ans. 41:
Rs.
(1) Budgeted contribution = Budgeted Profit + Budgeted Fixed
Cost
15,000 + 15,000 = 30,000
Plus Contribution quantity variance 1,800
Total Standard contribution 31,800
Standard Contribution per unit 3
Actual Sales Volume 10,600 units
(2) Actual Sales Volume 10,600 17 1,80,200
(3) Actual quantity of Raw Materials used
Standard consumption 10,600 5 2,000 Kgs.

Add: Material Usage Variance
.2
400

2,000 kgs.
Actual consumption 55,000 Kgs.
(4) Labour Efficiency variance
Standard labour cost for Standard hours (63,000 + 600) 63,600
Standard labour cost for actual hours 61,950
106
Labour efficiency variance 1,650 F
(5) Actual variable overhead

Selling Overhead variance Variable overhead Rs. 84,800 Rs. 1,800 = Rs. 83,000
(6) Variable Overhead efficiency variance
Actual hours (AH)
5 . 1
950 , 61

41,300 hours
Standard hours (SH) 60,600 4 42,400 hours
Standard rate per hour (SR)
4 600 , 10
600 , 63


Rs. 1.5
Efficiency variance SR (SH AH) = 2 (42,400 41,300) = 2,200F
(7) Actual fixed overheads: Budgeted Overhead + Fixed Overhead
variance = 15,000 + 600 = Rs. 15,600.

(8) Operating profit variance
If budgeted profit is considered (15,000 7,000) = Rs. 8,000 adverse
If standard profit is considered (16,800 7,000) = Rs. 9,800 adverse

Ans. 42:

Where RSQ
B
= Revised Standard Quantity of B = (Actual total qty of all DM used)

Standard Mix %age of B and

SQ
B
= Standard quantity of DM B for Actual Production = Standard quantity of all
DM allowed for actual output Standard Mix %age of B

Since Standard Mix %age is the same for both A and B (1: 1) we have, Total Yield variance
for A and B= T (Std price of A + Std price of B)
Where T = (Std qty of all DM allowed for actual output - Actual total qty of all DM used) 0.5

As Total Yield variance for A and B is given as Rs 270, we have

- Rs 270 = T Rs 24 + T Rs 30

Or T = - 5

Hence Yield Variance for A = - 5 24 = - Rs 120 and

107
Yield variance for B = - 5 30 = - Rs 150. Also

Similarly

(SQ
B
- RSQ
B
) 30 = - 150 or SQ
B
- RSQ
B
= - 5
Al ternati ve 1

Let total actual quantity consumed; X kg. Then, Quantity of
A = X 70
RSQ =
X
of A &
X
of B. (Since the Mix ratio is 1:1)

2 2
The Standard input for both A and B will be 0.5X 5

Since Cost Variance for A is given to be nil, we have, (SP
A
SQ
A
)
(AQ
A
AP
A
) = 0
i.e. 24 (0.5 X 5) (X 70) 30 = 0

or X = 110 Kgs

Therefore Actual Input for A = 110 70 = 40 Kgs

Al ternati ve 2

Let the standard input of A = X kg. Therefore, the total standard input for A + B= 2X Actual input = (2X
+ 10) Kgs. Actual input for A = (2X +10 70)= (2X 60)Kgs Forming the equation for nil cost variance
of A.
Rs. 24 X Rs. 30 (2X 60) = 0

Or X = 50 Kgs. Using this quantity in the Cost Variance of B, the actual price per kg. of
B (AP
B
) will be ,
50 30 70 AP
B
= 1,300

Or AP
B
= Rs. 40.

Al ternati ve 3

Let the actual input of A =X
Then the total actual input = (X + 70). Therefore, RSQ of A and B each = 0.5X + 35
and Standard Input of A and B each = 0.5X +30.

Forming the equation for nil cost variance of A, we have,

24 (0.5X + 30) 30 X = 0

Or X = 40 Kgs.
Standard Input will be 50 Kgs. Using this, quantity in the Cost Variance of B, the actual price per kg.
of B (AP
B
) will be,
50 30 70 AP
B
= 1,300

Or AP
B
= Rs. 40.

Substituting various values for quantity and price, we get the following table.


(1) (2) (3) (4)
108

Std. Price
SQ
Std. Price RSQ Std. Price
Actual Qty.
Actual Price
Actual Qty.
A 24 50 =
1200
24 55 = 1320 24 40 = 960 30 40 = 1200
B 30 50 =
1500
30 55 = 1650 30 70 = 2100 40 70 = 2800

2700 2970 3060 4000



(1) (2) (2) (3) (1) (3) (3) (4) (1) (4)

Yld variance Mix variance Usage variance Price variance Cost variance
A 1200 1320 =
120(A)
1320 960 =
360(F)
1200 960 =
240(F)
960 1200 =
240(A)
1200 1200 =
0
B 1500 1650 =
150(A)
1650 2100 =
450(A)
1500 2100 =
600(A)
2100 2800 =
700(A)
1500 2800 =
1300(A)

270A) 90A) 360A) 940A) 1300A)
Actual Output = 90 Kgs.

(Actual output and standard o utput are always equal numerically in any material variance analysis)
Standard output = Standard input Standard loss or 100 10 = 90 Kgs.

Ans. 43:

Working Notes
a) Computation of Standard Price per kg of Material
Let x be the standard price per kg
Direct material price variance = Rs. 15,750 (A) (given)
A.Q. (S.P. A.P.) = DMVP
63,000 kgs (x 3.25) = -15,750
63,000 x 2,04,750 = -15,750
63,000x = 1,89,000
x = 1,89,000 / 63,000 = 3
Standard price per kg of material is Rs. 3

b) Computation of Standard Quantity of material for actual output
Let x be the standard quantity
Direct material usage variance = Rs. 27,000 (A) (given)
S.P. (S.Q. A.Q.) = DMUV
3(x 63,000) = -27,000
3x 1,89,000 = -27,000
3x = 1,62,000
x = 1,62,000/ 3 = 54,000

Standard Quantity for actual output is 54,000 kgs.

c) Computation of Standard Labours hours per unit
Let x be the Standard labour hours per unit
D.L. rate variance + D.L. efficiency variance =D.L. Cost Variance
Rs. 6,840 (A) + Rs. 10,800 (F) = Rs. 3,960 (F)
Direct labour cost variance = Rs. 3,960 (F) (given)
Standard cost of Standard hours Actual cost of actual hours = Rs. 3,960 (F)
(x X Rs.6) (Rs. 2,12,040 = Rs. 3,960 (F)
6x = Rs. 2,16,000
x = 2,16,000 / 6 = 36,000

109
Standard hours for actual output is 36,000 hours
Standard hours per unit = 36,000 hours/ 18,000 units = 2 hrs.

d) Computation of Actual Hours per unit
Let x be actual hours
Direct labour efficiency variance = Rs. 10,800 (F) (given)
(Standard hours Actual hours) Std. rate = DLEV
[(18,000 units X 2) x] Rs. 6 = Rs. 10,800 (F)
2,16,000 6x = 10,800
6x = 2,16,000 10,800
x = 34,200

Actual labour hours are 34,200 for actual output
Actual labour hours per unit = 34,200 hrs / 18,000 units = 1.9 hrs.

e) Computation of Standard variable overhead per hour
Budgeted fixed overheads Actual fixed overheads = Fixed overhead expense variance
Let Budgeted fixed overheads be x
FOEV = Rs. 25,000 (A) (given)
x Rs. 3,25,000 = Rs. 25,000 (A)
x = 3,25,000 25,000 = 3,00,000

Budgeted fixed overheads is Rs. 3,00,000
Standard fixed overhead rate per unit = Rs. 3,00,000/ 20,000 units = Rs. 15 per unit Standard
fixed overhead rate per hour = Rs. 15 / 2 hours = Rs. 7.50 per hour

f) Computation of Budgeted selling price per unit
Let x be the budgeted selling price per unit
Sales price variance = Rs. 45,000 (F) (given)
Actual quantity (Actual selling price Budgeted selling price) = Sales price variance
18,000 units (Rs. 67.50 y) = Rs. 45,000 (F)
12,15,000 18,000y = 45,000
18,000y = 12,15,000 45,000
y = 11,70,000 / 18,000 = 65

Budgeted selling price is Rs. 65 per unit.
Budgeted Sales Actual Sales
Quantity
(Units)
Price
(Rs. p.u.)
Amount
Rs.
Quantity
(Units)
Price
(Rs. p.u.)
Amount
Rs.
20,000 65 13,00,000 18,000 67.50 12,15,000

Statement showing Standard Cost per unit and Budgeted Profit for 20,000 units.
Particulars Per Unit For 20,000 Units
Sales (a) 65 13,00,000
Costs:
Direct Material 9 1,80,000
Direct Labour 12 2,40,000
Variable Overhead 16 3,20,000
Fixed Overhead 15 3,00,000
Total Cost (b) 52 10,40,000
Standard Gross Margin 13 2,60,000

(ii) Computation of sales gross margin volume and fixed overheads volume variances
Sales Gross Margin Volume Variance
= Standard Margin per unit (Actual Quantity Budgeted Quantity)
= Rs. 13 (18,000 units 20,000 units) = Rs. 26,000 (A)
Fixed Overhead Volume Variance
= Standard fixed overhead rate per unit (Actual output Budgeted output)
110
= Rs. 15 (18,000 units 20,000 units) = Rs. 30,000 (A)

Operating Statement Reconciling the Budgeted Profit with Actual Profit (Rs.)
Budgeted Profit (20,000 units X Rs. 13 p.u.) 2,60,000
Sales Margin Volume Variance 26,000 (A)
Standard Profit 2,34,000
Sales Price Variance 45,000 (F)
2,79,000
Cost Variances: Favourable Adverse
Direct Material Price Variance - 15,750
Direct Material Usage Variance - 27,000
Direct Labour Rate Variance - 6,840
Direct Labour Efficiency Variance 10,800 -
Variable Overheads Expense Variance - 3,420
Variable Overheads Efficiency Variance 14,400 -
Fixed Overheads Expense Variance - 25,000
Fixed Overheads Volume Variance - 30,000
25,200 1,08,010 82,810 (A)
Actual Profit 1,96,190

Ans: 44:
Reconciliation Statement of Actual profit and Standard profit. (Rs)


Budgeted Profit (10,000 @ Rs.32)
Less: Sales volume variance (Adverse) (Rs.32 (8,000-10,000)
Standard profit (8,000 units @ Rs.32)
3,20,000
64,000
2,56,000

Cost Variances:
1. Direct Materials
(i) Price variance 16,500(Rs.20-Rs.24)
(ii) Usage Variance Rs.20 (16,000-16,500)
2. Direct labour
(i) Labour rate variance 1,70,000(Rs.2.00-Rs.2.04)
(ii) Labour efficiency variance Rs.2 (1,60,000-1,66,000)
(iii) Idle time variance (Rs.2.00 x 4,0000
3. Variable Overheads (Rs.8 x 8,000) Rs.60,000
4. Fixed Overheads
(i) Expenses variance (Rs.20 x 10,000) Rs.1,84,000
(ii) Efficiency variance Rs.20 (8,000 8,300)
(iii) Capacity variance Rs.20 (8,300 10,000)
Total
Less: Net Adverse variance
Actual profit for the period


Adverse
66,000
10,000

6,800
12,000
8,000
4,000

16,000
6,000
34,000
1,42,800



Favourable












20,000














1,22,800
1,33,200

Ans. 45:

(b)
Working notes:
Ravi Richard Rahim Roop Singh
(i) Standard sales units : 1,875 2,250 2,875 1,500
Sales quota Rs. 400

(ii) Standard selling expenses per unit (Rs.) 120 110 100 150

(Std. selling expenses/Std. sales units)
(iii) Actual sales units : 2,000 2,500 2,625 1,300
Actual salesRs. 400

(iv) Actual selling costs Rs. Rs. Rs. Rs.
Daily allowance 16,000 14,000 18,000 20,000
111
Conveyance allowances 30,000 27,000 27,000 45,000
Salaries 80,000 80,000 80,000 80,000
Free samples 9,000 7,500 5,375 8,000
Postage & stationery 8,000 9,000 10,000 6,000
Other expenses 9,000 5,000 4,000 10,000
Commission on sales 48,000 50,000 52,500 26,000
Corporate sales office expenses 60,000 75,000 1,05,000 52,000

Total actual selling cost 2,60,000

2,67,500

3,01,875

2,47,000
(v) Standard selling cost 2,40,000 2,75,000 2,62,500 1,95,000
(Actual units sold
Std. selling expenses per unit)
Since all the selling expenses have been related to sales units, only one variance can be calculated
by comparing the standard and actual selling costs as is shown in the schedule below:
Schedule showing the selling cost variances by salesman
Rs. Rs. Rs. Rs. Total (Rs.)
Standard Selling
expenses (Refer to Working Note (v)) 2,40,000 2,75,000 2,62,500 1,95,000 9,72,500
Actual selling
expenses (Refer to Working Note (iv)) 2,60,000 2,67,500 3,01,875 2,47,000 10,76,375
Selling cost variance (20,000) 7,500 (39,375) (52,000) (1,03,875) (A)
(F) (A) (A) (A)
A = Adverse
F = Favourable

Statement showing the computation of standard cost of production of shirts
Ans 46:

Lot no. Units Cost per Total standard
(Dozen) Dozen cost( Rs.)
45(UK) 1,700 531.00 9,02,700
46(US) 1,200 477.60 5,73,120
47(CAN) 1,000 531.00
Total
5,31,000

20,06,820
Cost per Dozen of 46 (US) lot. (Rs.)


Material cost 100% 264.00
Conversion cost 80%(80%of Rs.267)
Total
213.60

477.60
Statement of material used and its variance

Lot no. Output Std. Qty Total Total Variation
Dozen per Dozen Std. qty. Actual Qty.
( Mtrs.) ( Mtrs.) ( Mtrs.)
45(UK) 1,700 24 40,800 40,440 360(F)
46(US) 1,200 24 28,800 28,825 25(A)
47(CAN) 1,000 24
Total
24,000 24,100 100(A)

93,600 93,365 235(F)
Statement of labour hour worked and its variance

Lot no. Output Std.labour Total labour Total actual Variation
Dozen hour per hours labour hours (Hours)
Dozen
112
45(UK) 1,700 3 5,100 5,130 30(A)
46(US) 960 3 2,880 2,980 10(A)
(12000.8)
47(CAN) 1,000 3
Total
3,000 2,890 20(F)

10,980 11,000 20(A)

Calculation of variances

(1) Material price variance
actual quantity (standard rate actual rate)
= (95,000 metres *Rs. 11)-Rs.10,64,000
= Rs.10,45,000-Rs. 10,64,000 =Rs. 19,000(A)
(2) Labour rate variance
actual hour (Std. rate per hour actual rate per hour )
= 11,000 (Rs. 49-Rs.50) = Rs.11,000(A)
(3) Variable overhead efficiency variance
Std. variable overhead rate per hour (Std.hour actual hour)
= Rs. 24(10980-11,000) = Rs. 480 (A)
(4) Fixed overhead volume variance
Std. fixed overhead rate per hour (Std.hour for actual outputBudgeted hour)
= Rs. 16(10980-480003/12) = Rs. 16,320(A)

Working Notes :
(1) standard variance overhead rate per hour
= 40*60/100 = Rs.24
(2) standard fixed overhead rate per hour

= Rs. 40*40/100= Rs. 16



Ans: 47.
1. Sales variances
(5) Sales Volume Margin Variance
(Std. Margin on actual Sales Budgeted Margin)
=(Rs.25,000 units x Rs.6) (36,000 units x Rs.6)
=(Rs.1,50,000 Rs.2,16,000) =Rs.66,000 (A)

(6) Sales Price Variance
(Actual Sales at actual price Actual Sales at Std. Price)
=(25,000 Units x Rs.51.50)-(25,000 units x Rs.50)
=(Rs.12,87,500 Rs.12,50,000) =Rs.37,500 (F)

2. Material variances
(1) Material Price Variance
(Std Cost of Material Used- Actual Material Cost
=(96,000 kgs x Rs.2) (96,000 kg. x Rs.2.25)
=(Rs.1,92,000 Rs.2,16,000) =Rs.24,000(A)

(3) Material Usage Variance
Std Material cost of Actual production- Std. Cost of Material used)
=(1,00,000 kgs. x Rs.2) (96,000 Kgs. x Rs.2)
=(Rs.2,00,000 Rs.1,92,000) =Rs.8,000 (F)

3. Labour Variances
(1) Labour Wages Rate Variance
(Actual Labour hrs. at Std. rate- Actual Labour Wages)
=(1,60,000 hrs x Rs.4) (1,60,000 hrs. x Rs.4.10)
=(Rs.6,40,00 Rs.6,56,000) =Rs.16,000 (A)
113

(2) Labour Efficiency Variance
Std. Labour Wages for actual production Actual Labour hours worked at Std. rate)
=(1,50,000 hrs. x Rs.4) (1,54,000 hrs. xRs.4)
=(Rs.6,00,000 Rs.6,16,000) =Rs.16,000 (A)

(3) Idle Time variance
(6,000 hrs. x Rs.4 variance) =Rs.24,000 (A)

4. Variable Overhead Variances
(1) Total Variable overhead Variance
(Allowed Expenditure for actual hours-Actual variable overheads)
=Rs.(1,84,000 Rs.1,82,000) =Rs.2,800 (F)
(2) Variable overhead Efficiency Variance
( Allowed Expenditure for Std. hours- Allowed Expenditure for actual hours)
=(Rs.1,50,000 hrs. x Rs.1.20)- 1,54,000 hrs. x Rs.1.20)
=(Rs.1,80,000 Rs.1,84,800) =Rs.4,800 (A)
5. Fixed Overhead Variances
(1) Fixed Overhead Expenditure variance
(Budgeted Expenditure Actual Expenditure)
=(Rs.1,44,000 Rs.1,50,000) =Rs.6,000 (A)
(2) Fixed Overheads Efficiency variance
(Std. hours of production x Std. fixed overhead recovery rate per hour)-(Actual hours worked x
Fixed overhead recovery rate per hour)
=(Rs.1,50,000 hrs. x Re.0.80)-(1,54,000 hrs. x Re.0.80)
=(Rs.1,20,000 Rs.1,23,200) =Rs.3,200(A)
(3) Fixed Overhead Capacity variance
(Actual hours worked x Fixed overhead recovery rate per hour)-(Std. Fixed overhead recovery r
rate per hour x Budgeted capacity hours)
=(1,54,000 hrs. x 0.80)-(Re.0.80 x 1,80,000 hrs.)
=(Rs.1,23,200 Rs.1,44,000) =Rs.20,800(A)
A. Std. Variable Overhead Rate per hour.
=
Total Std. hours
Std. Variable Overheads

=(30,000 units x Rs.12)-Rs.1,44,000
1,80,000 units
=Rs.1.20

B. Std Fixed Overheads rate per hour
=
Budgeted hours
Budgeted Overheads
=Rs.1,44,000 / 1,80,000 hrs. =Rs.0.80

Statement of actual profit / loss for the second quarter of the year (Rs.)
Direct Material (96,000 kgs.@Rs.2.25)
Direct Wages (1,60,000 hrs. Rs.4.10)
Overhead
Total Cost
Sales Revenue (25,000 units @ 51.50)
Actual Profit

2,16,000
6,56,000
3,32,000
12,04,000
12,87,500
83,500

Operating Statement reconciling the budgeted profit with actual profit (Rs.)
Particulars Reference to
working note
Variance
Favourable Adverse
Actual
Budgeted profit (36,000 units x Rs.6)
1. Sales Volume Margin Variance
Price Variance
Profit before adjustment of Cost Variances
-
(1)
(2)

-
-
37,500

-
66,000
-

2,16,000
-
-
1,87,500
114
II Material - Price
- Usage
III. Labour - Rate
-Efficiency
-Idle time
IV. V. Overheads -Expenditure
-Efficiency
V. F. Overheads -Expenditure
- Efficiency
-Capacity
Actual Profit

(1)
(2)
(1)
(2)
(3)
(1)
(2)
(1)
(2)
(3)
-
8,000
-
-
-
2,800
-
-
-
-
10,800

24,000
-
16,000
16,000
24,000
-
4,800
6,000
3,200
20,800
1,14,800









1,04,000
83,500


Ans. 48:


Expenses
Overhead Expenses Schedule

Budget: 120 Std. Hours Actual: 156 Hours
Rate per hour
Rs.
Expenses
R s .
Rate per hour
Rs.
Expenses
R s .
Indirect material
Indirect labour
Maintenance
Power
Sundries
Total variable
overheads
Fixed overheads
Total overheads
0.40
0.60
0.40
0.30
0.30

48
72
48
36
36

0.50
0.60
0.45
0.32
0.29

78
94
70
50
45

2.00

2.00
240

240
2.16 337

250
480 587
Actual output = 12,160 units.
Hence standard hours produced or std. hours for actual production
=
Computation of variances:
A. Fixed expenses
(a) Charged to production (152 hours Rs. 2 per hours) Rs. 304
(b) Fixed expenses as per budget Rs. 240
(c) Actual fixed overheads Rs. 250
Volume variance = Fixed overhead recovery rate (Actual volume in std. hrs. Budgeted
volume in standard hrs.)
= Rs.2 (152 120) = Rs.64 (F)
Expenses variance = (Budgeted expenses Actual expenses)
= Rs.240 Rs.250 = Rs. 10 (A)
Total variance = (Fixed overheads absorbed Actual fixed overheads)
= Rs.304 Rs.250 = Rs.54 (F) Or
Volume variance: (a b) Rs.64 (F)
Expenses variance: (b c) Rs. 10 (A)
Total variance : (a c) Rs.54 (F)

B. Variable expenses
(a) Charged to production: (152 hours Rs.2) Rs.304
115
(b) Actual expenses Rs.337
Variable overhead cost variance (a b) Rs.33 (A)

Ans. 49:
(1) Statement showing standard and actual costs of material for 1,000 units of output and standard cost of actual
input
Basic Data:
Standard Cost Actual cost Standard cost of actual input =
(Actual quantity Standard
price)
Ma Qty. Price Amount Qty. Price Amount Actual
Qty.
Standard
Price/kg
Amount
Kg. Rs. Rs. Kg. Rs. Rs. Kg. Rs. Rs.
A 12,000 10 1,20,000 11,000 11 1,21,000 11,000 10 1,10,000
B 5,000 6 30,000 5,200 5.50 28,600 5,200 6 31,200
1,50,000 1,49,600 1,41,200
Standard yield (units) =
Kg. 17,000
units 1,000
16,200 kg. = 952.941764 units approx.
(2) Statement showing standard and actual labour cost of 1,000 units produced and standard cost of actual labour
hrs.
Hours Rate
p.h.
Amount Hours Rate
p.h.
Amount Hours Rate
p.h.
Amount
Rs. Rs. Rs. Rs. Rs. Rs.
5,000 3 15,000 5,500 3.1818 17,500 5,500 3 17,500
(3) Overheads
Fixed overheads (Rs.) Budgeted Actual
Hours 38,500 39,000
Output 5,500 5,500
Standard time p.u. (hrs.) 1,100 1,000
Standard fixed overheads p.u. (Rs.) 5
Standard fixed overhead rate p.h. (Rs.) 35
Computation of material variances (Refer to Basic data 1): 7
Computation of material variances (Refer to Basic data (1):
Material cost variance = Standard cost Actual cost
= Rs.1,50,000 Rs.1,59,500 = Rs.9,500 (Adv.)
Material price variance = Actual quantity (Std. price Actual price)
= 12,000 kg (Rs.10 Rs.11) + 5,000 kg (Rs.6 Rs.5.50)
= Rs.12,000 (Adv.) + Rs.2,500 (Fav.)
= Rs.9,500 (Adv.)
Material usage variance = Standard price (Standard quantity Actual quantity)
116
= Rs.10 (12,000 kg 11,000 kg) + Rs.6(5,000 kg5,200 kg)
= Rs.10,000 (Fav.) + Rs.1,200 (Adv.)
= Rs.8,800 (Fav.)
Material mix variance = Total actual quantity
)
`


kg 16,200
of price Std.
kg per mix Std.
of price Std.

= 16,200 kg
)
`


kg 16,200
0 Rs.1,41,20
kg 17,000
0 Rs.1,50,00

= Rs.1,741.18 (Fav.)
Material yield variance = Std. Rate (Actual yield Std. Yield
= Rs.150 (1,000 units 952.9411764 units)
= Rs.7058.82
Material purchase price variance:
= Actual quantity of material purchased (Std. Price per kg. Actual price per kg)
= 12,000 kg (Rs.10 Rs.11) + 5,000 kg (Rs.6 Rs.5.50)
= Rs.12,000 (Adv.) + Rs.2,500 (Fav.)
= Rs.9,500 (Adv.)
Computation of labour variances (Refer to basic data 2):
Labour cost variance = (Standard cost Actual cost)
= Rs.15,000 Rs.17,500 = Rs.2,500 (Adv.)
Labour rate variance = Actual hrs. (Std. Rate Actual rate)
= 5,500 (Rs.3 Rs.3.1818)
= Rs.1,000 (Adv.)
Labour efficiency variance = Std. rate p.h. (Std. Hours Actual hours)
= Rs.3 (5,000 hrs. 5,500 hrs.)
= Rs.1,500 (Adv.)
Computation of fixed overhead variance:
Total fixed overhead variance:
= Fixed overhead absorbed Actual fixed overhead
= 1,000 units Rs.35 Rs.39,000
= Rs.35,000 Rs.39,000 = Rs.4,000 (Adv.)
Fixed overhead expenditure variance:
= Budgeted fixed overhead Actual fixed overhead
= Rs.38,500 Rs.39,000 = Rs.500 (Adv.)
Fixed overhead volume variance:
= Std. Fixed overhead rate per unit (Actual output Budgeted output)
= Rs.35 (1,000 units 1,000 units) = Rs.3,500 (Adv.)
Efficiency variance:
= Std. fixed overhead rate per unit (Actual output Budgeted output)
117
= Rs.35 (1,000 units 1,000 units) = Rs.3,500 (Adv.)

Ans. 50:
1. Standard quantity and cost of raw material required for actual output
(i) Working Notes:
Actual output of EXE (units)
Standard output per kg. of raw material (units)
Standard quantity of raw material required for actual output (kgs.)
(4,680 units / 12 units)
Standard cost of 390 kgs. of raw material at Rs.60 per kg. (Rs.)
2. Basic data for the computation of labour variances:
Standard labour data for actual
output
Actual data
Std. Time hours Rate
p.h.
Amount Standard
cost of
actual
hours
Actual
cost
hours
Rate
p.h.
Amount
2,340 5 11,700 12,000 240 4.80 1,152
(4,680 units hr.) 320 5.20 1,664
5.00 1,840 9,200
2,340 11,700 12,000 2,400
3. Basic data for the computation of fixed overhead variances:
12,016
Budgeted / Std. data Actual data
Budgeted fixed overhead (Rs.)
(for 1 week)
24,400 Actual fixed overhead (Rs.) 19,800
Budgeted hours 2,400 Actual labour hours 2,400
(60 workers40 hrs. per week) Actual output (units) 4,680
Budgeted output (units) 4,800
Std. rate p.h. (Rs.) 8.50
Std. rate p.u. (Rs.) 4.25
(i) Computation of labour and overhead (variances):
Labour cost variance: (Refer to working Note 2)
= (Std. cost of labour Actual cost of labour)
= Rs.11,700 Rs.12,016 = Rs.316 (Adverse)
Labour rate variance:
= Actual hours (Std. rate Actual rate)
= Rs.12,000 Rs.12,016
= Rs.16 (Adv.)
Labour efficiency variance:
118
= Standard rate per hr. (Std. hours Actual hours paid)
= (Rs.11,700 Rs.12,000) = Rs.300 (Adv.)
= Total fixed overhead cost variance:
= (Fixed overhead absorbed Actual fixed overhead)
= [(4,680 units Rs.4.25) Rs.19,800]
= Rs.19,890 Rs.19,800 = Rs.90 (Fav.)
Fixed overhead volume variance:
= Std. fixed overhead rate per unit [Actual output Budgeted output]
= Rs.4.25 (4,680 units 4,800units)
= Rs.510 (Adv.)
Fixed overhead expenditure variance:
= [Budgeted fixed overhead Actual fixed overhead]
= [Rs.20,400 Rs.19,800] Rs.600 (Fav.)
(i i ) Statement showing total standard cost, standard profit and actual profit for the week.
Sales Rs. Rs.
4,680 units Rs.15 70,200
Less: Standard cost of :
Direct material 23,400
Direct labour 11,700
Overheads 19,890 54,990
(4,680 Rs.4.25)
(Refer to working notes 1 to 3)
Standard Profit 15,210
Less: Adjustment for variance:
Raw Material:
Price variance : 800 (A)
Usage variance : 600 (A) 1,400 (A)
Labour:
Rate Variance : 16 (A)
Efficiency variance : 300 (A) 316 (A)
Overhead:
Expenditure variance: 600 (F)
Volume variance: 510 (F) 90 (F) 1,626
Actual Profit 13,584

Sales variances (Sales Value Method)
Ans.51:
Budgeted Calculations:
Budgeted Sales Actual Sales

119
Product Qty.
Units
Rate
Rs.
Amount
Rs.

Qty.
Units
Rate
Rs.
Amount
Rs.
Actual
quantity

Budgeted price
Rs.
A 10,000 12 1,20,000 11,000 11.50 1,26,500 1,32,000
B 6,000 15 90,000 5,000 15.10 75,500 75,000
C 8,000 9 72,000 9,000 8.55 76,950 81,000
24,000 2,82,000 25,000 2,78,950 2,88,000
Computation of sales variances :

(1) Sales value variance = Actual sales Budgeted sales
= Rs. 2,78,950 Rs. 2,82,000
= Rs. 3,050 (A)
(2) Sales price variance = Actual quantity (Actual price Budgeted price)
= Rs. 2,78,950 Rs. 2,88,000

= Rs. 9,050 (A)
(3) Sales volume variance = Budgeted price (Actual Qty. Budgeted Qty.)

= Rs. 2,88,000 Rs. 2,82,000

= Rs. 6,000 (F)
(4) Sales mix variance = Total actual qty. (Budgeted price of actual mix

Budgeted price of budgeted mix)
=
= 25,000 units (Rs. 11.52 Rs. 11.75)

Rs. 5,750 (A)
(5) Sales quantity variance = Budgeted price of budgeted mix (Total actual
quantity Total budgeted qty.)





Check
=

=
Rs. 11.75 (25,000 24,000)
Rs. 11,750 (F)
Sales value variance = Sales price variance + Sales volume variance
Rs. 3,050 (A) = Rs. 9,050 (A) + Rs. 6,000 (F)
Sales volume variance = Sales mix variance + Sales quantity variance
Rs. 6,000 (F) = Rs. 5,750 (A) + Rs. 11,750 (F)
Alternative solution (sales margin method)
Basic calculations :
Budgeted margin Actual margin

Actual quantity
Budgeted margin

Product Qty. Rate Amount Qty. Rate Amount

Units Rs. Rs. Units Rs. Rs. Rs.
A 10,000 5 50,000

11,000 4.50 49,500 55,000
B 6,000 6 36,000

5,000 6.10 30,500 30,000
120
C 8,000 3 24,000

9,000 2.55 22,950 27,000
24,000 1,10,000 25,000 1,02,950 1,12,000
Computation of variances:
Sales margin variance = Actual margin Budgeted margin
= Rs. 1,02,950 Rs. 1,10,000
= Rs. 7,050 (A)
Sales price margin variance = Actual quantity (Actual margin Budgeted margin)
= Rs. 1,02,950 Rs. 1,12,000 = Rs. 9,050 (A)
Sales margin mix variance = Total actual quantity (Budgeted margin of actual mix Budgeted
margin of budgeted mix

Material Variances:
Basic Calculations
Standard and actual costs of material for actual output i.e. 11,000 units of A, 5,000 units of
B and 9,000 units of C and standard cost of actual input material.

Material Standard cost Actual cost Actual quantity
standard price


Q t y
Units
Rate
Rs.
Amount
Rs.
Qty.
Units

Rs.
Rate Amount
Rs.
X 51,000* 2 1,02,000 54,000 1,09,620

1,08,000
Y 74,000** 1 74,000 72,000 73,000 72,000
1,25,000 1,76,000 1,26,000 1,82,620 1,80,000
* 11,000 2 + 5,000 4 + 9,000 1 = 51,000
**11,000 3 + 5,000 1 + 9,000 4 = 74,000.

Computation of variances :
Material cost variance = Standard cost Actual cost
= Rs. 1,76,000 1,82,620 = Rs. 6,620 (A)
Material price variance = Actual quantity (Standard price Actual price)
= Rs. 1,80,000 Rs. 1,82,620 = Rs. 2,620 (A)
Material mix variance = Total quantity (Standard price of standard mix Standard price of
actual mix
121

Check:
Material cost variance = Material price variance + Material mix variance
+ Material yield variance

Rs. 6,620(A) = Rs. 2,620(A) + Rs. 2,592(A) + Rs. 1,408(A)

(i) Reconciliation statement showing which factor has contributed change in profit
Ans. 52
(Rs. in lacs)

Favourable Adverse
Increase in contribution
due to increase in volume

(Rs.280 lacs Rs.240 lacs) 40
(Refer to working note 3)

Sales price variance 140
(Refer to working note 3)

Material usage variance 52
(Refer to working note 4)

Material price variance 0
(Refer to working note 4)

Direct labour rate variance 28
(Refer to working note 4)

Direct labour efficiency variance

36


(Refer to working note 4)

Fixed overhead expenditure variance
(Refer to working note 3)



140
268 168
Total change in profit 100

=
160 lakhs
= Rs. 800 lakhs
Rs. 240 lakhs
100
Rs. 1200 lakhs
| |

|
\ .

Break-even sales (Year 2)
122
(Refer to working note 3) =
300 lakhs
= Rs. 962.50 lakhs
Rs. 480 lakhs
100
Rs. 1540 lakhs
| |

|
\ .

(iii) Percentage increase in selling price needed over the sales value of year 2 to
earn a margin of safety of 45% in year 2
P/V ratio = (Rs. 480 lacs/Rs. 1,540 lacs) 100 = 31.169%

If Margin of safety to be earned is 45% then Break-even point should be 55%
Revised contribution = 1,540 lacs 35.4193% = 545.45 lacs
Present contribution = Rs. 480 lacs
Increase in selling price required = Rs. 65.45 lacs (Rs. 545.45 lacs Rs. 480 lacs)

Working notes:
1 . Budgeted sales in year 2
If actual sales in year 2 is Rs. 110 then budgeted sales is Rs. 100.

3 . Statement of figures extracted from working results of a company
(Figure in lacs of Rs.)

Year 1 Year 2 Year 2 Total
Actual (Budgeted) Actual Variance
(a) (b) (c) d = (c) (b)
Sales : (A) 1,200 1,400 1,540 140 (Fav.)
123
(Refer to working note 1)

Variable costs :
Direct material 600 700 648 52 (Fav.)
(Refer to working note 2)

Direct wages and
variable overhead 360 420 412 8 (Fav.)
(Refer to working note 2)

Total variable costs : (B) 960 1,120 1,060 60(Fav.)
Contribution (C) = {(A) (B)} 240

280

480

200 (Fav.)
Less : Fixed cost 160

160

300

140 (Adv.)
Profit 80

120

180

60(Fav)
Total variable costs : (B) 960 1,120 1,060 60(Fav.)
Contribution (C) = {(A) (B)} 240

280

480

200 (Fav.)
Less : Fixed cost 160

160

300

140 (Adv.)
Total variable costs : (B) 960 1,120 1,060 60(Fav.)
Contribution (C) = {(A) (B)} 240

280

480

200 (Fav.)
Less : Fixed cost 160

160

300

140 (Adv.)
Profit 80

120

180

60(Fav)

(4) (i) Data for Material variances :

Standard data for actual output Actual data

Quantity Rate per Amount Quantity Rate per Amount
of material m/t

of material m/t

m/t

m/t


Rs. Rs.

Rs. Rs.
5,83,333 120 700 lacs 5,40,000 120 648 lacs
Material price variance = (Standard rate Actual rate ) Actual quantity = Nil

Material usage variance = (Standard quantity - Actual quantity) Standard rate per m/t

= (5,83,333 5,40,000) Rs.120 = Rs. 52 lacs (Fav.)

(ii) Data for labour variances overhead variances

Standard data for actual output Actual data

Labour Rate per Amount Labour Rate per Amount
hours hour

hours hour


Rs. Rs.

Rs. Rs.
87,50,000 4.80 4.20 lacs 80,00,000 5.15 412 lacs
Labour rate variance = (Standard rate Actual rate) Actual labour hours

= (Rs.4.80 Rs.5.15) 80,00,000 = Rs. 28 lacs (Adv.)

Labour and variable overhead efficiency variance :

= {Standard labour hours Actual labour hours} Standard rate per hour

= {87,50,000 80,00,000} Rs. 4.80 = Rs. 36 lacs (Adv.)
124


Ans. 53:
Equivalent Production in Units
Basic Calculations
Particulars Direct Material Labour & Overhead
Units completed 100 % 6,000 100 % 6,000
Work-in-progress 100 % 600 50 % 300
Total Equivalent Units 6,600 6,300

(a) Direct Material Variances

Material
Standard output 6,600 units
Actual
output 6,600
units

Qty. Rate (Rs.) Amount (Rs.) Qty. (kg) Rate (Rs.) Amount (Rs.)
A 13,200 3 39,600 14,850 2.90* 43,065
B 6,600 4 26,400 7,260 4.098* 29,750
19,800 66,000 22,110 72,815

*(Actual Cost/ Actual Quantity)

DMCV = Standard Cost for actual output Actual Cost
= 66,000 72, 815 = Rs. 6,815 (A)
DMPV = Actual Qty. (Std, Rate Actual Rate)
A = 14,850 (3 2.90) = 1,485 (F)
B = 7,260 (4 - 4.098) = 710 (A)
775
DMUV = Std. Rate (Std. Qty. for actual output Actual Qty.)
(F)
A = 3 (13,200 14,850) = 4,950 (A)
B = 4 (6,600 7,260) = 2,640 (A)
7,590
DMMV = Std. Rate (Revised Std. Qty. Actual Qty.)
(A)
A = 3
= 3 (14,740 14,850) = 330 (A)
B = 4
= 4 (7,370 7,260) = 440

(F)
110

(F)
DMYV = Std. Cost per Unit (Std. output for actual mix Actual output)
= 66,000
= 10 (7,370 -6,600) = Rs. 7,700 (A)

(b) Direct Labour Variances

DLCV = Std. Cost for Actual Output Actual Cost
= (6,300 X 20) 1,27,500 = Rs. 1,500 (A)

DLRV = Actual Time (Std. Rate Actual Rate)
= 32,000 [ = Rs. 500 (F)

ITV = Std. Rate X Idle Hours
= 4 X 200 = Rs. 800 (A)

DLEV = Std. Rate (Std. Time for actual production Actual time worked)
= 4 [(6,300 X5) 31,800]
= 4 (31,500 31,800) = Rs. 1,200 (A)

(c) Variable Overhead Variances

VOC = Recovered Overheads Actual Overheads
125
= 6,300 X 5 30,000
= 31,500 30,000 = Rs. 1,500 (A)

VOEXPV = Std. Variable Overheads Actual Variable Overheads.
= (31,800 X 1) 30,000
= 31,800 30,000 = Rs. 1,800 (F)
VOEEFV = Recovered Overheads Standard Overheads
= 1 X (31,500 -31,800) = Rs. 300 (A)

(d) Fixed Overhead Variances

FOCV = Recovered Fixed Overheads Actual Fixed Overheads
= (6,300 X 10) 80,600
= 63,000 80,600 = Rs. 17,600 (A)

FOEXPV = Budgeted Fixed Overheads Actual Fixed Overheads
= (8,000 X 10) 80,600 = Rs. 600 (A)

FOVV = Recovered Fixed Overhead Budgeted Fixed Overhead
= 63,000 80,000 = Rs. 17,000 (A)


Fixed Overhead Volume Variances may be segregated into the following:

FOEFFV = Std. Rate (Std. time for actual production Actual time booked)
= 2 (31,500 31,800) = Rs. 600 (A)

FOITV = Std. Rate per hour X Idle hours
= 2 X 200 = Rs. 400 (A)
FOCAPV = Std. Rate per hour (Actual time Budgeted time)
= 2 (32,000 40,000) = Rs. 16,000 (A)


(e) Sales Variances

SPV = Actual Qty. (Std. Price Actual Price)
= 6,000 = Rs. 5,000 (F)

Sales Volume Variance (Contribution loss) :
= Std. Rate of profit (Budgeted Qty. Actual Qty.)
= 5 (8,000 6,000) = Rs. 10,000 (A)


Operating Statement showing the Reconciliation between Budgeted and Actual Profit for the Month
(Rs.)

Budgeted Profit (8,000 X Rs. 5) Rs. Price 40,000
Sales Variances 5,000 (F)
Volume 10,000 (A)
Total 5,000 (A) 5,000 (A)

Cost Variances:
Direct Materials
Price 775 (F)
Yield 7,700 (A)
Mix 110 (F)
Direct Wages
Rate 500 (F)
Efficiency 1,200 (A)
Idle Time 800 (A)
126
Variable Overheads
Expense 1,800 (F)
Efficiency 300 (A)
Fixed Overheads
Expense 600 (A)
Efficiency 600 (A)
Idle Time 400 (A)
Capacity 16,000 (A)
Total Cost Variances 24,415 (A) 24,415 (A)
Actual Profit 10,585

Ans:54:
(a) Material Price variance
Computation of Variances
Material
(1)
Qty. Purchase
Kg. (2)
Std. Price Rs.
(3)
Actual Price
Rs.(4)
Std. cost Rs.
(2x3)=5
Actual Cost
Rs. (2x4)=(6)
Price Variance
Rs. (5-6)=(7)
A
B
9,000
5,000
10.00
3.00
10.25
2.75
90,000
15,000
1,05,000
92,250
13,750
1,06,000
2,250 (A)
1,250 (F)
1,000 (A)

(b) Material Usage Variance
Material
(1)

Std. Qty. for
actual output
(2)
Actual Qty.
(3)
Std. Price
(4)
Std. Cost of
Std. Qty.
(2x4)=5
Std. Cost of
Actual
(4x5)=6
Usage Variance
(5-6)=(7)

A
B
8,000
4,000
7,800
4,300
10
3
80,000
12,000
92,000
78,000
12,900
90,900
2,000 (F)
900 (A)
1,100 (F)

(C ) Labour Rate Variance
Actual Hours
(1)
Std. Rate
(2)
Rs.
Actual Rate
(3)
Rs.

Std. Wage
(4)=(1x2)
Rs.
Actual Wages
(5)=(1x3)
Rs.
Rate Variance
(6)=(4-5)
Rs.
4200 3 2,875 12,600 12,075 525 (F)

(d) Labour Efficiency Variance
Std. Hours
for actual
output
(1)
Actual Hours
(2)
Std. Rate
(3)
Rs.
Std. Cost of
Std. Hours
(4)=(1x3)
Rs.
Std. Cost. Of
Actual Hours
(5)=(2x3)
Rs.
Efficiency
Variance
(6)=(5-6)
Rs.
4,000 4,200 3 12,000 12,600 600 (A)

Overhead Variances
Basic calculations

(a) Budgeted overheads for November = 10,800 X 25
12
=Rs.22,500
(b) Std. hours produced for November = 800 units X 5 hrs per unit=4,000
(c ) Fixed production overheads per hour = 25/5=5
(d) Recovered overhead = 4,000 X 5 =Rs.20,000
(e) Actual overheads = Rs.23,500
(f) Standard overheads = 4,200 X 5 =Rs.21,000

Variances
Overhead Cost variance
=Recovered Overheads- Actual Overheads
=20,000-23,5000 =Rs.3,500 (A)
Overhead Expenditure Variance
=Budgeted Overheads-Actual overheads
=22,500-23,500 =Rs.1,000 (A)
127
Overheads Volume variance
=Recovered Overheads-Budgeted Overheads
=20,000-22,500 =Rs.2,500 (A)
Overhead Volume Variance may be segregated into:
(a) Overhead Capacity Variance
=( Std. Overhead rate per hour) X (Actual hours-Budgeted hours)
= Standard Overheads-Budgeted Overheads
=21,000-22,500 =Rs.1,500 (A)
(b) Overhead Revised Capacity variance
= ( Std. rate per hour ) X (Std. hrs. produced Actual hours)

Or

= Recovered overheads- Std. overheads
=20,000-21,000 =Rs.1,000 (A)

(ii) Operating Statement (Rs.)
(a) Sales (800 X Rs.200) 1,60,000
Less: std. Cost of Sales (800 X Rs.155) 1,24,000
Standard profit
(b) Variances Favourable Adverse
Materials
Price - 1,000
Usage 1,100 -
Direct Labour
Rate 525 -
Efficiency - 600
Fixed Overheads
Expenditure - 1,000
Capacity 1,500 (A)
Efficiency 1,000 (A) - 2,500

( c) Actual Profit
1,625 5,100


36,000











3,475 (A)
32,525


(iii) In the solution given the price variance has been calculated at the point of purchase. In case it is calculated at the
point of issue the variance will be as follows: (Rs.)
A 7,800 X (10-10.25)
B 4,300 X ( 3-2.75)

Present variance
Hence difference
Actual profit as in (ii) above
Price variance difference
Actual profit as per question
1,950 (A)
1,075 (F)
875 (A)
1,000 (A)
125
32,525
125
32,650

Statement showing the computation of standard cost per unit (Rs.)
Ans: 55:
Particulars Actual 960 units Variance (-)
Adv. (+) Fav.
Standard
960 units
Standard
cost per unit
Direct Material
Direct Wages
Variable overhead
Fixed overhead
Total Cost
Profit
792
1,192
1,940
1,040
4,964
976
5,940
(-)24
(-) 40
(-) 20
(-) 40
(-)124
(+)56
(+)180
768
1,152
1,920
1,000
4,840
920
5,760
0.80
1.20
2.00
1.04
5.04
0.96
6.00
Balancing figure
128
Original Budget and Flexible budget for sales achieved (Rs.)
Particulars Standard Cost
(per unit)
Original budget
(1,000 units)
Flexible budget
(960 units)
Direct Material
Direct Wages
Variable overhead
Fixed overhead
Cost of Sales
Profit
Sales
0.80
1.20
2.00
1.04
5.04
0.96
6.00
800
1,200
2,000
1,040
5,040
960
6,000
768
1,152
1,920
1,040
4,880
880
5,760

(i) Flexible budget for May 2004
Ans: 56:
Units Original Budget
20,000
Flexible Budget
for May 2004
18,000
Actuals may
2004
18,000
Variance
1 2 3 4 5
Sales Variable costs
Direct Materials
Direct Labour
Factory Overheads
Selling overheads
Total
Contribution (A)
Fixed Cost
Factory overheads
Selling overheads
Total (B)
Profit (A-B)
Volume variance
Net Loss
24,00,000
6,00,000
8,00,000
2,00,000
3,00,000
19,00,000
5,00,000

1,00,000
2,00,000
3,00,000
2,00,000
2,00,000
-1,50,000
21,60,000
5,40,000
7,20,000
1,80,000
2,70,000
17,10,000
4,50,000

1,00,000
2,00,000
3,00,000
1,50,000
-

22,00,000
5,20,000
7,56,000
1,84,000
2,88,000
17,48,000
4,52,000

1,16,000
1,84,000
3,00,000
1,52,000
40,000 F
20,000 F
36,000 A
4,000 A
18,000 A
38,000 A
2,000 F

16,000 A
16,000 F
-
2,000 F
50,000 A
(48,000)


(ii) Variance Analysis

(1) Sales Std. Price =Rs.24 lakhs /20,000 =Rs.120
Std. profit =Rs.2 lakhs / 20,000 =Rs.10
Actual quantity =18,000 and standard price =Rs.120
Turnover on Std. Price =18,000 X 120 =Rs.21,60,000
Actual turnover is given at Rs.22 lakhs.
: Price Variance =40,000 (F)
Std. Qty X Std. Profit =20,000 X 10 =Rs.2 lakhs
Actual Qty .X Std. Profit =18000 X 10 =Rs.180 lakhs
Quantity Variance =Rs.20,000 A
(2) Direct Materials
Std. Cost =Rs.6,00,000/20,000 =Rs.30
Actual Qty.=18,000 AQ X SC =18,000 X 30 =Rs.5,40,000
Total Actual Cost =Rs.5,20,000
Material Price Variance =Rs.20,000 (F)
(3) Direct Wages
Std. Time per unit =1,00,000/20,000 =5 hours
Std. hourly rate =8,00,000/1,00,000 =Rs.8/hr.
Std. Hours produced =18,000 units X 5 hrs. =90,000 hrs.
Std. Hours=90,000 Actual Hours=95,000 Std. Rate Rs.8
(a) Std. Hrs. X Std. rate =90,000 X 8. =Rs.7,20,000
(b) Actual Hrs. X Actual Rate =Rs.7,56,000
Actual Hrs. X Std. Rate =95,000 X 8 =Rs.7,60,000
Efficiency variance =(a)-( c)=Rs.40,000 (A)
129
Rate Variance =( c) (b)=Rs.4,000 (F)
(4) Factory Variable overheads:
Std. Rate =Rs.2,00,000/1,00,000 =Rs.2/hr.
(a) Charged to production =90,000 X2 =Rs.1,80,000
(b) Std. cost of actual hours =95,000 X 2 =Rs.1,90,000
(c ) Actual overheads =Rs.1,84,000
(a) (b) =Rs.10,000 (A) Being efficiency variance
(b) (c ) =Rs.6,000 (F) Being expense variance
(5) Selling variable overheads:
Std. Rate =Rs.3,00,000/20,000 =Rs.15 / unit
(a) Std. cost of output =18,000 X 15 =Rs.2,70,00
(b) Actual overheads =2,88,000
Adverse Variance =18,000
(6) Factory overheads- Fixed:
Std. Rate = Rs.1,00,000/1,00,000 =Re.1/hr.
(a) Std. cost of output of 90,000 =Rs.90,000
(b) Std. cost of actual hours. (95,000) =Rs.95,000
(c ) Budgeted =Rs.1,00,000
(d) actual
Efficiency variance : (a) ( b) =Rs.5,000 (A)
Capacity variance : (b) ( c) =Rs.5,000 (A)
Expenses variance : (c )- (d) =Rs.16,000 (A)
(7) Selling overheads : Fixed:
Standard =Rs.2 lakhs / 20,000 =Rs.10 per unit
(a) Std. cost of output =18,000 x 10 =Rs.1,80,000
(b) Budget =Rs.2,00,000
(c )Actual =Rs.1,84,000
Volume variance = (a) (b) =Rs.20,000 (A)
Expense variance = (b)-( c) =Rs.16,000 (F)

Ans: 57:
1. Sales Variances
Working Notes:
(1) Sales Volume Margin Variance
(Actual Sales Volume Budgeted Volume ) x Standard Margin
=(22,000 units 20,000 units) x Re.1 =Rs.2,000 (F)
(2) Sales Margin Price Variance
Actual Sales Volume x (Actual Selling Price Budgeted Selling Price)
=(14,000 units (Rs.5 Rs.5) + ( 8,000 units x (Rs.4.75 Rs.5) =Rs.2,000 (A)

2. Material Variances
(1) Material Price Variance
(Std. Price Actual Price) x Actual Quantity
A : (0.30 0.20) x 16,000 kg. =Rs.1,600 (F)
B : (0.70 0.80) x 10,000 kg. =Rs.1,000 (A)

=Rs.600 (F)
(2) Material Mix Variance
Total Actual Quantity (S.C. of Std. mix per kg. S.C. of actual mix per kg.)

=
.10000 .11800
26000
20000 26000
Rs Rs
kg
kg kg
(

(


=Rs.1,200 (F)
(3) Material Yield Variance
Std. rate per kg. of output (Actual Yield Std. Yield )
= 0.50 ( 24,000 kg. 26,000 kg.) =Rs.1,000 (A)
(3) Labour Variance
(1) Labour Rate Variance
(Std. rate p.h. Actual rate p.h. ) x Actual hours
Skilled Labour : (Rs.3 Rs.2.95 ) x 13,000 hrs. =Rs.650(F)
130
Unskilled Labour : (Rs.2.50 Rs.2.60 ) x 6,300 hrs. =Rs.630(A)
(2) Labour Efficiency Variance
=Rs.20(F)
(Std. hrs. for Actual output Actual hours ) x Std. rate p.h.
Skilled Labour : (Rs.10,800 hrs-12,000 hrs.) x Rs.3 =Rs.3,600 (A)
Unskilled Labour : (6,240 hrs. 6,300 hrs.) x Rs.2.50 =Rs.150 (A)
(3) Idle Time Variance
=Rs.3,750 (A)
(Idle hours x Standard Wage rate p.h)
Skilled Labour : 1,000 hours x Rs.3 =Rs.3,000 (A)
(4) Variable Overhead Variance
(1) Variable Overhead Expenditure Variance
(Variable Overhead recovered on actual output Actual Variable Overhead)
= (24,000 units x Re.0.50) Rs.15,000 =Rs.3,000 (A)
(5) Fixed Overhead Variances
(1) Fixed Overhead Expenditure Variance
(Budgeted Expenditure Actual Expenditure)
= (Rs.20,000 Rs.18,020) =Rs.1,980 (F)
(2) Fixed Overhead Volume Variance
(Budgeted Volume Actual Volume ) x Std. rate per unit
= (20,000 units 24,000 units ) x Re.1 =Rs.4,000 (F)

Statement reconciling Actual Profit and Budgeted Profit
Particulars Reference to
working note
Variance
Favourable Adverse
Actual
Budgeted profit
(as per Budgeted income statement)
1.Sales Variances
Sales Volume Margin Variance
Sales Volume Margin Variance

Profit before adjustment of Cost Variances
II Material - Price
- Mix
- Yield
III. Labour Variance
- Rate
-Efficiency
-Idle time
IV. V. Overheads -Expenditure

V. F. Overheads -Expenditure
-Volume

Actual Profit

-


(1)
(2)


(1)
(2)
(3)

(1)
(2)
(3)
(1)

(1)
(2)




2,000
-


600
1,200
-

20
-
-
-

1,980
4,000
7,800





2,000


-
-
1,000

-
3,750
3,000
3,000

-
-
10,750


20,000





20,000











2,950
17,050

Ans. 58:
Zonal Sales Officers
(1) Statement showing the amount of sales target fixed and the actual amount of contribution earned.

(Rs.000)
A B C D
Commission earned 29.9 23.5 24.5 25.8
Actual sales:
(Commission earned / 5%) 598 470 490 516
Sales price variance 4 (F) 6 (A) 5 (A) 2 (A)
Sales volume variance 6 (A) 26 (F) 15 (F) 8 (F)
131
Sales target / Budgeted sales 600 450 480 510
Standard cost of sales target 500 375 400
Standard margin/ Budgeted margin
425
100 75 80 85
Sales margin mix variance 14 (A) 8 (F) 17 (F) 3 (A)
Sales price variance 4 (F) 6 (A) 5 (A)
Actual margin
2 (A)
90 77 92 80
Note: As there is no information about sales margin quantity variances, therefore for calculating actual
contribution the same has been assumed to be zero.
(2) Statement to evaluate the performance of zonal sales officers
Zonal Sales Officers
S. No. Base factor to
evaluate performance
A B C D
Efficiency towards the
target sales:

1. (a) Whether target
achieved
No Yes Yes Yes
(b) Actual sales to
Target sales ratio
(Actual / target) (%)
99.67
|
.
|

\
|
600
100 598

104.44
|
.
|

\
|
450
100 470

102.98
|
.
|

\
|
480
100 490

101.18
|
.
|

\
|
510
100 516

(c) Ranking IV I II III
2. (a) Contribution
earned (in Rs.000)
90 77 92 80
(b) Ranking II IV I III
3.. (a) Standard margin/
sales target ratio
16.67 16.67 16.67 16.67
(b) Actual margin /
Actual sales ratio (%)
15.05 16.38 18.78 15.50
(c) Ranking IV II I III
Recommendati on:
A review of performance of four officers based on three based factors, shows that the performance of officer C is the
best.

Ans. 69:
Financial Prospective Increase in operating income by charging higher margins on Maharaja.
Kitchen Kings Score card should describe its product differentiation strategy. The key points that
should be included in its balance score card are
Customer Prospective Market share in high-end kitchen range market and customer satisfaction.
Internal business perspectives: Manufacturing quality, order delivery time, on time delivery and new
product feature added.
Learning and Growth prospective: Development time for designing new end product and improvement
in manufacturing process.
Operative Income:

(Amount in 000
Rs.)
2003 2004
Revenue (400001000:
420001100) 40000 46200
132
Direct Material 12000 13530
Conversion cost 10000 11000
Selling and Customer service 7200 7250
Total cost 29200 31780
Operative Income 10800 14420
Change in operating Income 36, 20,000 (F)
A. Growth Component
(a) Revenue effect = Output Price in 2003{Actual units sold in 04 Actual units sold in 03}
= Rs.1, 000 (42,000 units 40,000 units)
= Rs.20, 00,000 (F)
(b) The cost effect = Input price in 2003{Actual units of input to produce 2003 output less Actual units
of input which would have been used to produce year 2004 output on the basis of 2003}
(i) Direct Material = Rs.100 [1, 20,000sqft 1, 20,000sqft

= Rs.6, 00,000 (A)
(ii) Conversion cost and selling and customer service will not change since adequate capacity
exists in 2003 to support 2004 output and customers.
Hence variance
Conversion cost = 200(50000 50000) = 0
S & Customer Service = 25000(300 300) = 0
Increase in operating effect of Growth component is Rs14, 00,000 (F)
B. Price recovery Component:
(i) Revenue effect = Actual output in 2004 [Selling price per unit in 2004 less Selling price per unit in
2003]
= 42,000units (Rs.1, 100 Rs1, 000) = Rs.42, 00,000 (F)
(ii) Cost effect = Unit of input based on 2003 actual that would have been used to produce 2004
output {Input prices per unit in 2003 less Input prices per unit in 2004}
(a) Direct material = 1, 26,000sqft (Rs.100/sqft Rs.110/sqft)
= Rs.12, 60,000 (A)
(b) Conversion Cost = 50,000 units (Rs.200/unit Rs.220/unit)
= Rs.10, 00,000 (A)
(c) S & Custr Service = 300 customers (Rs.24, 000 Rs.25,000)
= Rs.3,00,000 (A)
= Rs.25, 60,000 (A)
Increase in Operating income due to Price Recovery is Rs.16, 40,000 (F) {Rs.42, 00,000 Rs.25, 60,000}
(C) Productivity Component
Productivity component = Input Prices in 04 {Actual units of input which would have been used to
produce year 2004 output on the basis of 2003 actual less Actual Input}
(i) Direct Material: Rs.110/sqft (1, 26,000 units 1, 23,000 units) = Rs.3, 30,000(F)
(ii) Conversion Cost: Rs.200/unit (50,000 units 50,000 units) = 0
(iii) Selling & Customer = Rs.25, 000 (300 customers 290 customers)
= Rs.2,50,000 (F)
= Rs. 5,80,000 (F)
The change in operating income from 2003 to 2004 is analyzed as follows:
(Amount in 000 Rs.)
2003 Growth component Price recovery Cost effect of productivity component 2004
Revenue 40000 2000 (F) 4200 (F) ------------ 46200
Cost 29200 600 (A) 2560 (A) 580 (F) 31780
Operating
Income 10800 1400(F) 1640 (F) 580 (F) 14420


40000 units
42000 units]
133
Ans.1: Statement showing ranking
Key Factor, Throughput Accounting & Budgeting
Products
Particulars P Q R
Selling Price/unit (Rs.) 25.00 30.00 35.00
Variable cost/unit (Rs.)
Direct material 11.00 16.25 21.00
Direct labour 2.50 2.50 2.50
Other variable costs 1.50 2.25 3.50
Contribution per unit (Rs.) 10.00 9.00 8.00
Machine hours/unit 0.67 0.33 0.4167
Contribution/machine hour 15 27 19.2
Ranking III I II

Ans: 2
Working Note
The limiting factor in the company is the No. of labour hours in department II. Hence, contribution
per labour hour of department II has to be found and products ranked on that basis.
A B C
Selling price / unit
Less: Variable cost:
Direct materials
Direct Labour:
Department I
Department II
Department III
Variable overhead

Contribution per unit
Time taken in department II
Contribution per labour hour of
Department II 20/0.5 =
Ranking for allotment of department
II labour hour
100

40

10
6
12
12
20
80

0.5 hr.

40

II
130

50

12
12
15
11
30
100

1 hr.

30

III
175

64

15
12
18
16 125
50


1 hr.

50

I

Solution
(a) Current mix profit and total labour hour in dept. IIs
Product No. of units Contribution
/ unit
Total
contribution
Labour time
in
department
II per unit
Total labour
time in
department
II
A 30,000 Rs.20 Rs.6 lakhs 0.5 hr. 15,000 hr.
B 40,000 Rs.30 12 lakhs 1.0 hr. 40,000 hr.
C 25,000 Rs.50 12.50 lakhs 1.0 hr. 25,000 hr.
Total
FOH
30.50
25.00


80,000 hr.
134
Profit 5.50

The suggested product mix is the optimum one because the first ranked product C is proposed to be
produced & sold to the maximum of 30,000 units. Similarly, the second ranked product A can be
produced and sold up to 50,000 units. The balance hours can be utilized to produce B to the extent of
25,000 units only. This will be optimum mix as indicated below:

Product Ranking No. of Units No. of hours in Dept. II
C
A
B
I
II
III
30,000 (Maximum)
50,000 (Maximum)
25,000 (Balance)
30,000
25,000
25,000 (Balance)
Total 80,000
(b) Statement of increase in profit
Product No. of Units Contribution per unit Amounts (Rs.lakhs)
C
A
B

Less: FOH
Profit
Profit under proposed
plan in question
Increase in profit
30,000
50,000
25,000
50
20
30
Total

15.00
10.00
7.50
32.50
25.00
7.50

5.50
2.00


If the suggestion for optimum product mix is implemented, the increase in profit would be Rs.2.00
lakhs.

Ans: 3
Working Notes
Statement of contribution per machine hour (Limiting factor ) and ranking
Particulars PIE SIGMA
Selling price
Less: Variable cost
Contribution per unit
Contribution per machine hour =
20
11
9
9/1
=Rs.9.00
30
16
14
14/2
Rs.7.00
Ranking I II
Solution
(a) Best combination:
Pie should be produced fully one lakh units. Then , sigma should be produced within the balance
machine hours. This combination will give optimum contribution as follows:
Product Ranking No. of Units No. of
Machine
Hours
CPU Total
contribution(Rs.)
135
Pie
Sigma


Less: Fixed
Profit
I
II

Total
1,00,000
1,50,000
(300000 /2 )
1,00,000
3,00,000
4,00,000
(Balance)
9.00
14.00

9,00,000
21,00,000

30,00,000
(Optimum)
26,00,000
4,00,000

(b) There is market for Sigma for one lakh units (i.e., 2,50,000 1,50,000 units). Two machine
hours are required per unit of production of Sigma. That is 1,00,000 units at 2 hours =
2,00,000 machine hours required. For this purpose, 7 machines are to be taken on rental
basis. Then, the profit will improve as follows:
(Rs.lakhs)
Pie 1 lakh units at Rs.9 9.00
Sigma 2.5 lakh units at Rs.14
Total contribution 44.00
35.00
Less: Fixed cost 26.00
Rent 7 X 1.5 = 10.50
Profit
36.50

7.50
(c) There is no change in number of machines required on rental basis. Total rental charges will
come down and profit will improve further as follows: (Rs.lakhs)
Total contribution (as calculated above) 44.00
Less: Fixed cost 26.00
Rent 7 X 1.25 = 8.75
Profit
34.75

9.25
Ans. 4:
Working Notes Products
Particulars X Y Z
Selling Price/unit (Rs.) 1900 2400 4000
Variable cost/unit (Rs.) 700 1200 2800
Contribution per unit 1200 1200 1200
Machine hours/unit 3 2 1
Contribution/machine hour 400 600 1200
Ranking III II I

(b) Machine hours available will be only 20000 hours
Product Ranking No. of units DLH CPU Total contribution
Z I 1000 1000 1200 1200000
Y II 2000 4000 1200 2400000
X III 5000 (15000/3) 15000 (B.F.) 1200 6000000
Total 20000 Rs. 9600000

Ans. 5: Statement of Ranking
Working Notes Products
136
Particulars X Y Z
Selling Price/unit (Rs.) 30 40 50
Variable cost/unit (Rs.)
Direct material(@Rs. 8 p.kg) 5.6 3.2 12
Direct labour(@Rs. 8 p.h.) 8 16 12
Variable overheads(@Rs. 5.6 p.h.) 5.6 11.2 8.4
Sellling commission (10% of SP) 3 4 5
22.2 34.4 37.4
Contribution/unit 7.8 5.6 12.6

Raw material per unit (kg) 0.7 0.4 1.5
Contribution per kg (Rs.) 11.14 14 8.4
Ranking II I III

Statement of Ranking (if additional 4500kg are made of RM is available)
Products
Particulars X Y Z
Selling Price/unit (Rs.) 30 40 50
Variable cost/unit (Rs.)
Direct material(@Rs. 8 p.kg) 5.6 3.2 12
Direct labour(@Rs. 10 p.h.) 10 20 15
Variable overheads(@Rs. 7 p.h.) 7 14 10.5
Sellling commission (10% of SP) 3 4 5
25.6 41.2 42.5
Contribution/unit 4.4 (1.2) 7.5

Raw material per unit (kg) 0.7 0.4 1.5
Contribution per kg (Rs.) 6.28 (3) 5
Ranking I - II

(a) Raw material available will be only 10400 kg
Product Ranking No. of units RM (kgs) CPU Total contribution
Y I 6000 2400 5.6 33600
X II 8000 5600 7.8 62400
Z III 1600 (2400/1.5) 2400 (B.F.) 12.6 20160
Total 10400 Rs. 116160
Less: Fixed overheads 50000
Profit 66160

(b) Raw material available will be only 14900(10400+4500) kg
Product Ranking No. of units RM (kgs) CPU Total contribution
X I 8000 5600 4.4 35200
Z II 5000 7500 7.5 37500
Balance 1800 -
Total 14900 Rs. 72700
Less: Fixed overheads 75000
137
Profit (2300)
Hence firm should not go into further production

Ans. 6: Statement of Ranking
Working Notes Products
Particulars A B C
Selling Price/unit (Rs.) 20 16 10
Variable cost/unit (Rs.)
Direct material 6 4 2.00
Direct labour 3 3 1.50
Variable overheads 2 1 1.00
11 8 4.50
Contribution/unit 9 8 5.50
Units 10000 12000 20000
Total contribution 90000 96000 110000
Ranking III II I

Raw material per unit (kg) 0.6 0.4 0.10
Contribution per kg (Rs.) 15 20 27.50
Ranking III II I

DLH required per unit 0.20 0.20 0.10
Contribution per DLH Rs. 45 Rs. 40 Rs. 55
Ranking II III I

Solution
(a) Raw material available will be only 12100 kg
Product Ranking No. of units RM (kgs) CPU Total contribution
C I 20000 4000 5.50 110000
B II 12000 4800 8 96000
A III 5500 (3300/0.6) 3300 (B.F.) 9 49500
Total 12100 Rs. 255500
Less: Fixed overheads 138000
Profit 117500

(b) Direct labour hours available will be only 5000 hours
Product Ranking No. of units DLH CPU Total contribution
C I 20000 2000 5.50 110000
A II 10000 2000 9 90000
B III 5000 (1000/0.2) 1000 (B.F.) 8 40000
Total 5500 Rs. 240000
Less: Fixed overheads 138000
Profit 102000

(c) No shortage of materials and labour: Ranking as per total contribution is to be considered.
Product Ranking No. of units CPU Total contribution
138
C I 25000 (20000 + 25%) 5.50 137500
B II 12000 9 96000
A III 10000 8 90000
Total Rs. 323500
Less: Advertisement cost 20000
Net contribution 303500
Less: Fixed overheads 138000
Profit 165500

Ans 7:
Working Notes
Statement of comparative contribution and Ranking (Direct labour Hour (DLH) is key
factor)
Particulars A B C
Selling
Less: Variable cost
Contribution per unit (CPU)
DLH per unit 10/10 =
Contribution per DLH
=CPU/DLH
28
23
5
1
5/1
=5.00
60
45
15
2
15/2
=7.50
125
95
30
5
30/5
=6.00
Ranking III I II

Solution
(a) Profit according to current plan

Product No. of Units DLH CPU Total
amount(Rs.)
A
B
C
D


Less
:Fixed
overheads
Profit
500 (Minimum)
500 (Minimum)
500 (Minimum)
1,400(from
surplus DLH)
Total


500
1,000
2,500
7,000
11,000
(Balance)
5
15
30
30
2,500
7,500
15,000
42,000

67,000
25,000
42,000

(b) Alternative plan for maximum profit

Product B is a Rank No. 1. Hence, instead of C Product. B should be manufactured by using
surplus labour hours. This will maximize the profit as follows:

Product No. of Units DLH CPU Total
amount(Rs.)
A 500 (Minimum) 500 5 2,500
139
B
C
D


Less :Fixed
overheads
Profit
500 (Minimum)
500 (Minimum)
3,500(from
surplus DLH)
Total


1,000
2,500
7,000
11,000
(Balance)
15
30
15
7,500
15,000
52,500

77,500
25,000
52,500
Note: This profit of Rs.52,500 is higher than current plan.

( C ) BEP (units and value)
At BEP, contribution is equal to fixed overheads, i.e., and C=F. In such case, the company
has to earn the contribution of Rs.25,000 in order to get BEP as follows:

Rank

Product No. of Units CPU Total
amount(Rs.)
I
II
II
Total
contribution
Less: Fixed
overheads
Profit
B
C
A



500 (Minimum)
500 (Minimum)
500 (Minimum)


15
30
5

7,500
15,000
2,500
25,000


25,000
Nil

BEP (Units and Value)
Product No. of Units Selling Price Per
unit
Sales Value at BEP
(Rs.)
B
C
A
Total
500
500
500
1500
60
125
28
30,000
62,500
14,000
1,06,500
BEP in terms of units: 1,500 units
BEP in terms of Sales Value : Rs.1,06,500
(d) Profit after tax (PAT) 24% on 1,00,0000 Rs.24,000
Tax Rate 50% Hence, Profit Before tax 24,000 x 100
50
Rs.48,000
Less: Tax at 50% Rs.
PAT
24,000

24,000
Note: By production and selling minimum quantities of A,B and C, BEP is achieved. Hence, in order
to earn profit before tax of 48,000, Rank No.1, Product B should be sold to the extent of 3,2000 units
(48,000 / CUP rs.15).

Then, the position will be as follows:
Product No. of Units DLH CPU Total
140
amount(Rs.)
A
B
C

B

Less: Fixed
overheads
Profit
500
500
500

3,200
Total



500
1,000
2,500

6,400
10,400
5
15
30

15
2,500
7,500
15,000
25,000
48,000
73,000

25,000
48,000
No. of Units and Sales value:

Product No. of Units Selling Price Per
unit
Sales Value (Rs.)
A
B
C
Total
500
3,700
500
4,700
28
60
125
14,000
2,22,000
62,500
2,98,500

The sales value of Rs.2,98,500 will earn the profit of Rs.48,000 (Profit Before Tax) as
worked out in the previous statement.
PBT 48,000
Less: Tax at 50% 24,000
PAT 24,000 (24% on capital employed of Rs.1,00,000)

Ans: 8
Working Notes
Statement of contributions per unit of raw material
(Key factor)
A B C
Contribution per
unit=
Contribution per
unit of Materials
2,00,000/20,000=Rs.10

10/4 = Rs.2.50
4,00,000/40,000=Rs.10

10/5 = Rs.2.00
3,00,000/20,000=Rs.15

15/6 = Rs.2.50
Ranking I II I

Solution
(i) Production / Sales mix.
Product Units Materials (Units) CPU Total Amount(Rs.)

A
C
B
20,000
20,000
20,000
20,000 X 4 = 80,000
20,000 X 6 = 1,20,000
Balance 1,00,000
10
15
10
2,00,000
3,00,000
2,00,000
Total
Less: Fixed Cost
Loss
60,000 3,00,000

(-)
7,00,000
7,50,000
141
50,000

(ii) Product No. of Units CPU Total Amount(Rs.)
A
C
B
B
20,000
20,000
20,000
40,000
10
15
10
6.25(Notes)
2,00,000
3,00,000
2,00,000
2,50,000
Total
Less: fixed Cost 7,50,000 + 50,000 =
Profit
1,00,000 9,50,000
8,00,000
1,50,000
Yes, The company can optimize production of 1,00,000 units with local substitute
materials.
Note 1. Imported Raw material cost Rs.3.00 per unit x 5 units = Rs.15.00
Local substitute materials 3.75 per unit x 5 unit =
Extra cost of materials
18.75
0.75 per unit
Contribution = 10.00-3.75= Rs.6.25 per unit
3.75

(iii) Product No. of Units CPU Total Amount(Rs.)
A
C
B

20,000
20,000
10,000

10
15
10

2,00,000
3,00,000
1,00,000

Total
Add: Lease amount

Less: Fixed cost
Profit
50,000 6,00,000
2,75,000
8,75,000
7,50,000
1,25,000
60,000-50,000 = 10,000
The company cannot enhance profits by leasing out a part of the plant.
Conclusion The proposal at (ii) will maximize the profit at Rs.1,50,000.

Ans:9
Working Notes Product A (Rs.per unit) Product B (Rs. per unit)
Sales
Less: Variable cost
Contribution

P/V ratio = C x 100 =
S
2,500
1,500
1,000
2,500
1,000
=40%
5,000
1,750
3,250
1,750
5,000
x 100
=35%
Solution
(i) When total sales in value is limited:
Product A is more profitable as its P/v ratio is 40% which is higher than that of B.
(ii) When raw material is in short supply:
Product A B
Raw material required per unit
Rs.500/50=
10 kg.

25 kg.
(Rs.1,250/50)
142
Contribution per kg of material
=Contribution per unit /kg

1,000/10 kg.
=Rs.100


1,750/25
Rs.70
In this case also, product A is more profitable as its contribution per kg of raw material is Rs.100
which is higher than that of B.
(iii) When Production capacity is the limiting factor:
Product A B
Direct Labour hours (DLH)
Required per unit = Rs.750/30
Contribution per DLH
=Contribution per unit/No. of DLH

25 hours

1,000 / 25 hours =Rs.40

1,500 / 30 = 50 hours

1,750 /50 hours =Rs.35
In this case also, Product A is more profitable as its contribution per DLH is Rs.40 which is
higher than that of B
(iv) Statement of Product Mix and Maximum profit:
Product Raw
Material (kg)
No. of Units. Contribution per Unit
(Rs.)
Amount (Rs.)
A
B

Total
Less: Fixed
Overheads
Profit (Maximum)
10,000
10,000
(Balance)
20,000
1,000
400
(10,000/25)
1,000
1,750
10,00,000
7,00,000

17,00,000
10,00,000
7,00,000

Ans:10 To maximize Profit.
(a) Statement of current profit (Rs.lakhs)
Products A B C Total
Direct Materials : 10,000 x 20
Direct labour : 10,000 x 12
Variable overheads : 10,000 x 8
2.00
1.20
0.80
0.80
0.70
0.50
1.44
0.96
0.48
4.24
2.86
1.78
Marginal cost
Sales 10,000 x 64
4.00
6.40
2.00
3.00
2.88
4.16
8.88
13.56
Contribution
Less: Fixed overheads
10,000 x 6
2.40

0.60
1.00

0.30
1.28

0.32
4.68

1.22
Profit 10,000 x 18 1.80 0.70 0.96 3.46
Ranking according to
profitability
P/v Ratio = C x 100
S
I

2.40
6.40
x 100

=37.5%

III

1.00
3.00
x 100
1
33 -- %
3
II

1.28 x 100
4.16

30.77%


143
( b) Though the contribution per unit of C is lowest, it should not be discontinued. Instead,
B should be discontinued. Total contribution from C is more than that of B.

Analysis:
Product A B C
Selling price
Less: Variable cost
64
40
60
40
52
36
CPU 24 20 16
If C is discontinued, Sales of A and B will increase by 50%.
Rs.lakhs
Contribution
A 10,000 + 50% = 15,000 units at 24= 3.60
B 5,000 + 50% = 7,500 units at 20=
5.10
1.50
Less: Fixed overheads
Profit
1.22
3.88
If B is discontinued, sales of A and C will increase by 50%



Contribution
A 3.60
C 8,000 + 50% = 12,000 units at 16 =
5.52
1.92
Less: Fixed overheads
Profit
1.22

4.30
Hence, C should not be discontinued. Product B should be discontinued. Then , the profit
will improve to Rs. 4,30,000.
Present profit 3,46,000
Proposed profit
Increase in profit
4,30,000

84,000
C. Product D:
Selling Price 48
Less: Marginal cost
Contribution per unit Rs.
25
23


Total contribution Rs.5,52,000 less contribution from a & C 3,68,000 = 1,84,000
Minimum sales = Rs.1,84,000/23 = 8,000 units are to be sold in order to ensure maximum
profit as per (b) above, i.e., Rs.4,30,000.
Statement of Profitability

Contribution from A (original level)
Contribution from C (original level)
Contribution from D ( proposed ) 8,000 x 23
Rs.lakhs
2.40
1.28
1.84
Total 5.52
144
Less: Fixed overheads 1.22
Profit 4.30

Ans:12
Working Note
Statement of contribution per labour hour (limiting Factor)
P Q R
Selling price / unit (Rs.)
Variable cost / unit (Rs.)
Contribution (Rs.)
Labour hrs/unit 20/10=
Contribution /labour hr(Rs.) 18/2=
Current sales (Units)
80
62
2
18
9
15,000
60
49
1.5
11
7.33
20,000

50
36
1
14
14
10,000
Solution
(a) Current Profit
Contribution: P : 15,000 x Rs.18 = Rs.2,70,000
Q : 20,000 x Rs.11 = Rs.2,20,000
R : 10,000 x Rs.14 = Rs.
Total contribution Rs.6,30,000
1,40,000
Less: Fixed overheads Rs.
Profit as per estimate Rs
5,50,000

. 80,000


(b) Labour is the limiting factor
Total Labour Hours utilized for the above production units : (Production and sales same).
P = 30,000 hrs.(15,000 x 2)
Q = 30,000 hrs.(20,000 x 1.5)
R = 10,000

hrs.(10,000 x 1)
70,000
Available hrs. 75,000 hrs.
hrs
Since contribution per labour hour is Maximum for R, and since labour hour is the limiting
Factor, normally this excess 5,000 hrs have to be allocated to R. But, increase in production / sales is
limited to 25% of current sales of any one of the products:
Product

(i)
Labour hours
available
(ii)
Production/sal
es possible
(iii)
25% of
current sales
(iv)
Lower of
the (iii) &
(iv)
Contributio
n per unit
(Rs.)
Total
contrib
ution
Rs.
P
Q
R
5,000
5,000
5,000

2,500
3,333
5,000
3,750
5,000
2,500
2,500
3,333
2,500
18
11
14
45,000
36,663
35,000
Contribution is highest for P.P should be chosen and after deduction of Rs. 30,000 for
advertisement, profit is Rs.15,000.
If selling price is reduced by 5% the position will be as follows:
145

Product Reduced
Selling price
Rs.
Variable cost
Rs.
Contribution
per unit Rs.
Labour hrs
reqd per
unit
Contribution
per labour
hour
Ranking
for
production
Rs.
P
Q
R
80-5%=76
60-5%=57
50-5% 47.50
62
49
36
14
8
11.50

2
1.5
1
7
5.33
11.50
II
III
I
Since labour hours are limited to 75000 hours only,product mix will be as follows:
Product No of units with
increase
Labour hrs. reqd. Total contribution
R
P
Q
15,000
22,500
10,000 (15,000/1.5)
15,000
45,000
15,000
75,000
(Bal.Fig)
@ Rs.11.5=1,72,500
@ Rs.14 =3,15,000
@ Rs. 8 =
5,67,500
80,000

Less: Fixed overheads
Profit
5,50,000
17,500
This proposal is not recommended because of lower profit.


Ans. 13:
146


147

148

Contribution per unit 120 125 121 -

Option 1: Units - 115 100 215

Contribution (Rs.) - 14,375 12,100 26,475 26,780 (305)

Option 2: Units 100 115 - 215

Contribution (Rs.) 12,000 14,375 - 26,375 22,000 4,375

Option 3: Units 80 - 135 215

Contribution (Rs.) 9,600 - 16,335 25,935 24,780 1,155

Best strategy is to produce 100 units of product A and 115 units of product B during off - season.

Maximum profit = Rs. 4,375.

(i) Best strategy for peak-season is to produce 202 units of A. (ii)
Maximum profit for off-season Rs. 4,375.


Ans:14
(a) Profit for the current year (Rs.)
Products A B C D Total
Sale Value Per acre 10
x 1000=
Variable cost per acre
Contribution per acre
Area occupied (acres)
Total contribution
25 x 5,300=
Less: Fixed overheads
Profit
10,000

4,700
5,300
25

1,32,500
10,000

5,100
4,900
20

98,000
13,500

5,950
7,550
30

2,26,500
16,200

6,600
9,600
25

2,40,000




100

6,97,000
5,40,000
1,57,000
(b) profit for the product mix
The land which is being used for A and B can be used for either items. A gives higher
contribution per acre. Hence, b should be produced to the minimum of 40 tonnes and in balance land
A should be produced.
Similarly, the land which is being used for C and D can be used for either items. D gives
higher contribution per acre. Hence, C should be produced to the minimum of 36 tonnes and in
balance land , D should be produced. Then, the position will be as follows:
A + B Area occupied = 25 + 20 = 45 acres.

B : Minimum production : 40 tonnes i.e., 40
8
= 5
Acres required.
149
A : Balance 40 acres : A should be produced

C + D : Area occupied = 30 + 25 = 55 acres

C : Minimum production = 36 tonnes, i.e., 36

= 4 acres required.
D : Balance 51 acres : D should be produced.
Then, the profitability will improve as follows:
Products A B C D Total
No of acres
Contribution per
acre
Total Contribution
Less: Fixed
Overheads
Profit
40
5,300
2,12,000
5
4,900
24,500

4
7,550
30,200
51
9,600
4,89,600
100
Rs.
7,56,300
5,40,000
2,16,300

The profit will improve from Rs.1,57,000 to Rs.2,16,300

Ans. 15:
Calculation of area to be cultivated in respect of each crop to achieve the largest total profit
Available information:
Land available for all four vegetables 340 hectares
Land available for peas and carrots
Total land available
140
Min. requirement of each variety 500 boxes
480
Max. requirement of each variety 113750 boxes
Potato Peas Carrots Tomatoes
Boxes per hectare 350 100 70 180
(a) Market price Rs. 30.76 Rs. 31.74 Rs. 36.80 Rs. 44.55
(b) Variable costs:
Direct material 2.72* 4.32 5.49 3.47
Labour Growing 5.12* 12.16 10.63 5.87
- Harvesting & Packing 7.20 6.56 8.80 10.40
Transport per box 10.40 10.40 8.00
Total variable costs
19.20
25.44 33.44 32.92
(c) Contribution per box (a)-(b)
38.94
5.32 (1.70) 3.88 5.61
Contribution per hectare
Boxes per hectare (c)
1862 (170) 271.60 1009.80
150
Ranking I IV III II
*Cost per hectare Boxes per hectare
Best cultivation plan: From 140 hectares for peas and carrots:
Peas: Minimum 5000 boxes = 5000100 = 50 hectares
Carrots: Balance land 140 hectares 50 hectares = 90 hectares
From 340 hectares all four vegetables:
Tomatoes: Minimum 5000 boxes = 5000180 = 28 hectares
(in terms of complete hectares)
Potatoes: Balance of land i.e. 340 -28 = 312 hectares

Area to be cultivated for each variety and total contribution
Potatoes Peas Carrots Tomatoes
Hectares 312 50 90 28
Contribution per
hectares
Rs. 1862 (170) 271.60 1009.80
Contribution Rs. 580944 (8500) 24444
Total contribution
28274.40
Rs. 625162.40
Less: Fixed expenses
Profit
424000.00


201162.40
(ii) Analysis to show whether land development should be undertaken
Carrot yield a lower contribution per hectare than Potatoes and Tomatoes, but it is grown in excess
of the requirement of 5000 boxes or 72 hectares i.e. 5000 boxes 700. Therefore, 18 hectares i.e., 90
hectares 72 hectares can be made available for Potatoes and Tomatoes by land improvement.
After land improvement the contribution per hectare of Tomatoes will be foloows:
Present contribution per hectare Rs. 1009.80
Saving per hectare after land improvement Rs. 2.60 180 boxes

460.00
Allocation of 18 hectares available
1477.80
Crop Maximum Sales
(Boxes)
Present
Production
(Boxes)
Addl. Reqt.
(Boxes)
Yield per
hectare
(Boxes)
Additional
hectares to be
allotted
Potatoes 113750 109200* 4550 350 13
Tomatoes 113750 5000 900 180 5(B.F.)
* 312 hectares X 350 boxes = 109200
151
Profit by revised Cultivation plan
Potatoes Peas Carrots Tomatoes Total
Hectares 325 50 72 33 480
Contribution per hectare Rs. 1862 (170) 271.60 1477.80
Total contribution Rs. 605150 (8500) 19555.20 48767.40 664972.60
Less: Fixed cost (revised)* 440200.00
Profit 2224772.60
*Capital expenditure = 18 hectares X 6000 = 108000
Interest ( 108000 X 0.15) = Rs. 16200
Existing fixed expenses 424000
440200
Conclusion: Since the profit after land development is greater, the company should implement the proposal to
develop 18 hectares of land.

Question 16: (i) Statement of Cost break-up
Sambalpur Bilaspur
Total cost (Rs.
Lacs)
Cost per M.T. of
output (Rs.)
Total cost (Rs.
Lacs)
Cost per M.T.
output (Rs.)
Material cost 198 1,650 240 1,600
(Refer to
working note)
(6,000 M. T.
Rs.1,800 +
3,600 M. T.
Rs.2,500)
(Rs.198 lacs/
12,000 M. T.)
(12,000 M. T.
Rs.12,000)
(Rs. 240 lacs/
15,000 M. T.)
Other variables 156 1,300 192 1,280
(156 lacs/
12,000 M. T.)
(192 lacs/
15,000 M. T.)
Fixed Cost 108 900 120 800
(108 lacs/
12,000 M. T.)
(120 lacs/
15,000 M. T.)
Total Cost 462 3,850 552 3,680
Working Note:
Sambalpur Bilaspur
Annual output (M. T.) 12,000 15,000
Maximum possible output (M. T.) 15,000 25,000
(12,000/80%) (15,000/60%)
Basic raw material requirement (M. T.) 9,600 12,000
(12,000 80%) (15,000 80%)
152
Material available locally (M. T.) 6,000 16,000
Possible output from local material (M. T.) 7,500 20,000
(6,000 / 80%) (16,000 / 80%)
(ii) Quantity of production at each unit from the availability of local supplies of basic raw material:
Sambalpur Bilaspur
Maximum output/ possible (M. T.) 15,000 25,000
(Refer to above working note)
Material cost/ M. T. of output from locals (Rs.) 1,440 (6,000
Rs.1,800) / 7,500
M T.
1,600
Other variables / M. T. of output from locals (Rs.) 1,300
[Refer to part (i)]
1,280

Total variable cost / M. T. of output 2,740
Possible output (M. T.) from local supplies of basic
raw material
2,880
7,500 19,500
(Balancing Figure)
(iii) Cost saving as per revised schedule of production :
Sambalpur Bilaspur Total
(Rs. lacs) (Rs. lacs) (Rs. lacs)
Total variable cost of output 205.5 561.6 767.1
(Refer to part ii) (7,500 M. T.
Rs.2,740)
(19,500 M. T.
Rs.2,880)

Fixed Cost 108.0 120.0
Total cost: (A)
228.0
313.5 681.6 995.1
Previous total cost: (B) 462.0 552.0
[as per (i) above]
1014.0

Cost savings: {(B) (A)} 148.5 (129.6)

18.9
Statement of cost per tonne and net profit earned in respect of each factory
Ans. 17
Lucknow Pune
Present production tonnes: (A) 7,200 10,800
Rs. Rs.
Cost of raw material (Rs. in lacs) 59.04 87.48
(Refer to working note 1)
Other variable costs (Rs. in lacs) 22.32 32.94
Fixed cost (Rs. in lacs) 18.00
Total cost (Rs. in lacs): (B)
24.84
99.36 145.26
Cost per tonne (Rs) : (C) = [(B) / (A)] 1,380 1,345
153
Selling price (Rs. Per tonne: (D) 1,450 1,460
Net profit per tonne (Rs.) : [(D) (C)] 70 115
Total net profit (Rs. in lacs) 5.04 12.42
(Rs.70 7,200 tonnes) (Rs.11510,800 tonnes)
Total profit of the company = Rs.15.46 lacs
(Rs.5.04 lacs + Rs.12.42 lacs)
Alternative production plan to earn optimum
Lucknow Pune
Maximum production capacity (tonnes) 9,000 11,880
Present production (tonnes) 7,200 10,800
Rs. Rs.
Cost per tonne of output: 800 810
Cost per tonne of output manufactured from locally
purchased raw material: (A)

(Refer to working note 2)
Cost per tonne of output manufactured from material
purchased from Bhopal : (B)
880 880
(Return to working note 3)
Other variable cost (Rs.) : (C) 310 305

tonnes 7,200
Lacs Rs.22.32

tonnes 10,800
Lacs Rs.32.94

Selling price per tonne (Rs.) : (D) 1,450 1,460
Contribution per tonne of Output : [(D){(A)+(C)}] 340 345
Contribution per tonne of Output : [(D) {(B)+(C)}] 260 275
(When material was purchased from Bhopal)
The priority to produce 18,000 tonnes of total output is as below as apparent from the above data:
Priority
Pune factory (Local purchase of raw material) 1
st
Lucknow factory (local) purchase of raw material) 2

nd
Pune factory (raw material purchased from Bhopal) 3

rd
Lucknow factory (raw material purchased from Bhopal) 4

th
Suggested alternative production plan :

Production priority Raw Material Output (in tonnes)
Input(in tonnes) Lucknow Pune Total
I 11,700 tonnes 13,000 -- 11,700 11,700
II 5,400 tonnes 6,000 5,400 -- 5,400
III (11,880 11,700) = 180 tonnes 200 -- 180 180
IV 720 tonnes balancing figure 800 720 -- 720
154
(18,000 17,280 tonnes)
20,000 6,120 11,880 18,000
Working Notes:
Lucknow Pune
1. Present production output (tonnes) 7,200 10,800
Total raw material required for present
production (tonnes)
8,000 12,000


90
100
200 , 7


90
100
800 , 10
Raw material produced locally (tonnes) 6,000 12,000
Raw material product from Bhopal 2,000 --
Cost of raw material purchased locally 59.04 87.48
and from Bhopal (Rs. in lacs) (Rs.7206,000+
Rs.792 2,000)
(12,000 Rs.729)
2. Cost per tone of output manufactured
from locally purchased raw material
800 810
(in Rs.)


90
100
720


90
100
729
3. Cost per tonne of output manufactured
from material purchased from Bhopal
880 880
(in Rs.)


90
100
792



Ans.: 20:

Throughout Accounting ratio is highest for Machine 2.
Machine 2 is the bottleneck
Contribution per unit of bottleneck machine hour :
A B C

Total Machine 2 hours available = 6,000

A. Contribution per unit (Rs.) 30 25 15

B. Machine 2 hours 15 3 6

C. Contribution per Machine 2 hours (A / B) 2 8.33 2.50

D. Ranking 3 1 2

E. Maximum Demand 500 500 500

Machine 2 hours required (B E) 7,500 1,500 3,000

Machine 2 hours available 1,500 1,500 3,000

Units 100 500 500


Ans. 21:



155
Production

A B C Total Mach TA

Capacity ratio
Demand (units) 200 200 200

Hrs. required in Dept.
Machine 1 2,400 800 400 3,600 3,200 112.5%
2 3,600 1,200 600 5,400 3,200 168.75%
3 1,200 400 200 1,800 3,200 56.25%
Machine 2 is the bottleneck

Note-2:
Through put contribution & rank

A B C
(a) Throughput Contribution 24 20 12
(b) MR/unit in Machine 2 18 6 3
(c) Contribution/hr. Machine 2 1.33 3.33 4
Rank III II I
Identification of product mix.

Hrs. in machine 2 units
Available 3,200
Less: Rank I C _ 200 600
2,600
Less: Rank II B 200 1,200
Less: Rank III A 18 77.77
i.e. 77 units


(a)
Ans. 22:

Machine Time required for products Total
Time
Time
Available
Machine
utilization
A B C D
1
2
3
2000
2000
2000
1200
1800
600
400
600
200
200
300
100
3800
4700
2900
3000
3000
3000
126.67%
156.67%
96.67%
Since Machine 2 has the highest machine Utilization it represents the bottleneck activity hence product,
ranking & resource allocation should be based on contribution/machine hour of Machine 2.

Allocation of Resources

A B C D Machine
Utilization
Spare
Capacity
156
Contribution per
unit
(Rs.)
Time required in
Machine 2
Contribution per
Machine hour (Rs.)
Rank as per
contribution
/ mach. Hour
Allocation of Machine
2
time
Production Quantity
Allocation Machine
1
time
Allocation of Machine
3
time
1500

10

150

3
r
d

20010 =
2000

200
2000

2000
1200

9

133.33

4
th

100
(balan
cing
figure)
100/9=11.1
1
11.116 =
66.66

11.113 =
33.33
1000

3

333.33

2
n
d

2003 =
600

200
400

200
600

1.5

400

1
st

2001.5 = 300

200
200

100









3000



2666.66

2333.33













333.34

666.67


Ans. 23:
Rs. p. u Rs. p. u.
W. Note 1
A B

Material 2 40

Variable production overhead cost 28 4

TVC 30 44

Selling price 60 70

(a) Contribution 30

26
(b) Limiting factor (hr./u) 0.25 0.15
(c) Contribution/hr. (a/b) Rs. 120 173.33
(d) Rank II II
(e) Budgeted production & sales 1,20,000 45,000
(f) Maximum demand 1,44,000 54,000
Total Fixed cost Rs 14,70,000
W. Note-2:

Fixed overhead recovery rate =(AmountBudgeted hours) = 14,70,000 36,750 = Rs. 40/hr.

Budgeted hours A 1,20,000 units @ Rs. 0.25 = 30,000 hrs.

B 45,000 units @ Rs. 0.15 = 6,750 hrs.

36,750 hrs.

(a) A B

Contribution per unit Rs. 30 26

157
Less: Fixed overhead per unit Rs. 10 6

(a) Profit per unit Rs.

20 20
(b) Units 1,20,000 45,000
Total (ab) 24 lakhs + 9 lakhs = 33 lakhs

Management is indifferent on the basis of profit per unit however this is wrong concept on selecting the product mix.

(b) A B

(a) Contribution per unit Rs.

30 26

(b) Limiting time/unit 0.02 0.015
Contribution /hr. (a/b) Rs. 1,500 Rs. 1,733
Rank II II
Statement of product mix & profit

Hrs. units Contribution/u Total
Available 3,075
Less: for Rank I 810 54,000 26 14,04,000
For Rank II

2,265/0.02 1,13,250 30 33,97,500
Product A

48,01,500
Less: Fixed cost 14,70,000

Profit 33,31,500
(c) Return per bottleneck hour = (selling price material cost)/ (Time on bottleneck resource)
Product A = Rs. 2,900 [(Rs. 60 Rs. 2)/ Rs. 0.02 hours]
Product B = Rs. 2,000 [(Rs. 70 Rs. 40)/ 0.015 hours]

Product A should be sold up to its maximum capacity of utilizing 2,880 bottleneck hours (1,44,000 units 0.02
hours). This will leave 195 hours for product B thus enabling 13,000 units (195/0.015) to be produced.
The maximum profit is calculated as follows:

Rs.


Throughput return from product A (1,44,000 Rs. 58)

83,52,000
Contribution from product B (13,000 Rs. 30) 3,90,000


87,42,000
Less: Variable overheads 35,40,000
Fixed overhead cost 14,70,000

Net profit

37,32,000

Note:

It is assumed that the variable overheads (e.g. direct labour) are fixed in the short term. They are derived from part
(a) [(120,000 Rs. 28) + (45,000 Rs. 4)]



Ans. 30:
Installed Capacity for the machine = 365 * 8 *3 * 500 = 43.8 lakh units
158

Practical Capacity = ( 365 52 - 13 ) * ( 8 - 1) * 3 * 500 = 31.5 lakh units

Out of the past five years, normal capacity is average of 3 normal years.
Normal Capacity = ( 30.1 + 29.7 + 30.2 ) / 3 = 30.0 lakh units

Actual Capacity Utilization = 30.1 lakh units = 68.7 %

Idle Capacity = ( 43.8 30.1) = 13.7 lakh unit = 31.3 %

Abnormal idle capacity = 31.5 30.1 = 1.4 lakh units


Ans. 31:
Capacity
Details of Computation Machine hours Production units @ 10
units per hour
1. Maximum capacity ( 365 days 8 hours per day) 2,920 29,200
2. Practical capacity
Maximum capacity (in hours) 2,920
Less: Idle capacity
Sundays: (52 days 8 hours) 416
Holidays (10 days 8 hours) 80
Plant maintenance 2,224 200 22,240
3. Normal capacity 2,000 20,000
4. Expected capacity 1,900 19,000

Determination of Factory overhead application rate
(a) Total Budgeted overheads
Fixed overhead costs Rs. 6,00,000
Variable overhead costs (2,000 hours Rs. 100) 2,00,000
8,00,000
(b) Normal Capacity (machine-hours) 2000
(c) (i) Factory overhead application rate (Rs. 8,00,0002,000) per hour 400
(ii) Factory overhead application rate (Rs. 8,00,0002,0000) per unit 40

Ans. 32

Working Notes:
(Amount in Rupees)
X Y Z

Selling price per unit (A) 135.00 140.00 200.00
Variable costs per unit
Direct material 32.00 76.00 58.50
Direct labour
Department 1 45.00 25.00 50.00
Department 2 15.00 12.00 21.00
Department 3 20.00 10.00 40.00
Variable overheads 8.00 4.50 10.50
Total variable costs (B) 120.00 127.50 180.00
Contribution per unit (AB) 15.00 12.50 20.00
159


(i ) Statement of budgeted profi tabi l i ty


X Y Z
Budgeted quantity (units) 19,500 15,600 15,600
Contribution per unit (Rs.) 15.00 12.50 20.00
Total contribution (Rs.) 2,92,500 1,95,000 3,12,000
Contribution fund (Rs.)
Fixed overheads (Rs.)
Profit (Rs.) 3,99,500


(i i ) Contri buti on per di rect l abour hour for Department 2


X Y Z
Contribution per unit (Rs.) 15.00 12.50 20
Direct labour hours per unit 5 4 7
Contribution per labour hour 3.00 3.125 2.857
Rank II I III
(i i i ) Total hours avai l abl e i n department 2

X 19,500 units 5 = 97,500 hours
Y 15,600 units 4 = 62,400 hours
Z 15,600 units 7 = 1,09,200 hours
Total = 2,69,100 hours


Y 19,500 2,69,100 4 19,500 78,000 1,91,100
X 23,400 1,91,100 5 23,400 1,17,000 74,100
Z 19,500 74,100 7 10,585 74,095 5


Opti mal profi t (Rs.) Contri buti on
(Rs.)
Y
19,500 Rs. 12.50 = Rs. 2,43,750

X 23,400 Rs. 15 = Rs. 3,51,000
Z 10,585 Rs. 20 = Rs. 2,11,700
Total Contribution

= Rs. 8,06,450
Less fixed cost = Rs. 4,00,000
160
Profit

= Rs. 4,06,450



Ans 33:
(a) Fl exible Budget

Output level (units) 50,000
(Rs. in lakhs)
80,000
(Rs. in lakhs)
1,00,000
(Rs. in lakhs)
Sales
Direct Material 12.5 per unit
(reduction for 1,00,000 units by
Rs.0.50)
Direct wages (5.00 per unit) Semi
variable cost (variable) Factory
overhead (V) Rs.5 per unit) Selling
and Adm. (25% variable) Total
variable cost
Contribution
Fixed factory overheads (560,000)
Selling and adm. (6 60,000)
Semi variable fixed part
Increase due to expansion
Interest
Depreciation
Special Advertisement exp.
Total fixed costs
20.00
6.25



2.50
0.25
2.50
1.00
12.50
7.50
3.00
3.60
.30





.
6.90
0.60
32.00
10.00



4.00
0.40
4.00
1.60
20.00
12.00
3.00
3.60
.30
2.00
.60
.50
.50
10.50
1.50
36.00
12.00



5.00
0.50
5.00
2.08
24.58
11.42
3.00
3.60
.30
2.80
.60
.50
.
10.80
0.62
Therefore activity level 80,000 units is most profitable level. Calculation of
Break even point
P/V ratio

7.5/20.00 100 = 37.5%, 12.00/32.00 100 = 37.5%, 11.42/36.00 100 = 31.72%BEP (value)
= 6.90/37.5% = Rs.18,40,000, 10.50/37.5% = Rs.28,00,000, 10.80/31.72% = 34,04,792

BEP (Units)
6.90
.15
lakhs
Rs

10.50
.15
lakhs
Rs

10.80
.15
lakhs
Rs

= 46,000 units = 70,000 units = 94,571 units
Al ternative Sol uti on (BEP in Sales)

Break Even Point in value of sales: (F x S) / (S V)


At 50000 units level : (6,90,000 x 20,00,000)/7,50,000 = Rs. 18,40,000

At 80000 units level : (10,50,000 x 32,00,000)/12,00,000 = Rs. 28,00,000

At 100000 units level : (10,80,000 x 36,00,000)/11,42,000 = Rs. 34,04,553


161
Ans. 34
Overheads
: Budget statement for April
Budget Actual Variance
Fixed Variable Total Adverse Favourable
Management Rs.30,000 - 30,000 30,000 - -
Shift premium - 3,600 3,600 4,000 400 -
ESI 6,000 7,920 13,920 15,000 1,080 -
Inspection 20,000 9,000 29,000 28,000 - 1,000
Supplies 6,000 6,480 12,480 12,700 220 -
Power - 7,200 7,200 7,800 600 -
Lighting and heating 4,000 - 4,000 4,200 200 -
Rates 9,000 - 9,000 9,000 - -
Repairs 8,000 5,400 13,400 15,100 1,700 -
Materials handling 10,000 10,800 20,800 21,400 600 -
Depreciation 15,000 - 15,000 15,000 - -
Administration 12,000 - 12,000 11,500 - 500
Idle time - - - 1,600 1,600 -
1,20,000 50,400 1,70,400 1,75,300 6,400 1,500
Rs.4,900 (A)

(b) E.S.I. This variance may be due to increase of E.S.I. rates. If this assumption is correct, then the
variance will be beyond the control of management. It should be noted that actual activity is less
than budgeted activity. It is , therefore, unlikely that increase is due to increase in the number of
labour hours worked. Another possibility is that E.S.I. Payment might have got increased due to
increase in E.S.I. rates.
Inspection: There is a possibility that standard inspection has been lowered, thus resulting in a
saving in costs. If this is not due to management policy, then the variance requires immediate
investigation. Another possibility is that a number of staff members have resigned and consequently
actual inspection is less than the budget.
Repairs and Maintenance: This increase may be due to unexpected repair, which might not have
been envisaged. The variance for this item over a period of several months should be studied to
form an opinion.
Idle Time: No Idle time has been included in the budget. Consequently this idle time must be of an
abnormal nature. Possible uncontrollable causes include a power failure or machine breakdown.
Controllable causes may include poor scheduling or lack of material.

(c ) (i) Calling for comments on variances in excess of a specific figure may not be satisfactory for
control purpose. For decision on whether to investigate or not, Cost of investigation should be
compared with benefits of investigation. Statistical tests may also be applied.
(ii) The statement could be improved by analyzing the expense items into their controllable and
non- controllable elements. Variances should be analysed according to whether they are due to
price and quantity changes. Analysis should include non- financial measures such as a comparison
of actual hours worked with standard hours produced.

(d) (i) Overhead absorbed = Rs.1,58,400, i.e.,36,000 hrs x Rs.4.40
(ii) Over spending = Rs.4,900
(iii) Actual production was 4,000 standard hours less than budgeted production and this decline
in output has resulted in a failure to recover Rs.12,000 fixed overheads. This under
recovery of Rs.12,000 is also known as the volume variance.


162
Ans. 35:
Analysis of the information required for preparation of cash budget (Rs.000)
A.Z. Limited
April May June July August
Sales receipts - 401.70 450.28 425.88 -
Variable cost of sales (60%) 240.00 270.00 312.00 252.00 288.00
Variable production costs:
In the month of sales (60%) 144.00 162.00 187.20 151.20 -
In prior month (40%) 108.00 124.80 100.80 115.20 -
252.00 286.80 288.00 266.40
Material costs 60% of production cost 151.20 172.08 172.80 159.84
Purchases:
In the month of production (50%) 75.60 86.04 86.40 79.92
In prior month (50%) 86.04 86.40 79.92
Payment to supplier 161.64 172.44 166.32
Labour costs
(Variable production cost x 0.3) 75.60 86.04 86.40 79.92
Variable overhead 25.20 28.68 28.80 26.64
(Variable production cost x 0.1)
Variable cost was paid as follows:
Paid in the month of incurrence (40%) 10.08 11.47 11.52 10.66
Paid in the following month (60%) 15.12 17.21 17.28
Variable overhead expenditure 26.59 28.73 27.94

Cash budget for the month of May to July 1997
May June July
Receipts from sales 401.70 450.28
Payments:
425.88

Materials 161.64 172.44 166.32
Labour 86.04 86.40 79.92
Variable overhead 26.59 28.73 27.94
Fixed costs (12,00,000-3,00,000)/12 75.00 75.00 75.00
Capital expenditure
Total expenditure 349.27 552.57 349.18
Net inflow (outflow) 52.43 (102.29) 76.70
Balance b/f 40.00 92.43 (9.86)
Balance c/f 92.43 (9.86) 66.84

Note. In this question language should be given particular attention:
(a) Variable production cost 60% in the same month 40% in the prior month.
Production cost relevant for cash budget for each month should be found.
(b) 60% of production cost is material 50% in the same month and 50% in the prior month.
30% of production cost is labour which is paid the same month.
10% of production cost is variable overhead, 40% is paid the same month.
60% is paid in the following months.
(c) This question illustrates the interaction of sales, purchase and manufacturing process and
requires the reader to think clearly about these relationships


Ans. 36
Note: Since question has not clearly specified that whether labour efficiency is lower by
163
ANOTHER 1% or by 1%, also it is unclear that efficiency is to reduced based on BUDGETED
EFFICIENCY OR ACTUAL EFFICIENCY, hence this question can be solved in following 3 ways
(after giving prompt assumption)


Solution Way 1
Production Cost Budget
(for 6 months ending 30th September, 2009)
30,000 units
Cost per unit Total
Rs. Rs.
Material cost 180 54,00,000
Labour cost 115.47 34,64,208
Variable overhead 23.65 7,09,500
Fixed overhead 23.2

6,96,000
342.34

1,02,69,708
Assumption : Here, difference in actual and standard time is also considered for calculating
the lower efficiency i.e. 3.74% + 1% = 4.74% based on budgeted efficiency

Working Notes:
I. Material cost
Material consumption per unit = 1,600MT 16,000 = 0.10 MT

Consumption for 30,000 units = 3,000 MT.
Cost of 3,000 MT @ Rs. 1,800 per MT = Rs. 54,00,000.

II. Labour cost can be calculated as follows:
2008 Total Budgeted Hour = 16,00,000 40 = 40,000 hours

Labour hour budget for each unit = 40,000 16,000 = 2.5

Actual time paid = 15,99,840 44 = 36,360 hours

Less: Standard labour hours for 14,000 units (i.e. 14,0002.5)= 35,000 hours
Difference in actual and standard hours = 1,360
3.74% = Difference in actual and standard hours Actual hours 100
= 1,360 hours 36,360 hours

Budget unit (2008) for each labour hour = 16,00040000 = 0.4 units
Less: (3.74% + 1%) = 4.74% for lower efficiency
Budget unit (2009) for each labour hour = 0.38104 units
= 0.01896 units

Time required for 30,000 units (30,000 0.38104) = 78,732 hours

Labour cost = 78,732 hours 44 per hour = Rs. 34,64,208.

III. Variable overhead
Actual rate = Rs.2,76,000 14,000 units = 19.71 per unit
Add: 20 % = 3.94
New rate 23.65
164

Total variable overhead = 30,000 23.65 = Rs. 7,09,500
IV. Fixed overhead
Actual = Rs. 5,80,000
Add: 20%

= Rs. 1,16,000

= Rs. 6,96,000
According to above the production cost budget will be as follows:

Production Cost Budget
Solution Way 2
(for 6 months ending 30th September, 2009)
30,000 units
Cost per unit Total
Rs. Rs.
Material cost 180 54,00,000
Labour cost 111.11 33,33,352
Variable overhead 23.65 7,09,500
Fixed overhead 23.2 6,96,000
337.96 1,01,38,652

Assumption : Here, lower efficiency of 1% is based on budgeted efficiency

Working Notes:
I. Material cost

Material consumption per unit = 1,600MT 16,000 = 0.10 MT
Consumption for 30,000 units = 3,000 MT.
Cost of 3,000 MT @ Rs. 1,800 per MT = Rs. 54,00,000.

II. Labour Cost:
2008 Total Budgeted Hour = 16,00,000 40 = 40,000 hours

Budget unit (2008) for each labour hour = 16,00040000 = 0.4 units
Less: 1% for lower efficiency
Budget unit (2009) for each labour hour = 0.396 units
= 0.004units

Time required for 30,000 units (30,000 0.396) = 75,758 hours

Labour cost = 75,758 hours 44 per hour = Rs. 33,33,352

III. Variable overhead
Actual rate = Rs.2,76,00014,000 units = 19.71 per unit
Add: 20 %
New rate
= 3.94
Total variable overhead = 30,000 23.65 = Rs. 7,09,500
23.65
IV. Fixed overhead
Actual = Rs. 5,80,000
Add: 20%

= Rs. 1,16,000

= Rs. 6,96,000
165
Production Cost Budget
Solution Way 3
(for 6 months ending 30th September, 2009)
30,000 units
Cost per unit Total
Rs. Rs.
Material cost 180 54,00,000
Labour cost 115.44 34,63,196
Variable overhead 23.65 7,09,500
Fixed overhead 23.2 6,96,000
342.29 1,02,68,696

Assumption : Here, lower efficiency of 1% is based on actual efficiency

Working Notes:
I. Material cost

Material consumption per unit = 1,600MT 16,000 = 0.10 MT
Consumption for 30,000 units = 3,000 MT.
Cost of 3,000 MT @ Rs. 1,800 per MT = Rs. 54,00,000.

II. Labour Cost:
2008 Total Actual Hour = 15,99,840 44 = 36,360 hours

Actual unit (2008) for each labour hour = 1400036360 = 0.385 units
Less: 1% for lower efficiency
Budget unit (2009) for each labour hour = 0.38115 units
= 0.00385units

Time required for 30,000 units (30,000 0.38115) = 78,709 hours

Labour cost = 78,709 hours 44 per hour = Rs. 34,63,196

III. Variable overhead
Actual rate = Rs.2,76,00014,000 units = 19.71 per unit
Add: 20 %
New rate
= 3.94
Total variable overhead = 30,000 23.65 = Rs. 7,09,500
23.65
IV. Fixed overhead
Actual = Rs. 5,80,000
Add: 20%

= Rs. 1,16,000

= Rs. 6,96,000

Ans. 37: (a) Cash Budget for October, November and December 1990

Opening balance of bank (overdraft)
Cash inflows Sales:
From cash sales of current month
From credit sales of previous month
Total Receipts (A)
Cash outflows:
October
Rs.35,000

5,000
15,000
55,000

November
Rs.(9,100)
-
6,000
18,000
14,900

December
Rs.(12,600)

8,000
20,000
15,400
166
Creditors for purchases of the
preceding month
Equipment
Wages
Administration
Rent
Dividend
Total payment (B)
Closing balance (Overdraft) (A-B)

40,000
16,000
3,000
1,500
3,600
-
64,100
(9,100)

23,000
-
3,000
1,500
-
-
27,500
(12,600)


27,000
-
3,000
1,500
-
15,000
46,500
(31,100)

(b) Budgeted Income Statement for three months ending 31
st
Sales
December 1990
Less: Cost of Goods Sold:
Material- Opening Stock
Add: Purchases (23,000 + 27,000 + 26,000)

Less: Closing stock
Cost of material consumed
Wages (3,000 x 3)
Gross profit
Less: Rent [ 3,600 x (3 / 12 ) ]
Administration (1,500 x 3)
Depreciation [3,000 x (3 / 12)]
Loss on sale of asset ( Rs.15,000 Rs.14,000)
Net profit


Rs.20,000
76,000
96,000
43,500
52,500
9,000

900
4,500
750
1,000

Rs.82,000






61,500
20,500



7,150
13,350



Working Notes:
(i) Total Sales Credit Sales Cash Sales Total
October 1990 Rs.18,000 Rs.5,000 Rs.23,000
November 1990 20,000 6,000 26,000
December 1990 25,000 8,000

33,000
63,000 19,000

82,000
For Cost of Sales:
(ii) Sales for the quarter Rs.82,000
Less: Gross Profit 25% of Sales
Cost of sales
20,500
(iii) For Material consumed:
61,500
Cash of sales for three months Rs.61,500
Less: Wages (3,000 x 3)
Cost of material consumed
9,000
52,500
(iv) For closing stock of material

Opening stock of material Rs.20,000
Add: Purchases (23,000 + 27,000 + 26,000) 76,000
96,000

Less: Material consumed
Closing stock of material 43,500
52,500
167
Ans. 38:


Shirt Short
Nov Dec Jan Feb Nov Dec Jan Feb
Cl. Stock( 40% 6120 6242 6367 - 8160 8320 8490 -
Of next month)
Sales 15000 15300 15919 15606 20000 20400 21224 20800
Total 21120 21542 21973 28160 28720 29290
Op. Stock 6000 6120 6242 8000 8160 8320
Production 15120 15422 15731 20160 20560 20970



Shirts Shorts
Opening stock 6000 8000
Sales

November 6000
= 15,000

40%
8000
= 20,000

40%
December 1.02 x 15,000 = 15,300 1.02 X 20,000 = 20400
January 1.02 x 15,300 = 15,606 1.02 X 20,400 = 20,808
February 1.02 X 15, 606 = 15, 919 1.02 X 20,808 = 21,224



Al ternative:


Opening Stock Shirts = 6000
= 40% of November Sales
Opening Stock of Shorts = 8000
=40% of November Sales
November sales 6000
= l5,000

40%
8000
= 20,000

40%
Dec. Sales l.02 x l5,000 = l5,300 l.02 x 20,000 = 20400
Closing Stock November 40% x l5, 300 = 6,l20 40% x 20,400 = 8l60
November Production =
Closing Stock + sales
Opening stock
l5, l20 20,l60
December Production l.02 X l5, l20 = l5422 l.02 X 20,l60 = 20, 560
January Production l.02 Xl5422 = l5, 73l l.02 X 20,560 = 20, 970

(a) Producti on Budget for product A and B
Ans39:
A uni ts B units
Inventory at the end of the year 1,000 2,000
Sales forecast 8,000 15,000
168
Total requirements 9,000 17,000
Less: Beginning inventory 3,000 5,000
Production 6,000 12,000

Budgeted requi rements of components P, Q and R
Components P Q R
For Product A: Production 6,000 units
P: 6,000 1 per unit 6,000
Q: 6,000 2 per unit 12,000
For Product B: Production 12,000 units
P: 12,000 2 per unit 24,000
Q: 12,000 1 per unit 12,000
R: 12,000 2 per unit 24,000
For comp R: Production 24,000 comp
Q: 24,000 1 per component R 24,000
Total requirements 30,000 48,000 24,000
(b) The company i s advi sed to adopt EOQ system.
P Q
2 30,000 15
EOQ
2 20%


2 48,000 15
0.8 20%


= 1,500 components = 3,000 components

(c) Calculation of savings arising from switching over to the new ordering system.
Existing situation:
P Q
Present order quantity (units)
(equivalent to 3 months consumption)
30,000 7,500 48,000 12,000
Average stock (units) 7,500 3,750 12,000 6,000
Investment in inventory of P & Q 3,750 Rs. 2 7,500 6,000 Re. 0.80 4,800
Total investment Rs. 7,500 + Rs. 4, 800 =Rs. 12,300
Carrying cost @ 20% p.a. of average
inventory investment
Rs. 12,300 20% Rs. 2,460
Ordering cost: P = 4Rs. 15 = Rs. 60
Total cost
Rs. 120
Rs. 2,580
After switching over:
P Q
Economic order quantity (units) 1,500 3,000
169
Average stock (units) 1,500 750 3,000 1,500
Investment in inventory of P & Q 750 Rs. 2 1,500 1,500 Re. 0.80 1,200
Total investment Rs. 1,500 + Rs. 1,200 =Rs. 2,700
Carrying cost @ 20% p.a. of average
inventory investment
Rs. 2,700 20% Rs. 540
Ordering cost: P = 20Rs. 15 = Rs. 300
Q = 16Rs. 15 = Rs. 240
Total cost
Rs. 540
Rs. 1,080
Savi ng i n costs: Rs. 2,580 Rs. 1,080 = Rs. 1,500
Reducti on i n worki ng capi tal : Rs. 12,300 Rs.2,700 = Rs. 9600

Ans. 42:
(showing quantities to be manufactured)
Production Budget
Chairs Tables Benches
Units to be sold (Note 1)
Add: Closing inventory as per budget

Less: Opening inventory as per budget
4,200
200
4,400
400
4,000
800
300
1,100
100
1,000
500
50
550
50
500
(b) Material Purchase Budget (in quantities)


Timber Upholstery
(cu. ft.) (Sq. yards)
Material required for production (Note 1) 4,450 1,000
Add: Closing stock as per budget 650
5,100 1,260
260
Less: Opening stock as per budget 600
Raw materials to b purchased
400
4,500

800
Materials Purchase (in rupees)

Quantities to be purchased Rate Amount
Timber (c.ft.) 4,500 50 Rs.2,25,000
Upholstery (sq. yds.) 860 20

17,200

2,42,200
(c) Direct wage Cost Budget

Total hrs. Rate p.h. Amount
Carpenters time and wages 4,625 6.00 Rs.27,750
Fixers and finishers time and wages 1,500 4.80 7,200


34,950
(d) Statement showing the variable cost of manufacture per unit of all three products.
Chairs Tables Benches
170
Raw materials Timer

Upholstery

Fixing and finishing materials cost
(Note 2)
Wages
Carpenters

Fixers and finishers
Rs.25.00
(0.5 x Rs.50)
5.00
(0.25 x 20)
1.50


4.50
(45/60) x Rs.6
1.20
(15/60) x 4.80
37.20
60.00
(1.2 x Rs.50)
-

3.00


6.00
(60/60) x Rs.6
1.20
(15/60) x 4.80
70.20
125.00
(2.5 x Rs.50)
-

6.25


7.50
(75/60) x Rs.6
2.40
(30/60) x 4.80
141.15

(e) Budgeted Net Income Statement
(For the quarter)


Selling price (per unit)
Less: Variable cost
Contribution per unit (A)
Units to be sold (B)
Total contribution
Fixed cost for the quarter
(Rs.8,000 x 30
Budgeted net income
Chairs
Rs.50.00
37.20
12.80
4,200
53,760
Tables
Rs.85.00
70.20
14.80
800
11,840
Benches
Rs.158.00
141.15
16.85
500
8,425
Total
Rs.



74,025

24,000
50,025


Working Notes:
1. Raw Materials, Carpenters Time and Fixers and finishers Time

Units to be manufactured
Timber (c. ft.)

Upholstery (sq. yards)

Carpenters time (hrs.)

Fixers and Finishers
time(hrs.)

Chairs
4,000
2,000
(4,000 x 0.5)
1,000
(4,000 x 0.25)
3,000
(4,000 x(45 /60)
1,000
4,000 x(15/60)
Tables
1,000
1,200
(1,000 x 1.2)
-

1,000
1,000 x (60/60)
250
1,000 x (15/60)
Benches
500
1,250
(500 x 2.5)
-

625
500 x (75/60)
250
500 x (30/60)
Total

4,450

1,000

4,625

1,500

2. Per unit cost of materials of fixing and finishing

Chairs Tables Benches
Total cost of Timber and Upholstery Rs.30 Rs.60 Rs.125
Fixing and Finishing Material will cost 5%
Of total cost of timber and upholstery 1.5 3 6.25
(5% of 30) (5% of Rs.60) (5% of Rs.125)

Ans. 43:Necessary Calculations
171
Statement showing total cost and selling price and sales in units for each product
(Working Note 1) Working A Working B Working C
Materials Rs. Rs. Rs.
M
1

(Rs.2x5 units) 10 - 2x12 24
M
2

- (4x10) 40 (4x9) 36
M
3
15 45 60
(Rs.1x5 units) 5 (1x5) 5

Labour
Department I (Rs.2.5x4) 10 (2.5x2) 5 (2.5x2) 5
Department II (Rs.2.0x6) 12 (2x2) 4 (2x3) 6
Department 1II (Rs.1.5x2) 3 (1.5x4) 6 (1.5x6) 9
Variable overhead 10 20 15

Fixed Cost(Working Note 2)
Department I (Rs.5x4 hrs.) 20 (5x2) 10 (5x2) 10
Department II (Rs.3x6 hrs) 18 (3x2) 6 (3x3) 9
Department 1II (Rs.6x2 hrs.) 12 (6x4) 24 (6x6)
Total production cost 100 120 150
36
Adm.(Based on 20% of production cost) 20 24 30
Selling and Distb. Cost (40% of prod. Cost) 40 48
Total cost 160 192 240
60
Profit (25% of total cost) 40 (12 % of 24 (16 2/3% of 40
Selling price per unit

total cost) total cost
200 216 280
Sales in rupees 15,00,000 10,80,000 16,80,000

Sales in units 7,500 5,000 6,000
Sales in rupees / Selling price (per unit)

(a) Production Budget for July 1986 A
(Units)
B
(units)
C
(Units)
Sales
Less: Closing stock (given)

Add: Closing stock : 20% reduction
(working Note 3)
Production

7,500
3,000

4,500
2,400
5,000
6,900
2,000

3,000
1,600

4,600
6,000
2,500

3,500
2,000
5,500

(b) Material Usage budget for July 1986
Product Units of
product
Qty. per
unit of
product
M
1
Qty. per
unit of
product
total
Qty reqd.
M
2
M
Total
Qty
reqd.
3
Total
Qty.
reqd.
Qty
per units
of
product
A
B
C
Total usage in unit
6,900
4,600
5,500
5
-
12
34,500
-
66,000
1,00,500
-
10
9
-
46,000
49,500
95,500
5
5
-
34,500
23,000
-
57,500
172

(c ) Material Purchase Budget
M M
1
M
2

3

Units Rs. Units Rs. Units Rs.
Usage 1,00,000 2,01,000 95,500 3,82,000 57,500 57,500
(price is given
Less: O/stock

24,500

49,000

20,500

82,000

17,500

17,500
76,000 1,52,000 75,000 3,00,000 40,000 40,000
(Add: C/stock)
(10% reduction)

22,050

44,100

18,450

73,800

15,750

15,750
98,050 1,96,100 93,450 3,73,800 55,750 55,750

(d) Budgeted profit and loss account for each product and in total
A B C Total
Sales Rs.15,00,000 Rs.10,80,000 Rs.16,80,000 Rs.42,60,000
Less: cost (Working Notes) 12,00,000 9,60,000 14,40,000 36,00,000
Profit 3,00,000 1,20,000 2,40,000 6,60,000

Working Notes
Note: 1. Price per unit of material and material units required for each product should be multiplied.

Note:2. Fixed overhead rate
Deptt. I = Rs.2,39,000
47,800
or Rs.5 per hour



Deptt. II = Rs.2,01,300
67,100
or Rs.3 per hour



Deptt. II = Rs.3,91,200
65,200
or Rs.6 per hour

Note:3. A = 3,000 x 80 or 2,400 , B = 2,000 x 80 or 1,600, C = 2,500 x 80
100 100 100
or 2,000


Note:4. A -7,500 x 160 =Rs. 12,00,000; B - 5,000 x 192 Rs.9,60,000;
C 6,000 x 240 =Rs.14,40,000

Ans. 44:
For the production manager
Responsibility Accounting Reports
Cutting Department Budgeted Rs. Actual Rs. Variance Rs.

Cloth 31,000 36,000 5,000 (A)
Cutting Labour 6,000 6,600 600 (A)
173
Cutting utilises 800 700
Total cutting Deptt. (A)
100 (A)
37,800 43,300
Sewing Department:
5,700 (A)

Thread 500 450 50 (F)
Sewing Labour 17,000 18,400 1,400 (A)
Sewing utilities 900 950
Total Sewing Dept. (B)
50 (F)
18,400 19,800
Total (A + B)
1,400 (A)
56,200 63,100 6,900 (A)
For the director-Manufacturing

Production Department * 56,200 63,100 6,900 (A)
Production engineering expenses 13,000 12,200 800 (F)
Production manager-office expenses 18,000 17,000
Total
1,000 (F)
87,200 92,300
(* As per responsibility accounting report for the production manager)
5,100 (A)
For the Direct-Marketing
Sales representative:

Travelling expenses 9,000 10,200 1,200 (A)
Sales commission 7,000 7,000
Total (A)
--
16,000 17,200
Sales Management:
1,200 (A)

Office expenses 16,000 15,700 300 (F)
Advertising 4,000 4,000
Total (B)

20,000 19,700
Credit Department:
300 (F)

Salaries 8,000 8,000
Credit reports 1,200 1,050 150 (F)
Bad debt Losses 5,000 3,000
Total
2,000 (F)
14,200 12,050
Total (A + B + C)
2,150 (F)
50,200 48,950 1,250 (F)
Note: F denotes favourable variance while A denotes adverse variance.

Performance Budget
Ans. 45:
Original Revised Actual Variance
Plan Rs. Budgeted Rs. Result Rs. Rs.

Revenue (5,00010) 50,000
(4,00010) 40,000
(4,00011) 44,000 4,000 (F)
Variable (5,0004) 20,000
Costs (4,0004) 16,000
(4,0004.5) 18,000 2,000 (A)
174
Contribution (5,0004) 30,000
(4,0006) 24,000
(4,0006.5) 26,000 2,000 (F)
Fixed costs 20,000 20,000 21,000 1,000 (A)
Net Profit 10,000 4,000 5,000 1,000 (F)
Summary Report on Profit Plan
Planned Income (from Project plan) Rs. 10,000
Activity variance (lost contribution margin due

to shortage of materials) (6,000)
Selling price variance (increased
Selling price of Re. 1/- per unit) 4,000
Variance cost variance (increased production
Costs at 0.50 per unit) (2,000)
Fixed cost variance (new research programme to
Develop raw materials and processes) (1,000)
Actual income (from income statement) 5,000


175

TRANSFER PRICING
Ans 9
(i ) In this case there are two options available
(a) Sell at the sub assembly stage (after completion of Div. A) @
Rs. 2000/-

Incremental cost in Div. A Rs 1,200/-
Contribution Rs 800/-
(b) Sell at the final product stage Rs. 3,000
Cost at Div. A and Div. B Rs(1200+1500) Rs 2,700
Contribution Rs 300
Therefore it is profitable to sell at the subassembly stage because of higher contribution, provided there is
a market.
Hence, if there is market at intermediate stage, first priority is to sell intermediary (sub
assembly).Therefore, 800 units should be sold as sale of intermediary.
The balance capacity available of (1000 800) = 200 units should be transferred to B and B should
complete the assembly and sell as final product, since the company can earn Rs. 300 per unit for each unit
of such sale.
(i i ) If B Div. receives the subassembly at market price of Rs. 2,000, plus its own incremental cost of Rs. 1,500
will give total cost of Rs. 3,500, thereby yielding a loss of Rs. 3500 Rs. 3000 = Rs. 500 per unit, whereas
the company makes a profit of Rs. 300 per unit.
In order to keep the manager of Div. B motivated, the profit earned of Rs. 300 per unit should be shared
between A and B. Hence transfer price will be variable cost of Div. A + 50% of profit earned in the final
product = 1200 + 150 = Rs. 1,350
(i i i ) Both Div. A and the Company make higher contribution by selling to intermediate market. If the market
demand increases to 1,000 units, the full quantity should be sold outside as intermediary and nothing
should be transferred to Div. B

Ans.10:
Transfer Price is Rs. 4,500 for each consulting day.
Profit mark-up = 150%
Let cost = x
Profit = x
100
150
= 1.5x
Cost + profit = Transfer price
x + 1.5x = 4,500
2.5x = 4,500
x = 1,800
Cost = Rs. 1,800
and profit = 1.5x = 1.51,800 = Rs. 2,700

Variable cost (80%) = Rs. 1,800 80% = Rs. 1,440
Fixed cost (20%) = Rs. 1,800 20% = Rs. 360.

Scenario (i):
Every consultancy team is fully engaged. There is no idle time or spare capacity. Hence, transfer
price = Marginal cost plus opportunity cost
Marginal cost = Rs. 1,440
Saving for internal work = Rs. 200
Net Marginal Cost = Rs. 1,240

176
Opportunity cost is the lost contribution.
Lost contribution = Contribution from external client
= Fee charged from external client Variable cost
= Rs. (4,500 1,440)
= Rs. 3,060.
Transfer price = Rs. 1,240 + 3,060
= Rs. 4,300 per consulting day per team.

Scenario (ii):
One team is idle. Idle time has no opportunity cost. Variable cost for internal work is Rs. 1,240
per consulting day. Second team is busy. Hence opportunity cost is relevant in case of second
team. Hence charge of second team is Rs. 4,300 per consulting day per team.
Average of charge of two teams = Rs. (1,240 + 4,300) / 2
= Rs. 2,770 per consulting day per team.

Scenario (iii):
New client offers a fee of Rs. 15,84,000
Duration: 5 days of 48 weeks 2 teams = 480 days
Fee per day 15,84,000 / 480 = Rs. 3,300
Variable cost = Rs. 1,440
Contribution Rs. (3,300 1,440) = Rs. 1,860

Fee for consulting day for internal work:
Variable cost = Rs. 1,240
Contribution lost = Rs. 1,860
Fee to be charged = Rs. 3,100 per consulting day per team.

Ans.11:

100% capacity 4,000 tones (Maximum)
Distribution market
Processing unit
2,000 Tones
2,000 Tones

80% capacity 3,200 tones
Market 2,000 Tones
Processing unit 12,00 Tones

(a) 80% capacity price Rs. 400 per ton (Rs.)
Particulars Basic unit Particulars Processing unit
Sales (3,200 * 400) 12,80,000 (24,000 * 40) 9,60,000
Raw materials (3,200 * 70)
Variable cost (3,200 * 140)
Fixed overhead
2,24,000
4,48,000

9,72,000
3,00,000
Tr. Price (1,200 *
400)
(1,200 * 170)


4,80,000
2,04,000

8,04,000
1,20,000
Profit 3,08000 1,56,000
Total profit of the company = Rs. 4, 64,000

(b) 100% capacity price Rs. 400 per ton (Rs.)
Particulars Basic unit Particulars Processing
unit
Sales (4,000 X 400)
Raw materials (4,000 X 70)
Variable cost (4,000 X 140)
Fixed overheads
16,00,000
2,80,000
5,60,000
3,00,000
14,00,000
(4000 X 320)
Tr. Price (2,000 X 400)
(2,000 X 170)
12,80,000
8,00,000
3,40,000
1,20,000
12,60,000
Profit 4,60,000 20,000
Total Profit of the Company = Rs. 4,80,000


177
(c ) 80% capacity- Market price @ Rs.360 & Transfer price to processing @ Rs. 400 per tonne
(Rs)
Particulars Basic unit Particulars Processing unit
Sales
(2,000 X 360) + (1,200 X 400)
Raw materials (3,200 X 70)
Variable Cost (3,200 X 140)
Fixed overheads

2,24,000
12,00,000
4,48,000
9,72,000
3,00,000

(24,000 X 40)
Tr. Price (1,200 X 400)
(1,200 X 170)

4,80,000

9,60,000
2,04,000
8,04,000

1,20,000
Profit 2,28,000 1,56,000
Total Profit of the Company = Rs. 3,84,000

(d) 100% capacity- Price Rs. 360 per tonne (Rs.)
Particulars Basic unit Particulars Processing units
Sales (4,000 X 360)
Raw material (4,0000 X 70)
Variable overheads (4,000 X 140)
Fixed overheads
2,80,000
14,40,000
5,60,000
11,40,000
3,00,000

Tr Price (2,000 X 360)
(2,000 X 170)
7,20,000

12,80,000
3,40,000
11,80,000

1,20,000
Profit 3,00,000 1,00,000
Total profit o the Company = 4,00,000

Comments :
At Rs. 400 per tonne, the processing unit will not be interested in buying more than 1,200 tonnes because the
profitability of the processing unit will be reduce from Rs. 1,56,000 to Rs. 2,000.
When the market price reduce to Rs. 360 per tonne the processing unit will not be interested in purchasing more than
1,200 tonnes because at this level it can maintain the same level of profit.
Even if the price is reduced to Rs.360 for the processing unit, it may not be interested in buying more than 1,200
tonnes as its profitability will be reduced from Rs.1,56,000 to Rs.1,00,000.
When the market price reduced to Rs.360 per tonne and the transfer price is maintained at Rs.400, the processing
unit may get its suppliers of 1,200 tonnes via open market at the price less than Rs.400 per tonne. This will increase
the profitability of the processing unit but reduced the profitability of the basic unit.
Thus the present policy market price for transfer pricing does not offer incentive to the processing unit. Hence cost
plus method should be restored to.

Ans. 12
(i) (a) At 80% level (in Rs)
-Textile unit -Process house
Sales (4,00,000 6) 24,00,000 Sales(1,50,000/100) 825 12,37,500
Less Less
Raw material (4,00,000 3) 12,00,000 Transfer Price (1,50,000 6) 9,00,000
Variable cost (4,00,000 1.2) 4,80,000 Variable cost (1,500 80) 1,20,000
Fixed cost 4,12,000 Fixed cost 1,00,000
Profit 3,08,000 Profit 1,17,500
Overall profit = 3,08,000 + 1,17,500 = Rs 4,25,500
At 100% level
Sales (5,00,000 6) 30,00,000 Sales (2,50,000/100) 725 18,12,500
Less Less
Raw material (5,00,000 3) 15,00,000 Transfer Price (2,50,000
6)

15,00,000
Variable cost (5,00,000
1.2)
6,00,000 Variable cost 2,00,000
Fixed cost 4,12,000 Fixed cost 1,00,000
Profit 4,88,000 Profit 12,500
Overall profit = 4,88,000+12,500 = Rs 5,00,500
(b) At 80% level (market price 5.60 and transfer price 6/-) (in Rs)
Textile unit Process house
Sale (2,50,000 5.6) 1400000
178
(1,50,000 6.0) 900000
23,00,000
Less
Raw material (4,00,000 3) 12,00,000
Variable cost (4,00,000 1.2) 4,80,000
Fixed cost 4,12,000
Profit 2,08,000 Profit 1,17,500

Overall profit = 2,08,000+1,17,500 =Rs 3,25,500
(c) Sales 100% level at (5.60) (in Rs)


Sale (5,00,000 5.6) 28,00,000 Sales(2,50,000 725) 18,12,500
Less Less
Raw material (5,00,000 3) 15,00,000 Transfer Profit (2,50,000
5.6)

14,00,000
Variable cost (5,00,000 1.20) 6,00,000 Variable cost (2,500
80)
2,00,000
Fixed cost 4,12,000 Fixed cost 1,00,000
Profit 2,88,000 Profit 1,12,500
Overall profit = 2,88,000 + 1,12,500 =4,00,500
(ii) Comments on the profitability of processing units:-
Transfer price (Rs) Profit (Rs)
(a) 80% capacity 6.00 1,17,500
100% capacity 6.00 12,500
(b) 80% capacity 6.00 1,17,500
(c) 100% capacity 5.60 1,12,500
Processing house will not be interested to buy more than 1,50,000 meters from textile units.


Ans.: 13 (Rs.)
Particulars BRITE LITE TITE
Selling Price
Variable costs
Contribution
300
150
60
40
700
590
150 20 110
Alternative I
Division AD
(Rs)
Contribution
(15,000 units of BRITE X Rs.150)
(40,000 units of LITE X Rs.20)
Total Contribution
Fixed Expenses
Profit (a)

22,50,000
8,00,000
30,50,000
20,00,000
10,50,000

Division CD
(Rs)
Contribution (5,000 units of TITE X Rs.110)
Fixed Expenses
Profit (b)
5,50,000
4,00,000
1,50,000
Overall profit of the company (a + b) Rs.12,00,000

Alternative II
Division AD
(Rs)
179
Contribution
(15,000 units BRITE outside customer @ Rs.150)
(5,000 units of BRITE Division CD @ 150)
(20,000 units of LITE (limited capacity) @ Rs.20)
Total contribution
Fixed expenses
Profit (a)

22,50,000
7,50,000
4,00,000
34,00,000
20,00,000
14,00,000

Division CD
Extra cost of labour Rs.50 and variable cost Rs.640
Hence contribution Rs.700 - Rs.640 = Rs.60
(Rs)
Contribution (5,000 units @ Rs.60)
Fixed Expenses
Loss (b)
Overall profit of the company (a-b)
300,000
400,000
100,000
13,00,000

Alternative III
Division AD
Price of BRITE to CD reduced by Rs.50
Hence Contribution/unit Rs.250 Rs.150 = Rs.100
(Rs)
Contribution
(15,000 units of BRITE outside party @ Rs150)
(5,000 units of BRITE to CD @ Rs100)
(20,000 units of LITE to capacity @ Rs20)
Total contribution
Fixed expenses
Profit (a)

22,50,000
5,00,000
4,00,000
31,50,000
20,00,000
11,50,000
Division CD
BRITE from AD Rs.250 contribution Rs.700-Rs.590 = Rs.110 per unit
Lobour and overhead Rs.340, Variable costs Rs.590 (Rs.)
Contribution (5,000 units @ Rs.110)
Fixed expenses
Loss (b)
Overall profit of the company (a+b)
5,50,000
4,00,000
1,50,000
Rs.13,00,000

Alternative 1V
Division AD (Rs.)
Contribution
(15,000 units BRITE outside customer @ Rs.150)
(10,000 units of BRITE to CD @ Rs.250 i.e., contribution @ Rs.100)
Total contribution
Fixed expenses
Profit (a)

22,50,000
10,00,000
32,50,000
20,00,000
12,50,000

Division CD (Rs.)
Contribution
(10,000 units with BRITE of AD @ Rs.110)
(2,000 units with imported component @ Rs.110)
Total contribution
Fixed expenses
Profit (b)

11,00,000
2,20,000
13,20,000
11,70,000
1,50,000
Overall profit of the company (a+b) Rs.14,00,000
Recommendation on best alternative
Alternative (iv) seems to be the best because it leads to the maximum profit of Rs. 1400000 for the company. But
management should consider whether stopping the production of Lite altogether will, in any way, be detrimental to
companys interests.
Negotiated price of Rs. 240 per unit.
The price of Rs. 240 per unit will be acceptable to AD because it will lead to a contribution of Rs. 22.50 per hour i.e.
(Rs. 240-Rs.150)4 hours. If this proposal is not accepted AD will have to produce Lite which will yield a
contribution of only Rs. 20 per hour, i.e. (Rs. 60-Rs.40)1 hour.
180

AJ
Ans.: 14: Alternative I
Rs. DJ Rs.
Sales : outside (18,000x15)
DJ (2,000x10)
Total
Less V. Costs( 20,000x8.50)
Net Contribution
2,70,000
20,000
2,90,000
1,70,000
1,20,000
Sales (2,000 x 105)
Variable Costs (2,000 x 99)
Contribution
Interest
Net Contribution
2,10,000
1,98,000
12,000
1,000
11,000
Total Group contribution =Rs.1,31,000
Alternative II
Sales : (20,000x15)
Variable costs(20,000x8.50)
Contribution
3,00,000
1,70,000
1,30,000

Sales (2,000 x 105)
Variable Costs (2,000 x104)
Contribution
Interest
Net Contribution
2,10,000
2,08,000
2,000
1,000
1,000
Total Group contribution = Rs.1,31,000

Alternative III
Sales : (20,000x15)
Variable costs(20,000x8.50)
Contribution
3,00,000
1,70,000
1,30,000

Sales (2,000 x 105)
Variable Costs (2,000x 104)
Contribution
Interest
Net Contribution
2,10,000
2,08,000
2,000
1,000
1,000
Total Group contribution = Rs.1,31,000

Alternative IV
Sales : (22,000x15)
Variable costs 1,87,000
Over time
Contribution
4,000
3,30,000

1,91,000
1,39,000

Sales (2,000 x 105)
Variable Costs(2,000 x 104)
Contribution
Interest
Net Contribution
2,10,000
2,08,000
2,000
1,000
1,000
Total Group contribution = Rs.1,40,000

Comments:
Alternative 1: AJ can supply part 35 to DJ at Rs.10 because the variable cost is Rs.8.50 only and
by this transaction a contribution of Rs.1.50 is available. But the overall contribution which
would have been Rs.13,000 if the part has been sold to outside buyers, would come down to
Rs.1,20,000. DJ however, will earn a net contribution of Rs.11,000. Thus the divisional
performance of AJ will go down and that of DJ will boost up at the cost of AJ.
Alternative 2: AJ will maintain its performance but DJs performance will be reduced to a
contribution of Rs.1,000 only.
Alternative 3: AJ will maintain its performance but DJs performance will be reduced to a
contribution of Rs.1,000 only. In these three cases the group income will not change but the
performance of the individual divisions will vary.
Alternative 4: AJs performance will boost up but DJs performance will remain at the low level
.DJ cannot show better performance except at the cost of AJ. Hence AJ should not reduce the
price particularly when it has an assured market for part 35 at Rs.15 each.

Ans.: 15:
Statement showing profitability of two divisions at two different levels of output
using different transfer prices

181
No. of bottles 8,00,000 12,00,000
Rs. Rs.
Sales value (Packed Product) : (A) 91,20,000 1,27,80,000
Less : Costs
Product Manufacturing Division 64,80,000 96,80,000
Bottle Manufacturing Division 10,40,000 14,40,000
Total costs : (B) 75,20,000 1,11,20,000
Profit :{(A) (B)} 16,00,000 16,60,000
Profit pro-rated to Bottle Mfg. Division and Product
Mfg. Division.

Share of Bottle Manufacturing Division:
16,00,000 10,40,000/75,20,000 2,21,276
16,60,000 14,40,000/1,11,20,000 2,14,964
Balance profit relates to Product Mfg. Division 13,78,724 14,45,036
16,00,000 16,60,000
Rs. Rs.
Transfer prices of bottles

Costs 10,40,000 14,40,000
Profit as computed above 2,21,276 2,14,964
Total price 12,61,276 16,54,964
Transfer price per bottle Rs. 1.577 Rs. 1.379
From the above computations, it is observed that shared profit relative to the cost involved is Rs.
2,21,276 (Re. 0.2766 per bottle) at 8,00,000 production level and Rs. 2,14,964 (Re. 0.179 per
bottle) at 12,00,000 production level. The profit of Product Mfg. Division is Rs. 13, 78, 724
(Rs. 1. 723 per bottle) at 8, 00, 000 production level and Rs.
14,45,036 (Rs. 1.2042 per bottle) at 12,00,000 production level.

Profitability based on market price
No. of bottles 8,00,000 12,00,000

Bottle Mfg. Division Rs.

Rs.
Market price 14,00,000 20,00,000
Less: Cost 10,40,000 14,40,000
Profit (i) 3,60,000 5,60,000
Product Mfg. Division

Sales 91,20,000 1,27,80,000
Less: Bottle cost 14,00,000 20,00,000
Product cost 64,80,000 96,80,000
Profit (ii) 12,40,000

11,00,000
Total profit : (i) + (ii) 16,00,000

16,60,000

Profit based on Profit based on
cost (Rs.Lakhs) Market price (Rs.Lakhs)

Production level Bottle Product Bottle Product
Mfg. Div. Mfg. Div. Mfg. Div. Mfg. Div.
8,00,000 bottles 2.21 13.79 3.60 12.40
12,00,000 bottles 2.15 14.45 5.60 11.00
Observations:
1. Market price methods gives a better profitability to Bottle Mfg. Division at both the
production levels.
2. Market price method gives a lower profitability to Product Mfg. Division as compared to Bottle
182
Mfg. Division.
3. Under Cost-based method, there is a better profit at lower level of production in Bottle
Mfg. Division. However in Product Mfg. Division 12,00,000 production level gives a higher
profit. But in Market price method, the position is quite reverse.

Ans. 16
(a) When component is purchased by Division B from outside (Rs.)
(i) Statement of contribution
Division A
Division B Sales (2000x 400)
Less: Cost of Purchase (2000x 200) 400000
Variable costs (200x 150)
Companys total contribution
300000

8,00,000

3,80,000

Nil


1,00,000
1,00,000

(b) When component is purchased from Division A by Division B (Rs.)
Division A
Sales (2000x 220)
Less: Variable costs (2000x 190)
Division B
Sales (2000x 400)
Less: Variable Costs:
Purchase cost in Division A (2000x 220) 440000
Variable cost in Division B (2000x 150)
Companys total contribution
300000

4,40,000
7,00,000

8,00,000


7,40,000



60,000




60,000
1,20,000

Thus, it will be beneficial for the company as a whole to ask Division B to buy the component
from Division A.
(ii) Statement of total contribution if Division A could be put to alternative use:
Division A:
Contribution from alternative use of facilities
Division B:
Sales (2000x 400)
Less: Variable costs:
Cost of purchase (2000x 400) 400000
Division B (2000x 150)
Companys total contribution
300000



8,00,000


7,00,000



30,000



1,00,000
1,30,000


Since, the companys contribution when component is purchased from outside, shows as
increase of Rs.30,000 as compared to when there is inter departmental transfer. Hence, it will be
beneficial to purchase the component from outside.


(iii) Statement of total contribution when component is available from outside at Rs. 185
Division A:
Division B:
Sales (2000x 400)
Less: Variable costs:
Cost of purchase (2000x 185) 370000
Division B (2000x 150)
Companys total contribution
300000


8,00,000


6,70,000

Nil




1,30,000
1,30,000


If the component is purchased by Division B from Division A, the contribution is only
Rs.1,20,000 as calculated under above. Hence it will be beneficial to buy the component from
outside.
183
(iv) Fixations of transfer price
(a) When there are no alternative uses of production facilities of Dept. A:
In such a case the variable cost i.e. Rs.190 per component will be charged.
(b) If facilities of Division A can be put to alternative uses:
(Rs.)
Variable cost
Opportunity cost
Transfer price
190
15
205


(c) If market price gets reduced to Rs.185 and there is no alternative use of facilities of Division
A. the variable cost of Rs.190 per component should be charged.

Ans.17
Total overheads
For the budgeted level of activities and expenses of LD the various costs and prices can
be worked out as follows:
(Rs.)
Less: Variable overheads
Fixed overheads per year
7,56,000
4,20,000
3,36,000

LX
Variable overheads 4,20,000 x
LY
90,000 4,20,000 x
2,10,000 2,10,000
1,20,000
1,80,000 2,40,000

Fixed overheads per year 3,36,000 x 90,000 3,36,000 x 1,20,000
At the budgeted level of activities 2,10,000 2,10,000

1,44,000 1,92,000

The costs and selling prices of the products of LD for normal sale to outside parties will be as
under: (Rs.per kg.)
Particulars LX LY
Direct material
Direct wages
Variable overheads
Total Variable cost:
Fixed costs
Total costs
Add: Mark-up 50%
Selling price
36
30
60
126
48
174
87
261
28
20
40
88
32
120
60
180

Labour hours calculated as under:
Particulars LX LY
Direct wages
Wages rate (Rs./hr.)
Direct labour hr.
30
5
6
20
5
4

Committed production of LY of 6,000 kg. would involve labour of 6000 x 4 = 24,000
Balance labour available for:

Production of LX = 42,000-24,000 = 18,000 Hrs.
Production of LY = 18,000 hrs. / 6 DLH = 3,000 Kg.

184
Cost estimate of KX it KD purchase Lx from LD at normal prices
(Rs.)
Cost of LX
Processing materials & Wage costs
Variable Overheads
Total Variable Cost
261
30
4
295

Profit Statement of LD & KD
(1) Transfer price based on total cost
LD Rs. KD Rs.
Sales LX (3000 x 261)
LY (6000 x 180)
Total Sales
Variable cost
LX (2000 x 122)
(1000 x 126)
LY (6000 x 88)
Total variable cost
Fixed costs
Total costs
Profit
7,83,000
10,80,000
18,63,000

2,44,000
1,26,000
5,28,000
8,98,000
3,36,000
12,34,000
6,29,000
Sales KX (2000 x 300)


Variable cost (2000 x 295)




Fixed cost
Total cost
Loss

6,00,000


5,90,000




1,00,000
6,90,000
(-)90,000

Total profit for the company = 6,29,000 90,000 =Rs.5,39,000

(ii) Transfer price based on total Cost after adjustment for selling expenses
LD Rs. KD Rs.
Sales LX (2000 x 257)
(1000 x 261)
LY (6000 x 180)
Total Sales
Less: Costs as above
Profit

5,14,000
2,61,000
10,80,000
18,55,000
12,34,000
6,21,000

Sales (2000 x 300)



Total costs (690000-4 x 2000)
Less
6,00,000



6,82,000
(-)82,000


(iii) Total profit to the company =6,21,000-82,000 =Rs.5,39,000
LD Rs. KD Rs.
Sales LX (2000 x 122)
(1000 x 261)
LY (6000 x 180)

Total Sales
Less: Total Costs as above
Profit

2,44,000
2,61,000
10,80,000

15,85,000
12,34,000
3,51,000

Sales KX (2000 x 300)

Variable cost (2000 x 156)
Fixed costs
Total costs

Profit
6,00,000

3,12,000
1,00,000
4,12,000

1,88,000


(iv) Total profit for the Company =3,51,000 + 1,88,000 =Rs.5,39,000
LD Rs. KD Rs.
Sales LX (3000 x 152) (a)
(Including Rs.30 oT)
(3000 x261)
LY (6000 x 180)
Total Sales
3,04,000

7,83,000
10,80,000
Sales KX (2000 x 300)(a)


Variable cost (2000 x 186)

6,00,000


3,72,000
1,00,000
185
Variable cost
LX (2000 x 152)
(3000 x 126)
LY (6000 x 88)

Total variable cost
Fixed costs
Total costs (b)
Profit (a-b)
21,67,000

3,04,000
3,78,000
5,28,000
12,10,000
3,36,000
15,46,000
6,21,000







Total costs (b)
Profit (a-b)







4,72,000
1,28,000
Total profit for the company =6,21,000 + 1,28,000 Rs.7,49,000


Ans.18 (i) Department A
By product BYEA Production 3000 Tonnes
Sales Income (30% of 3000 Tonnes @ Rs.200)
(70% of 3000 Tonnes @ Rs.1200)
Total
(Rs.)
1,80,000
25,20,000
27,00,000

(ii) Department B
Production of RESP (3000 x 200,i.e.,600000 litres)
Sales (600000 litres @ Rs.15) (a)
Costs: Opportunity Cost of BYEA
Variable Costs (600000 @ Rs.4)
Fixed Costs
Total (b)
Profit (a-b)
(Rs.)
90,00,000
27,00,000
24,00,000
12,00,000
63,00,000
27,00,000

(iii) Department C (ltrs.)
Production of POTS (600000 x 1.6)
5% wastage
9,60,000
48,000
9,12,000

(a) Sales
Pack (ML) % Litres No.of packs Price/Pack
Rs.
Sales Value
Rs
200
300
Total
75
25
6,84,000
2,28,000
9,12,000
34,20,000
7,60,000

2.50
3.50
85,50,000
26,60,000
1,12,10,000

(b) Costs
(Rs.)
RESP (600000 x 15)
Mfg. Cost (912000 x 1.50)
Total
90,00,000
13,68,000
1,03,68,000

Profit (a-b) 8,42,000
(iv) Total Profit under the existing arrangement
A-27,00,000 + B-27,00,000 + C-8,42,000 =Rs.62,42,000
Under the new proposal
(Ltrs.)
Total quantity of RESP purchased (3000 x 120)
Production of POTs (360000 x 1.60)
3,60,000
5,76,000
186
Amount of Saleable POTs (576000 x 95/100) 5,47,200


(a) Sales
Pack (ML) % Litres No.of packs Price/Pack
Rs.
Sales Value
Rs
200
300
Total
75
25
4,10,400
1,36,800
5,47,200
2,05,200
4,56,000

2.50
3.50
51,30,000
15,96,000
67,26,000

(b) Costs (Rs.)
RESP (360000 x 6.25) 22,50,000
Mfg. Cost (547200 x 1.50) 8,20,000
Fixed Overhead of Dept. B 12,00,000


42,70,800

Profit (a)- (b) 24,55,200
Analysis : Since under the new proposal profit gets lowered from Rs.62,42,000 to Rs.24,55,200
the proposal is not acceptable.


Ans.19. The transfer price will be notional revenue to S and notional cost to T.
(a) S will continue to produce more output until the costs of further production exceed the
transfer price revenue.
(b) T will continue to want to receive more output from S until its net revenue from further
processing is not sufficient to cover the incremental transfer price costs.

Output Units Division S Incremental Cost
Rs.
Division T Incremental Costs
Rs.
600
700
800
900
1000
1100
1200
-
100
140
160
200
250
350
-
300
280
250
220
200
150

Since S will continue to produce more output if the transfer price exceeds the incremental
costs of production, a price of at least Rs.200 per 100 units (Rs. 2 per unit ) is required to
persuade the manager of S of produce as many as 1,000 units, but a price in excess of
Rs.250 per 100 units would motivate the manager of S to produce 1,100 units (or more).
By a similar argument, T will continue to want more output from S if the incremental revenue
exceed the transfer costs from S. If T wants 1,000 units the transfer price must be less than
Rs.220 per 100 units. How ever, if the transfer price is lower than Rs.200 per 100 units, T
will ask for 1100 units from S in order to improve its divisional profit further.
In summary
(a) The total company profit I maximised at 1,000 units of output.
(b) Division S will, want to produce 1,000 units, no more and no less, if the transfer price is
between Rs.2 and Rs.2.50(Rs.200 to Rs.250 per 100 units).
(c) Division T will want to receive and process 1,000 units, no more and no less, if the
transfer price is between Rs.2 and Rs.2.20
(d) A transfer price must therefore be selected in the range Rs.2.00 to Rs.2.20 per
unit(exclusive).
187
Thus, if a price of Rs.2.10 per unit is selected, profits at 1,000 units of output would be;
(Rs.)
Particulars Division S Division T Total
Sales/Net revenue
Costs
Profit
2,100
1,200
900
4,000
2,100
1,900
4,000
1,200
2,800

At a transfer price of Rs.2.10 any increase in output above 1,000 units, or shortfall in output
below this amount, would reduce the profits of company as a whole, but also the divisional
profits of S and T.

Ans.20. (a)The problem
The overall company interest is obviously to produce 1,400 units which will given the maximum
profit. The problem is to fix the transfer price (TP) with which both X and Y will find 1,400 units
to be the optimum output for them severally.
Let us analyse and examine the incremental costs at X and the incremental revenue at Y
Level of output Incremental Cost for
X
Incremental Net
revenue for Y Rs.
Company profit
1,000
1,100
1,200
1,300
1,400
1,500
1,600
-
100
120
130
150
180
220
-
300
240
190
170
130
80
3,100
3,300
3,420
3,480
3,500
3,450
3,310

A price of at least Rs. 150 per 100 units (Rs.1.50 per unit) is required to induce the manager of X
to produce as many as 1,400 units; but the price must not exceed Rs.180 per 100 units, for in that
event X would like to produce 1,500 units (or more)
Similarly, Y will keep producing so long as the incremental revenues exceed the transfer cost
from X. in order that Y wants 1,400 units, the TP must be lower than Rs.170 per 100units; but it
shall not be lower than Rs.130,for Y will then ask for 1,500 units from X to increase his (Ys)
divisional profit further.
If the TP is selected at Rs.1.60 per unit, profits at 1,400 units of output would be
(Rs.)
Particulars X Y Company
Sales / Net revenue
Costs
Profit
2,240
1,400
840
4,900
2,240
2,660
4,900
1,400
3,500

At a TP of Rs.1.60 any increase in output above 1,400 units or shortfall in output below this level
would reduce the profits of the company as a whole and also the divisional profits of X and y.
With Rs.1.60 as TP, neither X or Y will like to deviate from 1,400 units, which incidentally is
also wanted y the corporate Management.

Ans. 21. (i) Calculation of transfer price to be quoted by Alfa to Beta based on residual
income
(Rs.)
Fixed Costs
Return on capital employed (Rs.750 lakhs x 12/100)
Residual income desired
Total
80
90
100
270
188
Desired contribution per unit =Selling price p.u.-Variable cost p.u. =Rs.180- Rs.60 =Rs.20
p.u.
Total desired contribution =12,00,000 units x Rs.20 p.u =Rs.240 lakhs
Minimum contribution to be earned from sale of additional 3 lakh units.

Rs.270 lakhs-Rs.240 lakhs =Rs30 lakhs.

Contribution p.u. on additional 3,00,000 units =Rs.30,00,000/3,00,000 units = 10 p.u.

Variable cost of modification per unit =Rs.5

Hence, the minimum transfer price per unit to be quoted will be
=Rs.160 + 10 + 5 =Rs.175
(ii) If Beta can buy from outside at less than the variable cost of manufacture, Rs.165, than only
the decision to transfer at the price of Rs.175 will become sub-optimal for the group as a whole.

Ans.22. Working Notes:
(i) Computation of Sales revenue from Foam Division
(Rs.)
Sales of Foam Division to outside customers (Rs.1,600-Rs.200)
Less: Variable Mfg. Costs (Rs.1,200-Rs.200)

Mark-up on outside Sale (Rs.400/Rs.1000)x 100=40%
Transfer Price of Foam to Upholstery Division
Sales of Foam Division to outside Customers
Total
1,400
1,000
400

280
1,400
1,680
(ii) Variable Mfg. Cost of Upholstery Division (Rs.000)
=(Rs.680-Rs.200 + Rs.280) =Rs.760

(iii) Computation of Traceable Administration Expenses ( Rs.000)
Divisions Foam Carpets Upholstery Total
Given Administration expenses
Less: Common expenses (10%
of Gross Profit)
Traceable Administration
Expenses
134

40

94
116

40

76
172

50

122
422

130

292

(iv) Computation of Traceable Selling Expenses ( Rs.000)
Divisions Foam Carpets Upholstery Total
Given Selling expenses
Less: Common expenses (2.5%
of Sales)
Traceable Selling Expenses
202

40
162
210

30
180
232

30
202
644

100
544

(a) Revised operating statement (using Contribution approach) (Rs.000)

Divisions Foam Carpets Upholstery Total
Sales Revenue
Less: Variable Mfg. Costs
Contribution (i)
Traceable Costs:
Fixed Mfg. Costs
Admn. Expenses
1,680
1,200
480

-
1,200
700
500

100
76
1,200
760
440

20
122
4,080
2,660
1,420

120
292
189
Selling Expenses
Total (ii)
Operating Income (i)-(ii)
Less: Common expenses
Net Income of the Company
94
162
256
224
180
356
144

202
344
96
544
956
464
230
234

(b) (i) Computation of contribution Margin (Rs.000)

Contribution Margin Ratio % = Sales
Contribution X 100

(Ranks)
Foam
Carpets
Upholstery
(Rs.480/Rs.1680) x100
(Rs.500/Rs.1200) x 100
(Rs.440/Rs.1200) x 100
28.57%
41.67%
36.67%
III
I
II

(ii) Computation of Net Contribution Ratio (Rs.000)

Net Contribution Ratio (%) =
Sales
Net Contribution X 100

Foam
Carpets
Upholstery
(Rs.224/Rs.1680) x100
(Rs.144/Rs.1200) x 100
(Rs.96/Rs.1200) x 100
13.33%
12%
8%
III
I
II

It is observed from the above analysis that foam Divisions Manager argument I correct when we
look at the calculation given above which shows that even though contribution margin ratio of
Foam Division is lower, the divisions ranking is higher based on the Net Contribution Ration.

The use of contribution approach for reporting is more realistic for assessing the performance of
various divisions as it considers variable and traceable costs only and avoids common costs while
finding out profitability. This approach enables the management to rightly interpret the
information. Further, pricing of internal transfers at market price will give due credit to specific
profits centre i.e. transferor.

The desired rate of return is 28% on investments. Investments include: (i)
Fixed assets after depreciation
Ans. 23
(ii) Net working capital.
In the question, current assets and debtors are given but current liabilities and creditors are not
indicated. Therefore, these are assumed to have nil value.
Investments

Fixed assets

5,00,000
Net working capital

Rs.

Current assets

3,00,000

Debtors

2,00,000

5,00,000
Total investments 10,00,000
The desired rate of return is 28%
The profit margin will be Rs. 280000
Budgeted volume 400000unit
Rs.
Profit margin per unit (Rs. 280000 400000 units) 0.70
Fixed cost per unit 2.00
Variable cost per unit 10.00
190
Transfer price per unit 12.70


Ans.24 (i) Profit =20% return on average assets employed
Average assets (Rs.Lakhs)
Sundry Debtors
Inventories
Plant & equipment
Total
2
5
5
12

Profit =Rs.12,00,000 x 20 /100 =Rs.2,40,000
(2) Budgeted sales revenue (2,00,000 units of component X) (Rs.Lakhs)
Fixed cost
Variable cost (2,00,000 units @ Rs.1)
Profit
Total Sales
5.00
2.00
2.40
9.40

Selling price per unit of component X =Rs.9,40,000/2,00,000 units =Rs.4.70 per unit
Options in hand with Division A
Option 1 -Sell 1,50,000 units in market and transfer 50,000 units to Division B
Option 11 -Sell only 1,50,000 units in market.
Statement of profitability of Division A under two options (Rs.)
Particulars Option-I Option-II
Sales (1,50,000 units @ Rs.4.70)
Transfer to Division-B (50,000 units @ Rs.2)
Total Sales revenue
Less: variable overhead
Contribution
Less: Fixed Cost
Profit (a)
Capital employed (b)
Return on capital employed (a)/(b)X100
7,05,000
1,00,000
8,05,000
2,00,000
6,05,000
5,00,000
1,05,000
12,00,000
8.75%
7,05,000
-
7,05,000
1,50,000
5,55,000
4,75,000
80,000
10,00,000
8%

Analysis : From the analysis of the above it is observed that under Option-I, Division As, Profit
and ROCE is increased by Rs.25,000 and 0.75% respectively. Hence Option-I is suggested for
Division-A.

Ans. 25
(i ) The company as a whole will not benefit if Division C bought the component from an outside supplier at
Rs.135/- per unit.
Rs.
Purchase cost from outside supplier 1,35,000
(1,000 units Rs.135 per unit)
Less: Saving in variable cost of division
A by reducing Divisions output 1,20,000
(1,000 units Rs.120 per unit)
Net cost (benefit) to the company as a whole 15,000
The company as a while will not benefit, as it will be required to incur an additional cost of Rs.15,000 if
Division C bought the component from outside supplier.
191
(i i ) The company will be benefited if C purchased the component from an outside supplier and Division A uses
the facilities for other activities.
Rs. Rs.
Purchase cost from outside supplier 1,35,000
(1,000 units Rs.135)
Less: Saving in variable cost of Division A for the units
purchased by Division C from outside
1,20,000
(1,000 units Rs.120 per unit)
Cash operating saving of Division A for the use of facilities
for other activities
18,000
Net cost (benefit) to the company as a whole
1,38,000

It is advisable that Division C should purchase the component from outside sources as this decision will
benefit the company by Rs.3,000.
(3,000)
(i i i ) The company will be benefited if C purchase the component from an outside supplier and there is no
alternative use of Division As facilities.
Rs.
Purchase cost from outside supplier 1,15,000
(1,000 units Rs.115)
Less: Saving in variable cost of Division A by reducing divisions
output
1,20,000
(1,000 units Rs.120)
Net cost (benefit) to the company
.
It is advisable that the Division C should buy the component from outside as this decision will benefit the
company by Rs.5,000.
(5,000)

Ans 26 (i) Working notes:
1. Contribution per hour of Super-chips and Okay-chips:
Super-chips Okay-chips
Selling price per unit (Rs.) 600 120
Less: Variable cost per unit (Rs.) 300 80
Contribution per unit (Rs.) 300 40
Hours required per unit 2 0.5
Contribution per hour 150 80
(Rs.300/2 hrs) (Rs.40/0.5 hrs)
2. Details of hours utilized in meting the demand of 15,000 units of Super-chips and utilizing the
remaining hours for Okay-chips out of available hours of 50,000 per annum:
Rs.
Hours utilized for manufacturing 15,000 units of Super-chips 30,000
(15,000 units 2 hours)
Hours utilized for manufacturing 40,000 units of Okay-chips
(40,000 units 0.5 hours)
20,000


3. Contribution of a process control unit (using an imported complex circuit board):
50,000
Rs.
192
Selling price per unit: (A) 1,400
Variable costs
Circuit board (Imported) 600
Other parts 80
Labour cost (5 hours Rs.100) 500
Total variable costs: (B) 1,180
Contribution per unit (Rs.) : [(A) (B)] 220
4. Contribution of process control unit (using a Super-chips):
Rs.
Selling price per unit: (A) 1,400
Variable costs
Super-chip 300
(Material + Labour costs)
Other parts 80
Labour (6 hours Rs.100) 600
Total variable costs: (B) 980
Contribution per unit (Rs.) : [(A) (B)] 420
5. Incremental contribution per unit of a process control unit, when instead of using imported
complex circuit board Super-chip is used:
Rs.
Incremental contribution per unit (Rs.420 Rs.220) (Refer to W. N. 3&4) 200
(ii) Super-chips to be transferred to Mini Computer Division to replace Circuit Boards:
Out of 50,000 available hours 30,000 hours are utilized for meeting the demand of 15,000 unit of
Super-chips, the rest 20,000 hours may be used for manufacturing 40,000 Okay-chips, which yields a
contribution of Rs.40 per unit or Rs.80/- per hour (Refer to working note 1) or a contribution of Rs.160
per two equivalent hours.
In case the company decides to forego the manufacturing of 20,000 units of Okay-chips in favour of
5,000 additional units of Super-chips to be used by Mini-Computer division (instead of complex
imported Circuit Board) for manufacturing process control units. This decision would increase the
existing contribution of Mini-computer Division by Rs.200/- per two-equivalent hours (Refer to working
note 5).
Hence the entire requirement of 5,000 units of Super-chips be produced and transferred to Mini-
Computer Division.
(ii) Minimum transfer price of Super-chip to Mini Computer Division:
Variable cost of a Super-chip + Opportunity cost of foregoing the production
of an Okay-chip and using craftsmen time for
Super-chip
= Rs.300 + 2 hours Rs.80
= Rs.460
(iii) Super chips to be produced for the production of 12,000 units of process control units:
After meeting out the order of 15,000 Super-chips per year, the concern is left out with 20,000 hours.
Use of Super-chips for control units production would increase the existing contribution of Mini-
Computer Division by Rs.200/- per unit. Out of the remaining 20,000 craftsmen hours, 10,000 units of
Super-chips can be made, which may be used for the production of 10,000 process control units.

Ans 27
193
(i ) Statement of the overal l profi t of the company
(By harvesti ng 2,000 kgs of oi l seeds, processi ng i t i nto edi bl e oi l & sel l i ng the same i n 2 kg cans)
Harvesting
Division
Oil Mill
Division
Marketing
Division
Total Rs.
Output of each
department
2,000 kgs of
oil seed
1,000 kgs. of
oil produced
500 cans of 2
kg each

Total costs
Variable cost (Rs.) : (A) 5,000 10,000 1,875 16,875
(2,000 kgs
Rs.2.50)
(1,000 kgs
Rs.10)
(500
Rs.3.75)

Fixed cost (Rs.): (B) 10,000 7,500 4,375 21,875
(2,000 kgs
Rs.5)
(1,000 kgs
Rs.7.50)
(500
Rs.8.75)

Total cost (Rs.): (C) =
[(A)+(B)]
15,000 17,500 6,250 38,750
Sales revenue (Rs.): (D) 75,000
(500 cans Rs.150)
Profit (Rs.) [(D) (C)] 36,250
(i i ) Working note:
(a) Total Contribution = (Sales revenue total variable cost)
= Rs.75,000 Rs.16,875 = Rs.58,125
(b) Amount of shared contribution in relation to variable costs:
Harvesting Division = Rs.58,125
Rs.16,875
Rs.5,000
= Rs.17,222
Oil Mill Division = Rs.58,125
Rs.16,875
Rs.10,000
= Rs.34,445
Marketing Division = Rs.58,125
Rs.16,875
Rs.1,875
= Rs.6,458
Computation of Transfer Price (for internal transfers) under the following pricing methods:
(1) Shared contribution in relation to variable costs:
Transfer price from harvesting Division to Oil Mill Division
= Variable cost of Harvesting Division + Shared contribution of Harvesting Division in relation to
variable costs
= Rs.5,000 + Rs.17,222 (Refer to working note 2) = Rs.22,222
Transfer price from Oil Mill Division to Marketing Division
= Transfer price from Harvesting Division to Oil Mill Division + Variable cost of Oil Mill
Division + Shared contribution of Oil Mill Division in relation to variable costs
(Refer to working note 2)
= Rs.22,222 + Rs.10,000 + 34,445
= Rs.66,667
(2) Market price:
Transfer price from Harvesting Division to Oil Mill Division
= Market price of 2,000 kgs of Oil seeds transferred to Oil Mill Division
194
= 2,000 kgs. Rs.12.50 = Rs.25,000
Transfer price from Oil Mill Division to Marketing Division
= Market price of 1,000 kgs of edible oil
= 1,000 of kgs Rs.62.50 Rs.62,500
(i i i ) Statement of profi tabi l i ty (under di fferent transfer pri ces method)
From Harvesting
Division to Oil
Mill Division
From Oil Mil to
Marketing Division
From Marketing
Division to market
(500 cans of 2
Kgs.)
Rs. Rs. Rs.
Shared contribution method
Transfer price: 22,222 66,667 75,000
(Refer to (1) above)
Less: Transfer price __ 22,222 66,667
(Refer to (ii) above)
Less: Variable cost 5,000 10,000 1,875
Less: Fixed cost 10,000 7,500 4,375
(Refer to (i) above)
Profit 7.222 26,945 2,083
Market price method
Transfer price 25,000 62,500 75,000
(Refer to (2) above)
Less: Transfer in price __ 25,000 62,500
(Refer to (ii) above)
Less: Variable cost 5,000 10,000 1,875
(Refer to (ii) above)
Less: Fixed cost 10,000 7,500 4,375
(Refer to (i) above)
Profit 10,000 20,000 6,250
Deci si on: Divisional Manager of Harvesting Division would prefer the use of market price method for
transferring 2,000 kgs of oil seeds to Oil Mill Division because its usage increases the profit by Rs.2,778
(Rs.7,222) over the shared contribution method.
Whereas Oil Mill Division manager would prefer the use of shared contribution method over the market
price method because its use would increase its profit by Rs.6,945 (Rs.26,945 Rs.20,000). Similarly
Marketing Divisional Manager would be benefited to the extent of Rs.4,167 (Rs.6,250 Rs.2,083) by using
market price method.

Ans 28
(i ) Statement of profi tabi l i ty of Di vi si on X
No. of components Transfer price for the
component to
Department Y@
Rs.90 per unit
Total cost of
components (Rs.)
Profit / (Loss) (Rs.)
(a) (b) (c) (d) = {(b) (c)}
5,000 4,50,000 5,62,500 (1,12,500)
10,000 9,000 9,00,000 __
195
15,000 13,50,000 12,37,500 1,12,500
20,000 18,00,000 15,75,000 1,25,000
25,000 22,50,000 19,12,500 3,37,500
30,000 27,00,000 22,50,000 4,50,000
Statement of profitability of Division Y
No. of
Components
Sale
revenue on
average
price basis
Component
cost
(Transfer
price) to
Dept. Y
Manufacturing
cost in
division Y
Total cost Profit/(Loss)
Rs. Rs. Rs. Rs. Rs.
(a) (b) (c) (d) (e)={(c)+(d)} (f)={(b)-(e)}
5,000 19,68,750 4,50,000 14,06,250 18,56,250 1,12,500
10,000 29,85,000 9,00,000 16,87,500 25,87,500 3,97,500
15,000 37,12,500 13,50,000 19,68,750 33,18,750 3,93,750
20,000 41,70,000 18,00,000 22,50,000 40,50,000 1,20,000
25,000 45,00,000 22,50,000 25,31,250 47,81,250 (2,81,250)
30,000 45,00,000 27,00,000 28,12,500 55,12,500 (9,90,000)
(i i ) Profi tabi l i ty of the company as a whol e
(a) At 30,000 units level, at which Division Xs net profit is maximum Rs.
Profit of Division X 4,50,000
Profit of division Y
Operating profitability / (Loss) of the company
(9,00,000)
(b) At 10,000 units level, at which Division Ys net profit is maximum Rs.
(5,40,000)
Profit of division X NIL
Profit of division Y
Operating profitability of the company
3,97,500
(i i i ) Profi tabi l i ty of the company, i f i t i s not organi sed on profi t centre basi s
3,97,500
No. of
components
Sales
revenue on
average
basis
Cost of
component
to division X
Manufacturing
cost in
division Y
Total cost Profit/
(Loss)
(Rs.) (Rs.) (Rs.) (Rs.) (Rs.)
(a) (b) (c) (d) (e)={(c) +
(d)}
(f)={(b)(e)}
5,000 19,68,750 5,62,500 14,06,250 19,68,750 -
10,000 29,85,000 9,00,000 16,87,500 25,87,500 3,97,500
15,000 37,12,500 12,37,500 19,68,750 32,06,250 5,06,250
20,000 4170,000 15,75,000 22,50,000 38,25,000 3,45,000
25,000 45,00,000 19,12,500 25,31,250 44,43,750 56,250
30,000 45,22,500 22,50,000 28,12,500 50,62,500 (5,40,000)

The level of output, the company will earn maximum profit, if the company is not organized on profit centre basis
is 15,000 components.
196

Ans.29. Statement showing contribution P.U. of ranking (Rs.)
Particulars Product
A B C D
Market Price P.U.
Less: Variable Production Cost P.U
Contribution P.U.
Labour hours P.U.
Contribution per labour hour (i)/(ii)
Ranking
150
130
20
3
6.67
IV
146
100
46
4
11.5
III
140
90
50
2
25
I
130
85
45
3
15
II

(i) Allocation of 20,000 labour hours
C (2,300 units x 2 L.H.)
D (1,600 units x 3 L.H.)
B (2,500 units x 4 L.H.)
A (Balance) (200 units x 3 L.H.0
4,600
4,800
10,000
600
20,000

Product D can be transferred to Division Y, but the maximum Quantity that might be required for
transfer is 2,500 units of D.
Time required for 2,500 units of D =2,500 units x 3 L.H =7,500 L.H
2,500 units of Product D for Division Y can be met by sacrificing as follows:
(Labour hours)
Product A (200 units x 3 L.H.)
Product B (Balance) (1,725 units x 4 L.H.)
600
6,900
7,500

Transfer price to be charged by Division Z to Division y on supply of 2,500 units of product
D.
(Rs.)
Variable cost (2,500 units x Rs.85)
Add: opportunity cost of contribution foregone
Product A (200 units x Rs.20)
Product B (1,725 units x Rs.46)
Transfer Price
Transfer Price P.U. (Rs.2,95,850 / 2,500 units)
2,12,500

4,000
79,350
2,95,850
118.34

(ii) Allocation of 30,000 Labour Hours

C (2,300 units x 2 L.H.)
D (1,600 units x 3 L.H.)
B (2,500 units x 4 L.H.)
A (2,800 units x 3 L.H.)
Idle Labour (Balance)
Total
4,600
4,800
10,000
8,400
2,200
30,000

2,500 units of Product D for Division Y can be met by sacrificing as follows:
Idle labour hour
Product A (1,725 units x 3 L.H.)
Total
2,200
5,300
7,500

Calculation of transfer price (Rs.)
197
Variable cost (2,500 units x Rs.85)
Opportunity cost of Contribution foregone of Product A (1,767 units x Rs.20)

Transfer price P.U. (Rs.2,47,840 / 2,500 units)
2,12,500
35,340
2,47,840
99.14

Ans. 30
Working Notes:

(i) Hours required to meet maximum demand:

External sales Hours reqd. Total Hrs. per unit

(i) (ii) (iii) = (i) (ii)

X 800 units 3 2,400
Y 500 units 4 2,000
Z 300 units 2

600
Total
(ii) Contribution per unit:
5,000

Product X Y Z

Rs. Rs. Rs.
Selling price 48 46 40
Less : Variable cost 33 24
Contribution per unit : (A)
28
15 22 12
Labour hours required per unit : (B) 3 4 2
Contribution per hour (Rs) : (A) / (B) 5 5.5 6
Ranking III II I

(a) If only 3,800 hours are available in Division A.

300 units of Z (maximum), which will take* 600 hrs.
500 units of Y (maximum), which will take 2,000 hrs.
400 units of X to use remaining hrs. 1,200 hrs.
3,800 hrs

.
*Note: Labour hours required per unit are given in the question. If 300 units of Y are to be
transferred to B division, then 1,200 hours will have to be used for production of
Y instead of X. It means Division A will sacrifice production of
400 units of X, which are yielding Rs. 5 per hr. Given above is the optimum
mix for Division A for 3,800 hrs. If 300 units of Y are to be transferred to B division
with time constraint of 3,800 hours, then additional 300 units of Y will have to be
produced sacrificing the production of 400 units of X which is yielding contribution.

Transfer price Rs.
(i) Variable cost of Y 24.00
Opportunity cost
(ii) Contribution relating to X forgone for
producing additional units of Y
(4 hrs Rs. 5*) 20.00
*Y takes 4 hours and in each hour production of X would have generated contribution of
44.00
198
Rs. 5.

(b) If 5,600 hours are available
Maximum time required to meet
external sales (Refer to Working note 1) 5,000 hrs.
Hours now available 5,600 hrs.
(i) It means 600 hrs can be easily used for the production of Y and transfer price will be variable
cost only
i.e. (600 hrs. 4 hrs) Rs. 24 Rs. 3,600
Note: Y takes 4 hours per unit
(ii) For producing additional 150 units,
production of X will be disturbed.
Variable costs

(i) 150 units of X @ Rs. 24 Rs. 3,600
Opportunity cost
(ii) Contribution of X units
forgone (600 hrs. Rs. 5) Rs. 3,000*

6,600
Total price for 300 units 10,200
Average transfer price should be Rs. 34 per unit
*Contribution per hr. of X forgone.

Ans.31. (1) Maximum hours required to meet the present outside market requirement
Maximum sales units Hours required per
unit
Total hours
Vx
X1
Xt
900
300
600
3
2
4
2,700
600
2,400
Maximum total hours required to meet the outside market requirement
5,700

(2) Contribution per unit, per hour and ranking (Rs.)
Product V X X
Selling price per units
Less: Variable cost per unit
Contribution per unit
Labour hours required per unit
Contribution per hour
Ranking
24
17
7
3
2.33
II
23
12
11
2
5.5
I
20
14
6
4
1.5
III

(3) Utilisation of 4,800 available hours according to ranking (hours)
300 units of products X1 (300 units x 2 hours)
900 units of products Vx (300 units x 3 hours)
375 units of products Xt (300 units x 4 hours)
Total hours
600
2,700
1,500
4,800

(a) computation of transfer price for each unit of Vx if total labour hours available in
Department x are 4,800
According to the ranking 4,800 available hours are utilized to produce 300 units of X 900
units of Vx and 375 units of X. The aforesaid product mix would give rise to optimum mix
for optimum profit.



199


In case 400 units of Vx are to be supplied to Department y in addition to existing outside sale
then the production of product X is to be curtailed partially and the hours thus obtained will
be utilized for the production of 400 additional units of Vx. The new product mix will be as
follows:
(Hours)
300 units of products X1 (300 units x 2 hours)
1,300 units of products Vx (1,300 units x 3 hours)
75 units of products Xt (75 units x 4 hours)
Total hours
600
3,900
300
4,800

Computation of transfer price per unit (Rs.)
Variable cost of one unit of Vx
Contribution foregone (opportunity cost) per unit due to the curtailment of Xt(3
hours x Rs.1.5)
Transfer price per unit
17.00

4.50
21.50

(b) Computation of transfer price for each unit of Vx, if total labour hours available in
Department x are 6,200
Hours required to meet the present outside market requirement 5,700
Remaining hours available for producing 400 additional units of Vx 500
After meeting the present outside market requirement (6,200 hours -5,700 hours)
Computation of transfer price per unit: (Rs.)
Total variable cost on the production of 166.67 units of Vx
(500 hours / 3 hours) @ Rs.17 per unit by utilizing 500 remaining available
hours
Total variable cost of 233.33.units of Vx @ Rs.17 per unit
(400 units 166.67 units) produced by curtailing the production of Xt
product to the tune of 700 hours.
Contribution foregone (opportunity cost ) on the diversion of 700 hours of
Production of Xt for producing 233.33 units of Vx (700 hours x Rs.1.50)
Total cost for producing 400 additional units of Vx
Transfer price for one unit of Vx (Rs.7,850 / 400 units)
2,833


3,967


1,050

7,850
19,625

Ans. 32
(a) When component is purchased by Division B from outside (Rs.)
(i) Statement of contribution
Division A
Division B Sales (2000x 400)
Less: Cost of Purchase (2000x 200) 400000
Variable costs (200x 150)
Companys total contribution
300000

8,00,000

3,80,000

Nil


1,00,000
1,00,000

(b) When component is purchased from Division A by Division B (Rs.)
Division A
Sales (2000x 220)
Less: Variable costs (2000x 190)
Division B
Sales (2000x 400)
Less: Variable Costs:
Purchase cost in Division A (2000x 220) 440000
Variable cost in Division B (2000x 150)
Companys total contribution
300000

4,40,000
7,00,000

8,00,000


7,40,000



60,000




60,000
1,20,000
200
Qty.
Unit
Rate
Rs.
Value
Rs.
Q t y
Unit




Sale in open market 2,400 12.00 28,800

9,600 2
Transfer to painting shop 9,600 12.00 1,15,200
Total sales : (A) 12,000

1,44,000

9,600




Thus, it will be beneficial for the company as a whole to ask Division B to buy the component
from Division A.
(ii) Statement of total contribution if Division A could be put to alternative use:
Division A:
Contribution from alternative use of facilities
Division B:
Sales (2000x 400)
Less: Variable costs:
Cost of purchase (2000x 400) 400000
Division B (2000x 150)
Companys total contribution
300000



8,00,000


7,00,000



30,000



1,00,000
1,30,000


Since, the companys contribution when component is purchased from outside, shows as
increase of Rs.30,000 as compared to when there is inter departmental transfer. Hence, it will be
beneficial to purchase the component from outside.


(iii) Statement of total contribution when component is available from outside at Rs.185.
Division A:
Division B:
Sales (2000x 400)
Less: Variable costs:
Cost of purchase (2000x 185) 370000
Division B (2000x 150)
Companys total contribution
300000


8,00,000


6,70,000

Nil




1,30,000
1,30,000


If the component is purchased by Division B from Division A, the contribution is only
Rs.1,20,000 as calculated under
(2) above. Hence it will be beneficial to buy the component from outside.
(v) Fixations of transfer price
(a) When there are no alternative uses of production facilities of Dept. A:
In such a case the variable cost i.e. Rs.190 per component will be charged.
(b) If facilities of Division A can be put to alternative uses: (Rs.)
Variable cost
Opportunity cost
Transfer price
190
15
205


(c) If market price gets reduced to Rs.185 and there is no alternative use of facilities of
Division A. the variable cost of Rs.190 per component should be charged.

Fastners Limited
Ans. 33
(a) Present profitability of individual shops and overall profitability

Particulars Welding shop Painting shop









201
Less: Variable cost : (B) 1,14,000 (9600 units Rs.20) 1,92,000
(12,000 units 9.50)
Contribution : {(A) (B)}

30,000

48,000
Less: Fixed cost 25,000 30,000
Profit

5,000

18,000
Overall profit for the company (Rs. 5,000 + Rs. 18,000) = Rs. 23,000
(b) (i) When painting shop purchases all its requirement from open market at a price
of Rs. 10 per unit
Welding shop Painting shop

Qty. Unit Rate
Rs.
Val
ue
R

Q t y
Unit
Rate
Rs.
Value
Rs.
Sale 2,400 12.00 28,800

9,600 25.00 2,40,000
Less: Variable cost 2,400 9.50 22,800

9,600 18.00* 1,72,800
Contribution 6,000 67,200
Less: Fixed cost

25,000

30,000
Profit/(Loss)
(19,000) 37,200

Overall profit for the company Rs. 37,200 Rs. 19,000 = Rs. 18,200
*It is given in the question that cost of painting including transfer price from welding shop is Rs.
20 per unit. The transfer price from welding shop is Rs. 12 per unit. Therefore, the variable cost
of Rs. 8 (Rs. 20 Rs. 12) is incurred by painting shop exclusively. The painting shop will be
purchasing its requirement from open market at Rs. 10 per unit. Therefore, the variable cost per
unit in painting shop will be Rs. 18 (Rs. 10 + Rs. 8). This point should be noted carefully.
(b) (ii) When all the requirements of painting shop is met by transfer from welding shop at
a transfer price of Rs. 10 per unit
Welding shop Painting shop

Qty.
Unit
Rate
Rs.
Value
Rs.
Qty
Unit
Rate
Rs.
Value
Rs.
Sale in the open

market 2,400 12.00 28,800

9,600 25.00 2,40,000
Transfer to painting

shop 9,600 10.00 96,000

Total sales 12,000

1,24,800

Less:Variable cost (12,000 unitsRs.9.50) 1,14,000 (9,600 unitsRs.18) 1,72,800
Contribution

10,800

67,200
Less: Fixed cost

25,000

30,000
Profit/(Loss) (14,200) 37,200

Overall profit of the company = Rs. 37,200 Rs. 14,200 = Rs. 23,000
For the purpose of comparison, the results of the three alternatives are summarised below:
Welding shop Painting shop
Rs. Rs.
Profit under (i) 5,000 18,000
Profit/(Loss) under (b)(i) (19,000) 37,200
Profit/(Loss) under (b)(ii) (14,200) 37,200
Rs.
The overall profit under (a) 23,000
202
Sales Centre (S) Rs.

Rs.

Rs.
New Board Sold
Selling price 35,000
Purchase price 29,000
Gross margin

6,000
b(i) 18,200
b(ii) 23,000
Alternative (b)(ii) should be accepted due to the following reasons:
(a) It gives a maximum overall profit of Rs. 23,000. The discussion is confined to either b(i) or
b(ii).
(b) Each shop is treated as a separate cost centre and not a profit centre.
(c) The policy of overall goal congruence of the company is followed.
Neither selling price nor total sales is given. Division A of Better Margins Ltd. expects a return
of 25% on average assets employed i.e., Rs. 12,00,000.
Ans. 34
Total sales will be: Rs.

(a) Profit (25% of 12,00,000)
(b) Fixed overhead
(c) Variable cost (2,00,000 Re. 1)
Total sales 9,00,000
Sales per unit (Rs. 9,00,000

2,00,000 units) Rs. 4.50



Transfer to Division B Sale to outside and
sale to outside parties parties only

Sales (units) 2,00,000 1,40,000


Sales value (1,40,000 units @ Rs. 4.50)
(60,000 units @ Rs. 2.25)


Less: Variable cost

Rs.
6,30,000
1,35,000
7,65,000


Rs.
6,30,000
Nil
6,30,000
(Re. 1 per unit) 2,00,000 1,40,000
Contribution 5,65,000

4,90,000
Less: Fixed overhead 4,00,000

3,60,000
*

Net profit 1,65,000

1,30,000
Average assets employed 12,00,000 10,00,000
Return on investment 13.75% 13.00%
If the component is transferred to Division B as well as sold to outside parties, it is more
profitable as the contribution, net profit and return on investment is more than the existing
proposal. Therefore selling the components to Division B at Rs. 2.25 per unit is in the overall
interest of the company.
*Reduction in selling and administration expenses (fixed in nature) by Rs. 40,000.
Statement showing the contribution to profit for each assuming that all estimates and budgets
materialised as expected
Ans. 35








203



















(ii) Assuming Additional Costs It is noticed that all estimates and budgets are materialised except
that repairs undertaken by R took an extra 10 hours and Rs. 100 of materials due to a problem not
noticed by B or R.
R is responsible for giving correct repair costs and, therefore, he has to bear the additional cost:
Rs. Rs.
Repair Centre (R)s contribution

540
Less: Extra cost of materials 100

Extra D.L. variable cost (10 hrs Rs. 6) 60

160
Revised contribution

380
However, full details are not given in the question. B is a middleman passing on Rs costs to S
and as such should not bear additional costs. Had the item been noticed originally then S would have
paid the cost and perhaps it should be passed back. This would be particularly so if R had
insufficient opportunity for a complete inspection. In that case extra cost should be:
Rs.

Material 100

Labour (10 hrs. Rs. 15) 150

250

Reduced contribution of S = Rs. 3,800 Rs. 250 = Rs. 3,550

Rs. Original
contribution of R 540
Add.: Saving in variable cost

[10 hrs (Rs. 15 Rs. 6)] 90

Increased contribution of R 630

Note: Other solutions are equally acceptable if well argued and logically justified.

(a) (i) AB sells product at external market
Ans. 36:
Selling price (Rs.) 30 45 60
Less Variable cost 18 18 18
Contribution (per unit) 12 27 42
Demands (units) 60,000 40,000 20,000
Total contribution 7,20,000 10,80,000 8,40,000

Optimal output is 40,000 units at a selling price of Rs.45

AB transfer at Rs.42 to XY division then contribution of XY
204
Selling price (Rs.) 120 135 150
Less Variable cost V+TP
(42+60) 102 102 102
Contribution (per unit) 18 33 48
Demands (units) 15,000 10,000 5,000
Total contribution 2,70,000 3,30,000 2,40,000

Manager will choose out put level 10,000 units at a selling price of Rs.135.

Overall profit when transfer made at Rs.42
Division AB contribution on 10,000 units [42 (18 -3)] = 2,70,000
Division XY contribution 10,000 (135 102)
Total contribution = 6,00,000
= 3,30,000
Division AB contribution from external market sale
Total profit
10,80,000

16,80,000
(ii) AB transfer at variable cost
Selling price (Rs.) 120 135 150
Less Variable cost (15+60) 75 75 75
Contribution (per unit) 45 60 75
Demands (units) 15,000 10,000 5,000
Total contribution 6,75,000 6,00,000 3,75,000

Optimal is 15,000 units at the rate of 120 per unit.
If AB transfer at Variable cost (Rs.15) then no contribution will be generated by AB division

XY division choose 15,000 units level gives contribution 15,000 45 = 6,75,000
Division AB contribution from external market sale
Total contribution
= 10,80,000

= 17,55,000
(iii) Contribution AB division by selling 10,000 units to new external market at Rs.32
and XY division purchasing at Rs.31.

Contribution (32 18) 10,000 = 1,40,000
XY contribution [135 (31 + 60)] = 4,40,000
Division AB contribution from external market sale
Total contribution
= 10,80,000

= 16,60,000
Ans. 37

(a) The variable costs per unit of output of sale outside the company are Rs.11 for the
intermediate product and rs.49(Rs.10 for A+Rs.39 for B) for the final product. Note that selling
and packing expenses are not incurred by the supplying division for the transfer of the
intermediate product. It is assumed that the company has sufficient capacity to meet the demand
at the various selling prices.
Optional output of intermediate product for sale on external market.
Selling Price (Rs.)
Unit contribution (Rs.)
Demand (units)

20
9
15,000
30
19
10,000
40
29
5,000
Total contribution (Rs.) 1,35,000 1,90,000 1,45,000

Optimal output is 10,000 units at a selling price of Rs.30.
Optimal output for final product
Selling Price (Rs.)
Unit contribution (Rs.)
Demand (units)

80
31
7,200
90
41
5,000
100
51
2,800
205
Total contribution (Rs.) 2,23,200 2,05,000 1,42,800
Optimal output is 7200 unit at a selling price of Rs.80.
Optimal output of Division B based on a transfer price of Rs.29.
Division B will regard the transfer price as a variable cost. Therefore, total variable cost per unit
will be Rs.68(i.e.,29+39) and Division Bs contribution will be as follows:
Selling Price (Rs.)
Unit contribution (Rs.)
Demand (units)

80
12
7,200
90
22
5,000
100
32
2,800
Total contribution (Rs.) 86,400 1,10,000 89,600

The manager of Division B will choose an output level of 5,000 units at a selling price of Rs.90.
This is sub-optimal for the company as a whole. Profit for the company as a whole from the sale
of the final product are reduced from Rs.2,23,200 (72,00 units) to Rs.2,05,000 (5000 units).
Rs.2,05,000 profits would be allocated as follows:

Division A Rs.95,000 (5000 units at Rs.19 i.e.,Rs.29-Rs.10)
Division b Rs.1,10,000

(b) At a transfer price of Rs.12 the variable cost per unit produced in Division B contribution will
be as follows:

Selling Price (Rs.)
Unit contribution (Rs.)
Demand (units)

80
29
7,200
90
39
5,000
100
49
2,800
Total contribution (Rs.) 2,08,800 1,95,000 1,37,200

The manager of Division B will choose an output level of 7200 units and a selling price of
Rs.80.This is the optimum output level for the company as a whole. Division A would obtain a
contribution of Rs.14,400 (7200 units @ Rs.2 (I.e.,Rs.12-Rs.10) from internal transfers of the
intermediate product whereas Division B would obtain a contribution of Rs.2,08,800 from
converting the intermediate product and selling as a final product. Total contribution for the
company as a whole would be Rs.2,23,200. Note that Division A would also earn a contribution
of Rs.1,90,000 from the sale of the intermediate product to the external market.

Ans. 38:

Opticals Ltd manufactures P( lenses) and Q ( swimming goggles ).

Division P has option to supply to Division Q or sell to outside market.

Division Q has option to buy from Division P or purchase from outside market.

However, both divisions have to work within their individual capacity.

Variable Cost for product P in Division P = Rs 60.

Variable cost for product Q in Division Q ( excluding 2 Nos P's) = Rs 80.

Division P has better market price of its product P than the market price offered to Q division.

For maximizing profit of the organization :

Rs
P division should optimise its profit by selling maximum units to outside market.

Contribution per unit for sale to outside for division P

40
Contribution per unit for Div Q as follows :

Sale price - Variable cost ( excluding lenses)

330
Max Contribution per unit ( if procured from P div at its variable cost i.e Rs 60)

210
206
Min Contribution per unit ( if procured at Rs 90 per unit from outside)

150
Contribution per unit at transfer price of Rs 70 i.e minimum market price

190
Option 1 : Division Q buys 5001 units from market @ Rs 70 and meets its capacity. Division P
sells 3000 units to outside market @ Rs 100

Sale / Transfer
Contrib.

/unit
Contribution in
thousand rupees
207

Rs P Div Q Div Total
DivP :Sale of 3000 units to outside market @ Rs 100 40 120

120
DivQ: Sale of 2500 units with P from market @ Rs 70 190

475 475
Less : cost of rejection of one unit of product P

-0.07 -0.07
Total

120 474.93 594.93
Option 2 : Division P sells 3000 units to outside market, transfer 4000 units to div Q and
Division Q buys 1000 units from outside market to work within the capacity

P Division agrees to a transfer price so that profitability of Q is not affected. To maintain the same
profitability of Q, contribution required from 2000 units for Div Q is Rs 400,000 i.e contribution per unit
Rs 200 i.e transfer price per unit of P is Rs 65 per unit to make cost of lences Rs 130

Sale / Transfer
Contrib
/unit
Contribution in
thousand rupees
208

Rs P Div Q Div
Total

Div P : Sale of 3000 units to outside market 40 120

120
Div P : Transfer of 4000 units to div Q at Rs 65 5 20

20
Div Q :Sale of 2000 units with P from P div @ Rs 65 200

400 400
Div Q : Sale of 500 units with P from market @ Rs 90 150

75 75
Total

140 475 615
Under Option 1, both divisions worked dis-jointly without caring for capacity utilization resulting
lower profitability of the organization.

Under Option 2, both divisions worked with mutual advantages for optimizing their
individual profits and overall profit for the organization has gone up by effective
utilization of capacity.
Product P from Division P fetches higher price from open market indicating good quality of
product. Moreover, supply from P division is well assured in the long run which is the
justification of establishment of two parallel divisions.
Hence, Option 2 is
suggested.

(ii) Division functioning as profit centers strive to achieve maximum divisional
profits, either by internal transfers or from outside purchase. This may not
match with the organisations objective of maximum overall profits. Divisions
may be commercial to advice overall objects objectives, where divisional
decisions are in line with the overall best for the company, and this is
goal congruence. Div isions at a disadvantage may be given due weightage
while appraising their performance. Goal incongruence defeats the purpose of
divisional profit centre system.
(b) In an assignment minimization problem, if one task cannot be assigned to one
person, introduce a prohibitively large cost for that allocation, say M, where M
has a high the value. Then, while doing the row minimum and column
minimum operations, automatically this allocation will get eliminated.


(a) Div A B B
Ans. 39

Rs. / unit Rs. / unit Rs. / unit
Direct Material (Other than A) 50 24
Direct Labour 25 14
Variable Overhead (Production) 20 2
Variable Production Cost (excl. A) 95

40 40
From A 144
From Outside ____ 160
Variable production Cost / unit

184 200
Selling Price
From outside 160 300
Less: Selling Overhead 13 26
209
Net Selling Price (outside) 147

274

Net Selling Price to B 144

Net Selling Price to S

250



Net Selling Price (outside)


147



274



274
Variable Production Cost 95 184 200
Contribution / unit (outside) 52

90

74

(Sale to B & S respectively)

144


250


250
Variable Production Cost 95 184 200
Contribution / unit 49

66

50

Best strategy
A = Maximise Production; Sell maximum no. of units @ 18,000 52 = 9,36,000
52 / unit (outside)


(To B) remaining units 2,000 49 = 98,000
Total Contribution for A

10,34,000

Best strategy for B:

Maximise contribution / unit by selling outside and procuring from A 90 / unit

Contribution 2,000 units
Balance units can yield contribution of either 74/ unit for outside or Rs. 50 / unit to S Ltd.
Production Capacity = 28,000.

Option I Option II
Outside Sales Sales to S Outside Sales contribution /

unit
20,000 74 = 14,80,000 6,000 50 = 3,00,000 24,000 74 = 17,76,000
2,000 90 = 1,80,000 2,000 90 = 1,80,000
16,60,000 3,00,000

Total Contribution (16,60,000 + 3,00 ,000)19,60,000 19,56,000

(B) Choose Option I i.e. get 2,000 units from A, sell 6,000 units to S and 20,000 to outside.
Make 28,000 units @ full capacity. Total Contribution Rs19,60,000.
If A and B are allowed to act independent of the group synergy,
Rs.

Total contribution A 10,34,000
B 19,60,000
Total contribution for X Ltd. 29,94,000

Cost from X Ltd. s Perspecti ve

Variable Cost of production Div A Rs. 95
Div B
Variable cost of production other than A 40 40
A supplied by Division 95
A Variable Cost
210
A purchased ____ 160
135 200

Option I Outside 26,000 units Option II

Outside 20,000 (274 135)

27,80,000 20,000 (274 135) 27,80,000
2,000 (274 200) 1,48,000 6,000 (274 200) 4,44,000
22,000

S Ltd. 6,000 units (250 200) 3,00,000 _________

32,28,000

32,24,000

Choose Option I

Contribution = Rs. 32,28,000 for X Ltd. as a whole

Transfer (2,000 units)

Make A transfer all output to B. Sell 6,000 units of B to S and 22,000 units to out side market.
This will make X Ltd. better off by 32,28,000 29,94,000 = Rs 2,34,000
(i.e. 18,000 units of A sold to outside increases contribution to A by 3 Rs. / unit and decreases
contribution to B by 16 Rs. / unit Net negative effect = 13 18,000
= Rs.2,34,000).

Ans. 40: (i) Division As best strategy 2011
Maximum Manufacturing capacity = 50,000 units

Per unit External
Market
Spl order Transfer to B
partially
Transfer to
B full
Demand (units) 30,000 15,000 < 45,000 45,000
Selling price 65 55 55 60
Variable Prod cost 35 35 35 35
Variable Selling cost 10 - - -
Total Variable cost 45 35 35 35
Contribution Rs. 20 20 20 25

Transfer to B in full gives maximum contribution. Hence, 45,000 units to be
transferred.

Balance 5000 will be sold to the external market.

Partial fulfilment of Special order will not be possible.

Statement of profitability for best strategy in 2011 :


Rs
Transfer 45000 units to B @ Rs 60 Per Unit : Contribution : 25 x 45000 11,25,000
Supply to external market : Contribution : 20 x 5000 units 100000
Total Contribution 12,25,000
Annual fixed cost Rs 4,30,000
Step fixed cost Rs 2,00,000
Fixed selling costs Rs 50,000



6,80,000
Profit in 2011 5,45,000
(ii) Companys best strategy for 2010

211
For Division A External
Market
Special
Order
B Partial B - Full
Variable cost 45 35 35 35
Price 65 55 45 50
Contribution to Division A 20 20 10 15
Margin for Division B 5 0

It is clear from the above table that the Company will have more profitability if A
first satisfies external market demand and special order and then supply to B.

As quantity for special order and transfer is more than 10,000 units, Div A will
always opt for fixed cost of 50,000 instead of variable selling cost of Rs 5 / unit.

The companys strategy for Division As production, sales/ Transfer will be :


External
Market
Special
Order
B
Partial
Total
Strategy I : As sale/ transfer (units) 25,000 10,000 5,000 40,000
Contribution of A & B ( Rs laks) 5.00 2.00 0.75 7.75
Fixed Cost Rs Lakhs( 4.30 +
1.00 +0.5)
5.80
Net for company Rs lakhs 1.95
Strategy II : from A ( units) 25000 10000 15000 50,000
Contribution of A & B : RS Lakhs 5.0 2.00 2.25 9.25
Fixed Cost Rs Lakhs ( 4.30 +
2.00 +0.5)
6.80
Net for company Rs. Lakhs 2.45

Thus, the strategy II will be the one for the Company for the year 2010.



(iii) B s negoti ating range i n 2011 :

Upper limit: The effective price of Rs. 60 for procurement from outside source.
Lower Limit : Minimum price A will look for
i.e Variable cost + Maximum possible contribution from other source +
additional fixed cost

= Rs ( 35 + 20 + ( 50000/45,000) = Rs 56.11

Thus, Price range for negotiation without changing As strategy is Rs 56.11 to
Rs 60 per unit.

(a) (i) Contribution per unit against sale to outside = Rs ( 200-120-20) = Rs 60
Ans.: 41
In case of transfer, good units and rejected units are in proportion of 9:1
In case of transfer, contribution per good unit = Rs (190 120) = Rs 70
In case of transfer, contribution per rejected unit = Rs (150 120-100) = Rs -70
Thus, effective contribution per unit of transfer = Rs (70 x 0.9 70x 0.1) = Rs 56

As contribution per unit against outside sale is higher, the best strategy should be to
sell maximum number of unit to outside marker.

Contribution from outside market from sale of 900 units = Rs 54,000 {Rs.(900 x 60)}

212
Contribution from transfer of 300 units to B {Rs (300 x 56)} = Rs 16,800
Total Contribution from best strategy = Rs 70,800

(ii) If Bs demand is 540 units, total production required (540 /0.9) = 600 units.

Taking outside market demand of 600, it is within production capacity of 1200 units.
Now contribution from 600 units of outside sale Rs (600 x 60 ) = Rs 36,000
Contribution from rejected 60 units Rs (60 x 70) =
= Rs 31,800
Rs (4,200)

To keep same level of contribution as in (i), the contribution required from transfer of
540 unit to B (Rs 70,800 31,800) = Rs 39,000

Thus, contribution required per unit Rs 39,000 /540 = Rs 72.22
Hence price to be charged p. u. against transfer to
B Rs (120 + 72.22) = Rs 192.2

Alternative Solution:
Let x be the number of units sold outside and y be the number of units sold to B,
before B returns 10% as defectives.

Then, x + y = 1,200, is the limitation on production capacity of A.

Department A
Outside to B
Rs. Rs.
Selling Prices 200 190
Variable Cost Production 120 120
Variable Cost Sale ___20 ___
Total Variable Cost 140 120
--
Contribution 60 70
Contribution on x units sold outside = 60x
Out of y units to B, 10% =
10
1
y. 1 = .1y is returned to A. If A scraps, amount got =
30 per unit.

If A reworks and sells, it gets 150 100 = 50/unit.
Decision to reworks all defectives. i.e. (.1) (y)
Contribution on good units of B = 0.9y 70 = 63y
Contribution on reworked units of B = (.1) (y) 50 = 5y
Amount of material lost on manufacture of defectives to B =12y(.1)(y)120
Contribution on y gross units transferred to B = 56y
63y + 5Y 12y
Total contribution earned by A = 60x +
56y
Where x + y = 1200

To maximize contribution, maximize units sold outside.
900 units sell outside.

Balance 3001,200 units (gross transfer to B, of which B gives back defectives)

Contribution: Rs.60 (900) + Rs.56 (300)
= Rs.54,000 + Rs.16,800
213
Contribution = Rs.70,800
Fixed Cost
(i) Profit
= Rs.36,000

= Rs.34,800
(ii) Outside demand = 600 units
Contribution = 600 Rs.60 = Rs.36,000
Balance to be got

= Rs.34,800

= Rs.70,800
Out of Rs.34,800, defectives of B will give
Rs. 3,000 60 50
Rs. 31,800 charge to B for 540 units

Contribution to be obtained from 540 units of B = Rs. 31,800
Add: Production cost of 600 units @ 120/-
Amount changed for 540 units
= Rs. 72,000

= Rs.1,03,800
Price to be charged to B = 1,03,800540 = 192.22

Per good unit transferred, to maintain the same level of profit as in (a).

Ans 42: B will not pay A anything more than 13, because at 13, it will incur additional cost of
Rs.2/- to modify it, 13 + 2 = 15, the outside cost.
A B C
Outside
sale
Transfer
to B & C
Divisional variable cost of
production
Transfer from A
Modification
Total Variable Cost of production
Selling Price
Contribution
7 7 19


13
2
25


13
7
15
8
7
13
6
34
40
6
38
50
12


Option for C, Purchase all units from A @ 13: Any other option is costlier.


A B C
214
Maximum external demand
Exiting capacity
Maximum capacity that can
be added
Total maximum that can be
produced
Additional fixed cost on
expansion
Units that must be
sold/transfer to get this
amount as contribution
External demand not
covered by existing capacity
Decision
3,750
5,000
5,000


10,000


24,000


24,0006
= 4,000

-


Expand make
10,000 units
3,750 outside
3,750 B
2,500 C
5,000
2,500
1,250


3,750


6,000


6,0006
= 1,000




Expand make
2,500 + 1,250
= 3,750 units
4,000
2,500
2,250


4,750


18,700


18,7006
= 1,558.33




Do not expand
make only 2,500
units.

A B C
Outside
sale
Transfer
to B & C
Units



Contribution / unit
Contribution (Rs.)


Additional Fixed Cost
Net revenue addition
3,750



8
30,000
3,750 +
2,500 =
6,250
6
37,500
3,750



6
22,500
2,500



12
30,000
67,500
24,000
22,500
6,000
30,000
-
43,500 16,500 30,000
Individual strategy is the companys best strategy.
215
Ans. 43




216

Manager of divi sion X wil l sell 14,000 uni ts outside at 110 Rs. per uni t and earn contribution of Rs.
3.50 lakhs.

Excess capacity of 6,000 units can be offered to Y at a price between 70 (the variable
manufacturing cost at X) and Rs. 95 (the maximum amount to equa l outside
contribution). But Y can get the material outside @ 85. So, y will not pay to X anything above
(Rs.85 6) = Rs. 79 to match external available price.

X will be attracted to sell to Y only in the range of 71 79 Rs. per unit at a volume of
6,000 units.

At Rs. 70, X will be indifferent, but may offer to sell to Y to use idle capacity.

Z will not buy from Y at anything above 135. If X sells to Y at 70 per unit, Y can sell to Z at 134
and earn no contribution, only for surplus capacity and if units transferred by X to Y at Rs. 70
per unit.


Y Z


Provided X sells to
Y at Rs. 70 per unit
Sell 4,000 units to Z at 134
(Indifferent)
Buy 4,000 units from y at
134 (attracted)
Sell 4,000 units to Z at 135 (willingly
for a contribution of Re. 1)
Indifferent, since market
price is also 135
For buying from X at 71 79 price range, Y will be interested in selling to Z only at prices
136 143, which will not interest Z.

Thus Y will sell to Z only if X sells to Y at Rs. 70 per unit and Y will supply to Z maximum
4,000 units.

Ans. 44: Capacity of X division = 7000 units

X has the following option to sell following number of units:

Option Domestic Market Export Transfer Hiring out (equivalent unit)
I 6000 800 200

II 5000 800 l200

III 5000

2000

IV 5000 800 400 800
According to the condition given in (iii) for procurement policy of Y,

For 7000 units, maximum amount Y is agreeable to pay at market rate i.e Rs 900 per unit

= 7000 x Rs 900 = Rs 63,00,000

If X transfers l200 units to Y, It has to incur expenses for 5800 units from market =

= 5800 x Rs 920 = Rs 53,36,000

It means for l200 units from X, Y will pay = Rs ( 63,00,000 53,36, 000)

217
= Rs 9,64,000 = Rs 803.33 per unit

If X transfers 2000 units to Y and Y buys 5000 units,, Y can pay to X only

= Rs ( 63,00,000 5000 X 920) = Rs l7,00,000 = Rs 850.00 per unit

If transfer of less than l000 units to Y, X can claim transfer price of Rs 900 per unit


Realization ( Rs)
Option I 6000 x l000 + 800 x 900 + 200 x 900 Rs 69,00,000
Option II 5000 x ll20 + 800 x 900 + l200 X 803.33 Rs 72,84,000
Option III 5000 x ll20 + 2000 x 850 Rs 73,00,000
Option IV 5000 x ll20+ 800 x 900 + 400 x 900 plus
contribution from hiring out
Rs 66,80,000 plus
Above table shows that Option III is preferable in comparison to Option I and II .
If Option III for X, transfer price will br Rs 850.00 per unit.

For taking a decision on option IV, contribution from equivalent unit from hiring out has to
be compared with contribution from minimum sales realization of Rs 775 because sales
realization of Rs 775 per unit from equivalent 800 units gives the amount of Rs
6,20,000 which makes up the gap between option III and option IV. In that case, transfer
price will be Rs 900 per unit.



218

Decision Making
Answer: 11

1. Material A is not yet owned. It would have to be purchased in full at the replacement cost of `6.00 per
unit. Relevant cost is therefore 1,000 units at the replacement Cost.
2. Material B is used by the Company regularly. There is already existing a stock of 600 units. If these
are used in the contract, a further 400 units would have to be purchased.
3. Material C: 1,000 units of material C are required. 700 units are already in stock. If it is used for the
contract, a further 300 units will have to be purchased at a replacement cost of `4.00 each. The
existing stock of 700 units will not be replaced. If they are used for the contract, they cannot be used
@ `2.50 each unit. The realisable value of these 700 units @ `2.50 per unit represent opportunity
cost.
4. Material D is already in stock and will not be replaced. There is an opportunity cost of using D in the
contract. It has following two uses:
It can be sold to fetch `1,200 i.e. 600 X `2
It can also be used for E, which would cost `1,500 i.e. 300 X `5.
Since substitution is more useful, `1,500 is the opportunity cost.

Summary of Relevant Costs:

`
Material A 1,000 units X `6 6,000
Material B 1,000 units X `5 5,000
Material C 700 units X `2.5 1,750
300 units X `4 1,200
Material D 300 units X `5 1,500
Other expenses 550
Total Relevant Cost

16,000
Contract should be accepted since offer is of `22,000 in relation to relevant Cost of `16,000.


Answer: 12

Retain Present
Machine
Buy New
Machine
Relative
Benefit of
Replacement
Variable Costs: (20,000 units @ `0.30 for 3
Years
18,000 12,000 (6,000)
Sale Proceeds of Old Machine - (4,000) (4,000)
Capital Cost of New Machine - 7,000 7,000
18,000 15,000 (3,000)

Thus, it is advantageous to replace the equipment.
Note. Depreciation charge and loss on sale of old machine should be ignored for this decision.


Answer: 13

Relevant costs of producing one unit of the finished product
`
Cost of material M (realisable value) 80
Cost of labour (Being sunk cost) 0
Out-of-pocket expenses 30
110
Allocated overhead is not relevant for the decision. The customer should be charged `110 per unit.

Answer: 14

219
(i ) The down payment of `2,50,000 represents a sunk cost.

The lost profit from subletting the shop of `1,20,000 per annum arrived as: (18,000 12) 96,000 =
1,20,000 is an example of an opportunity cost.
The salary amount is not given is also an opportunity cost lost.

(i i ) The relevant information for running the shop is:

(`)
Net Sales 22,20,000
Less: Costs (22,02,000 2,50,000) (sunk cost excluded for decision making purpose) 19,52,000
Gross Margin 2,68,000
Less: Opportunity cost from subletting 1,20,000
Profit 1,48,000
As profit is more than opportunity cost, the most profitable decision is to carry on business in the shop.

Ans. 15: Analysis of Cost and profit:
`(lakhs) `(lakhs)
Direct material 3.60
Direct labour
Prime cost 10.00
6.40
Overhead:
Variable factory overhead 2.20
Fixed factory overhead 2.60
Administration overheads 1.80
Selling commission 1.00
Fixed selling overheads 0.40
Total cost 18.00
8.00
Profit
Rate of profit on costs (2/18) = 1/9
2.00

Overhead absorption rate based on direct wages = (8.00 / 6.40) 100 = 125% of direct
wages

Break up of new order: `
Direct Materials 36,000
Direct Labour 64,000
Overheads 125% of direct wages 80,000
Total costs 1,80,000
Profit 1/9 20,000
Selling Price 2,00,000
The following points emerge:

(i) Factory overheads only are to be recovered on the basis of direct wages.
(ii) The special order is a direct order. Hence commission is not payable.
(iii) The budgeted sales are achieved. Hence all fixed overheads are recovered. Hence, no
fixed overheads will be chargeable to the special order.

Based on the above, the factory variable overheads recovery rate may be calculated as
under:

Total variable factory overheads `2.20 lakhs
Direct wages `6.40 lakhs
220
Factory overhead rate = (2.20 / 6.40) 100 = 34.375%

Applying this rate the cost of the special order will be as under:
`
Direct materials 36,000
Direct labour 64,000
Overheads 34.375% of direct wages 22,000
Total costs 1,22,000
Price offered 1,50,000
Margin 28,000 (more than 1/9)

Hence, the order is acceptable at the price of `1,50,000.

Answer: 16
Statement of minimum price which the company can afford to
quote for the new customer
(based on relevant cost)
Cost to be incurred to bring the equipment in its original condition. 29,700
Opportunity cost of the direct material 2,250
Direct wages:
Dept. A : 15 man days `120 1,800
Dept. B : 25 man days `100 2,500
Opportunity cost of contribution lost by department B (`2,500 `2.30) 8,000
Variable overheads 1,075
25% (`1,800 + `2,500)
Delivery costs 1,350
Supervisory overtime payable for modification 1,050
Control device to be used in another job (Refer to working note 1) (10,350)
Net loss on material cost savings, in the original equipment (Refer to
working note)
11,700
Opportunity cost of remaining materials which can be sold as scrap 11,400
Opportunity cost of sale drawings
Total minimum price which may be quoted
1,500
Working notes:
61,975

1. Cost of control device to be used in another job:
`
Cost of control device 10,500
Less: Dismantling & removal cost of control mechanism 120
(1 man day `120)
Less: Variable cost )25% `120)
Balance cost of control device
30
10,350
2. Net loss on material cost saving of equipment:
Loss on material cost saving of equipment 12,000
Less: Conversion cost
(2 man days `120) 240
Less: Variable overheads (25% `240)
Net loss on material cost saving of equipment
60
11,700
221


Answer: 17
Working Notes:
1. The book value of Material K `40,000 is a sunk cost and is not relevant for decision making.
2. The Scrap Value of Material K `10,000 will affect the cashflow and is relevant.

Alternative I
Relevant Costs

(`)
Material A (Replacement Cost) (600kgs. X `70) 42,000
Direct Labour Skilled (200 hrs X `6) 12,000
Contribution Lost (Opportunity Cost) (2000 X `2) 4,000
Unskilled (not relevant) -
Variable Overheads 2,000
Total Relevant Cost 60,000

Cost per unit = `60000 / 500 units = `120 p.u.
Selling Price = `150 p.u.
Profit = 500 units (`150 `120) = `15,000

Alternative II
1. The Cost of substitute material `8,000 is relevant.
2. The regular profit of a job `6,000 is not relevant.

Analysis: From the above analysis it is suggested to convert the materials into a specified product.

Answer:18
Working Notes:
1. Relevant cost of labour
Grade : Nil, labour cost for Grade 1 labour as it will not be affected by
the decision.
Grade 2 : `20 per hour
2. Relevant cost of material
Material A : `100 per unit, the replacement cost because the material is
widely used.
Material B : `250 per unit, the net realisable value, being the opportunity
cost.
3. Statement of loss of contribution from the reduction in the sale of product Y.
` `
Sales revenue per unit: (A) 700
Variable cost per unit
Grade 2 labour: (4 hour `20) 80
Materials relevant variable costs 120
Variable production overheads: (B) 120
(4 hours `30)
320

Contribution per unit: [(A) (B)] 380
222
Loss of contribution from the reduction in sale of 5,000
units
19,00,000
(5,000 units `380)
Less: Avoidable fixed factory overhead cost
Net Loss
5,90,000

Relevant costs and benefit analysis from the acceptance of the contract.
13,20,000
(`000)
Sales revenue: (1) 20,000
(20,000 kgs. `1,000)
Relevant costs:
Labour:
Grade 1 NIL
Grade 2 2,400
(20,000 kgs. 6 hours 20)
Material A (20,000 2 units `1,000) 4,000
Material B (20,000 kgs. 1 litre `250) 5,000
Variable production overhead (20,000 kgs. 8 hours
`30)
4,800
Total variable cost 16,200
Incremental fixed costs 2,280
18,480
Add: Loss of contribution on product Y
(Refer to working note 3)
Total relevant cost: (ii) 19,800
Excess of relevant revenue over relevant cost: 200
Advice to A Limited: to accept the contract, as it will enhance the pre-tax operating income by `2,00,000

Answer: 19:Working Notes:
Calculation of contribution margin
The company expects that each per cent point increase in on-time performance will result in revenue increase
of `18,000 p.a.
Additional revenue increase = `18,000 X 10 = `1,80,000
Contribution margin on additional revenue = `1,80,000 X 45/ 100 = `81,000

Costs incurred annually on the installation of new scheduling and tracking system (`)
Additional annual cost 1,50,000
Interest Foregone on Fixed
deposit
(Opportunity Cost) (10% X `2,00,000) 20,000
Total Costs 1,70,000

Expected Savings in costs on the installation of new scheduling and tracking system (`)
Contribution margin from additional annual revenue (45% X `1,80,000) 81,000
Decrease in variable costs due to reduced numbers of
carton lost
(3,000-1,000) X `50 1,00,000
223
Total savings in cost 1,81,000
Net saving (1,81,000 - 1,70,000) 11,000

Suggestion: The expected savings are more than annual costs, hence it is suggested to install a new
scheduling and tracking system.


Answer: 20
Statement showing Revised Cost Estimates:
1. Steel Sheets (`12/kg. x 5,000kg.) `60,000
2. Steel Rods (1,000 kg. @ `17 kg.) 17,000
3. Bearing, hardware items, etc. 15,000
4. Labour Cost Nil
5. Overheads:
Fabrication Shop (500 hrs @ `25) 12,500
Welding Shop (300 hrs @ `16) 4,800
Planning engineers cost Nil
Design engineers cost
Total Estimated Relevant Cost
Nil

1,09,300
Relevant costs are estimated future costs pertinent to a decision.
Imputed costs do not form part of relevant costs. All costs accumulated for stock valuation purposes may not
be relevant cost.
Reasons for Variation in the Cost Elements
1. Current rate of steel sheets is quite relevant. Past rate of `12 per kg has no impact on the decision
and therefore not adopted in the cost estimates.
2. Steel rods purchased five years ago cannot be used (non- moving) and as such it represents sunk
cost. This material can now be substituted for alloy steel rods (`17/kg). Alloy rods are cheaper than
steel rods and therefore relevant to the decision.
3. Fixed costs are past costs, not relevant to the decision. Labour costs are fixed in nature.
4. It is assumed that Fabrication Shop is working at optimum level. Therefore rate charged from
outsiders (`25 per hour) is relevant.
5. It is assumed that Welding shop is not working at full capacity. Therefore variable cost of `16 per
machine hour is adopted.
6. Planning and design engineers costs are fixed cost and, therefore, irrelevant.


Answer: 21
Revised Cost Estimate

1. Direct Material:
- Paper 2,500
- Ink 3,000 5,500
2. Direct Labour (Skilled)
Normal (250 hrs x `4) 1,000
Overtime (125 hrs x `1) 125 1,125
3. Variable Overhead (350 hrs x `4) 1,400
4. Printing 600
Revised Cost Estimate 8,625

Working Notes:

1. With no alternative use, the paper would not be replaced; the alternative, therefore, being to scrap the
stock receiving proceeds of `2,500.
2. The surplus ink could not be used or sold and therefore the whole cost of the ink purchased should be
charged to the cost of the programme.
3. The direct employees are currently usefully employed, therefore, their wage cost is being recovered
from an existing customer. Before, transferring them to the work on the programme, the ability of the
programme work to bear this cost must be determined.
224
4. The overtime premiums are directly caused by the programme work, which should be able to bear this
additional cost.
5. There is no additional cost associated with the employment of the unskilled labour.
Current idle time 200 hrs
Printing Work 75 hrs

(No additional cost)
Week-end work 25 hrs
125 hrs
Paid time off 50 hrs
The 50 hrs of paid time off is more than covered by the 125 hrs of idle time, which is also paid for and,
therefore, there is no additional cost.
6. Variable overhead is the incremental cost.
7. The variable overhead and other variable costs associated with running the printing press have been
separately dealt with. The additional recovery required is, therefore, the lost contribution associated
with 200 printing press hours.
8. Fixed production overheads are not associated with incremental cashflows, and therefore should be
ignored.
a) The cost of estimating time is a small cost, since it has already been incurred. It does not involve
incremental cash flow. Therefore, it has been ignored.
b) In short-term decision making, resources usage is best measured by using variable cost which
change in proportion to changes in output. When variable cost is matched with the sales revenue
with which it is associated, the resulting difference or contribution gives a good indication of the
expected benefit to the organisation of any course of action. If fixed assets are unaffected by a
decision, contribution will be close approximation of cash flow and therefore, it is very real figure
which may also be usefully used as a basis for ranking alternatives where limiting factors are
involved.
c) For evaluating the economic benefit derived from a product, it is necessary to match the revenue
generated with he cost incurred. Opportunity cost represents the benefit forgone for taking one
course of action rather than alternative. It gives a measure of sacrifice made in order to generate
income. Conventional contribution approach normally extracts variable costs from the internal
costing records (i.e., stock accounts, etc.). Opportunity costs may be derived from internal or
external sources depending on such factors as whether there are alternative uses for internal
resources consumed and whether, if used, they would be replaced.

Answer:22:
Research Project


Particulars Relevancy Reason
Amount
(Rs000s)
Project cost till date Not relevant Sunk cost

Sale price of the project Relevant Incremental
revenue/opportunity gain
400
Cost of materials received Not relevant Sunk cost

Cost of disposal of materials Relevant Avoidable/opportunity cost 15
Cost of labour Not relevant Common costs

Contribution lost on
the alternative use
Relevant Opportunity cost
[Sales (Prime cost
labour)

(125)
Absorbed Fixed overheads Not relevant Sunk cost

Cost of Research Staff Relevant Incremental / out of pocket (160)
Redundancy and severance
pay
Not relevant Common costs

Share of General Not relevant Sunk costs
225
B ildi
Total incremental inflow
if the project is
proceeded with
130


Decision: Better to continue the project.

Answer. 23
Statement of cost of product NP (`)
Particulars Total cost
(100000
units)
Cost per unit
Direct materials
A (1,00,000 X 2.50) 2,50,000
B (60,000 X 1.00) 60,000
(40,000 X 3.00) 1,20,000
C (1,00,000 X 6.00)
Direct labour
6,00,000
Skilled (25,000 hrs X `3) 75,000
Unskilled
Opportunity loss (25,000 X `2)
Variable overhead (1,00,000 X 1.50)
50,000
Fixed Overheads:
Factory overheads: -
Addl. Overheads- Foreman 36,000
Supervisor
Depreciation: Type P 30,000
24,000
Type Q
Total Costs
5,000
profit
Sales




10,30,000



1,25,000
1,50,000



60,000

35,000
14,00,000
4,00,000
18,00,000




10.30



1.25
1.50



0.60

0.35
14.00.
4.00
18.00

Working Notes:
1. Cost of Direct Material
Material A- It is in regular use and hence replacement cost of `2.50 will be charged.
Material B- Total requirement is of 1,00,000 units:
Stock available 60,000 units - opportunity cost `1.00 each
40,000 units - purchase price `3.00 each.
Material C- Purchase price of `6.00

2. Cost of Direct Labour
Skilled Labour: (i) 1,00,000 units at `0.25 per hour
(ii) Loss of contribution on existing product opportunity cost 25,000 X
2=`50,000
Unskilled labour: Available in surplus and is to be paid even without work. Hence, not relevant
3. Cost of Additional Staff
(`)
Foreman
Supervisor
Total
36,000
24,000
60,000

4. Variable Overheads `1.50 per unit is relevant cost
5. Fixed Overheads Not relevant hence excluded
6. Depreciation
Type P: The machine is used on other product.
Hence, replacement cost is relevant
Depreciation =`1,60,000-1,30,000 =`30,000
Type Q: Since it can be sold if not used resale value is relevant.
Depreciation =`22,000-`17,000 =`5,000
226
7. Market Survey Costs: It is a sunk cost. Hence it is not a relevant cost.

Answer: 24
Working Notes:
1. Machine manufacturing cost
Costs of `50,000 incurred to date in manufacturing the machine is irrelevant for the decision, since It
is a sunk cost. The payment of `15,000 received from the customer prior to the liquidation is also not
relevant for decision making.
2. Material Cost.
The purchase cost of `6,000 of materials bought in the past is irrelevant for decision making. Only the
scrap value of materials i.e.`6,000 is relevant for decision making since it is the opportunity cost of
materials bought in the past.
3. Labour Costs.
Opportunity cost of labour when the workforce, is in short supply, and switched to another job,it could
fetch the additional contribution of (`30,000-`8,000-`12,000)=`10,000.
4. Consultancy fees (`)
Cost of completing the work
Cost of canceling the contract
Incremental cost of completing of work
4,000
1,500
2,500

5. General Overheads
The general overheads are absorbed on allocation and therefore, these costs are not relevant for the
decision.
Statement showing economics of proposition (`)
Revenue from completing work
Less:
Materials (opportunity cost)
Labour: Actual costs 8,000
Opportunity costs
Cost of consultancy (Incremental cost)
10,000
Additional profit by accepting the offer of new customer in completion of the
work.


2,000

18,000
2,500

34,000




22,500
11,500

In view of incremental profit of `11,500, the offer of new customer can be accepted.

Answer: 25:
For solving this question, it is necessary to take the following into consideration. SV Ltd. Has two departments
A and B. Dept. A is manufacturing FLOTAP, but Dept. B is manufacturing the containers for this product. It
also stores this product. This is the existing situation. Now three alternatives are given.
Alternative 1.- Close Dept. B and manufacturing & storing may be given to PH Ltd.
Alternative 2 Continue Dept. B and manufacturing may be given to PH Ltd and storing to Dept. B.
Alternative 3 Continue Dept. B, Manufacturing may be done by Dept. B but storing may be given to PH Ltd.
Company should either select one of the alternative or continue the existing practice.
Working Notes:
(i) (`)
Direct Materials including germicide
Use of germicide (1/5
th
Direct materials other than germicide
of `6,00,000)
4,20,000
1,20,000
3,00,000

This material will be avoidable cost if Division B is to Close-down.
(ii) 10% of all materials = 10% of `3, 00,000
(a) Savings: `3, 00,000-`30, 000=`2, 70,000 if manufacture is given to PH Ltd. And storage is
with SV Ltd.
(b) Savings: 3, 00,000- 90% of `3, 00,000=`30, 000. If manufacture is done by SV Ltd and
storage given to PH Ltd.

(iii) (`)
Direct Labour cost
Less: Terminal benefit if B is closed
Avoidable cost, if Dept .B is closed (saving)
3,00,000
45,000
2,55,000
227

If manufacturing is given to PH Ltd. And SV Ltd. continues to store the product, saving on account of labour
retrenchment will be only `15,000.(It means in this alterative 3,00,000-15,000=2,85,000 will be spent any way
and avoidable cost will be only `15,000).
If manufacturing is done by SV Ltd. Then Labour force will continue. It means impact of labour cost in 3
rd


alternative will be nil.
(iv) Supervisory staff will be transferred to another department in the lst alternative. It means cash flow will not
be affected. In the second and third alternative, supervisory staff will be retained and it means no
additional cash flow or relevant cost due to decision.
(v) Depreciation does not affect the cash flow. Therefore it is not relevant for these decisions.

(vi) The hire charges of warehouse is `54,000 per annum. The remaining space of the warehouse is idle. It
means, when department B is closed, cash outflow of `54,000 will be avoided. Therefore `54,000(and not
`27,000) is the avoidable cost for this decision. If Department B continues, this expenditure of `54,000
continue. Therefore cash flow for alternatives 2 and 3 will not be isturbed on this account.

(vii) Maintenance of machine is required for manufacturing. If means `21,600 will be avoidable cost for
alternative 1 and 2.
In 3
rd

alternative this cost will continue to be there. Besides this machine will not be required in alternative
1 and 2. It will be sold at `1,50,000.It will be a one time cash inflow for alternatives 1 and 2.
(viii) Miscellaneous overhead of `94,500 will be avoidable cost for alternative 1. For 2
nd
alternative 80 % of
this i.e `75,600 will be avoidable cost. For 3
rd

alternative 20% of `94,500 i.e. `18,900 will be avoidable
cost.
(ix) Germicide- Stock: (`)
Stock in 2002
Used last year (1/5
th
Balance Stock
)
6,00,000
1,20,000
4,80,000

It is given that original price is `3,000

Therefore, `4,80,000/`3,000=160 tonne Germicide is there.

(x) Germicide-value
Alternative 1 : Storage is done by PH Ltd. Therefore it will be sold at `2,400 per tonne. Cash inflow
will be 2,400 X 160=`3,84,000.
Note that original price and replacement cost are irrelevant for the decision.


Alternative 2 : 10% of all material will be used. It means 90% of 160 tonne will be sold. Cash inflow
will be 160 X 0.90 X `2,400= `3,45,600

Alternative 3 : In this situation storage is done by PH Ltd. Therefore only 10% of whole quantity of
160 tonne will be sold in market at `2,400 per tonne . Cash inflow will be 16 X `2,400
`38,400.
(The replacement cost is irrelevant information in the question and it will be relevant only, when germicide has
competing demands.)
(xi) Machine is used for manufacturing of containers. It is not required in alternatives 1 and 2. Therefore , it will
be sold and there will be one time cash inflow of `1,50,000 under alternatives 1 and 2. Written down value
is irrelevant for decision under consideration.
(`)
Alternative 1 Alternative 2 Alternative 3
Division B
Manufacture of containers
Storage of product
Close
PH Ltd
PH Ltd.
Continue
PH Ltd.
SV Ltd.
Continue
SV Ltd.
PH Ltd.
Cash Inflows
(Including avoidable cost)






228
Direct materials other than germicide
Direct labour
Rent of a part of warehouse
Maintenance of machine
Miscellaneous overhead
Total avoidable cost p.a. (A)
Cash outflows
Contract fee to PH Ltd.
For Manufacturer
For packing and storage
Total outflow (B)
Net Cash outflow p.a. (A-B)-( C )
Total cash outflows for 4 years ( C X 4)
One time income
Sales of germicide
Sale of machine
Net cash outflow
3,00,000
2,55,000
54,000
21,600
94,500
7,25,100


7,50,000
9,00,000
1,50,000
(1,74,900)
(6,99,600)

3,84,000
1,50,000
(1,65,600)

2,70,000
15,000
-
21,600
75,600
3,82,200


7,50,000
-
7,50,000
(3,67,800)
(14,71,200)

3,45,600
1,50,000
(9,75,600)

30,000
-
-
-
18,900
48,900


-
1,50,000
1,50,000
(1,01,100)
(4,04,400)

38,400
-
(3,66,000)

Recommendations: All the alternatives result in net cash outflow. Therefore it is interest of SV Ltd. To
continue and to manufacture containers and store them in Division B.

Answer: 26:
Comparative Statement of Relevant Costs for use of own distribution division or use of Countrywide
distributions. (`000)
Particulars Own Distribution Countrywide Distribution
95-96 96-97 97-98 95-96 96-97 97-98
Relevant Cash outflow:
Operating Costs
Sub-Contract costs
Total
Less: Relevant cash inflow:
Sale of delivery vehicle
On 1-4-2002
On 31-3-2005
Net Relevant Cash outflows:
Total

2,100
-
2,100


-
-
2,100


2,100
-
2,100


-
-
+2,100


2,100
-
2,100


-
240
+1,860
=6,060

-
1,950
1,950


600
-
1,350


-
1,950
1,950


-
-
+1,950
=5,250

-
1,950
1,950


-
-
+1,950

Suggestion: From the above comparative statement it is observed that the net relevant cash outflow is more in
case of own distribution. Hence, selection of countrywide distributors is recommended. It is based on the
assumption that no portion of the common corporate cost of which `3,00,000 is apportioned to distribution
division which would be avoided even if, the distribution division is closed down.
(b) Reasons for reluctancy to accept countrywide distributors in distribution of Soft Drinks.
(1) Loss on Sale of Delivery Vehicles presently owned by the company: (`)

Cost of Vehicles (8 Vehicles on 1-4-2003)
Less: Depreciation for 2003-04
Book Value on 1-4-2004
Less: Sales realization (8 Vehicles X `75,000)
Book Loss on sale of Vehicles
19,20,000
4,20,000
15,00,000
6,00,000
9,00,000

6. Possibility of reduction in reported income as per Security Analysts recommendation
Forecast of operating income as per Security Analyst (`000)
Particulars 1995-96 1996-97
Estimated Profit when own distribution division is used
Net income if the offer of countrywide distributors is accepted
630
630
660
330

Working Notes: (`000)
Projected Profit for 95-96 660
229
Add; Depreciation avoided
Add: Saving in operating cost (`2,100- `1,950)

Less: Book loss on the disposal of delivery vehicles
Net income, if Countrywide distributors selected

420
150
1,230
900
330

Analysis: In view of short- run benefit, countrywide distributors can be opted. But when the long-run
benefits are recognized, and to focus on customer needs, the companys own distribution function is
recommended.

Answer: 27:
Statement showing value of total work undertaken by X Ltd. at customers price
(`000)
Material costs (for appliances covered under agreement) 825
[Rate to working note 1 (i)]
Material costs (for appliances not covered under agreement) 275
[Refer to working note 2 (i)]
Labor cost (for appliances covered under agreement) 1,000
[Refer to working note 1 (ii)]
Labour cost (for appliances not covered under agreement) 240
[Refer to working note 2 (ii)] _____
Total receipts 2,340
Break up of receipts:
Big appliances 60% 1.404
Small appliances 40% 936
Profitability Statement
(`000)
Option 1 Option 2 Option 3
Income
Big appliances 129.6
(60%`216)
1,404 1.404
Small appliances 936
.
86.4

936 (40%`216)
Total receipts: (A)
.
1,065.6 1,490.4 2,340
Costs:
Material 320 480 800
40%(825+275) 60%(825+275)

(825+275)
137.5% 137.5% 137.5%
Heat, rent, light etc. 125 50 150
Management costs 108 83 150
Service staff costs 230 440 750
Transport costs 25 220
Total costs: (B)
230
808 1,273
Profit: [(A) (B)]
2,080
257.6 217.4
Recommendation:
260
Option 3 is most profitable one.
Working Notes:
1. Material and labour cost (for appliances under after sales agreement):
230
`
(i) Cost of Material per unit charged to customers by X Ltd.
(`100 + 10% x `100 + 25% x `110) 137.50
Cost of material charged to customers by X Ltd.

|
.
|

\
|
Rs.10
Rs.60,000
`137.50 8,25,000
(ii) Cost of labour charged to customer by X Ltd.

|
.
|

\
|
Rs.10
0 Rs.1,00,00
`100 10,00,000
2. Material and labour cost (for appliances not covered under sales agreement):
`
(i) Cost of material charged to customer by X Ltd.

|
.
|

\
|
Rs.10
Rs.20,000
`137.50 2,75,000
(ii) Cost of labour charged to customer by X Ltd.

|
.
|

\
|
Rs.15
Rs.36,000
`100 2,40,000

Answer: 28
Statement of relevant cost of Mahila Griha Udyog Industries
If the contract is accepted/rejected
Decision Relevant costs (if
contract is
accepted) `
Relevant costs (if
contract is rejected)
`
Cash inflows
Contract price 18,00,000 -
Sale of material Y - 2,10,000
(Refer to working note I) .
Total cash inflows: (A)
.
18,00,000
Cash outflows
2,10,000

Material X substitute 1,35,000 -
(Refer to working note 2)
Adaptation required for the use of
obsolete material X
- 27,000
Material Z 3,00,000 -
Replacement of semi-skilled labour by
skilled labour
5,70,000 -
(Refer to working note 3)
Non-skilled labour cost 3,00,000 -
(Refer to working note 4)
Supervisory staff cost 35,000 -
(Refer to working note 5)
Avoidable overheads 1,25,000 -
231
(Refer to working note 6)
Total cash outflows: (B) 14,65,000 27,000
Net cash inflows: (A) (B) 3,35,000 1,83,000
The net benefit on accepting the contract is : `3,35,000 `1,83,000 = `1,52,000.
Conclusion
The contract should be accepted as it yields a net incremental cash inflow of `1,52,000.
Working notes:
1. Material Y will have to be paid for whether or not the contract is accepted, therefore its cost is irrelevant.
The relevant cost figure here is that which has an opportunity cost of `2,10,000. This means that the
company can resell material Y at this price.
2. Regarding material X, if the contract is accepted, alterative material will have to be purchased for the
other product at a cost of `1,35,000. If the contract is rejected material X will be adapted for a product not
included in the list of special range of namkeens at a cost of `27,000.
3. The relevant skilled labour cost of `5,70,000 is the extra cost to the company because of this contract. It is
the replacement cost of semi-skilled labour by skilled labour.
4. Non-skilled labour cost is the incremental cost of the contract.
5. If the company accepts the contract it will have to pay `35,000 for the two position that the supervisory
staff can replace.
6. Only `1,25,000 of avoidable overheads are relevant to this contract.

Answer: 29

M/s Ranka Builders
Statement of relevant costs on the
Acceptance of contract form Excel Ltd.
(Figure in lakh of `)
S.No. Particulars Basis for the
cost to be
relevant
Relevant cost if
contract is
accepted `
Irrelevant cost if
the contract is
accepted `
1. Land cost
(Refer to working note 1)
20
2. Drawings and design - 7
(Sunk cost)
3. Registration Incremental 10 -
4. Materials :
Cement and sand Replacement 8
Bricks and Tiles Opportunity 5
Steel Incremental 10
Others
(Refer to working note 2)
9
5. Labour :
Skilled Opportunity 2
Unskilled Incremental 8
232
Supervisors Salary 5
(Sunk Cost)
6. Overheads :
General Relevant
(avoidable)
4
Depreciation - 6
(Sunk Cost)
Replacement cost of
machine
7
7. Estimated profit foregone
on other project
Opportunity
foregone
10
Total 93

Decision : Since the offer price of contract is `1 crore and its total relevant cost is `93 lacs; these figures
clearly shows that the offer should be accepted.
Working notes :
1. `(Lacs)
Total cost of 3 grounds of land 60
Cost of ground of land will be borne by Excel Ltd. 40
Cost of 1 ground of land will be borne by M/s Ranka Builders 20
2. Others material cost is `10 lacs, it includes material worth `2 lacs, relating to interior decoration, which is
a sunk cost, this material can be sold for `1 lac, (which is a relevant opportunity cost) and `8 lacs,
material is an incremental cost. Hence total relevant cost of others material is `9 lacs. (`8 lacs,
incremental + `1 lac, opportunity cost).
3. Since the equipment can also be used on ths contract. Its current replacement price is `32 lacs, and after
one year its cost will be `25 lacs. Therefore the relevant opportunity cost of machine is : (`32 lacs `25
lacs).

Answer: 30
Alternative 1 (Conversion versus immediate sale)
` ` `
Sales revenue 900 units at `300 per unit (Refer
to working note 1)
2,70,000
Less: Relevant costs
Material XY opportunity cost (Refer to working
note 2)
21,000
Material A units @ `90 per unit (Refer to
working note 3
54,000
Material B 1,000 units @ `45 per unit (Refer to
working note 4)
45,000
Direct Labour :
Unskilled 5,000 hours @ `3 per hour 15,000
Semi-skilled Nil
Highly skilled 5,000 hours @ `11 (Refer to 70,000 55,000
233
working note 5)
Variable overheads 15,000 hours @ Re.1 (Refer
to working note 6)
15,000
Extra selling and delivery expenses 27,000
Extra advertising 18,000 45,000
Fixed advertising
2,50,000
Nil
(To remain same, not relevant)
Excess of relevant revenues
.

Alternative 2 (Adaptation versus Immediate Sale)
20,000

Saving on purchase of sub-assembly
Normal spending 1,200 units @ `900 per unit 10,80,000
Less: Revised spending 900 units @ `1,050
per unit (Refer to working note 7)
9,45,000 1,35,000
Less: Relevant costs:
Material XY opportunity cost (Refer to working
note 2)
21,000
Material C 1,000 units @ `37 (Refer to working
note 8)
37,000
Direct labour
Unskilled 4,000 hours @ `3 per hour 12,000
Semi-skilled Nil
Highly skilled 4,000 hours @`11 per hour
(Refer to working note 5, 6)
56,000 44,000
Variable Overheads 9,000 hours @ Re.1/- per
hour (Refer to working note 6)
1,23,000 9,000
Fixed overheads Nil
Net relevant savings
.


12,000
Evaluation :
The evaluation of two alternatives clearly shows that Alternative 1, yields higher net revenue of `8,000
(`20,000 `12,000). Hence because of higher net revenue of Alternative 1, it is advisable to convert material
XY into a specialized product.
Working notes :
1. There will be a additional sales revenue of `2,70,000 if Alternative 1 is chosen.
2. Acceptance of either Alternative 1 or 2 will mean a loss of revenue of `21,000 from the sale of the
obsolete material XY and hence it is an opportunity cost for both of the alternatives. The original purchase
cost of `75,000 is a sunk cost and thus not relevant.
3. Acceptance of Alternative 1 will mean that material A must be replaced at an additional cost of `54,000.
4. Acceptance of Alternative 1 will mean diversion of material B from the production of product Z. The
excess of relevant revenues over relevant cost for product Z is `180 (`390 `210) and each unit of
product Z uses four units of material B. The lost contribution (excluding the cost of material B which is
incurred for both alternatives) will therefore be `45 for each unit of material B that is used for converting
the obsolete materials into a specialised product.
234
5. Unskilled labour can be matched exactly to the companys production requirements. Hence acceptance of
either alternative 1 or 2 will cause the company to incur additional unskilled labour cost at `3 for each
hours. It is assumed that the semi-skilled labour will be able to meet the extra requirements of either
alternatives at no extra cost to the company. Hence, cost of semi-skilled labour will not be relevant. Skilled
labour is in short supply and can only be obtained by reducing the production of product L, resulting in a
loss of contribution of `24 (given) or `6 per hour of skilled labour. Hence the relevant labour cost will be `6
(contribution lost per hour) + `5 (hourly rate of skilled labour) i.e. `11 per hour.
6. It is assumed that for each direct labour of input, variable overhead will increase by Re.1 hence for each
alternative using additional direct labour hours, variable overheads will increase.
7. The cost of purchasing the sub-assembly will be reduced by `1,35,000 if the second alternative is chosen
and so these savings are relevant to the decision.
8. The company will incur additional variable costs, of `37 for each unit of material C that is manufactured,
so the fixed overheads for material C viz. `18/- per unit is not a relevant cost.

Ans. 31
Calculation minimum price to be quoted for a quotation, based on relevant costs only (`)
Opportunity cost of:
(1) Retaining materials already in the original machine
- Sale of Brass scrap
- Sale of Steel scrap
- Balance material , cost of scrapping )saved)
(2) Conversion materials
- Department M
- Department A

(3) Conversion work
(a) Department M
Labour 60,000
Variable overhead 12,000
Contribution foregone
(b) Department A
1,80,000
Labour Nil

Variable overhead 6,000
Off-loading cash flow foregone
(4) Sales proceed of design and specifications
57,000
(5) Incremental fixed overhead-cost of supervision
Minimum price to be quoted


1,00,000
25,000
(5,000)

12,000
3,000





2,52,000




63,000
60,000
10,000
5,20,000
Note: For the above minimum price of `5,20,000 profit can be added. The existing overheads are committed
costs and are not relevant for decision making.

Answer: 32
1. Value of Material X in stock : (which can be used as substitute for other materials)
= `54,000 X 90 / 100 = `48,600

2. Value of Material X for which firm order has been placed
= `76,000 X 90 / 100 = `68,400

3. Value of Material Y in stock
= 2 times x `62,000 = `1,24,000

4. Irrelevant Costs: Following costs are irrelevant therefore, they have been ignored
Site manage costs being fixed costs
Depreciation of plants
Interest on capital
Notional interest in estimated working capital
235
Head office expense allocated to contracts.

Comparative statement of Net Benefit resulting from each contract (`)
Particulars Contract
AX
Contract BX
Material X in stock 48,600
Material X firm orders placed 68,400
Material X not yet ordered (at current cost) 1,50,000
Material Y - in stock - 1,24,000
Material Z not yet ordered (at replacement
cost)
- 1,78,000
Labour to be paid 2,15,000 2,75,000
Travel and other expenses (future outflow) 17,000 14,000
Income from the hire of plant (15,000)
Penalty for rescinding the contract AX is
relevant
- 70,000
Total Cost 4,84,000 6,61,000
Contract Price 7,20,000 8,80,000
Expected net benefit 2,36,000 2,19,000

Advice- Since the expected net benefit of contract AX, is more than Contract BX, it is suggested to continue
with Contract AX.

Answer: 33:Relevant Cost of Jeet bicycle
Material 300.00
Labour 200.00
Variable Overhead (0.4 X 300) 120.00
Cost of Capital (0.15 X 6,00,000) / 25,000 3.60
623.60

If Star Bicycle company accept the offer of making Jeet for the chain stores the loss in contribution due to
sale of Smart is going down by 1,00,000 units is relevant, which causes a loss of `(899-300-200-120)= `279
The price of Jeet then should be `623.60 + 279 = `902.60. This is higher than the price of `800 as offered by
the chain store. So, the offer cannot be accepted.
Since the chain store has decided to launch a product like Jeet, it will do so whether or not Star Bicycle
Company accepts the proposal as there is excess capacity in the industry it will be able to do so. In that case,
the loss of contribution is `279 is not relevant and Star Bicycle Company can accept the proposal of the chain
store. Star Bicycle Company should have a closure look in the market condition and the chain stores ability to
get a replica of Jeet from other manufacturer before Star Bicycle Company reaches a final decision.

Answer 34:
Minimum recommended price per unit of 5,000 units of a product (obsolete model) of ACE Ltd.
(i) Historical cost of `11.50 per unit of 5,000 units of a product is irrelevant (as it is a sunk cost) for
determining the recommended price per unit.
(ii) If at all this model is sold in the market through normal distribution channels it will entail a variable selling
and distribution cost of `3 per unit.
(iii) If the stock is disposed off by asking someone to take them on as is where is basis, the company would
have to spend `5,000 over 5,000 units i.e. `1/- per unit.
In view of (ii) and (iii) the option of selling 5,000 obsolete units of the model using regular channels will nave a
differential cost of `2 (`3 Re.1) per unit.
Recommendation:
Hence, if the company can get anything more than `2/- per unit, then it is worthwhile to sell the stock of 5,000
units and earn an additional contribution.

236
Answer: 35
Statement of Increment Cost and Incremental Revenue
Capacity
in units
Unit cost ` Total cost
`
Incremental
cost `
Unit price
`
Total price
`
Incremental
revenue `
(a) (b) (c)=(a)(b) (d) (e) (f)=(a)(e) (g)
200 40 80,000 - 100 2,00,000 -
3000 35 1,05,000 25,000 95 2,85,000 85,00,000
(`1,05,000
`80,000)
(`2,85,000
-
`2,00,000)
4000 34 1,36,000 31,000 94 3,76,000 91,000
`1,36,000

1,05,000)
(`3,76,000

`2,85,000)
5,000 32 1,60,000 24,000 - - -
(`1,60,000

`1,36,000)

6,000 31 1,86,000 26,000 - - -
(`1,86,000

`1,60,000)

Decision:
At 4,000 units capacity told sales revenue is `3,76,000 and the total cost is `1,36,000 leaving a profit of
`2,40,000. The profit figure at this level clearly shows that the fixed expenses stand fully recovered. Hence,
we have to take incremental cost for further level levels of output. For an additional sales of 2,000 units
incremental cost is `50,000 (`1,86,000 `1,36,000) and the cost per unit is `25
|
.
|

\
|
units 2,000
Rs.50,000

Since the price quoted per unit is `28, which is more than `25, therefore, the order should be accepted.

Answer: 36
ABC Ltd is facing Direct material constraint and special steel plates are in short supply but the stock is
available only 500 M.T.
Alternatives available to maximize profit
Alternative I: - Manufacture and Supply only 20,000 cylinders at the risk of reduced order in future.
Alternative II: - Make 40,00 upper halves, buy 40,000 button halves from outside and supply 40,000
cylinders.

Profitability Statement

No. of Cylinders Alternatives
Differential
Cost
I
20,000
II
40,000
Sales Realisation @ `700 140 280 140
Welding and other costs @ 30 (6) (12) (6)
Transportation, loading etc. (at `5 per half) - (2) (2)
Net Differential Income 134 266 132

The additional net income when 40,000 halves are purchased is `132 lakhs which is the maximum price that
ABC Ltd. Can afford to pay keeping for itself at least the contribution it would earn by its own operation (a).

237
i.e. The Price = ` 120 Lakhs
40,000
= `330 per bottom half.

Answer: 37
Option 1: Profitability to continue only in season period (`)
Particulars Gift shop Restaurant Lodge Total
Incremental Revenue (i) 6,000 8,000 20,000 34,000
Differential cost:
Cost of Sales
Supplies
Electricity Charges
Total (ii)
Incremental revenue over differential cost (i)-(ii)
Less: Cost of advertisement
Net incremental revenue

3,300
300
40
3,640

2,360
4,400
800
160
5,360

2,640
-
1,600
400
2,000

18,000
7,700
2,700
600
11,000
23,000
12,000
11,000


Working Notes:
(1) Incremental revenue (`)
Gift shop (`48,000 X 10/80)
Restaurant (`64,000 X 10/80)
Lodge (`1,80,000 X 10/90)
Total
6,000
8,000
20,000
34,000

(2) Differential cost of sales (`)
Gift shop (`6,000 X 55/100)
Restaurant (`8,000 X 55/100)
Total
3,300
4,400
7,700

(3) Differential cost of supplies (`)
Gift shop (`6,000 X 5/100)
Restaurant (`8,000 X 10/100)
Lodge (`20,000 X 8/100)
Total
300
800
1,600
2,700

(4) Differential cost of Electricity Charges (`)
Gift shop (`900-`640) X 10/80)
Restaurant (`3,200 `1,920)X 10/80)
Lodge (`13,500-`9,900) X 10/90)
Total
40
160
400
600

Option 2: Profitability to continue throughout the year including season and off season periods (`)
Particulars Gift shop Restaurant Lodge Total
Incremental Revenue:
Season Period
Off Season period
Total (i)
Differential Cost
Cost of Sales
Supplies
Salaries
Electricity Fixed
Electricity- Variable
Total (ii)
Net incremental Revenue (i)-(ii)

-
34,200
34,200

19,800
1,800
9,600
1,280
240
32,720
1,480

-
45,600
45,600

26,400
4,800
9,600
3,840
960
45,600
-

-
80,000
80,000

-
12,800
40,800
13,800
3,200
70,600
9,400

-
1,59,800
1,59,800

46,200
19,400
60,000
18,920
4,400
1,48,920
10,880

Working Notes:
(a) Incremental Revenue in off season period (`)
238
Gift shop (`48,000 X 2 X 30 / 80X 95/100)
Restaurant (`64,000 X 2 X 30 / 80X 95/100
Lodge (`1,80,000 X 2 X 40 / 90X 50/100
Total
34,200
45,600
80,000
1,59,800

(b) Differential Cost of Sales (`)
Gift shop (`36,000 X 55/100)
Restaurant (`48,000 X 55/100)
Total
19,800
26,400
46,200



(c) Differential cost of supplies (`)
Gift shop (`36,000 X 5/100)
Restaurant (`48,000 X 10/100)
Lodge (`1,60,000 X 8/100)
Total
1,800
4,800
12,800
19,400

(d) Differential cost of Salaries (`)
Gift shop (`4,800 X 2)
Restaurant (`4,800 X 2)
Lodge (`25,200-`4,800) X 2)
Total
9,600
4,600
40,800
60,000

(e) Differential cost of Electricity (Fixed Element) (`)
Gift shop (`640 X 2)
Restaurant (`1,920 X 2)
Lodge (`6,900 X 2)
Total
1,280
3,840
13,800
18,920

(f) Differential cost of Electricity (Variable element) (`)
Gift shop (`900-`640) X 30 X2 /80)
Restaurant (`3,200 `1,920) X 30X 2/80)
Lodge (`13,500-`9,900) X 40 X 2 /90)
Total
240
960
3,200
4,400

Decision : By adopting the Option 1, the net increase in incremental revenue by `120 (i.e. `11,000-`10,880)
over the Option 2.Therefore, Option 1 is suggested to adopt.
Incremental profitability by adopting strategies of both advertisement insertions and operating during off
season period. (`)
Incremental Revenue with Advertisement
Incremental Revenue with the continue of operations during off season
Total incremental revenue
11,000
10,880
21,880
Therefore, both the strategies can be implemented simultaneously for increase of profitability of the
organization.


Answer: 38
(a) Consequences of undertaking: Nagpur & Delhi Contracts (`000)
Nagpur Contract Delhi Contract
Contract revenue
Sales of materials held for the Nagpur contract
(Note 1)
Saving in material purchases by alternative use of
materials of Delhi contract (Note2)
Hire of plant

Incremental costs:








170



48
2
220









180
24




204

239
Materials to be ordered (Note 3)
Project managers travel, lodging etc.
Local labour
Penalty for canceling the other contract

Excess of revenue/saving over incremental costs
40
4
70
8





122

98
34
4
56
16




110

94

Note:
(1) If the Delhi job is undertaken sales of materials no longer required for the Nagpur job would be
(`)
Materials held, at cost
Current money value (add 60%)
Sales price )x90%)
Less: transportation etc. costs (16.67%)
Net sales revenue
20
32
28.8
4.8
24.0

(2) If the Nagpur job is undertaken, the materials for the Delhi job might be refused on a different
contract, thereby saving the purchase of additional materials: (`000)
Materials held
Contracted for
Cost of unwanted materials
Saving in purchase on different contract (80%)
24
36
60
48

(3) The materials contracted for to carry out the Delhi job must be paid for whatever happens. Although
not yet received, they must be paid for whichever (if either) contract is undertaken. It is therefore not
an incremental cost chargeable to the Delhi contract.
For similar reason, materials already held are not an incremental cost to their respective contracts.
The alternative use of materials not required is , however, significant and this has been taken into
account on the revenue side of the analysis.
(4) It is assumed that the project manager salary is a fixed cost, whichever contract (if either) is
undertaken. Incremental labour costs are therefore travel, lodging etc. and local labour.
(5) The penalty cost of failing to undertake one contract should be treated as a consequential cost of
undertaking the other contract.
(6) The excess of revenue/ saving over incremental costs calculated for each contract shows the
comparative effect on profits of undertaking each job in preference to the other. The difference
between the two figures (`98,000 a and `94,000) shows that there is a difference between the two
project of `4,000 in favour of Nagpur job.
(c) The approach usef has assumed that one project or the other will be undertaken. Some costs have
already been incurred (some materials , plant): other costs have been committed (project managers
salary, head office administration) and others are notional (interest on plant).
These are not relevant to any decision about future action. The only relevant consideration should be:
(i) Future revenues or cash savings as a consequence of the decision.
(ii) Future costs, incurred as an additional expense as a consequence of the decision.
In the solution in part (a), incremental revenues are the revenues from the contract undertaken ,
alternative uses of materials held but not required and hire of plant. Incremental costs are only those
additional costs which would be incurred as a result of the decision to undertake one of the contracts.
The cost accounting profit or loss recorded for each contract might be:
Nagpur: `1,70,000-`1,60,000 = `10,000
Delhi: `1,80,000-`1,82,000 =(`2,000)
There figures are irrelevant to a decision because the costs include past , committed or notional costs,
and other revenues and penalty costs to the company are ignored.
( c) Other factors to consider are:
(i) The constraints on working which make the contract mutually exclusive. If there is a shortage of
labour, funds etc., it might be possible to overcome and carry out both projects:
(ii) The likelihood of another contract being offered for the same period of time, which is more
profitable than either the Nagpur or Delhi jobs.
(iii) Loss of goodwill and future contracts by not undertaking either projects:
(iv) Reliability of the prospective customer in each contract:
240
(v) Reliability of costs forecasts, lobour availability etc. on both contacts. The net difference between
the two jobs, `4,000 is relatively small and sensitivity / risk analysis will be very important:
(vi) The preference for the Nagpur contract (by `4,000) has assumed that the alternative use for the
Delhi contract materials will exist. It is only a likelihood, however. Failure to obtain this saving
would shift the preference strongly in favour of accepting the Delhi job.

Answer: 39
Working Notes:
Calculation of Balance Capacity
Products Units Labour Hours
(per unit)
Total Labur
Hours
Capacity utilized
(%)
AB
CD
Total
5,000
10,000
5
4
25,000
40,000
65,000
25
40
65
At 65% Capacity = 65,000 Labour hours used
At 100% Capacity = Labour hours used would be 1,00,000
Balance Capacity = 1,00,000 hours-65,000 hours = 35,000 hours
(i) Statement of Profit for 2003-04 (`)
Products AB CD Total
Production & Sales (Units) 5,000 10,000
Sales Revenue (i)
Variable Costs:
Direct Material
Direct Labour
4,00,000 (@`80)

50,000 (@`10)
1,25 ,000 (@`25)
10,00,000(@`100)

300000 (@`30)
200000 (@`20)
14,00,000

3,50,000
3,25,000

Products AB CD Total
Variable overheads
( 100% on Wages)
Total (ii)
Contribution (i)-(ii)
Less: Fixed Costs
Profit
1,25,000

3,00,000
1,00,000
2,00,000

7,00,000
3,00,000
3,25,000

10,00,000
4,00,000
2,25,000
1,75,000
Working Notes:
Proposals
(1) Utilise balance capacity to Produce AB
(2) Utilise balance capacity to Produce CD
(3) Utilise balance capacity to produce a new product EF
Additional Units= Balance Capacity / Labour hours per unit
AB = 35,000 hrs. / 5 hrs = 7,000 units
CD = 35,000 hrs. /4 hrs. = 8,750 units
Less: Decrease in 1,400 units
Efficiency by 16%

=
EF = 35,000 hrs. / 7 hours 5,000 units
7,350 units

Statement showing utilization of Balance Capacity

Products Proposal (a0 Proposal (b) Proposal ( c )
AB - Existing
- Additional


5,000
7,000
5,000
-
5,000
-
12,000 5,000 5,000
CD Existing
Additional

10,000
-
10,000
7,350
10,000
-
10,000 17,350 10,000
EF New Units - - 5,000
241

Statement showing contribution per unit of Products AB CD AND EF.

Product AB CD EF
Units Existing
5,000
Addl.
7,000
Existing
10,000
Addl.
7,350
New
5,000
Selling Price (i) 84.00 80.00 104.00 104.00 145.00
Variable Costs:
Direct Material 10.50 10.50 31.50 31.50 40.00
Direct Labour 26.25 26.25 21.00 25.00 36.75
Variable Overheads
(100% of Wages)
26.25 26.25 21.00 25.00 36.75
Total Variable Costs
(ii)
63.00 63.00 73.50 81.50 113.50
Contribution (i) (ii) 21.00 17.00 30.50 22.50 31.50

Note:
1. The selling price of additional units of Product CD is assumed to be `104 as is for existing units.
2. The direct labour cost per unit of additional units of Product CD is calculated as below:
Time taken for each additional unit of Product CD = 35,000 hours/ 7,350 units = 4,762 hours
Direct Labour Cost per unit = 4,762 hours x `5.25 per hour = `25000

The variable cost per unit of Products AB and CD were `60 and `70 respectively in the year 2003-04. In the
year 2003-04 it became `63 and `73.50 respectively. Then the differential cost for product AB for 5,000 units
comes to `3 per unit and for product CD for 10,000 units comes to `3.50 per unit. The differential cost per
unit for each additional unit produced during unutilised capacity is equal to its variable cost.

Profitability Statement using incremental revenue and differential cost approach (`)
Products Units Incremental
Revenue per
unit
Total
Incremental
revenue
Differential
cost per unit
Total
Differential
cost
Difference
Proposal (a)
AB 5,000 4.00 20,000 3.00 15,000 5,000
7,000 80.00 5,60,000 63.00 4,41,000 1,19,000
CD 10,000 4.00 40,000 3.50 35,000 5,000
Total 6,20,000 4,91,000 1,29,000
Proposal (b)
AB 5,000 4.00 20,000 3.00 15,000 5,000
CD 10,000 4.00 40,000 3.50 35,000 5,000
7,350 104.00 7,64,400 81.50 5,99,025 + 1,15,375
50,000 (*)
Total 8,24,400 6,99,025 1,25,375
Proposal (c)
AB 5,000 4.00 20,000 3.00 15,000 5,000
CD 10,000 4.00 40,000 3.50 35,000 5,000
EF 5,000 145.00 7,25,000 113.50 5,67,500+ 1,27,500
30,000(**)
Total 7,85,000 6,47,000 1,37,500
* Selling and Distribution Expenses ** Special Advertising Expenses
The Profit as per Statement of Profit for 2003-04 is `1, 75,000. By utilising the Balance capacity 35,000 hours
in manufacture of product EF the said profit will increase by `1,37,500

Statement of Profit for 2004-05 with the selection of Proposal (C) to Introduce Product EF (`)
Existing Profit on manufacture of Products AB and CD 1,75,000
Add: Profit from Product EF by utilising to balance capacity 1,37,500
Total Profit 3,12,500


Answer: 40
242
Differential Cost of the job
Increase ` Decrease
`
Material cost 50,000 20,000
Labour cost 90,000 22,500
Additional Overheads 10,000 -
Other expenses - 2,250
Total 1,50,000 44,750
Net differential cost of the job : `1,05,250
(`1,50,000 `44,750)
Note: Depreciation, rent, heat and light and power are not going to affect the costs.
(b) Full Cost of the jobs:
`
Cost as above at (a) 1,50,000
(i.e. increased costs)
Depreciation 9,000
Power 1,000
Rent 2,500
Heat & Light 250
Total 1,62,750
(c) Opportunity cost of taking the order:
` `
Sale of Product A 62,500
Less:
Material 20,000
Labour 20,500
Power 1,000
Other expenses 2,250
Total
45,750
(d) Sunk cost of the jobs:
16,750
`
Depreciation 9,000
Power* 1,000
Rent 2,500
Heat & Light 250
Total 12,750
*If a student treats power as a relevant cost, in that case it would not appear here. Advice regarding the jobs :
ZED Ltd. should not accept the job as there will be a chase disadvantage of `42,750/- as computed below:
` `
243
Incremental revenue
5,000 units @ `25 1,25,000
Less: Sale of product A 62,500
Differential costs (a)
62,500
Cash disadvantage
1,05,250

42,750
Ans 41:Working Notes:
Contribution per hour in manufacturing Product B is as follows: (`per unit)
Selling Price
Less: Variable Cost
Contribution per unit
100
60
40
Contribution per machine hour =`40/5 hours =`8
(`)
Relevant cost per unit 10+(2M.H. X `8)
Suppliers price per unit
Excess of relevant cost over suppliers price.
26
25
1
Analysis-The relevant cost of production of component is higher by Re 1 over the purchase price of
component part X-100.therefore buy decision is recommended.
Ans. 42:
Selling price per unit of product A 50
`
Less: marginal cost per unit 35
Contribution per unit 15
Contribution per hour of product A 3
Since one unit of product B needs 2 hours, therefore if a unit of B is produced, then the contribution lost
by not producing A = 2 hours `3 = `6
Real cost of producing one unit of product B
`
Marginal cost per unit 5
Add: Contribution lost per unit 6
Total cost of producing a unit of Product B 11
As the suppliers price per unit of product B is `10 and that of producing in the factory is `11, therefore it
is suggested that it is better to buy product B from outside.

Ans. 43:
Component
Calculation of total number of hour required in department P and Q
Particulars A B C Total
Demand units 900 900 1350
Department P:
Hours per unit 2 2 1.5
Total hour required 1800 1800 2025 5625
Component
Particulars A B C Total
Department Q:
Hours per unit 3 3 1
Total hours required 2700 2700 1350 6750
244
From the above, we can observe that department Q is facing the capacity constraint of 750 hours
Statement showing the qualities of components to be purchased to maintain cost
Particulars A C
Purchase cost 129 70
Less: variable cost of manufacture 99 50
Saving in manufacture 30 20
Hours required per unit in dept. Q 3 1
Saving in manufacture per hour 10 20
Suggestion: since the saving in manufacture per hour is more in case of component C, component A should be
purchased from outside.
No. of components of A to be purchased from outside =750 hrs/3 hrs =250 units

Ans. 44:
(a) Selling price per unit 600
Less: Variable cost of `
Component A 32
Component B 54
Component C 58
Component D 12
Component E 4
Assembly 40
Contribution per unit 400
200
Total contribution for 132 units ` 52800
Less: Fixed cost 132316
Net profit
41712
(b) The company may buy any one of the components. The number of units that can be produced
under the three options:
11088
Buy component A Buy component B Buy component C
Component Machine
Hrs reqd
Component Machine
Hrs reqd
Component Machine
Hrs reqd
A -
B 14
C
Total machine
12
Hrs/unit
Total machine hours available
is 4752 under all options
26

Number of units that can be
manufactured, if A is bought
= 4752/26 = 182.77 units
Additional capacity that can be
created
(182.77 132) 100
38.5%
132

=
A 10
B -
C

12


22


Number of units that can be
manufactured, if B is bought
= 4752/22 = 216 units
Additional capacity that can be
created
(216 132) 100
63.6%
132

=
A 10
B 14
C

-


24


Number of units that can be
manufactured, if C is bought
= 4752/24 = 198 units
Additional capacity that can be
created
(198 132) 100
50%
132

=
(c) If the increase in demand during the next period is 50% it is not possible to meet it by buying
Component A as additional capacity created is only 38.5%. Of the remaining two options,
the cheaper one has to be accepted.
Buy B BuyC
` `
245
Market price 160 125
Less: Variable cost if made by
the company
54
Additional cost to be incurred
58
106 67
Machine hours saved 14 12
Cost per hour 7.57 5.58

Since it is cheaper to increase capacity by buying C this option has to be exercised.

(d) Profitability statement
Selling price per unit of equipment `600
Less: Variable cost of:
Making A `32
Making B `54
Buying C `125
Making D `12
Making E `4
Assembly `
267
40
Contribution per unit
267
Total contribution for 198 units (Note 1) 65934
333
Less: Fixed cost (as worked out above)
Net profit
41712
Net increase over period for current period
27222
Note: 1. Maximum capacity = 4752 machine hours.
13134
Machine hours reqd for one unit of equipment : 36 hours.
No. of equipment that can be produced = 4752/36 = 132 Nos.
Marketing department of the company anticipates 50% increase in demand during the next period.
i.e. 132 + 50% = 198 Nos.

Ans. 45:
1. Present demand of components (in batches) from 10,800 (maximum) available machine hours and
projected estimates of components demand (in batches) in the next year.
Working Notes:
Maximum available machine hours 10,800
Machine hours needed to manufacture components. A, B and C (Per batch of
ten numbers) of water purifier
Components Total
A 20 Machine hours
B 28 Machine hours
C 24 Machine 72 hours
Present demand (in batches) of components A, B and C (10,800 hours/ 72 hours) 150
Projected estimate of demand of components A, B and C (add 50% increase) in 225
the next year
2. Present and future fixed costs:
Present fixed cost of 150 batches @ `200/- per batch 30,000
246
Add: Increase in fixed cost to meet 50% increase in demand
Total future fixed cost for 225 batches
10,000
3. Expected purchase cost of components
40,000
View point Probability Component
A B C
Expected
price
Expected
Price
Expected
Price
` ` `
Pessimistic 0.25 30 50 40
(`1200.25) (`2000.25) (`1600.25)
Most likely 0.50 55 65 70
(`1100.50) (`1300.50) (`1400.50)
Optimistic 0.25 20 35 30
(`800.25) (`1400.25) (`1200.25)
Total 105 150

140

4. Present contribution (per batch)
` `
Selling price (per batch) 800
Less: Variable production cost 320
Less: Variable assembly cost 50 370
Contribution (per batch) 430
Total Present contribution on 150 batches 64,500
(i) Maximum number of batches that could be produced in 10,800 machine hours each of the three
alternatives namely buying A or B or C is considered respectively.
(a) Buy component A (from outside) No machine hour required
Make component B 28 Machine hours required
Make component C 24
Total
Machine hours required
Number of batches that could be produced internally 207.69 batches
52
(10,800 hours/52 hours)
(b) Buy component B (from outside) No machine hour required
Make component A 20 Machine hours required
Make component C 24
Total
Machine hours required
Number of batches that could be produced internally 245.45 batches
44
(10,800 hours/744 hours)
But in view of projected (expected) market demand of 225 batches, production would be
restricted to 225 batches only.
(c) Buy component C (from outside) No machine hours required
Make component A 20 Machine hours required
Make component B 28 Machine hours required
247
Total 48
Number of batches that could be produced internally 225 batches
(10,800 machine hours 748 hours)
(ii) Statement of financial implication when purchases of component A, B and C are made from
outside(in view of the fact that production capacity will be limited to 50% increase)
Component bought A B C
` ` `
Total variable cost per batch (I) 64 108 116
Expected purchase cost (II) 105 150
(Refer to working note 3)
140

Increase I variable cost per
batch (III) = (II I)
41 42 24
Present contribution per batch
(IV)
430 430 430
(Refer to working note 4)
Revised contribution per batch
(V) = (IV III)
389 388 406
Total revised contribution 80,791 87,300 91,330
(207.69
batches
`389)
(225 batches
`388)
(225 batches
`406)
Advise: Purchase component C from outside as it gives maximum contribution on manufacturing A and B
internally.
(iii) Profit Statement
(When C is bought from outside and A, B were manufactured internally and extra production is
made and sold)
Per Batch ` Total (for 225
batches)
`
Sales revenue: (I) 800.00 1,80,000 (225
batches `800)
Less: Variable costs (`(Per batch) : (II)
Production cost of A `64
Production cost of B `108
Production cost of D `24
Production cost of E `8
Production cost of C `140
(Refer to working note 3)
`344
Assembly cost `50 394.00 88.650
(225 batches
`394)
Contribution : (III) (-II) 406.00 91,350
Less: Fixed costs 177.78
(`40,000 / 225 batches)
40,000

248
(Refer to working note 2)
Profit 228.22

51.350
Ans: 46:The components are made in a machine shop using three identical machines each of which
can make any of the three components.
Total machine hours required for 3 components
= 4+5+6 = 15 hours
Total capacity of 3 machines is 12,000 machine hours per month and is just sufficient to meet the
current demand.
Hence, the current demand is 12,000/15 = 800 units of product z per month.
Profit made by the company for current month.
Sale price 300
Less: Variable cost 48+60+80+30= 218
Contribution per unit 82
Total contribution 800 x 82= 65,600
Less: fixed cost per month
Profit for current month RS.
50,000
15,600
(a) From next month onwards, the company expects the demand for z to rise by 25% i.e.,
800+25% = 1,000 units per month. One component should be bought from the market.
Which component ?


Statement of extra cost of component per unit
Component A B C
Market price
Less: Variable cost
64
48
75
60
110
80
Extra cost of buying one unit 16 15 30
Machine hours required per unit
Extra cost per machine hour 16/4=
4
`4
5
15/5=`3
6
30/6=`5
Ranking II I III
Because of Ist rank (lowest extra cost), component b should be bought from the market.
Manufacturing Hours
C 1,000 units x 6 hours = 6,000
A 1,000 units x 4 hours = 4,000
B 400 units x 5 hours = 2,000 (
Total
Balance)

12,000
Balance 600 units of B should he bought from the market.
( c) Profit made by the company
Component Element of cost Cost per unit No. of units Amount(`)
A
B
B
C
Assembling
Variable cost
Variable cost
Market price
Variable cost
Variable cost
48
60
75
80
30
1,000
400
600
1,000
1,000
48,000
24,000
45,000
80,000
30,000
Total variable cost
Add: Fixed cost
Total cost
Sales 1,000 units at
`300 per unit
2,27,000
50,000
2,77,000

3,00,000
249
Profit on 1,000 units 23,000

Ans. 47:
Statement showing Profit / Loss of company
i)
(If it accepts the order of manufacturing moulded toys)
Total available machine hours: (A) 18,000
(8 machine 7.5 hours / day 300 days)
Machine hours required for producing
4,20,000 cans: (B) 14,000
(4,20,000 cans /30 cans)
Balance machine hours: {(A) (B)] 4,000
Total number of production of moulded toys in balance hours 60,000
(4,000 hours 15 toys / hour)
Total contribution on 60,000 moulded toys (`) 6,00,000
(60,000 `10)
Less: Fixed expenses of mould (`) 2,25,000
Net profit (`) 3,75,000
Decision: It is advisable for the company to accept the order of 60,000 moulded toys as it will
increase its profit by `3,75,000.
(ii) Statement showing Profit / Loss
(If the order of manufacture of cans increase to 5,40,000)
If 5,40,000 cans are produced, no machine hours would be available for manufacturing toys
`(Lacs)
Total contribution on 5,40,000 cans 32.40
5,40,000 cans `6)
Less: Fixed cost
Profit
20.00
Alternatively, the production would be 4,20,000 cans and 60,000 moulded toys
12.40
`(lacs)
A. Profit from 4,20,000 cans:
Contribution 25.20
(4,20,000 cans `6)
Less: Fixed cost
Profit
20.00
B. Profit from 60,000 moulded toys 3.75
5.20
(Refer to (i) above)
Total profit: (A + B) 8.95
250
Decisions: The production of 1,20,000 additional cans instead of 60,000 moulded toys will result an
additional profit of `3.45 lacs (`12.40 lacs `8.95 lacs). Therefore, the company is advised not to accept
the order of manufacturing moulded toys.
(iii) Let the minimum excess capacity needed to justify the manufacturing of any portion of the
moulded toys order be x.
If toys are manufactured, the profit is = (`60 `50) x `2,25,000
and, if toys are sub-contracted, the profit is = (`60 `57.50) x
Indifference point would be 10x `2,25,000 = 2.5x
or x = 30,000 moulded toys
Toys produced per hour =15 toys
Therefore, 2,000 (30,000 toys / 15 toys) excess machine hours are required to justify manufacturing of
toys by the company, instead of sub-contracting.
(iv) Profit under existing production plan:
(`Lacs)
Contribution from 4,50,000 cans 27.00
(4,50,000 `6)
Contribution from 45,000 toys 4.50
(45,000 `10)
Total contribution 31.50
Less: Fixed cost 22.25
(20 lacs + 2.25 lacs)
Profit 9.25
Profit from 15,000 sub-contracted toys
(15,000 `2.50)
0.375
Total profit
If demand was accurately forecasted & 4,80,000 cans were manufactured, excess machine hour capacity
available was 2,000 hrs, such excess being the pint of indifference i.e. profit from toys order would be the
same by either manufacturing 30,000 toys or sub-contracting them along with the rest of 30,000 toys.
9.625
(v) Profit under properly negotiated production plan:
(`Lacs)
Contribution from 4,80,000 cans 28.80
(4,80,000 `6)
Less: Fixed cost 20.00
Profit 8.80
Profit from Toys
60,000 Nos. sub-contracted
1.50
(60,000 `2.5)
Total profit 10.30
251
Therefore, the loss for improper prediction and negotiation is `10,30,000 `9,62,500 or `67,500.

Ans. 48:
1. (i) Fixed manufacturing overhead per unit
Working Notes:
XY 100; `3,00,000 / 5,000 units or `60
XY 200; `3,00,000 / 12,000 units or `25
(ii) Variable manufacturing overhead per unit
XY 100; (`180 `60) or `120
XY 200; (`60 `25) or `35
2. Variable costs of production of XY 100 and XY 200
Product Per unit
XY 100 XY 200
` `
Direct material 200 200
Variable machine operating costs 150 50
Variable manufacturing overheads 120
Total variable costs per unit
35
470
3. (i) machine hours for the production of one unit of each of the two products.
285
XY 100; `150/-`100 per hour = 1.5 hours.
XY 200; `50/- `100 per hour = 0.50 hours.
(ii) Total machine hours available
5,000 units 1.5 hours = 7,500 hours
Ranking between manufactured XY 100 and manufactured XY 200
Manufactured Manufactured
XY 100 XY 200
` `
Variable cost of production 470 285
(Refer to working note 2)
Variable marketing and administrative cost 80 60
Total variable cost per unit: (A) 350 345
Selling price per unit: (B) 900 600
Contribution per unit: [(B) (A)] 350 255
Contribution per hour 233 510
[Refer to working note 3(i)] (`3.50/1.5 hrs) (`255/0.5 hrs)
Ranking II I
Ranking between manufactured XY 100 and purchased XY 100
Manufactured Purchase
252
XY 100 XY 100
` `
Variable cost of production 470 --
(Refer to working note 2)
Purchase price -- 650
Variable marketing and administrative cost 80 40
Total variable cost per unit: (A) 550 690
Selling price per unit: (B) 900 900
Contribution per unit: [(B) (A)] 350 310
Ranking II I
XY 200: 12,000 units 0.50 hours or 6,000 hours
XY 100: (7,500 6,000) hours = 1,500 hours
Quantity of each product that XYZ Limited should manufacture and / or purchase to maximise
operating income
Manufactured XY 200 12,000 units
Manufactured XY 100: 1,500 hours / 1.5 hours 1,000
Purchased XY 100 6,000
Maximum number of units
Which ABC can supply.

Ans. 49: (i) Profitability as per original Budget
Rs
(000s)
Rs(000s)
Sales(1,80,000 units Rs 130) (A) 23,400
Direct Material (1,80,000 units Rs 30) 5,400
Component EH ( variable cost = Rs 7.20 per
unit)
1,296
Direct wages (1,80,000 units Rs 28) 5,040
Variable factory overheads (1,80,000 units
Rs 24 50% )

2,160

Variable selling & distribution (1,80,000 units
Rs 24 50% )

2,160

Total variable cost (B) 16,056
Contribution (A B) 7,344
Fixed factory overheads 2,160
Fixed selling & distribution overheads 720
Component EH @2.20 396
Administrative overhead 900 4,176
Profit 3,168
(ii) Export order
Rs per Unit Rs per Unit
253
Direct material 56
Direct labour (10 hours Rs 7 per hour) 70
Variable factory overhead ( Rs 3 10 labour hours) 30
Selling and distribution overheads 14
Total variable cost 170
Selling price (export) 175
Contribution 5
Since the product earns contribution of `5 per unit, it should be accepted.
Total units 500(per month) = 6000 units(per annum)
Therefore additional contribution (6000 units Rs 5) = `30,000
Total hours on product 43 grade (1,80,000 units 4) = 7,20,000 Hrs
Total hours on component EH (1,80,000 units 0.5*) = 90,000 Hrs
*
hour per rate Labour produced units of No
cost Labour Direct

=
hour per 7 Rs units 15,000
52,500 Rs

=
0.5 Hrs
Total hours utilised at 90% capacity = 7,20,000 hours + 90,000 hours = 8,10,000
hours
100% capacity hours =
90
100 hours 8,10,000
= 9,00,000 Hrs
Balance hours available = 90,000 hours p.a
Hours required for export order 60,000 hours.
Both contribution per unit of export order and availability of capacity confirm its
acceptance.
(iii) Component EH make or buy
(per 15,000 units) Make (`) Buy (`)
Direct material 30,000
Direct labour 52,500
Variable factory overhead 25,500
Total 1,08,000 1,18,500
Per unit 7.20 7.90
If the company makes the component the out of pocket cost is `7.20 per unit whereas if the
component is bought , the out of pocket cost is `7.90.
Decision : If the capacity remains idle it is profitable to make.
(iv) Alternative use of the spare capacity
Units required = 1,80,000 units and hours required = 1,80,000 0 .5 = 90,000
Hrs
Cost of buying component EH = (1,80,000 units Rs 7.90) =Rs 14,22,000
Cost of making component EH = (1,80,000 units Rs 7.20) = Rs 12,96,000
Hence , excess cost of buying = `1,26,000
Rent income (90,000 hours Re1) = `90,000
Contribution per unit from making component GYP = Rs 8 -
Units 15,000
1,12,500 Rs
=
Rs 0.5 per unit.
Direct labour cost per unit of GYP =
Units 15,000
31,500 Rs
= `2.10 per unit.
254
No. of labour hours required for one unit of GYP =
7 Rs
2.10 Rs
= 0.3 Hrs
No. of units of GYP in 90,000 hours =
hours 0.3
hours 90,000
=3,00,000
Contribution from component GYP = 3,00,000 Rs 0.50 = Rs 1,50,000
Since the contribution from GYP is greater than the extra variable cost of
buying component EH , component GYP should be manufactured and
component EH should be purchased.

Hence, accept export order and buy the component.

(i) If the reliable suppliers offered to supply P44E at a guaranteed price of `50 p.u. variable manufacturing
cost p.u.
Ans. 50:
Direct material 14
Direct labour 12
Variable overheads 8
Total variable manufacturing cost 34

(`)
Purchase price 50
Less: variable manufacturing cost 34
Saving, if manufactured internally 16

(ii) If the company incur additional inspection and testing charges of `56,000 p.a.
= `56,000/`16 p.u. = 3500 units
The company can purchase, if yhe requirement of P44E, is less then 3500 units. If the requirement is more
then 3500 component, it can manufacture its own requirement.

(iii) when the direct labour hours is limiting factor :
Calculation of contribution per labour hour
Particulars Own manufacture of
P44E
Extra sale of another
existing product
Selling price - 90
Cost of purchase of P44E (saving) 50 -


Less: variable cost
50

34
90

50
Contribution (i) 16 40
Labour hours (ii) 4 8
Contribution per labour hour (i)/(ii) 4 5
Rank II I
Analysis: since the labour hours are the limiting factor, it is suggested to opt for extra sale of another existing
product then to manufacture component P44E.
(iv) The cost of the machine bought last year is a sunk cost and not relevant to the present decision of make
or buy.
Book value of the machine is merely an accounting treatment.

Ans. 51: (a)This is a make or buy decision so compare the incremental cost to make with the incremental cost
buy
255
Incremental cost per unit Make the Blades
Direct material `7.50
(`75000 10,000)
Direct labour `6.50
(`65000 10,000)
Variable Overhead `5.50
(`55000 10,000)
Supervision
(`35,000 10,000) `3.50
Total cost `23.00
Compare the cost to make the blades for 10,000 motors. `23.00, with the cost to buy, `25.00 There
is a net loss of `2.00 if X chooses to buy the blades.
(b) X will be indifferent between buying and making the blades when the total costs for making and
buying will be equal at the volume level where the variable costs per unit times the volume plus
the fixed avoidable costs are equal to the suppliers offered cost of `25.00 per unit times the
volume.

(Direct materials + Direct labour + Variable overhead) Volume + Supervision =, Cost to buy
Volume. Let volume in units = x
(7.50 + 6.50 + 5.50) x + 35,000 = 25.00x

19.50 x + 35,000 = 25.00 x

35,000 = 25.00 x 19.50 x

35,000 = 5.50 x x = 6,364 units
of blades
As volume of production decreases, the average per unit cost of in house production increases. If the
volume falls below 6,364 motors, then X would prefer to buy the blades from the supplier.
(c) If the space presently occupied by blade production could be leased to another firm for `45,000 per
year, X would face an opportunity cost associated with in house blade production for the 10,000
units of `4.50 per unit.
New cost to make = 23.00 + 4.50 = 27.50

Now X should buy because the cost to make, 27.50, is higher than the cost to buy, 25.00.
(i) Deciding whether B Ltd. Should accept the offer from an outside vendor instead of
manufacturing chains internally.
Ans. 52:
Price of chain offered by vendor `12
Less: Variable cost of (`5 + `2)
Excess of quoted price over variable cost
7
Total excess of quoted price over variable cost (24,000 x `5) `1,20,000
5
Less: Avoidable cost
Inspection, set-up, etc. `24,000 -
Machine rent 24,000 48,000
Excess of bought out price over variable cost and avoidable cost 72,000


Decision- B Ltd. Should not accept the offer from outside vendor, because this decision will lead
to reduction in profit by `72,000.
(ii) Deciding whether the use of internal facilities for upgrading the quality of chains would
be useful in comparison to purchase from outside.

Incremental revenue per unit `22
256
Less: Differential cost per cycle
Contribution
18
Total Contribution (24,000 x `4) `96,000
4
Less: Tooling costs
Net contribution
16,000

80,000
Decision B Ltd. Should accept the offer of alternative use of facilities for upgrading the
bicycle. It will lead to increase of `80,000 in contribution. This is more than the excess of
bought-out price over variable and avoidable cost [i.e.`72,000 as per (i)]. Thus company will
benefit by `8,000 i.e.,(`80,000 `72,0000)

(iii) Deciding whether use of internal facilities for upgrading the bicycle ( chain) internally
would be profitable, if batch size becomes 4,000 units in comparison to their purchases
from an outside vendor.
Bought- out price offered `12
Less: Variable internal cost
Excess of bought out cost over variable cost
7
Total excess of bought out cost over variable cost (24,000 x `5) `1,20,000
5
Less: Inspection cost `12,000 -
Machine rent 24,000
Excess of bought out price over variable and avoidable costs
36,000
Decision If inspection cost (Which varies with batch size) decreases, then excess of bought- out
price over variable and avoidable costs would be `84,000. In comparison to this, net contribution
from using the internal facilities for upgrading quality of chains will `80,000[refer to (ii).] There
fore, if batch size increases and inspection cost reduces, then use of internal facilities of updation
of quality of chain is advocated. If decision to update is taken in (ii), it will increase profit by
`4,000 (i.e..`84,000 `80,000)
84,000

Ans. 53:
For taking a make or buy decision, it is necessary to find out the relevant cost of both the decisions,
i.e. manufacturing vis--vis purchasing the component from outside.
Departmental Expenses Budget
(`000)
Items Total Allocation ratio Gadgets Components
Production
Variable Costs
Direct material
Direct labour
Indirect labour
Inspection and testing
Power

Fixed Costs
Lighting
Insurance
Depreciation
Misc. Fixed Exp.

Total cost
Variable cost per unit


3,840
1,536
720
480
480
7,056

40
30
96
54
220
7,276


80 : 20
75 : 25
80 : 20
75 : 25
75 : 25









24,000

3,072
1,152
576
360
360
5,520







`230
24,000

768
384
144
120
120
1,536







`64
257

(i) Variable cost of component is `64 per unit. The purchase price is `70 per unit. For each
unit net cash outflow will be `6. Therefore, company should take decision to make.
(ii) Evaluation of decision to export
Inflow
(a) Additional contribution due to export
12,000 units x (`245 230) `1,80,000
(b) Saving in variable cost of components (24,000 units x `64)
17,16,000
15,36,000
Less: Outflow
Payment to be made to supplier (24,000 units x `70)
Net Cash Inflow
16,80,000

36,000
(a)
Ans. 54

Demand 52,000 48,500 26,500 30,000
A B C D
Direct Material 64 72 45 56

M/c 48 32 64 24
Other Variable Cost 32 36 44 20
Total Variable Cost 144 140 153 100
Selling Price 162 156 173 118
Contribution (`/u) 18 16 20 18
M/s Hours per unit 6 4 8 3
Contribution (`/ M/c hr.) 3 4 2.5 6
Ranking III II IV I Sub-Contract
Cost `/u) 146 126 155 108
Cont (`/u) on (Subcontract) 16 30 18 8

I Division: It is more profitable to sub-contract B, since contribution is higher sub- contract.

1
st
Level of Operations: 1,50,000 hours, Produce D as much as possible. Hours
required = 30,000 units 3 = 90,000 hours
Balance hours available: 60,000 hours.

Produce the next best (i.e. A, Since B is better outsourced)

60,000 hrs
= 10,000 units of A.

6 hrs / u

1
st
Level of Operation:

Contribution (units) Contribution (`)

A Produce 10,000 units 18 1,80,000
A Outsource 42,000 units 16 6,72,000
B 48,500 units
Outsource fully 30 14,55,000
258
C 26,500 units
Outsource fully 18
D 30,000 units
Fully produce 18 5,40,000
Total Contribution: 33,24,000
Less: Fixed cost 10,00,000
Net Gain 23,24,000

2
nd
Level of Operation:

Both A and C increase contribution by own manufacture only by `2/ - per unit. 1,50,000
hrs can produce 25,000 units of A.
Contribution increases by 25,000 2 = 50,000

(Difference in Contribution sub -contract and own manufacturing) = 2

But increase in fixed Cost = 50,000

At the 2
nd
level of operation, the increase in contribution by own manufacturing is exactly set up by
increase in fixed costs by `50,000/-. It is a point of financial indifference, but other conditions like
reliability or possibility of the sub -contractor increasing his price may be considered and decision may
them but towards own manufacture.
3
rd
Level Additional: 1,50,000 hrs available

Unit of A that are needed = [52,000 25,000 (2
nd
Level) 10,000 (1
st
Level)]

= 17,000 units 6 hrs/u = 1,02,000 hrs.
Balance 48,000 hrs are available for C to produce 6,000 units. Increase in
Contribution over Level 1
st
or 2
nd
:

A: 17,000 2 = `34,000
C: 6,000 2 = `12,000

= `46,000
Increase in fixed costs = `50,000

Additional Loss = `4,000

4
th

Level Additional 150000 hrs. can give 150000 8
= 18,750 unit of C

Increase in Contribution 18,750 2 = ` 37,500

Increase in Cost = (`50,000) Level 3
rd
loss
c/fd = (` 4,000) Level 1
st
profit
will order by =(` 16,500) Advice: Do not
expand capacities; sell maximum
No. of units by operating at 1,50,000 hrs. capacity (level 1
st
) and gain `23,24,000.

Summary:
Product Produce
(Units)
Sub-Contract
(Units)
Contribution
(Production)
Contribution
(Sub-Contract)
Total
Contribution
A 10,000 42,000 1,80,000 6,72,000 8,52,000
B - 48,500 - 14,55,000 14,55,000
259
C - 26,500 - 4,77,000 4,77,000
D 30,000 - 5,40,000 - 5,40,000
33,24,000
Fixed Cost 10,00,000
Profit 23,24,000

Calculation of contribution per unit
Ans. 55
Particulars EXE WYE
(a) selling cost P.U. 375 540
Variable cost P.U.
Dept. 1 Direct materials 58 100
Direct labour 5 hours
50
7.5 hours
75
Variable overheads
(5 hrs*`2.40) 12 -
(7.5 hrs*`2.40) - 18
(i) 120 193
Dept.2 Direct materials 21 26
Direct labour 90 120
(7.5 hrs*`3.60) 27 -
(10 hrs* `3.60) - 36
(ii) 138 182
Total variable cost (i)+(ii) 258 375
Contribution P.U. (a)-(b) 117 165
Calculation of contribution per unit if facilities of Dept.1 were sub-contracted but facilities of Dept.2 used
internally
(`)
Particulars EXE WYE
Selling price per unit (a) 375 540
Cost of sub-contracting Dept.1 facilities 138 212
Cost of manufacture in Dept.2 internally 138 182
Total variable manufacturing cost per unit 276 394
Contribution per unit (a)-(b) 99 146


Calculation of contribution per unit if facilities of Dept.1 and Dept.2 are sub-contracted
Particulars EXE WYE
Selling price per unit (a) 375 540
Cost of sub-contracting P.U.
Dept.1
Dept.2

138
150

212
192
Total variable cost P.U. (b) 288 404
Contribution P.U. (a)-(b) 87 136
Statement showing number of units to be produced and sold to earn maximum profit by using own
manufacturing capacity
Particulars EXE WYE
Dept.1 (1,75,000 hrs/5 hrs)
(1,75,000 hrs/7.5 hrs)
Dept.2 (2,80,000 hrs/7.5 hrs)
(2,80,000 hrs/10 hrs)
35,000
-
37,333
-
-
23,333
-
28,000
Maximum unit can be produced and sold by using facilities of 35,000 23,333
260
both departments.
Maximum contribution
(35,000 units* `117)
(23,333 units*`165)
Les: fixed cost
(Dept.1 `5,00,000 + Dept.2 `10,00,000)

40,95,000
-

15,00,000

-
38,49,945

15,00,000
Maximum profit 25,95,000 23,49,945
Suggestion: by production and sale of 35,000 units of EXE is maximum, it is suggestion to manufacture EXE
internally.
Calculation of profit from EXE (`)
Contribution on internally produced units (35,000 units * `117)
Contribution when Dept.1 services were sub-contracted (2,333 units * `99)
Contribution when Dept.1 & Dept.2 services were sub-contracted (1,167 units * `87)
40,95,000
2,30,967
1,01,529
Total contribution of EXE
Less: fixed cost
44,27,496
15,00,000
Profit 29,27,496
Calculation of total contribution of WYE (`)
Contribution on internally produced units (23,333 units * `165)
Contribution when Dept.1 services were sub-contracted (4,667 units * `146)
Contribution when Dept.1 and Dept.2 services were sub-contracted (3500 units * `136)
38,49,945
6,81,382
4,76,000

Total contribution of WYE
Less: fixed cost
50,07,327
15,00,000
Profit 35,07,327
Suggestion: profit is maximum for product WYE. Hence 31,500 units of WYE should be produced to yield a
sum of `35,07,327 as profit.

Working notes:
Ans. 56:
1.
(a) Total normal and overtime hours available.
Department
A B
Normal capacity hours 600 520
Overtime hours 300 260
(50% of normal hours in each department)
Total available hours 900 780
(b) Total hours required to meet fully the market demand of 2,500 units of P and 2,000 units of Q.
Department
A B
Hours required for
manufacturing P 2,500
units of Product
250 500
(2,500 Units 0.1 hour) (2,500 Units 0.2
hour)
Hours required for
manufacturing Q 2,000
600 400
261
units of Product
(2,000 Units 0.3 hour) (2,000 Units 0.2
hour)
Total hours required 850 900
2. Sub-contracting should be resorted:
To meet the market demand of 2,500 units of product P and 2,000 units of product 850 and 900 hours
[Refer to working note 1(b)] are required in departments A and B respectively. In department B only 780
hours are available and thus does not meet fully the requirement of 900 hours. Hence, sub-contracting
should be resorted to meet the market demand fully.
3. (i) Contribution per unit;
Product P Q
Normal
hours
Overtime
hours
Normal
hours
Overtime
hours
Director material cost (`) 10.00 10.00 5.00 5.00
Direct labour cost Dept. A (`) 1.00 1.50 3.00 4.50
(`10
0.1 hr.)
(`15
0.1 hrs.)
(`10
0.3 hrs.)
(`15
0.3 hrs.)
Dept. B: (`) 2.40 3.60 2.40 3.60
(`12
0.2 hrs.)
(`18
0.2 hrs.)
(`12
0.2 hrs.)
(`18
0.2 hrs.)
Total variable cost per unit (`) : (A) 13.40 15.10 10.40 13.10
Sub-contract price per unit (`) : (B) 18.00 18.00 12.00 12.00
Contribution / cost saving / (Loss
per unit (`)
4.60 2.90 1.60 (1.10)
(C) = [(B) (A)]
(ii) Contribution per hour
Hours required per
unit

Dept. A 0.1 0.1 0.3 0.3
Dept. B 0.2 0.2 0.2 0.2
Contribution per
hour

Dept. A (`) 46 29 5.33 Loss
(`4.60/0.1 hrs.) (`2.90/0.1 hr.) (`1.60/0.3 hrs.) --
Dept. B (`) 23 14.50 8.0 Loss
(`4.60/0.2 hrs.) (`2.90/0.2 hr.) (`1.60/02. Hrs.) --
4. Utilization of normal and overtime available hours to meet fully monthly market demand of 2,500 units of P
and 2,000 of Q.
(i) An analysis of contribution statement (Refer to working note 3) clearly shows that 2,500 units of the
product P should be manufactured by utilising the normal capacity hours of departments A and B. The
262
manufacturing of 2,500 units of P will consume 250 normal hours of department A and 500 hours of
department B (Refer to working note 1(b).
(ii) For manufacturing 2,000 units of product Q, it is beneficial to utilise the remaining normal available
hours of departments A and B. The normal available hours in the department B are only 20 hours,
[520 hours 500 hours] and in department A 350 hours [600 hours 250 hours]. 100 units of product
Q can be manufactured by utilising the normal available hours of departments A and B. The
manufacturing of 100 units of Q in normal available hours will utilise 30 hours in department A and 20
hours in department B.
(iii) Now for manufacturing the remaining 1,900 units of product Q, we have 320 normal hours plus 300
overtime hours in department A and 260 overtime hours in the department B. The manufacturing cost
per unit of product Q comes to `11.60 when normal hours of department A and overtime hours of
department B are utilized.
{`5 (Material Cost) + `3 (Direct Labour in Department A) + `3.60 (Direct Labour in Department B)}
On comparing `11.60 with sub-contracting price of `12 per unit, we arrive at a contribution of 0.40 per
unit. Hence maximum number of units of product Q should be manufactured by using normal hours of
department A and overtime hours of department B. since 0.3 and 0.2 hours are required respectively
for manufacturing one unit of product Q in the two departments, therefore, utilising 320 normal hours
and 213 overtime hours in departments A and B respectively, 1066.66 units (or say 1,067 units) of
product Q are manufactured.
(iv) Finally, to manufacture remaining 833 units of Q, the available time is 300 overtime hours and 47
overtime hours in department A and B respectively. According to (working note 1) the available time in
department B is short by 120 hours (900 required hours 780 available hours) therefore 833 units of
Q cannot be made internally. But few units can be made by utilising the available overtime hours in
departments A and B. The manufacturing cost of 1 unit of Q by utilizing overtime hours in departments
A and B comes to `13.10 (Refer to working note 3) which on comparison with subcontract price of `12
gives rise to a situation of loss of `1.10 per unit {`13.10 `12}. Hence it is advisable not to
manufacture the remaining 833 units internally. These 833 units should be sub-contracted at a price
of `12/- per unit.
(i) Statement of quantity of each product to be manufactured / or to be sub-contracted for fulfilling
the market demand in most economical way.
Departments
A B
Normal
time hours
Overtime
hours
Normal
time hours
Overtime
hours
Available hours (Refer to
working note 1(i)
600 300 520 260
Production 2,500 units of P 250 -- 500 --
(Refer to working note 4 (i)) (2,500 units 0.1 hrs.) (2,500 units 0.2 hrs.)
100 units of Q 30 -- 20 --
(Refer to working note 4 (ii) (100 units 0.3 hrs.) (100 units 0.2 hrs.)
1067 units of Q 320 -- -- 213
(Refer to working note 4(iii)) (1,067 units 0.3 hrs.) (1,067 units 0.2 hrs.)
(ii) Statement Showing Total Cost
(Based on the solution in (i) above)
Products
263
Particulars P Q Sub contract
price
Total
` ` ` `
Direct Material Cost 25,000 5,835 - 30,855
(2,500 units
`10)
(1,167 units `5)
Direct Wages:
Dept. A 2,500 3,500 -- 6,000
(250 hours
`10)
(350 hours `10)
Dept. B 6,000 4,074 -- 10,075
(500 hours
`12)
(20 hours `12 + 213 hours `18
Fixed overhead 18,000 6,400 - 24,400
Cost of 833 units @
`12 per unit on sub-
contracting
-- -- 9,996 9,996
Total Cost 51,500 19,809 9,996 81,305

(i) Option
Ans. 57:
Statement of Profit (Loss)
(if the firm discontinue all the operations during notice period of 3 months)
(`Crores)
Products A B C D Total
Sales* - - - - -
Costs:
Material & Labour - - - - -
Allocated overheads:
Manufacturing 1.5 1.2 1.8 1.2 5.7
Admin. & Selling 0.6 0.3 0.9 0.6
Total allocated overheads during
notice period of 3 months
2.4
2.1 1.5 2.7 1.8
Profit / (Loss)
8.1
(2.1) (1.5) (2.7) (1.8) (8.1)
*The option (i) would not yield any revenue.
Conclusion: The option (i) will result in a loss of `8.1 crores due to the committed costs account of 3
months notice period.
(ii) Option
Working note:
Ascertaining profitable products (if their production is continued during 3 months of notice period)
(`Crores)
Products A B C D
Sales (X) 18 13.5 21 15
264
Variable cost:
Materials 12.0 7.5 13.5 9.0
Labour 4.5 3.0 7.5 7.5
Total variable costs: (Y) 16.5 10.5 21.0 16.5
Contribution: (X Y) 1.5 3.0 - (1.5)
A review of contribution figures in the above statement of four products A, B, C and D clearly reveals
that products A and B are only profitable.
Statement of Profit (Loss)
(If the firm continues the operations of profitable products A and B during 3 months of notice period)
(`Crores)
Products A B Total
Contribution (Refer to above working note) 1.5 3.0 4.5
Less: total manufacturing administrative & selling
overheads (Refer to part (i) above)

Profit / (Loss)
8.1

Conclusion: Under this option the total loss is (`3.6) crores which is less than the loss of option (i).
(3.6)
(iii) Option
Working Note:
Ascertaining profitable products (when notices are issued to the staff and the landlord only in the
manufacturing unit, resort to subcontracting only on profitable products)
(`Crores)
Products A B C D
Sales: (X) 72.0 54.0 84.0 60.0
Variable Cost:
Materials 48.0 30.0 54.0 36.0
Sub-contracting charges 16.0 10.5 27.0 26.0
(20 lacs (15 lacs
`80)
(30 lacs
`70)
(20 lacs
`90)
Total variable costs : (Y)
`130)
64.0 40.5 81.0
Contribution: (X Y)
62.0
8.0 13.5 3.0 (2.0)
A review of contribution figures in the above statement clearly shows that products A, Band C are only
profitable.
Statement of Profit / (Loss)
(If the firm resorts to manufacturing of profitable products by sub-contracting)
(`Crores)
Product Total
A B C
Contribution: (X) 8.0 13.5 3.0 24.5
(Refer to above working note)
Total manufacturing overheads of 3 months
notice period : (Y)
5.7
(Refer to option (i) above)
265
Total administrative & Selling overheads: (Z)
Profit/(Loss): {X (Y+Z)}
9.6

Decision:
9.2
Out of the three options the option (iii) is the most viable one. Not only it will help the company with a turn
around, but from the year 2002, the company can look forward to even higher profitability, since the
manufacturing overhead would no longer be incurred thereafter.

Working Notes:
Ans. 58:
1. FOB price of dismantled kit:
FOB price of dismantled kit (in$) 510
FOB price of dismantled kit (in `) 24,000
($510 `47.059)
2. Cost of a dismantled kit to Z Inc.
If `120 is the S. P. of kit to Z Inc. then its C `100
Re 1 =
Rs.120
Rs.100

If `24,000 is the S. P. then C. P. is =
Rs.120
Rs.100
`24,000
= `20,000
3. Cost of local procurements:
140% of the supplies made by Z Inc. or 140% `10,000* = `14,000
*Being 50% of cost of a dismantled kit to Z Inc.
4. Landed cost of a dismantled kit:
`
FOB price 12,000
(50% `24,000) (Refer to working note 1)
Add: Insurance & freight
CIF price 12,500
500
Add: Customs duty 3,750
(30% `12,500)
Landed cost of a dismantled kit
5. Cost of the standard items procured locally:
16,250
48% of the cost of locally procured goods
= 48% `14,000
= `6,720

6. Royalty payment per computer:
Let x = Selling price per unit of personal computer
y = Royalty paid per computer
Since 20% is the margin of profit on S.P. it main a margin of 25% on C.P.
Therefore we have
266
X = 1.25 (`32,250+ `150 + y)
Y = 10% {x (`6,720 + `16,250)}
On solving the above equations we get:
X = `43,000
Y = `2003.43 or `2,000 (Approx)
Statement showing the selling price of a personal computer in India `
A. Landed cost of a dismantled kit
(Refer to working note 4) 16,250
B. Cost of local procurement
(Refer to working note 3) 14,000
C. Cost of assembly and other overheads per computer
D. Total cost of manufacture: (A+ B + C) 32,250
2,000
E. Technology fee per computer 150
(`3,00,00,000 / 2,00,000 computer)
F. Royalty payment per unit
(Refer to working note 6)
G. Total cost (D + E+ F) 34,400
H. Profit (20% on selling price of 25% of total cost)
I. Selling price (per computer)
8,600

43,000
Ans. 59:
Capacity Output (units) FOB cost Total cost(`) Differential Differential
Statement of Differential cost
per unit (`) cost(`) cost per unit (`)
70% 70,000 97 67,90,000
80% 80,000 92 73,60,000 5,70,000 57
90% 90,000 87 78,30,000 4,70,000 47
100% 1,00,000 82 82,00,000 3,70,000 37
Statement showi ng gai n or l oss for vari ous export order

If proposal A is accepted the company will suffer a loss of `10,000 with an idle capacity of 5,000 units.

If proposals A and B are accepted, the company will suffer a loss of `10,000 with an idle capacity of `5,000
units.

If the company accepts all the three proposals, it will earn profit of `80,000 with an idle capacity of 5,000 units.
267
Therefore, the company should accept all three proposals.


Shut down point =
Ans. 60:
P/V Ratio
Avoidable Fixed cost - Shut down cost

= [120000-40000] - 0
1-0.8
= `400000


Ans. 61:

Continue Shut Down
Fixed expenses at 50% activity 30,000 -
Additional shut down cost 2,000
Fixed expenses during shut down

10,000
30,000 12,000
Additional fixed cost incurred due to continued operations = 18,000

If contribution from operation is less than 18,000, a shut down is recommended.

i.e. Contribution per unit <
18,000

5,000

i.e. Contribution < Rs 3.60 per unit
i.e Selling price variable cost < Rs 3.60 per unit
or S.P. 3.6 < Variable cost i.e. 14.00
3.60 < Variable Cost or variable
cost is more than Rs 10.40
For a variable cost more than Rs 10.40 per unit, a shut down is recommended.

Al ternative

Contribution from operation must be less than 18,000 `for a shut down. Sales value =
14x5,000 = 70,000
Sales variable cost < 18,000

or variable cost is more than 70,000-18,000 = 52,000

Variable cost of 5,000 units above `52,000

Or Variable Cost V.C. per unit >
52,000
= `10.40

5,000

For a variable cost per unit above `10.40, shut down is recommended.

Ans. 62:


If plant is continued If plant is shutdown
Sales 7,60,000 -
Less:Variable Cost - 5,70,000
Contribution 1,90,00


Less:Fixed Cost 3,50,000 1,30,00

Additional Cost

15,000
268
Operating Loss 1,60,00

1,45,00

A comparison of loss figures indicated as above points out that loss is reduced by
(16,000-14,500) `15,000 if plant is shut down.

Shut down point =
3,50,000 - 14,5000
=
20,500
= 1,02,500 units

8 - 6 2

Capacity level of shut down point:

At 100% level production is
95,000
= 1,18,750

0.80



Capacity level at shut down =
1,02,500
= 86.31%

1,18,750


Al ternative Sol uti on

`
If the plant is shut down, the sunk cost or fixed expenses 1,45,00

If it is working at 80% capacity, the fixed cost 3,50,000
Additional fixed expenses 2,05,000
Contribution (95000*2) 1,90,00
0
Incremental Loss on Continuing 15,00

Decision - better to shut down

Production at shut-down point

2 x 350000 = 1,45,000
2x = 2,05,000
x = 1,02,500 Units
Capacity % = 1,02,500/(95,000/0.8) =



(a) Contribution per tin = Selling Price Variable cost
Ans. 63:

= 21 (7.8 + 2.1+ 2.5 + 0.6)

= `8 per tin.

Loss on operation:

Fixed cost per annum = 2,00,000 units 4 per unit = 8 lakhs
Fixed cost for 1 quarter = 8/4 = 2 lakhs
`
Fixed cost for the quarter 2,00,000
Less: Contribution on operation (8 10,000)
Expected loss on operation
80,000
(1,20,000

)
Loss on shut down:
`
Unavoidable Fixed Cost 74,000
Additional shut down cost
Loss on shut-down
14,000
(88,000

)
269
Conclusion: Better to shut down and save `32,000.

Shut-down point (number of units) = Avoidable Fixed Cost Contribution per unit
= (2,00,000 88,000)/8
= 14,000 units.

The Directors,
Ans. 64:
XYZ Co.
New Delhi
Date.
Dear Sir
As desired, we have analysed the cost implications of the decision of temporary closure of the trade
recession. We find that if the factory is run at 50% capacity and with reduced sales revenue, the loss
likely to be incurred in one full year (the estimated period of recession), would be around `200000 as
detailed below:
`In000

Direct materials 300
Direct labour 400
Production overhead 240
Administrative over head 120
Selling & distribution overhead

130
1190
Loss
Sales
200
If the factory is closed, the following costs will be incurred: `In 000
990
Fixed costs 220
Settlement cost 150
Maintenance costs 20
Cost of resuming operations

80
It is obvious from the above, that despite the fact that running at 50% capacity would imply a loss of
`200000, it is better not to close down the factory since in that case the loss would be higher.
470
In our views, even if running the factory entailed a somewhat bigger loss as compared to the loss
incurred by closing it down temporarily, it may be better to keep the factory in operation. This is
because a closure, even if temporary, results in the loss of regular and old customers, suppliers and
skilled personal. This, coupled with a loss of goodwill in the market, may give rise to substantial
losses at the time of restarting the factory. We trust that the above analysis would be helpful to you
in reaching an appropriate decision in the matter. We shall be glad to be of any further assistance that
may be required in this regard.


Yours faithfully
X and Co.


Chartered Accountants.
270
Working Note:

Production Admn. Selling

overhead overhead overhead

(`Lakhs) (`Lakhs) (`Lakhs)
(i) Amount at 60% 2.52 1.24 1.36
(ii) Amount at 80% 2.76 1.32 1.48
(iii) Vari abl e cost for 20% 0.24 0.08 0.12
(iv) Variable cost for 60% 0.72 0.24 0.36
(v) Fixed Cost 1.80 1.00 1.00
(vi) Amount at 50% (iii2.5+v) 2.40 1.20 1.30

M/s supreme Ltd.
Ans. 65:
(i) Comparative statement of sales and profit under marginal costing

Details 2002 2003
Sales Revenue `6,00,000 5,62,500
Less: variable cost 4,50,000 4,50,000
Contribution 1,50,000 1,12,500
Less: fixed cost 1,20,000 78,750
Profit 30,000 33,750

(ii) Minimum sales required, if the firm decides to shut down in units in 2003:
Minimum sales required is the sales which should yield at least the contribution, which is sufficient to meet
increase in fixed cost.
Increase in fixed costs in 2003 = `78,750 60,000 = `18,750
Sales required to yield contribution equal to increase in fixed cost
X* P/V retio = `18,750
Or x = `18750 / 0.20 = `93,750
Working notes
1. Computation of variable costs, break even point, profit and fixed cost for the year 2002:
Sales revenue `6,00,000
P/V ratio 25%
Margin on safety 20%
So, margin of safety = sales * 0.20
= `6,00,000 * 0.20 = 1,20,000
We know that margin of safety * P/V ratio = Profit
So, Profit: `1,20,000 * 0.25 = `30,000
Total contribution = sales * P/V ratio
= `6, 00,000 * 0.25 = `1,50,000
Variable cost = sales contribution
So, variable cost = 6, 00,000 1,50,000 = 4,50,000
Fixed cost = contribution Profit
= 1,50,000 `30, 000 = `1,20,000
Break even sales * P/V ratio = fixed cost
So, BES = 1,20,000 / 0.25 = `4,80,000

2. Computation of sales revenue, variable cost, fixed cost and profit in 2003
Let sales revenue for the year 2003 be x. the variable cost for the year 2003 is `4,50,000
(no. change).
So, contribution = X `4,50,000
= 20% (given)
We know that P/V retio =
Sales
contribution
Or, 20
100 =

X
X 4, 50,000
271
Or, 20x = `100x `4, 50, 00,000
Or, x = `4, 50, 00,000/80 = 5, 62,500
Margin of safety = 30% (given)
So, margin of safety = sales * margin of safety ratio
= `5, 62,500 * 0.30 = `1, 68,750
We know that sales margin of safety = B.E. sales
So, B.E. sales = `5, 62,500 `1, 68,750
= `3, 93,750

Ans. 66 (i )


Option I
At 75% in Feb and close in
March and April (`)
Option II
At 25% each from Feb
April (`)
Direct Material 5,25,000 5,25,000
Direct Labour 5,23,600 5,19,750


Factory Overhead :
Indirect Material
Two months idle
Indirect Labour
Training cost
Indirect Exp. :
Repairs & Maintenance
Over hauling cost
Others Expenses
Idle 2
10,48,600 10,44,750


8,400
9,800
1,01,500
65,800


28,000
14,000
52,500
53,200


14,700


1,78,500




84,000


1,02,900
Office overhead:
Staff Salaries
Idle 67,550 2
Other overheads
Idle


1,48,400
1,35,100
28,000
22,400


2,94,000


59,850
Total overhead cost 6,67,100 7,33,950
Total cost 17,15,700 17,78,700


The more economic course of action is to operate at 75% capacity for a month only,
and close the plant for March and April. This option will save (`17,78,700 `17,15,700) = `63,000.

Ans. 68:
(i)
Statement of Profitability of E Ltd. in Existing Situation
A B C Total
No. of units 10,000 25,000 20,000
` ` `
Selling Price per unit 40 75 85
Less: Variable Cost per unit
Direct Material 10 14 18
Direct wages 8 12 10
Variable Overhead 8 9 10
272
Contribution per unit 14 40 47
Total Contribution 1,40,000 10,00,000 9,40,000 20,80,000
Less: 1,60,000 Fixed Cost 4,50,000 4,00,000 10,10,000
Net Profit -20,000 5,50,000 5,40,000 10,70,000

Calculation of overall profit under each proposal
(ii)(a) If Product A is discontinued and capacity released is utilized for either B, either C or for both B and C

Revised contribution of Product B and Product C.
B(`) C (`)
Selling Price per unit 73.50 80.75
(75 2% of 75) (85 5% of 85)
Less: Variable cost per unit
Direct Material 15.40 18.90
(14 + 10%of 14) (18 + 5%of 18)
Direct Wages 12.00 10.00
Variable Overhead 9.00 10.00

Contribution per unit 37.10 41.85

Profitability Statement
Option 1 Option 2 Option3- Both B and C
equally
Only B Only C B C
No. of Units (as per W.N.1) 6,666 8,000 3,333 4,000
` ` ` `
Additional contribution 2,47,308.6 3,34,800 1,23,654.3 1,67,400
2,91,054.3
Savings from Fixed Cost of A 1,60,000 1,60,000 1,60,000
Reduction in contribution from A 1,40,000 1,40,000 1,40,000
Net Increase in Profit 267308.6 3,54,800 3,11,054.3
Existing Profit 10,70,000 10,70,000 10,70,000
Total Profit 1337308.6 14,24800 3,31,054.3
Hence, it is better to produce Product C only.

(ii)(b) Discontinue Product A and divert the capacity to produce Product D
A B C Total
Sales (units) 10,000 25,000 20,000
Labour Hrs. per unit 4 6 5
Total Labour Hours 40,000 1,50,000 1,00,000 2,90,000
Idle Capacity (hours) 2,90,000 * 20 / 80 72,500
Capacity released of A 40,000
Total hours released 1,12,500
Hours per unit 4
No. of units that can be produced 28,125

Profitability Statement
No. of units 28,125
`
Selling Price per unit 60
Less: Variable Cost per unit
Direct Material 28
Direct wages 12
Variable Overhead 6
Contribution per unit 14
Additional Contribution (D) 3,93,750
Less: 1,05,500 Additional Fixed Cost
Additional Net Profit 2,88,250
Add: Existing Profit (B & C) 10,90,000
Total Profit 13,78,250
273

(c) If we hire out the idle capacity
`
Idle hrs. 72,500
Profit per hour (10,70,000 / 2,90,000) 3.69
Total Profit 2,67,500
Existing Profit 10,70,000
Total Profit 13,37,500
Decision : Better to produce product C as per proposal (a)

Working Note-1: Hours release on discontinuation of Product A = 10,000 * 4
Only B Only C B and C equally
40,000 / 6 = 6,666 40,000 / 5 = 8,000 B- 3333 and C- 4000


Ans. 69:
1. Quantity analysis
Input in process A total capacity given = 2, 00,000 kg
Less: loss in process A = 10% of Input = 20,000 kg
Balance transfer to process B = 1,80,000 kg
(NRV at `1/ kg = 20,000)
Less: loss in process B = 5% of Input = 9,000 kg
Balance good output available for sale =
(NRV at `2 / kg = 18,000)
2. Supplier Evaluation and Decision
1,71,000 kg
Supplier P Q R R
Condition Max. 1,20,000 kg Max. 1,60,000 kg Any Quantity Qtty = 2,00,000 kg
Price
Var. Transport cost
Total
10.00
1.20
11.20
11.20
1.00
11.60
12.20
1.00
11.00
12.60
1.00
12.00
The following can be planned in any of the following ways
Total Purchase = 2, 00,000 kg
Purchase entirely from R 2,00,000 kg Purchase first kg. from P(least cost) and
balance 80,000 kg from Q (Next least cost)
1,20,000 * `11.20 + 80,000 * `12.20)
Cost incurred = (2,00,000 * `12) = 23,20,000
= 24,00,000
Decision: hence the company should Buy 1,20,000 kg from P and 80,000 kg from Q
Fixed transport cost being constant is not relevant to the above decision.
3. Customer evaluation and decision
Customer k L M
Condition Upto 80,000 kg only Upto 1,60,000 kg only All 1,71,000 kg
Selling price
Less: discount 2%
Net selling price
Less: var. transport cost
Net realization
65.00
63.70
1.30
2.60
64.00
61.10
62.72
1.28
1.44
61.80
61.28
61.80
NIL
NIL
61.80
The sales can be made in any of the following way
Total sale Quantity = 1,71,000 kg

Sold entirely to M 1, 71,000 kg sell first 1,60,000 kg to L (max. revenue)and
Balance 11,000 kg to K (next max. revenue)
Amt realized = (1,71,000 * `61.80) (1,60,000 * `61.28 + 11,000 * `61.10)
= `1,05,67,800 = 1,04,76,900
Less: fixed delivery cost NIL = `60,000 (`5,000 * 12 months)
Net amount = `1,05,67,800 = 1,04,16,900

274
Decision: since revenue is higher, the company should sell the entire quantity to customer M.
4. Statement of process costs

Particular Process A Process B
Raw materials (`23,20,000 + fixed transport
2,00,000)
Transport from previous process
Direct wages
Overheads
25,20,000

22,00,000
9,56,000

56,56,000
21,00,000
13,45,800
Total process costs
Less: scrap value of normal loss ( as in WN Above)
56,76,000
20,000
91,01,800
18,000
Net process costs transferred to subsequent
process/FG
56,56,000 90,83,800
Net profit: sales revenue costs of production = 1,05,67,800 90,83,800 = `14,84,000

(i) Reorder level = Safety Stock + lead time consumption
Ans. 70:
= 100 units + (3600 units/12) = 400 units
(ii) Anticipated reduction in the value of the average stock investment
EOQ =
2 Annual consumption Buying cost per order
Cost of carrying one unit of inventory for one year



=
2ab
cs

Where a = Annual consumption
b= Buying cost per order
c= Storage and other inventory carrying cost rate

=
2 3600
0 2 100
Rs. 40
. .
units
Rs


The average stock to be held under new system:
= minimum lavel + Reorder quantity
= 100 + * 120 = 160 units
The average stock investment under new system:
= 160 units * `100 = `16,000
The average stock under old system:
= Minimum level + EOQ
= 0 + (1800 units) = 900 units
The average investment under old system = 900 * `100 = `90,000
Therefore, anticipated average reduction in value of average stock investment
= `90,000 `16,000 = `74,000
(iii) The anticipated reduction in total inventory costs (in the first and subsequent years)
Under new system:
Annual ordering cost ((3,600/120) * `40) = `1,200
Stock holding cost (0.20 * `16,000) =
Total inventory cost
3,200
Under old system:
4,400
Annual ordering cost (2 orders * `40) = ` 80
Stock holding cost (0.20 * `90,000) =
Total inventory cost
18,000
Anticipated reduction in subsequent year:
18,080
Thus anticipated reduction in total inventory cost is `13,680 (i.e., `18,080 4,400) in subsequent years.
Anticipated reduction in the first year = `13,680 `10,000 * = `3,680
275
* In the first year 100 units will have to be purchased.

Particular
Ans. 71:
Current Policy A Policy B Policy C
Sales
Less: variable cost at 70%
4,50,000
3,15,000
5,00,000
3,50,000
5,40,000
3,78,000
5,65,000
3,95,500
Contribution
Less: fixed cost (given)
1,35,000
10,000
1,50,000
10,000
1,62,000
10,000
1,69,500
10,000
Profit before tax
Less: tax at 40%
1,25,000
50,000
1,40,000
56,000
1,52,000
60,800
1,59,500
63,800
Profit after tax 75,000 84,000 91,200 95,700
Cost of good sold (VC + FC)
Inventory turnover ratio (given)
Average inventory (COGS /T/o ratio)
Carrying cost of inv. At 5% (a)
Opportunity cost at 20 % of capital blocked in
average inventory (b)
Total cost of inventory holding (a + b)
Net benefit = total cost of inventory
3,25,000
10 times
32,500
1,625

6,500
8,125
66,875
3,60,000
8 times
45,000
2,250

9,000
11,250
72,750
3,88,000
6 times
64,667
3,233

12,933
16,166
75,034
4,05,500
4 times
1,01,375
5,069

20,275
25,344
70,356
Decision: As net benefit is Maximum under policy B, it may be chosen (alternative assumptions exist)

Working Note:
Ans. 72:
Fixed overheads `
Present sale value: (A) 15,00,000
(15,000 units `100)
Direct materials 4,50,000
(30% of sale value)
Direct labour 3,00,000
(20% of the value)
Variable overheads 3,00,000
(`20 per unit)
Total variable costs (B)
..
Contribution: (C) = (A) (B) 4,50,000
10,50,000
Profit : (D) 2,25,000
(15,000 units `15)
Fixed overheads: (C) (D) 2,25,000
.
(current level)
Add: Additional fixed overheads due to price escalation
Total fixed overheads:
50,000
Statement of profitability for various alternatives
2,75,000
Alternatives I II III IV
Rejecting
the
Rejecting the
proposal for
Accepting the
proposal of
Accepting the
proposal of the
276
proposal
for the
purchase
of 10,000
units and
continuing
with
present
level of
sales only
the purchase
of 10,000
units from a
party and
attaining the
maximum
capacity by
incurring
additional
selling
expenditure
the party to
take 10,000
units @ `90
per units by
installing a
balancing
equipment
and
continuing
with present
level of sales
party to take
10,000 units @
`90 per cent by
installing a
balancing
equipment and
attaining sale of
maximum
available
capacity by
incurring
additional selling
expenditure
Sale (units) 15,000 20,000 25,000 30,000
` ` ` `
Sales Value:
(A)
15,00,000
(15,000
`100)
20,00,000
(20,000
`100)
24,00,000
(15,000
`100+10,000
`90
29,00,000
(20,00,000
`100 + 10,000
`90)
Variable costs
Direct
material
4,95,000 6,60,000 8,25,000* 9,90,000*
(33% of sales value)
Direct Labour 3,75,000 5,00,000 6,25,000* 9,90,000*
Variable
overheads
3,00,000 4,00,000 5,00,000 6,00,000
(@`20 per unit)
Total variable
costs: (B)
11,70,000 15,60,000 19,50,000 23,40,000
Fixed costs
Fixed
overheads
2,75,000 2,75,000 2,75,000 2,75,000
(Refer to working note)
Additional
selling
expenditure
- 50,000 - 50,000
Depreciation
for balancing
equipment
- - 1,00,000 1,00,000
Additional
administrative
expenses
- - 50,000 50,000
Total fixed
cost : (C)
2,75,000 3,25,000 4,25,000 4,75,000
Total cost D:
[(B)+(C)]
14,45,000 18,85,000 23,75,000 28,15,000
Profit: (A)-(D) 55,000 1,15,000 25,000 85,000
Note: For computing the material and labour cost under alternative III & IV the notional sale price of `100 is
taken for additional 10,000 units.
Recommendation: Alternative II is the best as it gives maximum profit.

277
Comparative profit Statement (based on Revised Cost Structure)
Ans. 73:
Proposal 1 Proposal 2 Proposal 3
Sell 20,000 units
only
Secure orders for
5,000 additional
units (unused
capacity) and sell
25,000 units
Accept the new
order for 10,000
additional units
and sell 30,000
units
` ` `
Total sales revenue (A) 20,00,000 25,00,000 29,00,000
(`20,000 units
`100)
25,000 units
`100)
(30,000 units +
`33)
Director Labour 5,00,000 6,25,000 7,50,000
(20,000 units
`25)
(`25,000 units
`10)
(30,000 units +
`10)
Variable overhead 2,00,000 2,50,000 3,00,000
(20,000 units
`10)
(25,000 units
`10)
(30,000 units +
`10)
Fixed overheads 4,40,000 4,40,000 4,40,000
(`4,00,000 +
`40,000)

Add: Administrative
charges
- - 60,000
Add: Sales promotion
expenses
- 50,000 -
Depreciation (New
equipment)
1,50,000
Total cots (B) 18,00,000 21,90,000 26,90,000
Profit (C) = [(A) (B)] 2,00,000 3,10,000 2,10,000
Analysis
An analysis of the profit figures of M/s Unique products under three proposals clearly shows that it is
maximum under proposal 2. Therefore, it is advisable for the concern to produce and sell 25,000 units @
`100/- per unit and utilise its full production capacity.

Ans. 74
(as originally envisaged by the company)
(a) Statement of Profitability for the year 1993-94
Products Ethylene EDC VCL Total
Annual Production Capacity (MT)
Annual Planned Productions (MT)
(Refer to Note -1)
Cost of production of annual planned
production
25,000

25,000

`
30,000

25,000

`
30,000

15,000

`




`
278
Variable costs (Refer to Note 2)
Fixed cost (Refer to Note 3)
Common cost (Refer to Note 4)
Cost of Ethylene
Cost of Ethylene (Used for EDC)
Cost of EDC (25,000 MT)
Cost of 10,000 MT of EDC (Refer to Note 5)
Cost of 15,000 MT of EDC for (VCL)
Cost of Sale (A)
Sales Revenue (B)
Profit (B-A)
5,00,000
5,00,000
2,50,000
12,50,000

7,50,000
9,00,000
4,50,000

12,50,000
33,50,000
13,40,000

13,40,000
15,00,000
1,60,000
6,00,000
12,00,000
6,00,000




20,10,000
44,10,000
45,00,000
90,000








57,50,000
60,00,000
2,50,000
Note: Only 25,000 metric tonne of ethylene is available and as such 25,000 metric tonne of EDC
could be produced. Out of this 15,000 metric tonne of EDC is consumed for VCL production and the
balance of 10,000 metric tonne of EDC is sold.
Working Note:
Note: 1 annual planned production
Ethylene EDC VCL
Proposed Sale
Production:
For EDC
For VCL
Total
-
-
10,000
15,000
25,000
10,000
10,000

15,000
25,000
15,000
15,000

-
15,000

2. Variable Costs Ethylene EDC VCL
25,000 MT x `20 25,000 MT x `30 15,000 MT x `40
=`5,00,000 =`7,50,000 =`6,00,000

3. Fixed Cost
(This will be based on 25,000 MT x 20 30,000 MT x 30 30,000 MT x40
Production capacity) =`5,00,000 =`9,00,000 =`12,00,000

4. Common Cost
(This will also be based 25,000 MT x 10 30,000 MT x 15 30,000 MT x 20
On production capacity =`2,50,000 =`4,50,000 =`6,00,000

5. Cost of 25,000 metric tones of EDC = `33,50,000
Cost of one metric ton of EDC = `33,50,000 * 25,000 = `134
Cost of 10,000 metric tones of EDC = 10,000 x `134 = `13,40,000
Cost of 15,000 metric tones of EDC = 15,000 x `134 = `20,10,000

6. Sales Revenue
EDC = 10,000 MT x `150 =`15,00,000
VCL = 15,000 MT x Rs,300 =`
Total
45,00,000

60,00,000
(b) Revised Statement of Profitability
(When the company decides to accept offer of X)
Products Ethylene EDC VCL Total
Annual planned productions (MT) 25,000 25,000 30,000
279

Cost of production Refer (a)
Variable cost (30,000 MT x `40)
Production Fixed cost (30,000 MT x `40)
Common Fixed cost (30,000 MT x `20)
Purchases cost of 5,000 MT of EDC
@ `125 per MT
Total of EDC used in VCL
Total cost (A)
Total Sales
(Refer to note 1 below (B)
Profit (B-A)
`
12,50,000
`
33,50,000



6,25,000

39,75,000




`

12,00,000
12,00,000
6,00,000


39,75,000
69,75,000

80,00,000
10,25,000
Comment Since the profit has increased the proposal of X should be accepted.
Note 1: Total Sales :
20,000 MT of VCL to X @ `250 per MT =`50,00,000
10,000 MT of VCL X `300 (in open Market) =`
Hours
30,00,000
Available capacity 20,000
First product D should be produced (2,800 x 6)
Balance hours 3,200
16,800
Second product A should be produced (2,000 x 1)
Balance hours 1,200
2,000
Third product B should be produced (600 x 2) 1,200

Thus, if 20,000 hours is the limiting factor, all requirements of D and A can be manufactured and
only 600 units of product B can be manufactured. The balance requirement of product B.
i.e.,3,500-600 =2,900 units will have to be bought out or manufactured in the second shift.
(b) Because purchase price of component c is `52 and cost of manufacturing is `57, it will not be
profitable to manufacture C even in second shift. It should be purchased form outside, purchased
from outside. The relative position is as follows:
Cost of producing 2,900 units of product B in second shift


Solution (a) Working Notes
Ans. 75:
(i) Press hours required
Components A B C D
Direct expenses
Direct hours per unit
`10
1
`20
2
`10
1
`60
6

(ii) Marginal cost per unit vs. bought-out prices per unit
Marginal costs:
Direct Materials
Direct wages
Direct expenses
Marginal costs
Bought out price
Excess of bought out price over
marginal cost
Process hours per unit

`37
10
10
57
60

3
1

`27
8
20
55
59

4
2

`25
22
10
57
52

(5)
1

`44
40
60
144
168

24
6
280
Excess of bought out price per unit
of limiting factor
Ranking

3
11

2
111

(5)
-
4

1

The bought out price of component C is lower than the marginal cost by `5 and for this reason
it should be purchased from outside. For the remaining products. Ranking is based on utilization
of limiting factor. Optimal product mix has been, calculated as follows:

Calculation of optimal product mix

Variable Cost `55
Increase in direct wages

2
Total variable cost (2,900 x 57) `1,65,300
57
Additional fixed cost
Hours required = 2,900 x 2=5,800 hours
Extra fixed cost of 5,800 hours at `500 for each 1,000 hours or part thereof
Total cost for producing 2,900 units of product B in second shift 1,68,300
3,000
Bought- out price for 2,900 units of product B will be 2,900 x `59
Disadvantage in buying B
1,71,100
2,800


For the above-mentioned reasons, it is in the interest of company to manufacture product B in the
Second shift instead of buying it from outside market .The disadvantage of the decision to buy
product B from outside will be `2,800 .
80,00,000
Components
Ans. 76:
P Q R S
i. Direct wages
ii. D.L.H. @ `8.75 p.h
iii. Variable Mfg. cost
iv. Purchase Price
v. Saving if components are
manufactured
vi. Saving per hour (5 * 2)
Ranking
`17.50
2
`99.75
105.00

5.25
2.625
2
`35.00
4
`96.25
103.00

6.75
1.6875
3
`17.50
2
`99.75
91.00

-
-
`105.00
12
`252.00
294.00

42.00
3.50
1

(i) Statement showing product-mix of the components to be manufactured
(Available hrs. = 40,000)
Component Qty. reqd. Hrs. / unit Production Hrs. Used Balance hrs.
S
P
Q
2,400
2,400
4,800
12
2
4

2,400
2,400
1,600
28,800
4,800
6,400
11,200
6,400
-
Components to be manufactured= S = 2,400
P = 2,400
Components to be purchased = Q = 1,600
Q = 3,200
R = 1,200
*6,400 hrs * 4 = 1,600
281

(ii) Statement showing impact of second shift working


Additional quantity of Q required = 3,200
Hours required to manufacture (3,200 x 4) = 12,800
Say = 13,000
Fixed cost (`875 * 1,0000 ) x 13,000 = `11,375
Fixed cost per component Q (11,375 * 3,200) =`3.55
Increase in labour cost (`35 x 25%)
Total 12.30
8.75
Saving in cost
Loss if component Q is manufactured
6.75

5.55
Hence, second shift operation is not recommended
Fixed cost given per 1,000 hours

Since S and Y are produced simultaneously from an input of raw material Z, therefore when additional
60,000 kgs. of Y will be produced then 30,000 of S will also be produced simultaneously. The input of
material Z required for these additional 60,000 kgs. of Y and 30,000 kgs. of S will be 90,000 kgs. of
material Z. Hence the cost of processing 90,000 kgs. of material will be as follows:
Ans. 77:
`
Cost of Raw material Z 2,70,000
(90,000 kgs. `3)
Variable processing cost 1,80,000
(90,000 kgs. `2)
Total cost of processing 4,50,000
Less: Sales revenue from 60,000 kgs. of Y 2,40,000
(60,000 kgs. `8)
Balance cost to be recovered 2,10,000
Current sales revenue from the sale of 3,00,000 kgs. of S 24,00,000
(3,00,000 kgs. `8)
Total sales revenue to be earned from the Sale of S 26,10,000
(3,00,000 kgs. + 30,000 kgs.)
Hence minimum reduced price per kg. of S to recover `26.10,000 from 7.91
the sale of 3,30,000 kgs. of S
(`26,10,000 / 3,30,000 kgs.)

Working notes:
Ans. 78:
1. Statement of total available, utilized and surplus capacity hours when 9,000 units of product X are
produced.
282
Departments Available Capacity
hours
Capacity utilized Surplus
Capacity hours
(in % (in hours)
(1) (2) (3) (4) = (2)(3) (5)=(2)-(4)
A 2,400
(300 days 8
hours)
75 1,800 600
B 2,400 100 2,400 NIL
C 2,400 70 1,680 720
D 2,400 50 1,200 1,200
2. Statement of total available, utilized and surplus capacity hours when 12,000 units of product X are
produced.
Production
Department
Available
capacity
hours
Capacity utilization on
9,000 units Hours
Balance
capacity
hours
Unit per hour Hours
required
for 3,000
additional
units
Surplus
capacity
hours
(1) (2) (3) (4)=(2)(3) (5) (6) (7) (8)=(5)-
(7)
A 2,400 75 1,800 600 5
|
.
|

\
|
hrs. 1,800
units 9,000

600 Nil
B 2,400 100 2,400 Nil 3.75
|
.
|

\
|
hrs. 2,400
units 9,000

800 Nil
C 2,400 70 1,680 720 5.36
|
.
|

\
|
hrs. 1,680
units 9,000

560 160
D 2,400 50 1,200 1,200 7.5
|
.
|

\
|
hrs. 1,200
units 9,000

400 800

Alternative I
Statement of net Revenue (Under Alternative I)
Production Surplus
capacity
hours
(Refer to
W.N.-1
Hire
charges
per hour
Total
revenue in
(`Lacs)
Incremental
costs per
hour `
Total cost
in (`Lacs)
Net
revenue
in (`)
(a) (b) (c)=(a)(b) (d) (e)=(a)(d) (f)=(c)-(e)
A 600 2,500 15.00 2,000 12.00 3.00
B 720 1,800 12.96 1,500 10.80 2.16
D 1,200 1,600 19.20 1,200 14.40 4.90
Total 47.16 37.20 9.96
Add: present income (10% of `1,800 lacs)
Total return
180.00
189.96
283
Return on investment
=
investment Total
return Total
100 =
1,800
189.96
100 = 10.553%
Alternative II
Statement of Net Revenue when 12,000 units of product X are produced and surplus plant capacity (hours)
in departments C and D hired out.
Production Surplus
capacity
hours
(Refer to
W.N.-2)
Hire
charges
per hour
Total
revenue in
(`Lacs)
Incremental
costs per
hour `
Total cost
in (`Lacs)
Net
revenue in
(`Lacs)
(1) (2) (3)=(1)(2) (4) (5)=(1)(4) (6)=(3)-(5)
C 160 1,800 2.88 1,500 2.40 0.48
D 800 1,600 12.80 1,200 9.60 3.20
Total 15.68 12.00 3.68
Add: Revenue (in lacs) earned on 3,000 additional units sale (3,000 units is `1,600)
Add: Present income on investment (10% `1,800 lacs)
48.00
Total Return (in lacs)
180.00
Return on investment =
231.69
lacs 2,200
lacs 231.68
100 = 10.53%
Evaluation of two alternative proposals :
Since the return on investment under alternative I is more than that under alternative II; therefore it should be
accepted.

(i) Statement of Profitability of three Joint Products resulting from the joint production process
of a popular line of colognes.
Ans. 79:
Evergreen Morning
Flower
Evening
Flower
Total
` ` ` `
Sales revenue 4,00,000 6,00,000 6,00,000 16,00,000
(10,000 units
`40)
(6,000 units
`100)
(4,000 units
`150)
--
Less: cost
after point of
split off
2,00,000
(10,000 units
`20)
2,40,000
(6,000 units
`40)
2,00,000
(4,000 units
`50)
6,40,000
--
Net realization
value at the
point of spilt
off
2,00,000 3,60,000 4,00,000 9,60,000
Less: Joint
cost
apportioned
(Refer to
working note)
1,16,667 2,10,000 2,33,333 5,60,000
Profit 83,333 1,50,000 1,66,667 4,00,000
284
Response to the Presidents question. Review of the above profitability statement clearly shows that
the concern is not selling its largest-volume product viz. evergreen at a loss. It yields a profit of
`83,333. In fact the figure of joint cost data given in the statement of the question is misleading. The
total joint cost viz. `5,60,000 should have been apportioned ever the three joint products by using net
realisable value method. The use of net realisable value method would give joint cost per-unit of
three respective joint products as `11,666; `35 and `58.33. (Refer to working note)
Working note:
Statement of Joint cost apportionment over three products obtained under a joint production process.
Evergreen Morning
Flower
Evening
Flower
Total
` ` ` `
Total Joint cost 2,80,000 1,68,000 1,12,000 5,60,000
(10,000
units
`28)
(6,000 units
`28)
(4,000 units
`28)
--
Joint cost
apportionment (One
the basis of net
realization value i.e.
(`2,00,000 :
`3,60,000 : `4,00,000
or (5:9:10)
1,16,667 2,10,000 2,33,333 5,60,000
--
Joint cost per unit 11,666
(`1,16,667
/ 10,000
units)
35 (`2,10,000 /
6,000 units)
58.33
(`2,33,333 /
4,000 units)

(ii) Should the company sell Morning Flower Cologne below cost: To compete successfully with the
other companys product, if the price of Morning Flower Cologne is reduced to `60, it will still
contribute `20 per unit (`60 `40) towards joint cost and profit. On a volume of 6,000 units it will
contribute `1,20,000 in total. Hence the company should do so and go ahead to sell Morning Flower
below cost.
(iii) Response to price reduction: (Refer to working note) A reduction in sales price of Morning Flower
fails to maintain a gross margin of 20% on sales of three products obtained from the joint production
process of a popular line of colognes. Hence the company cannot reduce the sales price of Morning
Flower to `60. A reduction in sale price would result in a loss of revenue of `1,40,000.
Working note:
`
Total joint cost (20,000 units `28) 5,60,000
Total cost after split off (10,000 `20 + 6,000 units `40 + 4,000
units `50)
Total cost
6,40,000
12,00,000
Add: Profit margin (20% on ales or 25% on total cost 3,00,000
Expected desired sales revenue
Less: Sales revenue of Evergreen and Evening Flower (10,000
units `40) + (4,000 `150)
15,00,000
10,00,000
Expected sales revenue from Morning Flower 5,00,000
By reducing sales price of morning flower to `60/- total sale
revenue received will be
3,60,000
Loss of revenue resulting from the sale of Morning Flower 1,40,000
(iv) Minimum price for Morning Flower
285
Expected Sales revenue from Morning Flower to maintain a gross
margin of 20% of sales: (`)
5,00,000
(Refer to (ii) part
Quantity (in units) 6,000
Hence minimum price per unit (`) 83.33
(`5,00,000 / 6,600 units)

(i) (a) Statement showing apportionment of joint costs sales value at split-off
Ans. 80:

Products Sales in tones (a) Selling price per ton
(`) (b)
Sales value (`)
(c) = (a) * (b)
Apportioned joint
cost (`)
Caustic soda
Chlorine
2,400
1,600
100
150
2,40,000
2,40,000
1,00,000
1,00,000
Total 4,80,000 2,00,000

*Apportioned joint cost = Total joint cost
Total sale value
* sale revenue of each product.

Apportioned joint cost to caustic soda = `2,00,000
`4,80,000
* `2,40,000 = 1,00,000

Apportioned joint cost to chlorine = `2,00,000
`4,80,000
* `2,40,000 = `1,00,000

(b) Statement showing apportionment of joint costs on physical measure (tons)
Products Sales in (tons) Apportioned ** joint costs (`)
Caustic soda
Chlorine
2,400
1,600
1,20,000
80,000
Total 4,000 2,00,000

**Apportioned joint cost = Total joint cost
Total sales (tons)
* sales of each product (tons)

Joint cost apportioned to caustic soda = `2,00,000
`4000 tons
* 2,400 tons = `1,20,000

Joint cost apportioned to chorine = `2,00,000
`4,000 tons
* 1,600 tons = `80,000

(c) Statement showing apportionment of joint costs by using estimated net realizable value method

Products Sales revenue (`) Further pro-cessing
cost (`)
Net realizable value
(`)
Apportioned ** joint
cost (`)
Caustic soda (2,400
tons * `100)
PVC (1,000 tons of
PVC * `400)
2,40,000

4,00,000
-

40,000
2,40,000

3,60,000
80,000

1,20,000
Total 6,00,000 2,00,000

***Apportioned joint cost = Total joint cost
Total net realizable value
* Net realizable value of each product.


Apportioned joint cost for caustic soda = `2,00,000
`6, 00,000
* `2,40,000 = `80,000

286
Apportioned joint cost for chlorine = `2, 00,000
`6, 00,000
* `3, 60,000 = `1, 20,000


(ii) Statement of gross margin percentage of caustic soda and PVC under sales value at split off: physical measure (tons)
and estimated net realizable value method
Sale value at split off (`) Physical measure (tons) (`) Estimated net realizable
value (`)
Caustic soda
Sale revenue : (A)
Joint cost allocated : (B)
Gross margin (C) : (A)-(B)

Gross margin (%) (C) *
(A)
100
(b) PVC:
Sales revenue (A)
Joint cost allocated
Further processing cost
Total cost
Gross margin (c) : (A)-(B)

Gross margin (%) (C)
(A)
*100

2,40,000
1,00,000
1,40,000

58.33%


4,00,000
1,00,000
1,40,000
40,000
2,60,000

65%

2,40,000
1,20,000
1,20,000

50%


4,00,000
80,000
1,20,000
40,000
2,80,000

70%

2,40,000
80,000
1,60,000

66.67%


4,00,000
1,20,000
1,60,000
40,000
2,40,000

60%

(iii) Consequence of the operating income of inorganic chemicals for November, 1998 by accepting the offer of daily
swimming pools Ltd. to purchase, 1,600 tons of chlorine
Incremental revenue (loss) due to processing of chlorine to PVC (`1, 60,000)
(1,600 tons * `150) (1,000 tons * `400 tons)
Saving on further processing cost of chlorine into PVC
Incremental operating income
40,000

(`1, 20,000)
The operating income of inorganic chemicals will be reduced by `1,20,000 in the month of November, 1998 if it accepts
the offer of daily swimming pools Ltd., to purchase 1,600 tons of chlorine in November, 1998 at `150 per ton.


Ans. 81:
(i) Statement showing the product to be manufactured and sold and the result contribution
Aristocrat deluxe

Maximum possible production in unit (Note1)
S. P. per unit `90.00 `80.00
Less: variable costs: Aristocrat deluxe
Direct material `10.00 `10.00
Variable costs:
Deptt. A (0.5*`50; 0.3 * `50) 25.00 15
Deptt. B (0.4 * `60; 0.45 * `60) 24.00
Total variable cost per unit
27.00
59.00 52.00 59.00 52.00
Contribution per unit 31.00 28.00


Total contribution per unit 6,800 * `31; 8,500 * `28 `2,10,800 `2,38000 Form the above,
it is apparent that sale of `8,500 units of deluxe model produces the maximum contribution of `2,38000 within the
capacity and material constraints. Therefore, 8,500 units of deluxe model should be produced.
(ii) statement showing the maximum contribution on the sale of aristocrat or deluxe models and hiring out the surplus
capacity in departments A and B
Aristocrat deluxe
Total contribution on sale of maximum possible production
as per (i) above `2,10,800 `2,38,000

Contribution on hiring capacity (Note 2):
287
Aristocrat Deluxe
Deptt. A Nil 850 * `40 - 34,000
Deptt. B 1,120 * `60 15 * `60 67,200
Total contribution
900
2,78,000
It is noticed that total contribution of the company would be maximum i.e. `2,78,000 on the sale of 6,800 units of
aristocrat model and hiring out the surplus capacity of the two departments.
2,72,900

(iii) Statement showing total contribution of company when 4,250 units of each product are manufactured and surplus
capacity of Deptt. A and/or Deptt. B hired out
Aristocrat Deluxe Total
(a) Production (units) 4,250 4,250
(b) Contribution per unit as at (i) above `31 `28
Total contribution (a) * (b) `1,31,750 `1,19,000 `2,50,750
Contribution earned on hiring the surplus
capacity of Deptt. B (Note 3)

13,650*
This proposal is less profitable then proposal at (ii) above
2, 64,400

Working Note:
Maximum capacity or production is given in hours. But part (i) required production to be stated in units. The same has
been worked out as under:
Deptt. A Deptt. B
Maximum capacity in hours 3,400 3,840
Aristocrat Deluxe
Maximum hour per unit - Deptt. A 0.50 0.30
Deptt. B 0.40 0.45
Maximum possible production (in unit) constant
Maximum capacity
Deptt. A: 3,400/0.5; 3,400/0.30 6,800 11,333
Deptt. B 3,840/0.40; 3,840/0.45 9,600 8,533
Maximum possible production (in unit) constant
Available material 17,000 kgs/2 kgs 17,000/2 kgs 8,500 8,500
Maximum possible production considering both
Capacity and material constants 6,800 8,500

2. Surplus capacities
Deptt.A Deptt. B
(a) Maximum possible hours 3,400 3.840
(b) Capacity used when 6,800 units of aristocrat model
are produced (0.50 * 6,800; 0.40 * 6,800) 3,400
(c) surplus capacity with aristocrat model
3,840
NIL
(d) Capacity used when 8,500 units of deluxe model are produced
1,120
(0.3 * 8,500; 0.45 * 8,500) 2,550 3,825
(e) Surplus capacity with deluxe model (a)-(d) 850 15
Deptt. A Deptt. B
3. Maximum possible hour as in unit Note 1 3,400 3,840
Hour utilized aristocrat 4,250 * 0.50; 4,250 * 0.45 (-) 2,125 (-) 1,700
Deluxe (-) 1,275
Surplus capacity (hours) NIL NIL
(-) 1912.5

Brightly
Ans. 82:
Unit
price
`
Contrib
ution
per unit
`
Volume
Units
Total
contributi
on
(`in 000)
Increment
al
contributi
on
(`000)
Labou
r
hours
Increme
ntal
labour
hours
Increment
al
contributi
on per
labour
hour
`
Rank
288
276 176 12000 2112 2112 2400
0
24000 88 2
272 172 14000 2408 296 2800
0
4000 74 6
268 168 16000 2688 280 3200
0
4000 70 7
264 164 18000 2952 264 3600
0
4000 66 8
260 160 20000 3200 248 4000
0
4000 62 9
254 154 22000 3388 188 4400
0
4000 47 10
Lightly
Unit price Contribution
per unit
Volume Total
contribution
(`in 000)
Incremental
contribution
(`000)
Labour
hours
Incremental
labour
hours
Incremental
contribution
per labour
hour
Rank
163 103 40,000 4120 4,120 40,000 40,000 103 1
162 102 42,000 4284 164 42,000 2,000 82 3
161 101 44,000 4444 160 44,000 2,000 80 4
160 100 46,000 4600 156 46,000 2,000 78 5
156 96 48,000 4608 8 48,000 2,000 4 11
152 92 50,000 4600 (8) 50,000 2,000 (4) Loss

As the labour time is scarce source (time available 78,000 hours), the decision has to be taken on the basis of
ranks based upon incremental contribution per labour hour.
Product Price Incremental
volume
Incremental
labour
hours
Balance
hours
Incremental
Contribution
(in 000 `)
Lightly 163 40,000 40,000 38,000 4120
Brightly 276 12,000 24,000 14,000 2112
Lightly 162 2,000 2,000 12,000 164
Lightly 161 2,000 2,000 10,000 160
Lightly 160 2,000 2,000 8,000 156
Brightly 272 2,000 4,000 4,000 296
Brightly 268 2,000 4,000 280
Total 7,288

Hence product mix is Brightly 16,000 units and Lightly 46,000 units
Optimal contribution per month `72,88,000
Fixed costs per month `60,00,000
Optimal profit per month `12,88,000
Working Notes:
Brightly Lightly
Variable cost
(p.u.)
100 Rs.
12,000) 000 , 16 (
4,00,000) 3 000 , 00 , 38 (
=

60 Rs.
0,000) 4 000 , 48 (
2,00,000) 6 000 , 80 , 66 (
=


Fixed cost (`) 22,00,000 38,00,000
Contribution = Unit selling price less variable cost per unit.
289

Statement showing computation of selling price per unit
Ans. 83:
Months 1-3 4-9 10-12 Total
Total number of unit
Produced (Note 4) `28,125 `60,000 `33,750
Variable cost `2,81,250 `6,00,000 `3,37,500 `12,18,750
1,21,875
Labour cost (Note 2) 3,00,000 6,00,000 3,37,500 12,18,750
Overheads 1,12,500@ 2,40,000@ 1,35,000@ 4,87,500
Total sami variable overheads (Note 3) 72,000#
Fixed overheads
Total costs
1, 92,300
Add: profit: (20% on selling price or 25% on cost)
32, 08,050
Sales revenue
8, 02,013
Selling price per unit ( `40,10,063/1,21,875 on cost)
40, 10,063

32.90

Working notes
1. Average installed capacity per month (in units):
= Total annual installed capacity/12 month (in units):
= 1, 50,000 units/ 12 months = 12,500 units per month.
2. Total labour cost at different capacity utilization:
Capacity utilization 75% 80% 90%
Expected production per month ( in units) 9,375 10,000 11,250
Labour cost of expected production (`) 93,750 1, 00,000 1, 12,500
Minimum lqbour cost per month (`) 1, 00,000 1, 00,000 1, 12,500
Capacity utilization (in months) 3 6 3
Total labour cost at different capacity levels `3, 00,000 `6, 00,000 `3, 37,500

@28125 `4; 60000 `4; 33750 `4
#This can also be taken based on average capacity utilization i.e. ( 121875150000) 100 = 81.25%. Therefore, semi-
variable overheads can also be taken as 68000 (refer note 3). In that case, selling price will be `32.87.


Ans. 84:
Part A Part B

Target Price (`) 145 115
Less : Variable Cost p.u. (`)
Material(1.6 kg. @ `12.5 p.kg.) (`) 20 20
Variable OH Machine A (0.6/0.25 hrs @ `80 p.h.) (`) 48 20
Variable OH: Machine B (0.5/0.55 hrs @ `100 p.h.) (`) 50 55
Total Variable Cost p.u. (`) 118 95
Contribution p.u. (`) 27 20

Number of parts can be manufactured on the basis of:
Alloy Available (13000kg 1.6/1.6) 8,125 8,125
Machine A (4000 hrs 0.6/0.25) 6,666 16,000
Machine B (4500 hrs 0.5/0.55) 9,000 8,181
Maximum units that can be manufactured 6,666 8,125

Total Contribution (6,666 units 27; 8,125 20) 179,982 162,500
Hence it is recommended to produce Part A.

(b) Parts A to be Manufactured 6,666 units
Hours utilized Idle hours
Machine A usage (6,666 0.6) 3,999.6 0.4
Machine B usage 3,333 1167
Compensation for unutilized machine hour (1167.4 @ Rs 60/ hour) `70,044
Revised contribution after reduction of 10% in S.P. [6,666 (145 0.9 118)] `83,325
290
Total Contribution `153,369


Ans. 85:
Cutting Finishing
Capacity (units) 10,000 5,000
Selling Price 1000
Material Cost 400
Throughput contribution 600 `/u.

(i) Throughput Contribution 600

Subcontracting changes

400

200
Increase in throughput contribution = 200 x 5000 = 10,00,000

(ii) Already cutting has surplus capacity. It is not a bottleneck. Do not outsource as there will be
no benefit, instead there will be reduction of or throughput contribution of outsourced.
(iii) Cutting has surplus capacity. Do not increase non-bottleneck capacity.

Contribution analysis:
Ans. 86:
Product X Product Y
` `
Selling price 288 432
Variable costs:
Direct materials 40 80
Direct Labour: 48 72
24 48
72
96
Variable overheads 32 28
Total variable costs 216 324
Contribution per unit 72 108
The direct labour hours required to manufacture the two products in each of the four departments at the wage
rate of `8 per hour are as under:
Department Product X Product Y
Wage cost Hours/unit Wage cost Hours/unit
1 48 6 72 9
2 24 3 48 6
3 72 9
4 96 12

Department 3 is used only for product X and department 4 is used only for product Y. Hence, these two
departments will determine the maximum production of these two products as under:
Department 3 : Maximum available hours:
Workers Hours/day Days/year
27 8 300 = 64,800 hours
291
Maximum possible production of product X:
unit per hrs 9
64,800
= 7,200 units
Department 4 : Maximum available hours:
Workers Hours/day Days/year
36 8 300 = 86,400 hours
Maximum possible production of product Y:
unit per hrs 12
86,400
= 7,200 units
The company can produce 7,200 units each of products X and Y provided departments 1 and 2 have capacity
to process this quantity of output.
We can check the capacity of departments 1and 2 as under:
Department 1: Maximum available hours:
Workers Hours/day days/year
45 8 300 = 1,08,000 hours
Hours required to produce 7,200 units each of X and Y:
Product X 7,200 6 hours = 43,200 hours
Product Y 7,200 9 hours = 64,800 hours
Total = 1,08,000 hours
Department 1 has capacity to produce 7,200 units each of products X and Y.
Department 2 : Maximum available hours:
Workers Hours/day days/year
24 8 300 = 57,600 hours
Hours required to produce 7,200 units each of X and Y:
Product X 7,200 3 hours = 21,600 hours
Product Y 7,200 6 hours = 43,200 hours
Total = 64,800 hours
Department 2 has scarce capacity.
Since department 2 capacity is scarce, link the contribution to the key factor of department 2 hours as under:
Product X Product Y
Contribution per unit 72 108
Department 2 hours per unit
Hours
3 6
Contribution per hour of Department 2 ` 24 18
Rank 1 2
Optimal product mix:
Product Max.
units
Lab.
Hours/unit
Prod.
units
Hours
used
Balance
hours
Cont./unit Total
cont.
` `
X 7,200 3 7,200 21,600 36,000 72 5,18,400
Y 7,200 6 6,000 36,000 108 6,48,000
Total optimal contribution 11,66,400
Fixed costs 5,00,000
Optimal profit 6,66,400
Alternative Solution:
292
The maximum possible production of product X is 7,200 units and that of product Y is 7,200 units. The
following two methods shall be used to determine the optimal profit:
(a) Produce 7,200 units of product X and use the balance capacity to produce product Y.
(b) Produce 7,200 units of product Y and use the balance capacity to produce product X.
Profitability based on (a):
Direct labour hours are scarce in Department 2.
Maximum available hours in Department 2 57,600
Product X requires 7,2003= 21,600 hours
Balance hours on Y 36,000
Production of Y 36,000 6= 6,000 units
Contribution: X 7,200 units
@ `72
`5,18,400
Y 6,000 units
@ `108
`6,48,000
Total `11,66,400
Fixed costs `5,00,000
Profit `6,66,400

Profitability based on (b):
Maximum available hours in Department 2 57,600
Product Y requires 7,2006= 43,200 hours
Balance hours on X 14,400
Production of X 14,400 3= 4,800 units
Contribution: X 4,800 72 `3,45,600
Y 7,200 108 `7,77,600
Total `11,23,200
Fixed costs `5,00,000
Profit `6,23,200
Profitability of (a) is better.

(a) Statement of Cash Receipts, Disbursements and cumulative difference in Cash flows for four years taken
together under both alternatives
Ans. 87:
(`in thousands)
Alternatives Keep old machine Buy new machine
Year
1
2
nd
3
rd

& 4
th
All 4
years
year
each
Year 1 2
nd
3
rd

& 4
th
All 4
years
year
each
Cumulative
difference in
cash flows
for four years
taken
together
Receipts
Sales revenue 150 150 600 150 150 600
Self of old - - - 8 - 8
293
equipment
Total receipts :
(A)
150 150 600 158 150 608
Disbursements
Annual operating
cost
15 15 60 9 9 36
Other cash costs 110 110 440 110 110 440
Purchase cost of
old machine
20 - 20 20 - 20
Purchase of
new machine
- - - 24 - 24
Total
disbursements :
(B)
145 125 520 163 119 520
Net cash in-
flows: (A)-(B)
5 25 80 (5) 31 88 08
(b) Statement of income for each of the four years and cumulative difference in operating income.
Alternatives Keep old machine Buy new machine
1
st
, 2
nd
3
rd
&
4
th
All
years year
each
Year 1 2
nd
3
rd

& 4
th
All 4
years
year
each
Cumulative
difference
operating
income
Income
Sales revenue 150 600 150 150 600
Total revenue : (A) 150 600 150 150 600
Costs:
Annual operating
cost
15 60 9 9 36
Other cash costs 110 440 110 110 440
Depreciation 5 20 6 6 24
(Refer to working note 1)
Loss on the
disposal of old
machine
- - 12 - 12
(Refer to working note 2)
Total costs: (B) 130 520 137 125 512
Operating income:
(A)-(B)
20 80 13 25 88 08
(c) The purchase of cost old machine `20,000; the sale revenue `1.50,000 and other cash costs of `1,10,000
as irrelevant items for the presentation in requirements (a) and (b) above. These items are irrelevant
because their amounts are common to both the alternatives.
(d) The net difference in requirements under (a) and (b) will not change if the cost of old machine becomes
`10,00,000 instead of `20,000. This is so because the cost of old machine is common for both the
alternatives.
(e) In the decision about eh replacement of machine the book value of the machine is irrelevant because it is
a past (historical) cost. All past costs are down the drains. Nothing can change what has already
happened. As apparent from (a) and (b) above; we can completely ignore the cost of old machine i.e.
`20,000 and still have a correct analysis.
294
Working note:
1. Depreciation (according to straight line method):
Old machine New machine
(i) cost of machine (`) 20,000 24,000
(ii) Terminal disposal value Zero Zero
(iii) Useful life 4 4
Depreciation
|
|
.
|

\
|
(iii)
(ii) - (i)
` 5,000 5,000
2. Loss on the disposal of old machine:
` `
Purchase price of old machine 20,000
Disposal value 10,000
Less: Removal cost 2,000

8,000
12,000
Evaluation of Make or Buy proposal
Ans. 88:
(All figures are in lakhs in rupees)
Year P.V. factors
at 10%
When the component is
manufactured
When the component is
bought from an outside
supplier
Cash outflow
(Capital cost +
manufacturing
cost +
opportunity
cost)
Present Value Cash
outflow
(Buying
cost)
Present
Value
` ` ` `
(a) (b) (c) (d)=(b)(c) (e) (f)=(b)(e)
0 1.000 4 4.000 - -
1 0.909 6+2 7.272 9 8.181
2 0.826 7+2 7.434 10 8.260
3 0.751 8+2 7.510 11 8.261
4 0.683 10+2 14 8.196
Total
9.562
34.412
(
(
(
(

(
(
(
(

=
outside. from
bought is component
the when outflow, cash
of value present Total
internally
ed manufactur is component
the when outflow, cash
of value present total
outside) from brought (when
outflow cash in Saving
24.264

= `24.412 `34.264
= `0.148 (lakhs)
Conclusion: Since there is a saving of `0.148 (lakhs) in buying the component from outside, therefore, we
should stick to this decision.
295
Note: The loss of `2 lakhs cash inflow for each of the four years due to the inability of the firm to operate
another machine if it manufactures the component has been treated as an opportunity cost.

Proposal I
Ans. 89:
Statement of sales revenue of mild
Year (a) Quantity of mild in
metric tonnes (b)
Price per metric
tonne (c)
Total amt of sales
in (`Lacs) (d) =
(b) * (c)
Discount factor
@ 12% (e)
NPV of sales
(`In lacs)
(f) = (d) * (e)
1
2
3
4
5
15,000
15,000
15,000
15,000
15,000
950
900
850
800
750
142. 5
135.0
127. 5
120.0
112. 5
0.89
0.79
0.71
0.64
0. 57
126.825
106.65
90. 525
76.800
64.125
464. 925
Proposal II

Year
(a)
Quantity of
medium in
metric tones
(b)
Price per
metric
tone
(c)
Variable
cost per
metric tone
(`) (d)
Net price per
metric tone (`)
(e) =(c) (d)
Net sales
revenue in
(`Lacs.) (f) =
(b) * (e)
Discount
factor @
12% (g)
PV of net sales
revenue
(`Lacs.) (h) =
(f) * (g)
1
2
3
4
5
1,000
2,000
3,000
4,000
5,000
1,200
1,300
1,400
1,500
1,600
200
200
200
200
200
1,000
1,100
1,200
1,300
1,400
10
22
36
52
70
0.89
0.79
0.71
0.64
0.57
8.90
17.38
25.56
33.28
39.90
Total 125.02
Note: since the selling price of medium is not given after second year, therefore an individual is free to talk any selling
price after second year. In view of this assumption the answer of each case may differ.

Year (a) Quantity of mild
in metric tones
(b)
Price per metric
tone (`) (c)
Sales revenue in
(`Lacs) (d) = (b)
* (c)
Discount factor
@ 12% (e)
PV of sales revenue
in (`Lacs) (f) = (d)
* (e)
1
2
3
4
5
14,000
13,000
12,000
11,000
10,000
950
900
850
800
750

133
177
102
88
75
0.89
0.79
0.71
0.64
0.57
118.37
92.43
72.42
56.32
42.75
Total 382.29
Total present value of sales of medium and mild under proposal II (`Lacs) 507.31
(`125.02 lacs `382.29 lacs)
Total net present value under proposal II
(`507.31 lacs `30 lacs)
The net present value under proposal I is `464.925 lacs, and that under proposal II is `477.31 lacs. A comparison of the
net present value under two proposal clearly shows that the proposal II is better as it yield a higher net present value of
revenue, therefore it should be accepted.

Ans: 90
(i) 15,000 tins scrapped per month can be converted into 75,000 lids. (Each rejected tin can
be converted into 5 lids) unusable tins are sold as scrap at `8 per unit. Hence, `8 can be taken as raw
material cost for conversion into lids.
15,000 tines at `8 1,20,000
Add: Conversion cost `50 per 100 pieces. i.e.
50 paise per piece. 15000 x 5
= 75,000 lids x 0.50 = 37,500
296
3,000 3,000
33,000
29,000
33,000
26,000
4,000 7,000
5,000
2,000

Total cost of 75,000 lids 1,57,500
Less: Value of scrapped lids and off-cuts.
Weight of tins: 15,000 kgs.
75,000 x 120 gms
1,000
=
9,000 kgs
Weight of scrap
.
Sales value of scrap 6,000 x 5
6,000 kgs.
Net cost of 75,000 lids
30,000
1,27,500
Cost of each lid 1,27,500 / 75,000 `1.70

Cost of buying one lid `2.00
Hence, there will be a saving of 30 paise on each lid converted instead of buying from outside. In
view of saving , the proposal should be accepted.
`lakhs
(ii) Saving in year: Buying 1,00,000 lids x
12 Months x `2.00 24.00

Less: Conversion cost:
75,000 lids x 12 months x 1.70 = 15.30
Cost of buying the balance lids
= 25,000 lids x 12 months x 2.00 = 6.00
Saving in a year
21.30

2.70
Or else, 75,000 lids x 12 months = 9,00,000 lids at Re. 0.30 each = `2,70,000 savings in a year
accrue to the company if the proposal is accepted.
Ans. 91:



Selling Price `/u
Order Qty
100-140 (`)
Order Qty
141-200 (`)
30,000 30,000
Commission @ 10%
Sales revenue p. u.
Less: Variable purchase cost Contribution / unit
(before shipping) Less: Shipping cost > 110 units
Contribution/ units after Shipping
(i) Upto 110 units, Reference will earn a contribution of `4,000/u.

(ii) Between 110 & 140 units, contribution of 4,000 will be wiped out by 5,000 on shipping
costs. Hence we should not consider 110 140 range.

(iii) 101 110 not to be considered since additional fixed costs 2,25,000 will not be covered
by 10 units.
(iv) Valid consideration, 100 units or 141 to 190 units. Fixed cost
of box of 50 cameras is `2,25,000

297
Units
No. of Camera Boxes
Cost of Cameras
(`)
Contribution (Rs/u)
`4,000
Contribution (`) first
110 units @ 7,000/u
Contribution (`)
Balance units @
2,000/u
Total Contribution
(F




A
B

C
D
E

F



100
2
4,50,000


400,000









4,00,000


- 50 000
141
3
6,75,000





7,70,000


62,000



8,32,000


1 57 000
150
3
6,75,000





7,70,000


80,000



8,50,000


1 75 000
190
4
9,00,000





7,70,000


1,60,000



9,30,000


30 000
Best strategy buy 150 units from Comp. sell 110 at store and 40 outside. BEP
should be between 151 191 units
Extra Camera box cost beyond 150 units = 2,25,000

Less: Profit for 150 units = 1,75,000

Extra profit acquired = 50,000

No. of units to cover this additional costs at contribution 2000 `/u =25


BEP = 150 + 25 = 175 units




298
Miscellaneous Theory Chapters

Ans. 6:
(a) Calculation of cost of per 100 units of good components:
(A)

X Ltd. Y Ltd.

If not inspected


Units required 10,000 10,000

Estimated defectives 300 500

(3%) (5%)
Cost
Purchase price (Rs.)
Production damage (Rs.)
Total Cost (Rs.)
Good component (units)
Cost per 100 good component (Rs.)
Rs.
18,000
540
18,540
9,700
191.13

Rs.
17,400
900
18,300
9,500
192.63


(B)


If inspected


Defectives not detected 30

50

Defectives detected 270

450

Components paid for 9,730

9,550

Cost Rs.

Rs.
Purchase cost 17,514

16,61
7
Inspection cost 2,400 2,400
Production damage 54

90
Total cost 19,968 19,107
Good components 9,700 9,500
Cost per 100 good components (Rs.) 205.86 201.3

Decision:

(i) On the basis of the cost per 100 good component calculated at (A) and (B) above, it is
concluded that inspection at the point of receipt is not justified.

(ii) It will be advantageous to purchase the component from X Ltd.

Ans.
1.
7:
2003 2004
a. Percentage of defective units
shipped
400
10,000
= 4%

330
11000
= 3%
b. Customer complaints as a
percentage of units shipped
500
10000
= 5% 517
11000
=4.7%
c. On-time delivery 8500
10000
= 85% 9900
11000
= 90%

d. Percentage of units reworked
during production
600
10000
=6% 627
11000
=5.7%

299
2. The calculations in requirement I indicate that ESCs performance on both quality and timeliness has
improved. Quality has improved because (a) percentage of defective units shipped has decreased from 4% to
3%,(b) customer complaints have decreased from 5% to 4.7% , and (c) percentage of units reworked during
production has decreased from 6% to 5.7% . Timeliness has improved as on time delivery has increased
from 85 % to 90% . Of course , there is a relationship between the improvements in quality and timeliness.
Better quality and less rework reduces delays in production and enables faster and on-time delivery to
customers.

3a. 2003 2004
The output per labor- hour
Between 2003 and 2004 10000 =0.11 11000
Can be calculated as follows 90000 110000
=0.10

3b . Output per labor-hour may have declined from 2003 and 2004 either because workers were less
productive or more likely because the initial implementation of the quality program may have resulted in lost
production time as employees were trained and became more adept at solving production quality problems.
As workers implement good quality practices and defects and rework decrease over time, it is possible that
both quality and productivity (output per labor-hour) will increase.

3c. it is not clear that the lower output per labor-hour will decrease operating income in 2004. the higher labor
costs in 2004 could pay off in many ways. Higher quality and lower defects will likely result in lower material
costs because of lower defects and rework. Internal and external failure costs will also be lower, resulting in
lower customer returns and warranty costs. Customer satisfaction will likely increase, resulting in higher sales,
higher prices, and higher contribution margins. Indeed the 10% increase in the number of units produced and
sold in 2004 may well have been due to quality improvements. Overall, the benefits of higher quality in 2004
may very well exceed the higher labor costs per unit of output.

Ans.

8: (i) Classification of Quality Costs Figures Rs. 000
2007
% of
sales 2008
% of
sales

Sales 6,000 6,000
Prevention
Quality training 75 1.25 150 2.5
Appraisal
Product Inspection 200 240
Materials
Inspection 80 60
280 4.67 300 5
Internal Failure
Scrap 600 300
Rework 500 400
1100 18.33 700 11.67
External Failure
Product warranty 300 5 150 2.5
1755 29.25 1300 21.67
(ii) Cost reduction was effected by 7.58% (29.25 21.67) of sales, which is an increase in profit by
Rs.4,55,000. (6 Marks) Nov/08-NC& ICWA-June/03 [Adapted]

Ans. 9
Ascertainment of Total cost
: Had there been no defectives for production of 1,00,000 pieces of P 1,00,000X5=5,00,000 units of
raw material would be required. In case of high quality material , defective being 10% total raw material
required is 5,00,000 units/0.90 =5,55,556 units. In case of lower quality material, defective being 20%, total
raw material requirement is 5,00,000 units/0.08 =6,25,000 units. Similarly labour and variable overhead
requirement are to be adjusted accordingly.
I. Using high quality materials (scrap 10%) (Rs)
Material (5,00,000 units/0.90X Rs.1.05) 5,83,333
Labour (2,50,000 hours/0.90X Rs.0.50) 1,38,889
Variable overhead (Rs.1,00,000/0.90) 1,11,111
300
Fixed overhead 50,000
Less: Scrap (5,00,000/0.90)-5,00,000)XRe.0.30 8,83,333
16,667
Cost of 1,00,000 pieces of P 8,66,666

II. Using lower quality materials (scrap 10%) (Rs)
Material (5,00,000 units/0.80X Rs.0.80) 5,00,000
Labour (2,50,000 hours/0.80X Rs.0.50) 1,56,250
Variable overhead (Rs.1,00,000/0.80) 1,25,000
Fixed overhead 50,000
Machine and Tooling cost 3,000
Additional laboour (1,00,000units X 0.5hours XRe.0.50) 2,500
Additional overhead for additional
labour

(1,00,000 units x 0.5 hours)X
(Rs.1,00,000/2,50,000 hours)
20,000

8,79,250
Less: Realizable value of scrap 5000
Cost of 1,00,000 pieces of P 8,74,250
Analysis: Hence the high quality material should be used.

Ans. 10
(I) If each components is tested before being sent to the agents for sales
:-Let the defectives bed
No: of components in a batch Rs.2000
Cost of testing each components Rs.20
Cost of rectification before dispatch Rs.200
Total Cost Rs.(2000x25)+200d
(II) If components dispatch without pre-testing and defectives received back for rectification under warranty.
Total Cost 400d
In difference point of two alternatives
(2000x25)+200d 400d
400d-200d 2000x25
200d 50,000
D 50000/20
250
Defective Components 250 components
Percentage of defectives to total components 250/2000*100 =12.5%
Analysis: If defectives exceed 12.5% of the total number of components per-testing is recommended.

Ans. 11
Cost per unit. (Rs.)
: Present Position (Based on 1,000 units Production)
Direct material 10
Direct wages (8 hours @ Re.0.50) 4
Overheads (8 hours @ Rs.1.75) 14
Total 28

Per unit Units Total
Particular Sales price Profit / Loss Profit Loss
Firsts 30 2 900 1,800 -
Seconds 20 (-) 8 50 - 400
Thirds 10 (-) 18 50 - 900
1,800 1,300
Net Profit 500
Reprocessing of Inferior units
(a) Additional expenditure for reprocessing per unit (Rs.)
Direct Material 4
Direct Wages 8 hrs. 4
Variable overhead @ 0.875 7
15
Total expenditure for 100 units Rs.1,500
301

.
(b) Additional Revenue (Rs.)
Second (Rs.30-Rs.20)x50units 500
Thirds (Rs.30-Rs.10)x50 1000
1500
Note: No change in the profit position hence this need not be considered.

Ans. 1

2: (a)

Existing
After TQM
Programme
i. Total production units
(Preinspection)

Total sales requirements 5,000 5,000
Specification losses 5% 250 2.5% 125
5,250 5,125
Downgrading at inspection

5 . 87
5 . 12
5,250
750

5 . 92
5 . 7
5,125
416
Total units before inspection 6,000 5,541
ii Purchase of material X(Sq
Mtr)

Material required to meet pre
inspection production
requirement 6,000 8 SqMtr
48,000
SqMtr
5,5418
SqMtr
44,328 SqMtr

Processing loss
96
4
48,000
2,000

5 . 97
5 . 2
44,328
1,137
Input to the process 50,000 45,465

Scrapped material
95
5

50,000
2,632

97
3
45,465
1,406
Total purchases 52,632 46,871
iii
Gross Machine Hours

Initial requirements 6,000
0.6
3,600 5,541 0.5 2,771

Idle time
80
20
3,600
900

5 . 87
5 . 12
2,771
396
Gross time 4,500 3,167
(b) Profit and loss statement
Rs Rs
Sales revenue 5,000 Units Rs
1,000
50,00,00
0
50,00,000
Sales downgraded 5,25,000 416 Units Rs 700 2,91,200
302
750 UnitsRs 700
55,25,00
0
52,91,200
Costs:
Material 52,632 Sq Mtr Rs 40 21,05,28
0
46,871Sq Mtr Rs
40
18,74,840
Inspection and storage costs
52,632 Sq Mtr Re 1

52,632

46,871Sq Mtr Re 1

46,871
Machine cost 4,500 Hrs Rs
400
18,00,00
0
3,167 Hrs Rs 400 12,66,800
Inspection and other cost 2,50,000 2,50,000 60% 1,50,000
Product liability (3%
50,00,000
1,50,000 1% 50,00,000 50,000
Sundry cost of selling,
distribution and administration.

6,00,000

6,00,000 90%

5,40,000
Preventive programme cost 2,00,000 6,00,000
51,57,91
2
45,28,511
Net profit 3,67,088 7,62,689

Ans. 13
(a) (i) Units
:
Components worked on in the process 6120
Less: planned defective units 612
replacements to customer (2% X 5400) 108
Components invoiced to customers 5400
Therefore actual result agree with planned results
(ii) Planned components cost = (3 X Rs.18 for material A) + (2 X Rs.9 for material B) + Rs.15 variable cost
=Rs.87
Comparing with the data in appendix:
Materials = Rs.440 640/6120 =Rs.72
Variable overhead = Rs.91 800/6120 = Rs.15
This indicates that prices were at the planned levels.
(b) Internal failure costs = Rs.53 244(612 units X Rs.87)
External failure costs = Rs.9396 (108 units X Rs.87)
(c) (i) Period 2 (units) Period 3 (units)
Components invoiced to customers 5500 5450
Planned replacement (2%) 110 109
Unplanned replacement 60 (170-110) -69 (40-109)
Components delivered to customers 5670 5490
Planned process defects (10% of worked on in
the process) 620 578
Unplanned defects (difference to agree with
with final row) -90 -288
Components worked on in the process 6200 5780

(ii) Period 2(Rs.) Period 3(Rs.)
Internal failure costs 46,110 (620-90) XRs. 87 25,230 (578-288) X Rs.87
External failure costs 14,790 (110+60) X Rs.87 3,480 (109-69) X Rs.87
Appraisal costs 10,000 15,000
Prevention costs 5,000 8,000
(iii) The following points should be included in the report:
303
1. Insufficient detail is provided in the statistics shown in the appendix thus results in the need to for an
improvement in reporting.
2. The information presented in (c) (i) indicate that free replacement to customers were 60 greater than
planned in period 2 but approximately 70 less than planned in period 3. in contrast, the in process
defects were 90 less than planned (approximately 15%) in period 2 and 288 less than plan
(approximately 50%) in period 3.
3. Internal failure costs show a downward trend from period 1-3 with a substantial declined in period
3.External failure costs increased in period 2 but declined significantly in period 3.
4. The cost savings arising in period 2 and 3 are as follows:
Period 2(Rs.) Period 3(Rs.)
Increase /decrease from previous period:
Internal failure costs -1734(Rs.53244-Rs.46110) -20880(Rs.46110-Rs.25230)
External failure +5394(Rs.9396-Rs.14790) -11310(Rs.14790-Rs.3480)
Total decrease -1740 -32190
The above savings should be compared against the investment of Rs.10000 appraisal cost and Rs.5000
prevention cost for period 2 and Rs.15,000 and Rs.8,000 respectively in period 3. it can be seen that the
cost exceed the savings in period 2 but the savings exceeds the cost in period 3. There has also been an
increase in the external failure cost from period 1 to period 2. Investigations should me made relating to
the likely time lag from incurring prevention/appraisal costs and their subsequent benefits.
5. The impact on customer goodwill from the reduction in replacements should also be explained.

Return of 12% net (after tax of 40%) on capital employed is equivalent to 12%(1-0.4) = 20%
(gross) on capital employed.
Ans. 27:

Let selling price per unit to be x
Since Total sales = Total cost + profit
i.e., 80,000x = 14,60,000+20% (12,00,000+0.580,000)
Or, 80,000x = 14,600+2,40,000+8,000x
Or, 72,000x = 17,00,000

Or, x =
000 , 72
000 , 00 , 17
= Rs. 23.61
Hence selling price per unit will be Rs. 23.61

Ans. 28:

(i ) Statement showing price of Product Z
Direct Material Deptt. A 30


Deptt. B 25 55
Direct Labour Deptt. A 30


Deptt. B 40 70
Variable overhead Deptt. A 36 18


Deptt B 43 12 30
Variable selling and distribution overhead 30,000/1,500 20

Total Variable Cost per unit 175

Total hours required for a target of 1,500 units of product Z
304

Deptt. A1500 3 4500 hours

Deptt. B1500 4 6000 hours

10500 hours

10500 hours represent 30% capacity

So total capacity per month 10500 / 0.30 = 35000 hours. Yearly
capacity is 35000 12 = 420000 hours.
Fixed capital employed in both department = 40.00 Lakhs

(25 lakhs + 15 Lakhs)

Expected return = 0.21 40,00,000 = 840000
Contribution per hour = 840000 / 4200000 = 2.00 per hour
Working Capital = 0.21 400000 = 84000
Contribution per unit 84000 / 18000 unit = 4.67 per unit

Total contribution required Rs.
To cover fixed cost 3 hours of A and 4 of B = 7 2 = 14.00
To working capital = 4.67

18.67

Fixed charges recovery is based on usage. Full capacity is not being used by product Z and
departments are also producing other products using same plant and machinery.
Price of Product = Variable cost + contribution required = 175 + 18.67 =
193.67 per unit.

(ii) Price of product when product is well established in market:

Variable Cost 175
Fixed Cost (24 + 16) 40
Total price 215
The product is first time launched in the market, and then variable cost Rs.175 should form the basis
for price fixation.

(a) Rs./u of alloy
Ans. 29:

Materials:

Iron 10kg @ Rs.5/- 50
Copper 5 kg @ Rs.8/- 40 90
Wages
X : 3 hrs @ 15 Rs./Hr. 45
Y : 5 hrs @ 12 Rs./Hr 60 105
Variable OH (Production)
X : 8 hrs 3 hrs 24
Y : 5 hrs 5 hrs 25 49
305
Variable OH Selling 20
Total Variable Cost 264
Fixed Off:
X : 8/hrs 3 hrs. 24
Y : 5/hrs 5 hrs 25 49
Total Cost 313
(i) If pricing strategy is to penetrate the market, the minimum price for a new product
should be the variable cost i.e. Rs.264/-. In some circumstances, it can also be sold below the
variable cost, if it is expected to quickly penetrate the market and later absorb a price increase.
Total Variable Cost is the penetration price.
(ii) When the alloy is well established, the minimum selling price will be the total cost including the
fixed cost i.e. Rs.313 per unit. Long run costs should cover at least the total cost.

Ans. 30:
Sales in X (rearranged for the purpose of ranking
XYZ Ltd.
Rank Category Stock(Rs.000) Cum. Sales(Rs.000) %
)
1 OTC 175 175 21.9
2 Toiletries 150 325 40.6
3 Photo 125 450 56.3
4 Food/ Drink 100 550 68.8
5 Baby 50 600 75.0
5 San. Prod. 50 650 81.3
5 Other 50 700 81.3
8 Foot Care 30 730 91.3
9 Cosmetics 25 755 94.4
10 Hair-care 25 780 97.5
11 Perfume 20 800 100.0

Stock in X (rearranged for the purpose of ranking)
Rank Category Stock(Rs.000) Cum. Sales(Rs.000) %
1 Toiletries 60 60 26.1
2 Cosmetics 40 100 43.5
3 OTC 35 135 58.7
4 Photo 20 155 67.4
4 Food/ Drink 20 175 76.1
6 Other 13 188 81.7
7 Baby 10 198 86.1
7 San. Prod. 10 208 90.4
7 Hair 10 218 94.8
7 Perfume 10 228 99.1
11 foot care 2 230 100.0

Sales in Z (Rearranged for ranking)
Rank Category Stock(Rs.000) Cum. Sales(Rs.000) %
1 OTC 120 120 24
2 Toiletries 100 220 44
3 Food/ Drink 75 295 59
4 Photo 60 355 71
5 Cosmetics 30 385 77
6 Baby 25 410 82
6 San. Prod. 25 435 87
6 Other 25 460 92
9 Foot care 20 480 96
10 Hair 10 490 98
11 Perfume 10 500 100

306
Sales in Z (Rearranged for ranking)
Rank Category Stock (Rs.000) Cum. Sales(Rs.000) %
1 Toiletries 65 65 30.2
2 Cosmetics 45 110 51.2
3 OTC 40 150 69.8
4 Food/ Drink 20 170 79.1
5 Photo 12.5 182.5 84.9
6 Perfume 7.5 190 88.4
7 Baby 5 200 93.0
7 San. Prod. 5 200 93.0
7 foot care 5 205 95.3
7 Hair 5 210 97.7
7 Other 5 215 100.0

Ans. 46:

Annual Relevant Costs of Current Production System and JIT Production System for Evans
Corporation.
Relevant

Relevant
Costs under

Costs under
Current JIT
Production Production
Relevant Items System
Annual tooling costs - Rs.1,50,000
System
Required return on investment:
12% per year x Rs.9,00,000 of average inventory per year Rs.1,08,000 -
12% per year x Rs.2,00,000 of average inventory per year - 24,000
Insurance , space, materials handling , and setup costs 2,00,000 1,40,000
Rework costs 3,50,000 2,80,000
a

Incremental revenues from higher selling prices - (90,000)
b

Total net incremental costs
c

Rs.6,58,000
Annual difference in favor of JIT production Rs.1,54,000
Rs.5,04,000
a
Rs. 200,000 (1-0.30) = Rs.140,000
b
Rs. 350,000 (1-0.20) = Rs.280,000
c

Rs. 3x30,000 units = Rs.90,000
(a) Personal observation by production line workers and managers is more effective in JIT plants than in
traditional plants. A JIT plants production process layout is streamlined. Operations are not obscured by piles
of inventory or rework. As a result, such plants are easier to evaluate by personal observation than cluttered
plants where the flow of production is not logically laid out.
Besides personal observation, non financial performance measures are the dominant methods of
control. Non financial performance measures provide most timely and easy to understand measures of plant
performance. Examples of non financial performance measures of time, inventory, and quality include:
Manufacturing lead time
Units produced per hour
Machine setup time / manufacturing time4
Number of defective units / number of units completed.

In addition to personal observation and non financial performance measures. Financial performance
measures are also used. Examples of financial performance measures include.
Cost of rework
Ordering costs
Stock out costs
Inventory turnover
(3b) The success of a JIT system depends on the speed of information flows from customers to manufactures
to suppliers. The Enterprise Resource Planning (ERP) system has a single database, and gives lower-level
managers, workers, customers, and suppliers access to operating information. This benefit, accompanied by
tight coordination across business function, enables the ERP system to rapidly transmit information in
response to changes in supply and demand so that manufacturing and distribution plans may be revised
accordingly.

Ans. 47: (i) Comparative Statement of cost for purchasing from Y Co Ltd under current policy & JIT
307
Particulars Current Policy JIT
Rs Rs
Purchasing cost 18,20,000 18,20,260
(13,000 140) (13,000 140.02)
Ordering cost 26 260
(213 orders) (2130 orders)
Opportunity carrying cost 10,500.00 1,050.15
(1/2100014015%) (1/2100140.0215%)
Other carrying cost(Insurance,
material handling etc) 1,550.00 155
(1/210003.10)
Stock out cost 200
(4 50)
Total relevant cost 18,32,076 18,21,925.15
Comments: As may be seen from above, the relevant cost under the JIT purchasing policy is lower than the
cost incurred under the existing system. Hence, a JIT purchasing policy should be adopted by the company.

(ii) Statement of cost for purchasing from Z Co Ltd.
Particulars Rs.
Purchasing cost 1,76,800
(13,000x13.60)
Ordering Cost 260
(2x130 orders)
Opportunity Carrying 102
Cost (1/210013.60 15%)
Other Carrying Cost 150
(1/21003.00)
Stock out Cost 2,880
(8x360)
Inspection Cost 650
(13,000 x .05)
Customer Return
Cost 6,500.00
( 13,000 x 2% x 25)
Total Relevant Cost 1,87,342
Comments : The comparative costs are as follows,
Under current policy Rs 18,32,076.00
Under purchase under JIT Rs 18,21,925.10
Under purchase from Z Co Ltd Rs 1,87,342.00
Packages should be bought from Z Co as it is the cheapest.

308
A B Rs.
4 2 10
1 1 4
80 60


1
Ans:7
Linear Programming
The problem may be summarized as follows:

Chemical Chemical Cost per mix


Supplier X
Supplier X
2


Units required


Let x
1
be the number of mixes to be purchased from supplier X
1
and x
2
be of those
to be purchased from supplier X
2
.
The conditions of the problem when symbolised, take the form:
Minimize Z = 10 x1 + 4 x2
Subject to the restrictions
x1 0, x2 0
4x1 + x2 80,
2x1 + x2 60.
For the line 4x1 + x2 = 80,
let x1 = 0,so that x2 = 80;
let x2 = 0,so that x1 = 20.
For the line 2x1 + x2 = 60,
let x1 = 0,so that x2 = 60;
let x2 = 0,so that x1 = 30.




Feasible region is shaded in the diagram which appears to be unbounded. We now try to
deter-
mine the additional hidden conditions in the problem for which the feasible region
becomes bounded.
The column vector for the values of the objective function is given by
309

Since 260 is the smallest element in EC, the minimum value is reached at the extreme point
E
2
, whose coordinates are (10,40).

Thus, to honour the contract and yet to minimize cost, the company should purchase 10
mixes from X
1
and 40 mixes from X
2
.

Ans.8:
Maximize z = 80x + 100y subject to x + 2y 720
5x + 4y 1800
3x + y 900
x 0 y 0
where x = No. of units of A
y = No. of units of B
By the addition of slack variables s1, s2 and s3 the inequalities can be converted into
equations. The problem thus become
z = 80x + 100y subject to x + 2y + s
1
5x + 4y + s
= 720
2
3x + y +s
= 1800
3
and x 0, y 0, s
= 900
1
0, s
2
0, s
3
Table I
0
80 100 0 0 0
Profit/unit Qty. X Y S S
1
S
2

3

S 0
1
720 2 1 0 0
360
2
720
=
S 0
2
1800 5 4 0 1 0 1800/4 =
450
S 0
3
900 3 0 0 1 900/1 = 900
Net evaluation
row
80 100 0 0 0
1800 720 4/2 = 360 900 - 7201/2 = 540
5 I2 = 3 3 - 1 = 5/2
4 2 2 =0 I 2 1/2 = 0
0 - I2 = - 2 0 I 1/2 =- 1/2
I - 02 = I 0 0 1/2 = 0
0 - 02 = 0 I- 01/2 = I

Table 2:
80 100 0 0 0
Program Profit/unit Qty. X Y S S
1
S
2

3

310
Y 100 360 I 0 0 3601/2=720
S2 0 360 3 0 2 1 0 3603=120
S3 0 540 5/2 0 1/
2
0 I 5405/2=216
Net
evaluation
row
30 0 50 0 0

360 360 1/6 = 300 540 360 5/6 = 240
- 3 1/6 = 0 5/2 3 5/6 = 0
1- 0 1/6=1 0 0 5/6 = 0
- -2 1/6 = 5/6 -1/2 - -2 5/6 = 7/6
0 1 1/6 = - 1/6 0 1 5/6 = -5/6
0 0 1/6 = 0 1-0 5/6 = 1

Table 3:
80 100 0 0 0
Program Profit/unit Qty. X Y S S
1
S
2

Y
3

100 300 0 I 5/6 -1/6 0
X 80 120 I 0 2/3 1/3 0
S3 0 240 0 0 7/6 -5/6 I
Net evaluation
row
0 0 -500/6
+160/3
+100/6
-80/3

0

=
6
180
=
6
60


All the values of the net evaluation row of Table 3 are either zero or negative, the optimal
program has been obtained.
Here X = 120, y = 300 and the maximum profit
= 80120 + 100 300 = 9600 + 30,000
= Rs. 39,600.
Ans. 9:
Formulation of Linear Programming (LP) model :

Let X
1
and X
2

be the units of products A and B respectively which were manufactured
and sold (within sales constraints) by the company in a month, by utilizing monthly
available budgeted capacity in department A and B so as to maximize the profit of the
company. The formulated LPP based on the given data is as under :
Max Z = 80 x
1
+ 100 x
2

(Refer to working note)
2 x
1
+ 4x
2

< 1,400 -- Department P Constraint
5x
1
+ 4x
2

< 2,000 -- Department Q constraint
X
1

< 400 -- Product A Sales constraint
311
X
2

< 400 -- Product B sales constraint
X
1

, X2 > 0
Graphical Solution :

Draw the above four constraints by selecting X1 and X2 axes as shown in the diagram.




X2 - AXIS

X1 < 400



(0,500) X2 <

400
(0,350)





P (200, 250)





0 (400,0) (700,0)




Put x
2
= 0 in (i), then x
1

= 700
Put x
1
= 0 in (i), then x
2

= 350
Put x
2
= 0 in (ii); then x
1

= 400
Put x
1

= 0 in (ii); then x2 = 500
The point of intersection of (i) and (ii) viz, P is given by (200, 250)

The marked area represents the feasible area (common to all of the four constraints).
The corner points of the identified feasible region are (0,0); (400,0); (200,250) and
(0,350).

According to Dantzig, the objective function is maximum or minimum at the corner
points of the feasible region.

312
Z (0,0) = 0,0

Z (400, 0) = 32,000

Z (0, 350) = 35,000

Z (200, 250) = 41,000

The objective function has maximum contribution viz, Rs. 41,000 at the point (200,
250). Hence, the concern should manufacture and sell 200 units of A and 250 units of
B product.

Optimal contribution (Rs) 41,000
(200 units x Rs. 80 + 250 units x Rs. 100)

Less : Fixed costs 34,000
(Rs. 14,000 + Rs. 20,000)
_____
Optimal profit

7,000
Working note :

Product A Product B

Selling price per unit : (i) 300 200

Variable manufacturing costs 160 60

Sales commission 60

40
Total variable cost per unit : (ii) 220

100
Contribution per unit : (ii I) 80

100
Ans. 11:
Let x1, x
2
x
3
Then the profit gained by the industry is given by
be the number of units produced of products A, B and C respectively.
Z = 3x
1
+ 8x
2
+ 2x
Here it is assumed that all the units of products A and B are sold.
3

In first operation, A takes 3 h of manufacturers time and B takes 4 h of manufacturers time.
Therefore, total number of hours required in first operation becomes.
3x
1
+ 4x
In second operation, per unit of A takes 2 h of manufacturers time and per unit B takes 5 h of
manufacturers time. Therefore, the total number of hours used in second operation becomes
2

3x
1
+ 5x
Since there are 18 h available in first operation and 21 h in second operation, the restrictions
become
2

3x
1
+ 4x
2
18 (1)
313
3x
1
+ 5x
2
Since the maximum number of units of C that can be sold is 5, therefore,
21 (2)
X
3
Further, the company gets three units of by product C for every unit of product B produced, therefore
5 (3)
X
3
= 3x
2
Now, the allocation problem of the industry can be finally put in the following linear programming
problem:
(4)
Maximise Z = 3x
1
+ 8x
2
+ 2x
Subject to the constraints
3

3x
1
+ 4x
2
3 x
18
1
+ 5x
2
x
21
3
5, x
3
= 3x
x
2

1
, x
2
, x
3
Ans. 15:
0
Let X
1
, X
2
and X
3
(i ) Phosphorus content must not exceed 0.03%
respectively be the amounts in tons of grades A, B, and C used. The constraints
are
.02 X
1
+ .04X
2
+ 0.3 X
3
.03 (X
1
+ X
2
+ X
3
2X
)
1
+ 4 X
2
+ 3X
3
3 (X
1
+ X
2
+ X
3
) or X
1
+ X
2
(i i ) Ash content must not exceed 3%
0
3X
1
+ 2 X
2
+ 5 X
3
3 (X
1
+ X
2
+ X
3
) or X
2
+ 2X
3
(i i i ) Total quantity of fuel required is not more than 100 tons. X
0
1
+ X
2
+ X
3
The Mathematical formulation of the problem is
100
Maximize Z = 12 X
1
+ 15X
2
+ 14 X
Subject to the constraints:
3

- X
1
+ X
2
- X
0
2
+ X
3
X
0
1
+ X
2
+ X
3
X
100
1
, X
2
, X
3
Introducing slack variable X
> 0
4
>0, X
5
>0, X
6

>0
12 15 14 0 0 0
C Y
b
X
b
Y
b
Y
1
Y
2
Y
3
Y
4
Y
5

0
6

Y 0
4
-1 1* 0 1 0 0
0 Y 0
5
0 -1 2 0 1 0
0 Y 100
6
1 1 1 0 0 1
Z -12 -15 -14 0 0 0

C Y
b
X
b
Y
b
Y
1
Y
2
Y
3
Y
4
Y
5

6

314
15 Y 0
2
-1 1 0 1 0 0
0 Y 0
5
-1 0 2 1 1 0
0 Y 100
6
2* 0 1 -1 0 1
Z -27 -14 15 0 0
C Y
b
X
b
Y
b
Y
1
Y
2
Y
3
Y
4
Y
5

15
6

Y 50
2
0 1 1/2 1/2 0 1/2
0 Y 50
5
0 0 5/2* 1/2 1 1/2
12 Y 50
1
1 0 1/2 -1/2 0 1/2
Z 0 0 -1/2 3/2 0 27/2
C Y
b
X
b
Y
b
Y
1
Y
2
Y
3
Y
4
Y
5

15
6

Y 40
2
0 1 0 2/5 -1/5 2/5
14 Y 20
3
0 0 1 1/5 2/5 1/5
12 Y 40
1
1 0 0 -3/5 -1/5 2/5
Z 0 0 0 8/5 1/5 68/5
The optimum solution is X
1
= 40, X
2
= 40 and X
3
Ans.16:
= 20 with maximum Z = 1360.
Table 1

C

j

Qty
40 60 0 0 0 Ratio
Cj Variable X1 X2 X3 X4 X5


0 X3 36 3 3 1 0 0 12
0 X4 60 5 2 0 1 0 30
0 X5 60 2 6 0 0 1 10

Zj 0 0 0 0 0 0
Zj-Cj -40 -60 0 0 0



Table 2

Cj

Qty
40 60 0 0 0 Ratio
Cj Variable X1 X2 X3 X4 X5
0 X3 6 2 0 1 0 - 3
0 X4 40 13/3 0 0 1 -1/3 120/13
60 X2 10 1/3 1 0 0 1/6 30
Zj 600 20 60 0 0 10
Zj-Cj -20 0 0 0 10


Table 3

Cj

40 60 0 0 0
315
Qty
Cj Variable X1 X2 X3 X4 X5
40 X1 3 1 0 0 -1/4
0 X4 27 0 0 -13/6 1 3/4
60 X2 9 0 1 -1/6 0 1/4
Zj 660 40 60 10 0 5
Zj-Cj 0 0 10 0 5

Since all Zj Cj are positive or zero, this is the optimum solution with, X1 =40 &
X2 = 60 and optimum Z = 660.

Note: Alternatively, Cj-Zj may be used whereby maximum positive value may
be considered.

Ans. 18:
Under the usual notations where S1,
S2, S3 are stock Variables, A4
= the artificial variable
S4 = Surplus Variable

We have,
Max. Z = 100x
1
+ 80x
2
+ 0S
1
+ 0S
2
+ 0S
3
+ 0S
4
M
A
4
.
S.t.

3x
1
+ 5x
2
+ S
1
=

150
x
2
+ S
2
=
20
8x
1
+ 5x
2
+ S
3
=
300
x
1
+ x
2
+ - S
4
+ A
4
= 25


x
1
x
2
S
1
S
2
S
3
S
4
A
4


Basis
C
j
C
B

100

80

0

0

0

0

- M

S
1

S
2

S
3

A
4
0

0

0

- M
3

0

8

1
5

1

5

1
1

0

0

0
0

1

0

0
0

0

1

0
0

0

0

-1
0

0

0

1
150

20

300

25







Z
j

C
j
-Z
j

- M

100+M
- M

80+M
0

0
0

0
0

0
M

-M
-M

0
-25M



Ans.20:
(i ) Si mpl ex Table


Basis
C
j

8 6 0 0 (1 mark)
C
B
x
1
x
2
s
1
s
2

316

X
1
X
2
S
1
S
2
X
1 8 X 1 = 8 8 X 0 = 0 8 X 1/3 = 2.2/3
8 X - 1/
1.1/
X
2 6 X 0 = 0 6 X 1 = 6 6 X - 1/6 = - 1 6 X 1/3
Adding Z
j
8 6 5/3 2/3

x
1
x
2
Z
j
NER
8
6


C
j
- Z
j
1
0

6
0
0
1

6
0

-

5
3

5
3

-


-




2 marks
1 marks

Note: Z
j
values are obtained by multiplying each row with cost and adding
the
values of the respective column as under:



6
=
3
=
2


Net Evaluation Row (NER) is obtained by deducting Z
j
from C
j
as under:
8 8 = 0 6 6 = 0 0 5/3 = - 5/3 0 2/3 = - 2/3

Since the values of NER are 0, the solution represented by this tableau is optimal.

(ii)
S
1
S
2 Rs.
M I X
1 60 x 5/3 100
M II X
2 48 x - 2/3 32
Total optimal contribution 132

Ans.21:
Let p
i
d
j
be the variable to denote the number of units of product from the ith plant to the
jth
destination, so that

P
1
d
1
= transport from plant P
1
to D
1

P
2
d
2
= transport from plant P
2
to D
2
etc.
Objective function
Minimize z = 400 p
1
d
1
+ 600 p
1
d
2
+ 800 p
1
d
3
+ 1000 p
2
d
1
+ 1200 p
2
d
2
+ 1400 p
2
d
3

+ 500 p
3
d
1
+ 900 p
3
d
2
+ 700 p
3
d
3
.

Subject to:

p
1
d
1
+ p
1
d
2
+ p
1
d
3
65

p
2
d
1
+ p
2
d
2
+ p
2
d
3
24
`
(Plant constraints)


p
3
d
1
+ p
3
d
2
+ p
3
d
3
111
)

and
p
1
d
1
+ p
2
d
1
+ p
3
d
1
60
p
1
d
2
+ p
2
d
2
+ p
3
d
2
65
`
(destination constraints)


317
p
1
d
3
+ p
2
d
3
+ p
3
d
3
75
)

all p
i
d
j
0


Ans.22:

Route I Residence HO 600 400 300

180
Residence


Route II Residence 500 300 200

40
Br. Residence
No. of vehicles 4 10 20
Max. capacity 80 80 100 220
No. of passengers
260


Let i be the ith route,

and j be the type of vehicle, so that

S
11
= no. of vans (vehicles on Route I, Type I)
S
12
= no. of 8 seater cars on Route I
S
13
= no. of 5 seater cars on Route I

S
21
= no. of vans on Route II

S
22
= no. of 8 seater cars on Route II S
23
= no. of 5 seater cars on Route II

Ans. 23:
Formulation. Let x
i
be the number of times cutting alternative i (j = 1,2, .....6) is employed.
Minimise (waste produced) Z = 1x
3
+ 1x
4
+ 1x
5
+ 1x
6
subject to
6x
1
+ 1x
3
+ 4x
6
3000

3x
1
+ 3x
2
+ 1x
3
+ 4x
5
+ 2x
6
2000

2x
2
+ 1x
3
+ 2x
4
+ 1x
5
+ 1x
6
1500

1x
3
+ 1x
4
1000

x
j
0, for
all j

Ans.24:
The profits for each arrangement are:

Economy = 6.00 4 (0.20) 2(0.25) 8 (0.15) = Rs. 3.50
May time = 8.00 8 (0.20) 5 (0.25) 10 (0.15) 4 (0.22) = Rs. 2.77
Spring colour = 10.00 9 (0.20) 10 (0.15) 9 (0.20) 6 (0.22) = Rs. 3.58
Deluxe rose = 12.00 12 (0.20) 12 (0.20) 12 (0.22) = Rs. 4.56
Let x
1
, x
2
, x
3
, x
4
be number of units arrangements of type Economy,
May time, Spring colour & Deluxe rose.
Then the objective is Maximise Z = 3.5x
1
+ 2.77x
2
+ 3.58x
3
+ 4.56x
4

subject to 4x
1
+ 9x
3
+ 12x
4
800

318
3 2
2x
1
+ 5x
2
456

8x
1
+ 10x
2
+ 10x
3
4000
8x + 9x+ 12x 920
4x
2
+ 6x
3
+ 12x
4
422

All x
i
's 0

Ans. 26: The information given in the question can be presented in the following tabular form.

Products
Raw material (in kg) required to produce one kg of product Selling price
(per kg)
X X
1
X
2

3

Y 1/2
1
1/4 1/4 Rs.90
Y 3/7
2
2/7 2/7 Rs.100
Y --
3
2/3 1/3 Rs.120
Cost of raw
material (Per kg)
Rs.30 Rs.50 Rs.120
Availability of
raw material
20 kg 15 kg 10 kg
From the above table, the cost of producing 1 kg of Y
1
, Y
2
and Y
3
Cost to produce 1 kg of Y
can be calculated as given below:
1
= Rs.15 + Rs.12.50 + Rs.30
= Rs.30 + Rs.50 + Rs.120
= Rs.57.50
Profit per kg of Y
1
Similarly, cost to produce 1 kg of Y
= Rs.90 Rs.57.50 = Rs.32.50
2
= 1/7 (Rs.90 + Rs.100 + Rs.240)
= 3/7 Rs.30 + 2/7 Rs.50 + Rs.120
= Rs.430/7 = Rs.61.43
Profit per kg of Y
2
and cost to produce 1 kg of Y
= Rs.100 Rs.61.43 = Rs.38.57
3
Profit per kg of Y
= 2/3 Rs.50 + 1/3 Rs.120 = Rs.220/3 = Rs.73.33
3
Let the manufacturer produce y
= Rs.120 Rs.73.33 = Rs.46.67
1
, y
2
and y
3
units of the products Y
1
, Y
2
and Y
3
Since the manufacturer wants to maximise the profit, the objective function is given by
respectively.
Maximise Z = 32.50 y
1
+ 38.57 y
2
+ 46.67 y
y
3

1
+ 3/7 y
2
20 or 7 y
1
+ 6 y
2
y
280
1
+ 2/7 y
2
+ 2/3 y
3
15 or 21 y
1
+ 24 y
2
+ 56 y
3
y
1,260
1
+ 2/7 y
2
+ 1/3 y
3
10 or 21 y
1
+ 24 y
2
+ 28 y
3
where Y
840
1
, Y
2
and Y
3

0
Ans. 27:
Let x
1
X
= No. of units of product 1 produced
2
= No. of units of product 2 produced
319
X
3

= Amount of money borrowed
The profit contribution per unit of each product is given by the selling price minus
the variable cost of production. Total profit ay be computed by summing up the
profits from producing the two products minus the cost associated with borrowed
funds (if any):-
The objective function is thus stated as
Maximize Z = (14 10 ) x
1
+ (11 8) X
2
- 0.05 X
3
= 4 x

1
+ 3 X
2
- 0.05 X
(Note that the interest rate is 20% per annum, hence 5% for a period of three
months)
3


Subject to the following constraints:

The production capacity constraints for each department, as given by table 1
are:

0.5x
1
+ 0.3X
2
0.3x
500 .(1)
1
+ 0.4X
2
0.2x
400 .(2)
1
+ 0.1X
2
The funds available for production include both Rs.3,00,000 cash that the firm
possesses and any borrowed funds maximum up to Rs.2,0,000. Consequently
production costs. The constraint expressing this relationship is
200 .(3)
Funds required for production Funds available.
i.e 10x
1
+ 8X
2
Rs. 3,00,000 + X
or 10x
3

1
+ 8X
2
- X
3

Rs. 3,00,000 .(4)
The borrowed funds constraint (from condition (iii) of the Question) is
X
3

Rs. 2,00,000 .(5)
The constraint based on the acid-test condition is developed as follows:-
Surplus cash on hand after production + Accounts receivable
Bank Borrowings + Interest accrued thereon
1


i.e. (3,00,000 +X
3
- 10x
1
8X
2
) + 14x
1
+ 11X
2
(X
1
3
+ 0.05X
3

)
or, 3,00,000 +x3 +4x1 +3x2 > (x3 +0.05x3)
Or, - 4x
1
- 3X
2
+ 0.05X
3

3,00,000 .(6)
Thus, the linear programming problem is given by
Maximize Z = 4x
1
+ 3X
2
- 0.05X

3
Subject to 0.5x
1
+ 0.3X
2
500

0.3x
.(1)
1
+ 0.4X
2
0.2x
400 .(2)
1
+ 0.1X
2
10x
200 .(3)
1
+ 8X
2
- X
3
X
Rs. 3,00,000 ..(4)
3
- 4x
Rs. 2,00,000 .(5)
1
3X
2
+ 0.05X
3
Rs. 3,00,000 .(6)
320

Where x
1
X
2
X
3

0.
Ans. 28:
Let x
1
, x
2
and x
3
Since the average yield of radish is 1,500 kg per acre, and the selling price for radish is Rs.5/kg
hence the selling amount which the agriculturist gets from one acre is:
be the number of acres allotted for cultivating radish, mutter and potato
respectively.
Rs.5 1,500 = Rs.7,500
To produce 100 kg of radish, the manure cost is Rs.12.50, so the manure cost per acre will be
Rs.12.50 1,500/100 = Rs.12.50 15.
Labour cost per acre for radish = Rs.40 6 = Rs.240
Profit per acre for radish = Rs.7,500 Rs.12.50 15 Rs.240 = Rs.7,072.50
Similarly, the selling price, manure cost, labour cost and profit per acre of land for mutter and potato
are also calculated and presented in the following table.
Per acre Radish Mutter Potato
Selling price Rs.5 1,500 =
Rs.7,500
Rs.4 1,800 =
Rs.7,200
Rs.5 1,200 =
Rs.6,000
Manure cost
100
1,500 Rs.12.50

100
1,800 Rs.12.50

80
1,200 Rs.12.50

Labour cost Rs.40 6 = Rs.240 Rs.40 5 = Rs.200 Rs.40 6 = Rs.240
Profit (Rs.7,500-Rs.187.50
Rs,240) = Rs.
7,072.50
(Rs.7,200 Rs.255 -
Rs.200) = Rs.6,775
Rs.6,000Rs.187.50
Rs.240) = Rs.
5572.50
Since, the agriculturist wants to maximise the total profit, hence the objective function of the problem
is given by:
Maximise Z = 7,072.5x
1
+ 6,775x
2
+ 5572.5x
Subject to following constraints:
3

x
1
+ X
2
+ X
3
6x
125
(1) (land constraint)
1
+ 5x
2
+ 6x
3
Where x
500 (2) (man day constraint)
1
, x
2
and x
3

0
Ans. 29:
Maximize Z = 60 (9x
1
+ 5x
2
) + 90 (7x
1
+ 9x
2
)


= 1170x
1
+ 1110x
2

Subject to 9x
1
+ 5x
2
500 commitment for
A


7x
1
+ 9x
2
300 commitment for B


5x
1
+ 3x
2
1500 availability of Q


321
7x
1
+ 9x
2
1900 availability of P


2x
1
+ 4x
2
1000 availability of R


and x
1
0, x
2
0.

Ans. 30:
Let x
1
, X
2
and X
3

denote the number of P III, P II and Celeron Computers respectively to the
manufactured in the company. The following data is given:
P III P II Celeron
Selling Price per unit (Rs.) 3,000 5,000 15,000
Labour, Material and other Variable Costs p.u. (Rs.) 2,000 4,000 8,000
Profit per unit (Rs.) 1,000 1,000 7,000
From the data given for time required for various models and the total number of hours available for
machine time and assembly time, we get the following constraints:
20x
1
+ 15x
2
+ 12x
3
5x
1,000 (Machine Time Restriction)
1
+ 4x
2
+ 3x
3
The level of operations in the company is subject to availability of cash next month i.e.; the cash
required for manufacturing various models should not exceed the cash available for the next month.
1,500 (Assembly Time Restriction)
The cash requirements for x
1
units of P III, x
2
units of P II and x
3
2,000x
units of Celeron computers are:
1
+ 4,000 x
2
+ 8,000x
3
The cash availability for the next month from the balance sheet is as below:
(1)
Cash availability (Rs.) Cash balance (Rs. 2,10,000)
Loan to repay to Nationalized bank (Rs. 50,000)
Interest on loan from XYZ cooperative bank and Nationalized bank (Rs. 1500)
Interest on long term loans
(


12
000 , 00 , 2 18 . 0

Salary to staff (Rs. 15,000)
Or, Cash availability = Rs. 2,10,000-(Rs. 50,000 + Rs. 1,500 + Rs. 3,000 + Rs. 15,000)
= Rs. 1,40,500 ... (2)
Thus, from (1) and (2),
2000 X1 + 4000 X2 + X3 < Rs. 1,40,500
The company has also promised to deliver 3 P III, 2 P II and 5 Celeron computers to M/s.
Kingspen Ltd.
Hence, X1 > 3, X2 > 2, X3 > 5
Since the company wants to maximize the profit, hence the objective function is given by: Maximize
Z = 1000X1 + 1000X2 + 7000X3- (Rs. 15000 + Rs. 3000 + Rs. 1500) The LP formulation of the given
problem is as follow:
Maximize Z = 1000 X1 + 1000X2 + 7000 X3 (Rs. 15000 + Rs.15000) Subject to the constraints:
20X1 + 15X2 + 12X3 < 1000
5X1 + 4X2 + 3X3 < 1500
322
2000 X1 + 4000 x2 + 8000 X3 < Rs. 1,40,500
X1 > 3, X2 > 2, X3 > 5
X1, X2 and X3 can take only positive integral values.

Ans. 31:
Let the firm produce x
1
units of product A, x
2
units of products B and x
3
The profit per unit of products A, B and C is Rs. 50, and Rs. 80 respectively.
Since the objective of the firm is to maximize the profit, therefore, the objective
function is given by
units of
product C.

Maximize Z = 50x
1
+50x
2
+80x

3

The firm uses two types of raw materials I and II of which 5,000 and 7,500 units
respectively are available. As per the given data, the raw material constraints
can be formulated as given below:

3x
1
+4x
2
+5x
3
and 5x
< 5,000 .. (i)
1
+3x
2
+5x
3

< 7,500 (ii)
The labour time for each unit of product A is twice that of product B and three
times that of product C. Also the entire labour force can produce the
equivalent of 3,000 units.
X
1
3 2
3 2 X X
+ + < 3,000
or, 6x
1
+3x
2
+2x
3

< 18,000 (iii)
The minimum demand of the three products is 600, 650 and 500 units
respectively.
Hence, x
1
> 600, x
2
> 650 and x
3

> 500
Since the ratios of the number of units produced must be equal to 2:3:4,
therefore,

x
1
= 1/3 x
2
, and 1/3 x
2
= x
or, 3x
3

1
= 2x
2
and 4x
2
=3x
3

(iv)
The linear programming model can be formulated as follows:
Maximize Z = 50x
1
+50x
2
+80X
3

(v)
Subject to the constraints:
3x
1
+4x
2
+5x
3
5x
< 5,000
1
+3x
2
+5x
3
6x
< 7,500
1
+3x
2
+2x
3
3x
< 18,000
1
= 2x
2
and 4x
2
x
=3x3
1
>600, x
2
> 650 & x
3

> 500.
Ans. 32:
Renco Foundries has to decide the amount of funds to be allocated to projects
A, B, C, D, E and money market instruments. Let us define the decision
variables as
323

a : Rs. Invested in investment A
b : Rs. Invested in investment B
c : Rs. Invested in investment C
d : Rs. Invested in investment D
e : Rs. Invested in investment E
S
i

: Rs. Invested in money market instruments at time i
(for i = 0,1,2)
The objective of Renco Foundries is to draw up the capital budget in such a
way that will maximize cash on hand at time 3.

Now at time 3, the cash on hand for Renco Foundries will be the sum of all
cash inflows at time 3.
Since the firm earns interest at 8% p.a. by parking the un-invested funds in
money market instruments, hence Rs. S
0
which are invested in these
instruments at time 0 will become 1.08 S
0
at time 1. Similarly an investment of
Rs. S1 at time 1 will become 1.08 S
1
at time 2, and an investment of Rs. S
2
at
time 2 will become 1.08 S
2
From the table giving the description of various investments, it can be
computed that at time 3,
at time 3.

Cash on hand = aRe. 0+bRe.1+c Re. 0+d Rs.1.9+ e Rs. 1.5+ 1.08S
= Rs. (b+1.9d+1.5e+1.08S
2

2

)
The objective of Renco Foundries is to maximize the cash on hand at time 3.
hence the objective function will be
Maximize Z = b+ 1.9d+ 1.5e +1.08 S
2

.(i)
It may be noted that
Cash available for investment at time t = cash on hand at time
t ..(ii)

At time 0, funds to the tune of Rs. 1,00,000 are available for investment. From
the table, it can be seen that funds are invested in investment A, C, D and S
0


at time 0.
Hence, a+c+d+S
0

= 1,00,000.(iii)
At time 1, Rs. 0.5 a, Rs. 1.2 c and Rs. 1.08 S0 will be the available returns as a
result of investments made at time 0. From the table Rs. B and Rs. S1 are
invested in investment B and money market instruments respectively at time 1.

Using equation (ii), we get
0.5a+ 1.2c+ 1.08S
0
= b+ S
1

.(iv)
At time 2, Re. 1 a, Rs. 0.5 b and Rs. 1.08S
1
will be available for investment.
However, Rs. E and Rs. S
2

are invested at time
2..(v)
Further, since the firm will not commit an investment exceeding Rs. 75,000 in
any project, we get the following constraints:

324
a < 75,000 (vi)
b < 75,000 (vii)
c < 75,000 (viii)
d < 75,000 (ix)
e < 75,000 (x)

Also a, b, c, d, e and S
i
(for
i

= 0, 1, 2) are all > 0
Combining all the constraints, the linear programming model for the Renco
Foundries is as given below:
Maximize Z = b+ 1.9d+ 1.5e+ 1.08S

2

Subject to following constraints
a+c+d+S0 = 1,00,000
0.5a +1.2c +1.08 S
0
= b+S
1a+ 0.5b +1.08S
1

1
= e+ S

2

a < 75,000
b < 75,000
c < 75,000
d < 75,000
e < 75,000

a, b, c, d, e and si (I =0, 1, 2) are all > 0

Ans. 33:
Let x
1
and x
2
Maximise Z = 0.1x
be the amount to be invested in first and second stock portfolio respectively. The
average rate of return for first portfolio is 10% and for second portfolio, it is 20%. Since the company
wishes to maximize the return from investment, the objective function is as given below:
1
+ 0.2x
The maximum amount available for investment is Rs.1,00,000.
2

Hence, x
1
+ x
2
Further, the maximum investment allowed in either portfolio set is Rs.75,000.
1,00,000 (1)
Therefore, x
1
and x
75,000 (2)
2
The first portfolio has a risk rating of 4 (on a scale from 0 to 10) and the second has 9. The company
will not accept a risk factor above 6.
75,000 (3)
Therefore, 4x
1
+ 9x
2
6 (x
1
+x
2
Further, the company will not accept an average rate of return below 12%.
) (4)
Hence, 0.1x
1
+ 0.2 x
2
0.12 (x
1
+ x
2
Also, x
) (5)
1
and x
2
The linear programming model for the given problem can now be formulated as follows:
0 (6)
Maximise Z = 0.1x
1
+ 0.2x
Subject to the constraints
2

x
1
+x
2
1,00,000 (1)
325
x
1
x
75,000 (2)
2
4x
75,000 (3)
1
+ 9x
2
6 (x
1
+ x
2
or 2x
)
1
+ 3x
2
0.1x
0 (4)
1
+ 0.2x
2
0.12 (x
1
+x
2
or 0.02x
)
1
+ 0.08x
2
where x
0 (5)
1
, x
2
The problem is solved graphically below:
0

The point of intersection for the lines
- 2x
1
+ 3x
2
and x
= 0
1
+ x
2
is given by B (60,000, 40,000)
= 1,00,000
The point of intersection for the lines
X
1
and x
= 75,000
1
+ x
2
is given by C (75,000, 25,000)
= 1,00,000
Similarly, the lines
x
1
and 0.02x
= 75,000
1
+ 0.08x
2
intersect at point D (75,000, 18,750)
= 0
Thus, the feasible region is bounded by ABCDA and feasible points are A (0, 0); B(60,000, 40,000);
C(75,000, 25,000) and D(75,000, 18,750).
Value of the objective function at the above mentioned feasible points is calculated below:
At A, Z=0
At B, Z=0.1 60,000 + 0.2 40,000
= 6,000 + 8,000 = Rs.14,000
At C, Z=0.1 75,000 + 0.2 25,000
= 7,500 + 5,000 = Rs.12,500
326
At D, Z=0.1 75,000 + 0.2 18,750
= 7,500 + 3,750 = Rs.11,250
We find that the value of the objective function is maximum (Rs.14,000) at point B(60,000, 40,000).
Hence, the company should invest Rs.60,000 in first portfolio and Rs.40,000 in second portfolio to
achieve the maximum average rate of return of Rs.14,000.

Ans. 35:

Contribution analysi s:

Products A B
(Rs.) (Rs.)
Selling price (A) 500 450
Variable costs:
Direct Materials 100 100
Direct Labour 80 40
Painting 30 60
Variable Overheads 190 175
Total variable costs (B) 400 375
Contribution (A B) 100 75
Direct Material per unit 100/25 = 4 kg. 100/25 = 4 kg.
Direct Labour hour per unit 80/20 = 4 hours 40/20 = 2 hours
Painting hour per unit 30/30 = 1 hour 60/30 = 2 hours
Let A be the units to be produced of product A and B be the units to be produced of
product B.


LP Problem formulation:

Z Max 100A + 75B Maximisation of contribution

Subject to:

4A + 4B 480 Raw material constraint

4A + 2B 400 Direct Labour hour constraint

A + 2B 200 Painting hour constraint

A, B 0 Non negativity constraint

Raw Material Constraint : Put B = 0, A = 120

Put A = 0, B = 120

Direct Labour Constraint : Put B = 0, A = 100

Put A = 0, B = 200
327





Painting Constraint : Put B = 0, A = 200

Put A = 0, B = 100

The graphical representation will be as under:



Q Intersects 4A + 2B = 400 (1)
and 4A + 4B = 480 (2)
Subtracting (2) from (1), we get 2B = 80
B = 80/2 = 40

Putting value of B in (1), we get 4A + 2 40 = 400

A =
400 80
= 80

4

R Intersects 4A + 4B = 480 (3) and A +
2B = 200 (4) Multiplying (4) by (2) and then subtracting
from (3), we get
2A = 80

A = 40

Putting the value of A in (4), we get 2B = 200 40

B = 80.
328





Evaluation of corner points:

Point Products Contribution Total
Contribution

A B A (Rs.)
100 per unit
B (Rs.)
75 per unit
Rs.
P 0 100 0 7,500 7,500
Q 80 40 8,000 3,000 11,000
R 40 80 4,000 6,000 10,000
S 100 0 10,000 0 10,000
Optimal product mix is Q

Product Units Contribution
Rs.
A 80 8,000
B 40 3,000
Total contribution 11,000
Less: Fixed costs 400 D.L. Hrs. Rs. 17.50 7,000

Optimal Profit 4,000


(iii) If the painting time can be sold at Rs. 40 per hour the opportunity cost is calculated as
under:

A

(Rs.)
B

(Rs.)
Income from sale per hour 40 40
Painting variable cost per hour 30 30
Opportunity cost 10 10
Painting hours per unit 1 2
Opportunity cost 10 20
Revised contribution 100 10 = 90 75 20 = 55


Hence, modification is required in the objective function.

Re-formulated problem will be:
329





Z Max.

Subject to:
90A + 55B Maximisation of contribution

4A + 4B 480 Raw Material constraint

4A + 2B 400 Direct Labour hour constraint

A + 2B 200 Painting hour constraint

A, B 0 Non-negativity constraint

Ans 40:
Dual:

Minimise 140u
1
+ 120u
2
+ 50u
3

S.T. 6u
1
+ 10u
2
+ 10u
3
100


4u
1
+ 10u
2
+ 12u
3
90


8u
1
+ 2u
2
+ 6u
3
40


4u
1
+ 6u
2
+ 2u
3
60

u
1
, u
2
u
3
u
4
0


330
Ans. 6
Transportation
(a)

3
--
9
----
6

4

4
---
6
---
8

3

5

40

50

30


(ii) Initial allocation under NW corner rule is as above.
Initial cost: 203 = 60
204 = 80
204 = 80
303 = 90
305 =

150
(a)
460

3
--
9
----
6

4

4
---
6
---
8

3

5

40 50 30

1 1 1

4 1 1

1 1

Initial solution 203 = 60
204 = 80
503 = 150
206 = 120
105 = __100
__


460
Checking for optimality
3

4 6



3 5
V1 =3 v2 = 3 v3 = 5



20
20

40

60

120
20 20
30 30
20
20 3

40 0 0 2

60 2 2 2

20 20
30 30
U1 = 0

U2 = 1

U3 = 0
331

Ui+ vj
3

5
4


3


3 3 5

ij = Cij- (ui-vj)

6 1
0
5
ij > 0 Solution is optimal


Conclusion:
The solution under VAM is optimal with a zero in R
2
C
2
which means that the cell C
2
R
2
which means
that the cell C
2
R
2
can come into solution, which will be another optimal solution. Under NWC rule
the initial allocation had C
2
R
2
and the total cost was the same Rs. 460 as the total cost under
optimal VAM solution. Thus, in this problem, both methods have yielded the optimal solution under
the 1st allocation. If we do an optimality test for the solution, we will get a zero for ij in C
3
R
2


indicating the other optimal solution which was obtained under VAM.
Ans. 8
The new transportation costs table, which consists of both production and transportation costs, is given in following table.
Store
P Q R S Supply
A 2+2=4 4+2=6 6+2=8 11+2=13 50
B 10+3=13 8+3=11 7+3=10 5+3=8 70
Factories C 13+1=14 3+1=4 9+1=10 12+1=13 30
D 4+5=9 6+5=11 8+5=13 3+5=8 50
Demand 25 35 105 20 200
185

Since the total supply of 200 units exceeds the total demand of 185 units by 200-185 =15 units of product, there fore a
dummy destination (store) is added to absorb the excess supply. The associated cost coefficients in dummy store are
taken as zero as the surplus quantity remains lying in the respective factories and is, in fact, not shipped at all. The
modified table is given below. The problem now becomes a balanced transportation one and it is a minimization problem.
We shall now apply Vogels Approximation method to fine an initial solution.





0

1

0
332
P Q R S Dummy Supply Difference
A 25
4
5
6
20
8
13

0 50/25/20/0 4 2 2 2 5

B 13 11 70
10
8

0 70/0 8 2 2 2 2 2

C 14

30
4
10 13 0 30/0 4 6 _ _ _ _

D 9 11 15 20 15 50/35/15/0 8 1 1 3 3 5
13 8 0
Demand 25/0 35/5/0 105/85/15/0 20/0 15/0 200
Difference 5 2 2 0 0
5 2 2 0 -
5 5 2 0 -
- 5 2 0 -
- - 2 0 -
The initial solution is shown in above table. It can be seen that 15 units are allocated to dummy store from factory D. This
means that the company may cut down the production by 15 units at the factory where it is uneconomical. We will now
test the optimality of the solution. The total number of allocations is 8 which is equal to the required m+n-1 (=8) allocation.
Introduce u
is,
v
j
s, i= (1,2,- - - - -4) and
j
=(1,2,- - - -5)
ij
=c
ij
-(u
i
+v
j
) for allocated cells. We assume that u
4
=0 and
remaining u
j
s, v
j
s and
ij

s are calculated below
P Q R S Dummy Supply U
A
i

25
4
5
6
20
8
13
+10
0
+5
50 U
1
= -5

B 13
+7
11
+3
70
10
8
+3
0
+3
70 U
2
=

C 14
+1
30
4
10
+4
13
+12
0
+7
30 U
3
= -7

D 9
0
11 15 20 15 50 U
4
= 0
0 13 8 0
Demand 25 35 105 20 15
V V
j

1
2 =9 2 0 0
Please not that figures in top left hand corners of the cell represent the cost and the one in the bottom right hand corner of
the non basic cell are the values of
ij
=c
ij
-[(u
i
+v
j
Since opportunity cost in all the unoccupied cells is positive, therefore initial solution is an optimal solution also. The total
cost (transportation and production together) associated with this solution is
)]
Total cost = 425+65+820+1070+430+1315+820+015
= 100+30+160+700+120+195+160
= Rs.1,465/-
Ans.9:
333
The given problem is an unbalanced transportation problem since the availability of trailers
(= 10+4+6+5=25) is less than the requirement (=13+10+6+6=35). Therefore, it is first converted into a balanced problem
by adding a dummy terminal with an availability of 10 trailers and cost elements for various plants as zero. The problem
becomes as given below.
Plants
Terminals A B C D Availability
U 20 36 10 28 10
V 40 20 45 20 4
W 75 35 45 50 6
X 30 45 40 25 5
Dummy 0 0 0 0 10
Requirement 13 10 6 6
The objective of the company is to minimize transportation cost. To achieve this objective, let us find an initial feasible
solution by applying Vogels Approximation Method to the above matrix.

Plants
Terminals A B C D Availability Difference
U 3
20

36
6
10
1
28

10/4/1/0

10/10/8/8
V
40
4
20

45

20

4/0

0/0/0/-
W
75
6
35

45

50

6/0

10/10/15/15
X
30

35

40
5
25

5/0

5/5/5/5
Dummy 10
0

0

0

0

10/0

0/-/-/-
Requirement 13/3/0 10/6/0 6/0 6/1/0
Difference 20 20 10 20
10 15 30 5
10 15 0 5
10 0 - 5
The initial solution is as given below which is tested for optimality.






334
Plants
Terminals A B C D Availability
U 3
20

36
6
10
1
28

10
V
40
4
20

45

20

4
W
75
6
35

45

50

6
X
30

35

40
5
25

5
Dummy 10
0

0

0

0

10
Requirement 13 10 6 6


The number of allocation is 7 which is one less than the required m+n-1 (=8) allocations. Introduce a very small quality e
in the least cost independent cell (Dummy, B0. Let us also introduce u
j
, v
j
; I- (1,2 5) j = (1,2,3,4) such that
ij =
cij-(u1+v
j
)
for allocation cells. We assume that u1=0 and remaining u
i
s, v
j
s and
ij

s are calculated as below:
Terminals A B C D u
i
s
U 3 +
20
16
36
6
10
1 -
28

0
V 20
40
4 -
20
35
45
-8 +
20

0
W 40
75
6
35
20
45
7
50

15
X 13
30
18
35
33
40
5
25

-3
Dummy 10 -
0
e +
0
10
0
-8
0

-20
vjs 20 20 10 28

Since some of the
ij
s are negative, the above solution is not optimal. Introduce in the cell (V,D) with the most negative

ij
an assignment . And the reallocated solution as obtained from above is given below. The values of u
i
s and v
j
s and

ij

s also calculated.



335
Terminals A B C D u
i
s
U 4
20
16
36
6
10
8
28

0
V 20
40
3
20
35
45
1
20

0
W 40
75
6
35
20
45
15
50

15
X 5
30
10
35
25
40
5
25

5
Dummy 9
0
1
0
10
0
0
0

-20
v
j
20 s 20 10 20 -20
Since all Ijs for non basic cells are positive, therefore, the solution obtained above is an optimal one. The
allocation of terminals to plants and their cost is given below.
Terminal Plant Cost
U A 4 Rs.20 = Rs.80
U C 6 Rs.10 = Rs.60
V B 3 Rs.20 = Rs.60
V D 1 Rs.20 = Rs.20
W B 6 Rs.35 = Rs.210
X D 5 Rs.25 = Rs.125
= Rs.555

Ans. 10:
Answer

(a) The problem may be treated as an assignment problem. The solution will be the same even if prices
are halved. Only at the last stage, calculate the minimum cost and divide it by 2 to account for fall in oil
prices.

A B C
X 15 9 6
Y 21 12 6
Z 6 18 9
Subtracting Row minimum, we get


A B C
X 9 3 0
Y 15 6 0
Z 0 12 3


Subtracting Column minimum,
336

A B C





No of lines required to cut Zeros = 3

Cost / u Units Cost Revised
Cost

Allocation: X B 9 10 90 45

Y C 6 10 60 30

Z A 6 10 60 30

210 105
Minimum cost = 105 Rs.

Al ternative Sol uti on I

Least Cost Method


X B Y C Z A







Test for optimality

No. of allocation = 3

No. of rows m =3, no. of column = 3

337
m + n 1 = 3 + 3 1 = 5

2 very small allocation are done to 2 cells of minimum costs, so that , the following table is got :

A B C

X

15

10
9


e
6


Y

21

12

10
6


Z

10
6


18

e
9

m + n 1 = 5

Now testing for optimality

u
i
9 e
0

6
0

6 e
0

v
j
6 9 6

u
i
+ v
j
for unoccupied cells


A B C
X 6 - -
Y 6 9 -
Z - 9 -

Diff = Cij (u
i
+ v
j
)

A B C
X 9 - -
Y 15 3 -
Z - 9 -

All
ij
> 0, Hence this is the optimal solution.







338
Original Costs Reduced Costs
due to Oil Price
Qty. Cost
X B
Y C
Z A
9

6

6
4.5

3

3
10

10

10
45

30

30

105
Total cost of transportation is minimum at Rs.105


Al ternative Sol uti on II




No. of rows + no. of column 1

m + n 1 = 5

No. of allocation = 3
339
3 - -

3 4.5 -

- 4.5 -



Hence add e to 2 least cost cells so that


Now m + n 1 = 5
Testing for optimality,
u
i
, v
j
table

A B C u
i
X 4.5 e
0

Y 3
0

Z 3 e
0

v
j
3 4.5 3

u
i
+ v
j
for unoccupied cells



3 - -

3 4.5 -

- 4.5 -

Cij u
i
+v
j


7.5 - -

11.5 6 -

- 9 -

ij
= C
ij
(u
i
+ v
j
)

4.5 - -

11.5 1.5 -

8.5 4.5 -

All
ij
> 0. Hence the solution is optimal.



Qty. Cost/u Total Cost
X B
Y C
Z A
10

10

10
4.5

3

3
45

30

30
Total minimum cost at revised oil prices 105
340
9

2

5

6

2

C1 C2 C3 C4 C5

8

6

4
11

2

8

6

2


10

9

9

12

9

6


8

7

6

3

7

7


2

0

2

9

3

5

6

11

12

8

8

8

4



Ans.11:
The concept tested in this problem is Degeneracy with respect to the transportation problem. Total of rows
and columns = (4 + 5) = 9. Hence, the number of allocations = 9
1 = 8. As the actual number of allocation is 7, a zero allocation is called for. To resolve this, an
independent cell with least cost should be chosen. R4C2 has the least
cost (cost = 3), but this is not independent. The next least cost cell R4C3 (cost = 5) is
independent.




Total


0R1 18



0R2 10


2R3 8



0R4 4

Total 40

Forming Equations through allocated cells

Basic equation Setting R1 = 0 other values
R1 + C2 = 2 Setting R1 = 0, C2 = 2
R1 + C4 = 6 C4 = 6
R1 + C5 = 2 C5 = 2
R2 + C1 = 9 R2 = 0
R3 + C3 = 3 R3 = 2
R4 + C1 = 9 C1 = 9
R4 + C3 = 5 C3 = 5
R4 + C4 = 6 R4 = 0

Evaluate unallocated cells

R1C1 = 11 0 9 = 2 R3C1 = 7 + 2 9 = 0
R1C3 = 8 0 5 = 3 R3C2 = 6 + 2 2 = 6
R2C2 = 9 0 2 = 7 R3C4 = 7 + 2 6 = 7
R2C3 = 12 0 5 = 7 R3C5 = 7 + 2 2 = 7
R2C4 = 9 0 6 = 3 R4C2 = 3 0 2 = 1
R2C5 = 6 0 2 = 4 R4C5 = 11 0 2 = 9
Since all the evaluation is 0 or +ve, the optimal solution is obtained.

Optimal cost = (8 2) + (6 6) + (4 2) + (10 9) + (8 3) + (2 9) + (0 5) + (2
6) = 16 + 36 + 8 + 90 + 24 + 18 + 10 + 12 = Rs. 204.

Note: As regards allocation of the zero values, the solution to the above problem is also obtained by
allocating the zero value in other independent cells such as R1C3, R2C2, R2C3, R3C1, R3C2, R3C4,
341
38 40 43 0 190/100/0 38/1/1/3
90/0 110/10/0 160/0 60/0
0 0 0 0
10* 2 6* 0

3 7 0
4 0 52 0
38 40

40 50 52


50

37

28

25 12

2 2 14


1

14

11

18 12

e
2
R3C5. In such situation there will be one more iteration.

Ans. 12
The optimum distribution for this company to minimize shipping costs
Availabilities = 160 +150 +190 = 500
Requirements = 80 +90 +110 +160 = 440
Availabilities Requirement = 500 440 = 60


Therefore, a dummy warehouse H is introduced, and initial solution is obtained below by
VAM in just one table.

D E F G H Available Diff.

160 e

A 42 48 38 37 0 160/0 37/1/1/1

80 10 60

B 40 49 52 51 0 150/90/10/0 48*/9/11*/1

90 100

C 39

Reg. 80/0

Diff. 0

1


since there are only 6 (one less than m+n 1) allocations, an infinitesimally small allocation e is placed
in the least cost and independent cell (1, 5). This solution is tested for optimality below. (N.B.: if
allocations were m +n 2 we would place two es, e ,
which are virtually zero in the 2 least cost independent cells). This device enables us to
apply to optimality test on (m +n 1) allocations.


V
j
0
0
12
V j 4 0 50 52 3 7 0



(u
i
+ v
j
) matrix








ij m atrix

Since there are ve
ij
s the initial solution is not optimal. Reallocation is done below by ticking the
most -ve
ij
cell (1, 3) and involving it in the loop.

mx
342

160 e Note that the maximum that can be
tansferred to the ticked cell is e. Since e is
infinitestimally small it leaves other corner
allocations unaffected. (Intermediate i.e.
non corner allocations are never altered in
the process of reallocations).
80

10

60

90 100



e 160

80

10

60

90 100



38 37

40

52

0

38 40


26 36

14

50

51

28

39 12

16 12

14

1

0

11

4 12

160
80

10

60


90 100+


e 160
80 10 60
80 110




38 37

40 49

0
0 - 3 8 40

e = 0



min

10 = 0



= e






Reallocation





This solution is tested for optimally below :

38

52

40

-12 -2 0 -1 -52




(u
i
+v
j
) matrix)






j matrix



Since there are ve , this solution too is not optimal. Reallocation is done below :
10 = 0

max = min

90 = 0








Reallocation



Since there are ve
ij
this solution too is not optimal. Reallocation is done below.

This solution is tested for optimality below:


u i

1 3

0

1 1
V i 40 49 51 50 0








343
27 26

13

51 50

29

39 11

15 1 2

13

1 1

10

4 11

3
j

( v i +v j )





ij m a t ri x



Since all
ij
s are +ve, this solution is optimal.

Ans. 15:


The initial solution is found by VAM below:
Factory Godown Availability Diff.
1 2 3 4 5 6
1 7 20 5

7 7 5 40 3 60/40/0 2/4/0
2 10 9

11 10 6 11 5 20/10/0 1/3
3 11

10 30 6 20 2 40 2 8 90/70/30/0 0/4/2/5
4 50 9

10 9 6 9 12 50/0 3/0
Demand 60
0
50 0
20 40
0
10 0
20
0
40
0
40
Diff. 2 5 0/1 4 3 2

The above initial solution is tested for optimality. Since there are only 8 allocations and we
require 9(m+n-1 =9) allocations, we put a small quantity in the least cost independent cell
(2, 6) and apply the optimality test. Let u= 0 and then we calculate remaining ui and v
v
j
u
Factory
i

Godowns
1 2 3 4 5 6
1 7

20
5

7

7

5
40
3
2 10
9

11
10
6

11


e
5
3
11

10
30
6
20
2
40
2

8
4 50
9

10

9

6

9

12
Vj 9 7 6 2 2 5

Now we calculate
ij
= c
ij
(u
i
+v
j
) for non basic cells which are given in the table below:

0 3 7 5
4 9

2 3 3
3 3 4 7 7

ij
matrix











- 2

0

0

0
344


Since all
ij
are positive, the initial solution found by VAM is an optimal solution. The final

allocations are given below:
Factory to Godown Unit Cost Value

1 2 20 5 100
1 6 40 3 120
2 1 10 9 90
2 3 10 6 60
3 3 30 6 180
3 4 20 2 40
3 5 40 2 80
4 1 50 9

450
Total cost Rs. =
The above solution is not unique because the opportunity cost of cell (1,2) is zero. Hence alternative
solution exists. Students may find that the alternative solution is as given below:
1,120
Factory to Godown Unit Cost Value
1

1 10 7 70
1

2 20 5 100
1

6 30 3 90
2

3 10 6 60
2

6 10 5 50
3

3 30 6 180
3

5 40 2 80
3

4 20 2 40
4

1 50 9

450

Total cost (Rs.)

1,120
Ans. 16
The given problem is a balanced minimization transportation problem. The objective of the company is to minimize the
cost. Let us find the initial feasible solution using Vogels Approximation method (VAM)
Outlets
Plants A B C D Capacity Difference
X
4
400
6

8
300
6
700/300/0 2 2 0 0

Y
3
50
5
350
2

5
400/50/0 1 2 0 0

Z 400
3

9

6
200 600/200/0 2 2 4 0

Requirement 400/0 450/400/0 350/0 500/300/0
Difference 0 1 4 0
0 1 - 0
- 1 - 0
The initial feasible solution obtained by VAM is given below:
345
Outlets
Plants A B C D Capacity
X
4
400
6

8
300
6
700

Y
3
50
5
350
2

5
400

Z 400
3

9

6
200
5
600

Requirement 400 450 350 500

Since the number of allocations = 6= (m+n-1), let us test the above solution for optimality. Introduce u
i
(i=1,2,3) and v
j

(1,2,3,4) such that
ij
= C
ij
(u
i
+v
j
) for allocated cells. We assume u
1
=0, and rest of the u
i
s, vjs and i
j
Outlets
s are calculated
as below:
Plants A B C D U
X
i

0
4
400
6
5
8
300
6
0

Y 0
3
50
5
350
2
0
5
-1

Z 400
3
4
9
4
6
200
5
-1

V 4
j
6 3 6
On calculating i
j
s for non-allocated cells, we found that all the i
j
The optimal allocations are given below.
0, hence the initial solution obtained above is optimal.
Plants Outlet Units Cost Total Cost
X B 400 6 = 2,400
X D 300 6 = 1,800
Y B 50 5 = 250
Y C 350 2 = 700
Z A 400 3 = 1,200
Z D 200 5 = 1,000
7,350

The minimum cost = 7,350 thousand rupees.
Since some of the i
j

s = 0, the above solution is not unique. Alternative solutions exist.
Ans.17:
The given problem is a transportation problem. The profit matrix for various factories and sales counters is calculated
below:
346
Factory Sales Centres Capacity (kgms)
1 2 3
A 3 2 4 100
B 0 -1 1 20
C 4 3 5 60
D 2 1 3 80
Demand (kgms) 120 140 60
Since this is an unbalanced transportation problem (demand > capacity), let us introduce a dummy factory with profit as
Rs.0 per unit for various sales centres and capacity equal to sixty units. The resulting matrix would be as below:
Factory Sales Centres Capacity (kgms)
1 2 3
A 3 2 4 100
B 0 -1 1 20
C 4 3 5 60
D 2 1 3 80
Dummy 0 0 0 60
Demand (kgms) 120 140 60

The above profit matrix can be converted into a loss matrix by subtracting all its elements from the highest payoff of the
matrix i.e. 5. The loss matrix so obtained is given below:
Factory Sales Centres Capacity (kgms)
1 2 3
A 2 3 1 100
B 5 6 4 20
C 1 2 0 60
D 3 4 2 80
Dummy 5 5 5 60
Demand (kgms) 120 140 60

The initial solution is obtained by applying Vogels approximation method.
Factory Sales Centres Capacity Difference
1 2 3
A 100
3

1

100/0

1 1 - 2
B
5
20
4

20/0

1 1 1 6
C
1

2
60
60/0

1 - - 0
D 20 60
347
3 4 2 80/60/0 1 1 1
Dummy
5
60
5

60/0

0 0 0 5
Demand 120/20/0 140/120/60/0 60/0
Difference 1 1 1
1 1 -
2 1 -

The solution obtained by VAM is as given below:
Factory Sales Centres Ui
1 2 3
A 100 0
3
E
1

3 2
B 0
5
20 0
4

6 6
C 0
1
0
2
60
2 0
D 20 60 0
2

4 3 4
Dummy 1
5
60 2
5

5 5
Vj -1 0 2

Since all
ij
From Factory
0 for the non allocated cells, hence the solution given by above matrix is optimal. The optional solution for
the given problem is given below:
To Sales Centre Quantity Profit per unit
(Rs.)
Total Profit (Rs.)
A 1 100 3 300
B 2 20 -1 -20
C 3 60 5 300
D 1 20 2 40
D 2 60 1 60
Dummy 2 60 0 0
Total Profit = 660
(Note: since some of the
ijs

are equal to zero, alternative solutions also exist.)
Ans.18:
The given problem is an unbalanced transportation problem which is converted into a balanced on by adding a dummy
investment as given below:
348
Year Net Return data (in paise) of Investment Dummy Amount
Payable
P Q R S
1 95 80 70 60 0 70
2 75 65 60 50 0 40
3 70 45 50 40 0 90
4 90 40 40 30 0 30
Maximum
Investment
40 50 60 60 20
The values in the table represent net return on investment of one rupee till the end of the fourth year. The objective of the
company is to maximize the net return. For achieving this objective, let us convert this maximization problem into
minimization problem by subtracting all the elements of the above payoff matrix from the highest payoff i.e. 95, and apply
Vogels approximation method for finding the initial feasible solution.
Year Loss Matrix Investment type Dummy Amount
Available
Difference
P Q R S
1 40
0
30
15

25

3
5

9
5

70/30/0

15/10 _ _

2
20
20
30
20
35

4
5

9
5

40/20/0

10/5/5/10

3
25

50
40
45
50
5
5

9
5

90/50/0
10/40/20
/0/0

4
35

55

55
10
6
5
20
9
5

30/20/0

10/3/0

Maximum
Investment
40/0 50/20/0 60/40/0 60/10/0 20/0
Difference 20 15 10 10 0
- 15 10 10 0
- 20 10 10 0
- - 10 10 0
solution obtained by VAM is as given below



349
Year Loss Matrix Investment type Dummy Amount
Available
P Q R S
1 40
0
30
15

25

35

95

70

2
20
20
30
20
35

45

95

40

3
25

50
40
45
50
55

95

90

4
35

55

55
10
65
20
95

30

Maximum
Investment
40/0 50/20/0 60/40/0 60/10/0 20/0

This initial solution is tested for optimality. There are 8 (=m+n-1) independent allocations. Let us introduce u
i,
v
j,

i=(1,2,3,4); = (1,2,3,4,5 such that Dij = cij = (u
i
+v
j
Year
) for allocation cell. We assume u1 = 0 and remaining u1s vjs and Dijs
are calculated.
Loss Matrix Investment type Dummy Amount
Available
P Q R S
1 40
0
30
15
5
25
5
35
35
95

0

2 5
20
20
30
20
35
0
45
20
95

15

3 0
25
10
50
40
45
50
55
10
95

25

4 0
35
5
55

55
10
65
20
95

35

v
j
0 s 15 20 30 60

On calculating A
ij
Year Invest in Net Return
s for non-allocated cells, we found that their values are positive, hence the initial solution obtained above
is optimal. The optimal allocations are given below:
1 Invest Rs 40 lacs in investment P 0.95xRs.40 lacs = Rs. 38,00,000
350
Rs 30 lacs in investment Q 0.80xRs.30 lacs = Rs. 24,00,000
2 Invest Rs 20 lacs in investment Q 0.65xRs.20 lacs = 13,00,000
Rs 20 lacs in investment R 0.60xRs.20 lacs = 12,00,000

3 Invest Rs 40 lacs in investment R 0.50xRs.40 lacs = Rs. 20,00,000
Rs 50 lacs in investment S 0.40xRs.50 lacs = Rs. 20,00,000
4 Invest Rs.10 lacs in investment S 0.30xRs.10 lacs = Rs.3,00,000
Total Rs.130,00,000

Ans. 19:
The given information can be tabulated in following transportation problem:
Profit
Sales offices
Capacity in
units
Plant 1 2 3 4 5
1
2
3
Demand
9
-1
8
80
11
3
9
100
6
1
10
75
5
9
14
45
5
1
4
125
150
200
125

Where entries in the cells of the above table indicate profit per unit received by selling one
unit of item from plant i (1 =1,2,3) to the sales office (i=1,2,3,4,5). The profit per unit is
calculated using the following formula.

Profit = sales price (production cost +Shipping cost)

The objective of the company is to maximize the profit. For achieving this objective, let us
convert this maximization problem into minimization problem by subtracting all the elements
of the above payoff matrix from the highest payoff i.e. Rs. 14.


Loss matrix
Sales offices Capacity in
units
Plant 1 2 3 4 5
1
2
3
5
15
6
3
11
5
8
13
4
9
5
0
9
13
10
150
200
125
Demand 80 100 75 45 125
The problem is an unbalanced transportation problem since capacity (=475 units) is 50 units
more than the demand. Hence a dummy sales office is added with cost equal to zero for all
plants and demand equal to 50 units. Now, let us apply Vogels Approximation method to
the resultant balanced matrix for finding the initial feasible solution.









351
Sales offices
Plant 1 2 3 4 5 Dummy Capacity Difference
1 50

5
100

3


8


9


9

0


150/50/0

3/3/2/2/4
2 25

15


11


13


5
125

13
50
0
200/150/125/0 5/11/2/2/2/2
5

75 45
3 6 5 4 0 10 0 125/80/5/0 0/4/1/1/4/4

Demand 80/30 100/0 75/0 45/0 125/0 50/0
/25/0
differ 1 2 4 5 1 0
1 2 4 -- 1 0
1 2 4 -- 1 --
1 2 -- -- 1 --
1 -- -- -- 1 --

The initial solution obtained by VAM is given below which is tested for optimality.


Plant 1 2 3 4 5 Dummy Capacity
in units
1 50
5
100
3

8

9

9

0

150
2 25

15

11

13

5
125
13
50
0

200
3 5
6

6
75
4
45
0

10

6

125
Demand
in units

80

100

75

45

125

50


These are m +n 1 =8 independent allocations. Let us now introduce ui, vj, I = (1,2,3); j =
(1,2-----6) such that ij = Cij (ui +vj) for allocation cells. We assume u2 = 0 and remaining
uis vjs and ijs are calculated as below:


Sales offices
Plant 1 2 3 4 5 Dummy U
i
s
1 50
5
100

3
5
8
10
9
6

9
10
0

-10
2 25 -

15
-2

11
10
13
-4 +

5
125
13
50
0

0
3 5 +
6
1
5
75
4
45 -

0
6
10
9
0

-9
V
j
15 s 13 13 9 13 0











352
Since some of the
ij
s are negative, therefore, the above solution is not optimal. Introduce in
the cell (2,4) with the most negative
ij
, an assignment. The value of and reallocated
solution as obtained from above is given below. The reallocated solution is again tested for
optimally. Hence, the values u
i
s v
j
s and
ij

s are again calculated.

Sales offices
Plant 1 2 3 4 5 Dummy Uis
1 50
5
100

3
5
8
10
9
2
9
6
0

-6
2 4
15
2
11
4
13
25
5
125
13
50
0

0
3 30
6
1
5
75
4
20
0
2
10
5
0

-5
11
Vjs
9 9 5 13 0

Since all
ij

s for non-basic cells are positive, therefore, the solution obtained above is an
optimal one. The allocation of plants to sales officers and their profit amount is given below:
Plant Sales units profit per profit
Office unit
1 1 50 9 450
1 2 100 11 1,100
2 4 25 9 225
2 5 125 1 125
2 Dummy 50 0 0
3 1 30 8 240
3 3 75 10 750
3 4 20 14
Total
280

3,170
Ans.20:

Convert the given profit matrix into a loss matrix by subtracting each element of the matrix from the
highest value viz.44.The resulting loss matrix is as follows:

Loss Matrix

Customer
------------------------------------------------
Factory A B C D supply


P 4 19 22 11 100
Q 0 9 14 14 30
R 6 6 16 14 70
Demand 40 20 60 30 150/200



The loss matrix, obtained as above is an unbalanced one, We introduce a dummy column to
make it a balanced one.




353
Loss Matrix


Customers

Factory A B C D Dummy Supply
______________________________________


P 4 19 22 11 0 100
Q 0 9 14 14 0 30
R 6 6 16 14 0 70
Demand 40 20 60 30 50 200/200


By using Vogals approximation method, the following initial feasible solution is found

Customers
Factory A B C D Dummy Supply



P 10 60 30 e 100

4 19 22 11 0
--------------------------------------------------------------------------------------------------------------
Q 30 30

0 9 14 14 0
----------------------------------------------------------------------------------------------------------------
R 20 50 70

6 6 16 14 0
-----------------------------------------------------------------------------------------------------------------
Demand 40 20 60 30 50 200/200



Since the number of allocations in the initial feasible solution are 6 and for applying
optimality test they should be equal to (m+n-1)=7, therefore we enter a very small
assignment equal to e in the minimum cost so that no loop is formed.

Let us introduce the variables Ui and Vj such that Ui + Vj = Cij for allocated cells.
We thus have the following relations:

U
1
+ V
1
= 4 U
2
+ V
1
U
= 0
1
+ V
3
= 22 U
3
+ V
2
U
= 6
1
+ V
4
= 11 U
3
+ V
5
U
= 0
1
+ V
5
Put U
+ 0
1
V
= 0,we get
1
= 4;V
3
= 22; V
4
= 11; V
5
= 0; U
3
= 0;V
2
= 6 and U
2

= (-4)
Compute: Cij (Ui + Vj) for non-allocated cells.
U
1
V
2
U
=19 - (0 + 6) = 13
2
V
2
= 9 - (- 4 + 6) =7
354
U
2
V
3
U
= 14 - ( - 4 + 22) = (-4)
2
V
4
U
= 14 - (- 4 + 11) = 7
2
V
5
U
= 0 - (- 4 + 0) = 4
3
V
1
U
= 6 - (0 + 4) = 2
3
V
3
U
= 16 - (0 + 22) = (-6)
3
V
4

= 14 - (0 + 11) = 3
Since the value of Cij - (Ui + Vj)is negative in two cells therefore the initial solution is not
optimal, Introduce an assignment 0 in the cell U3V3 and construct a loop shown as below,
after adjusting.



Customers


Factory A B C D Dummy Supply Ui
---------------------------------------------------------------------------------------------------------------------------
-----
P 10 60-0 30 e+0 100 U
1

=
0
4 19 22 11 0
30
Q 30 U
2
0 9 14 14 0
=
(-4)
20 0 50-0
R 70 U
3
6 6 16 14 0
=
0

Demand 40 20 60 30 50 200/200


V
j
V
1
= 4 V
2
= 6 V
3
= 22 V
4
=11 V
5


= 0

Maximum value of 0 = 50

Apply optimality test once again. Introduce U
i
and V
j

s and determine their values
Compute C
ij
- (U
i
+ V
j
) for non-allocated cells, since it comes out to be negative for U
2
V
3

cell, therefore we repeat the aforesaid process by introducing 0 in U
2
V
3

cell, the minimum
value 0f 0 is 10.







355


Customers



Factory A B C D Dummy Supply U


i

P 10+ 10- 30 50 100 U
1

=0
4 19 22 11 0
0
Q 30- 30 U
2

=(-4)
0 9 14 14 0

R 20 50 70 U
3

= ( -
6)
6 6 16 14 0

Demand 40 20 60 30 50 200/ 200

V
1
V
1
=4 V
2
=12 V
3
=22 V
4
=11 V
5


=0
The second improved solution obtained is as under:
Apply optimality test to the solution once again after determining the values of U
i
and V
j
.
Since C
ij
- (U
i
+ V
j


) for non-allocated cell is positive, therefore the following solution is
optimal one.
Customers


Factory A B C D Dummy Supply Ui
---------------------------------------------------------------------------------------------------------------------------
---
P 20 30 50 100 U1=0

4 19 22 11 0
10

Q 20 30 U2=(-4)

0 9 14 14 0

R 20 50 70 U3=(-2)

6 6 16 14 0

Demand 40 20 60 30 50 200/200

356
Vj V1=4 V2=8 V3=18 V4+11 V5=0

Transferring the solution to the original profit matrix, we get;



Customers


Factory A B C D Dummy Supply
-------------------------------------------------------------------------------------------------
P 20 30 50 100

40 25 22 33 0


Q 20 10 30

44 35 30 30 0

R 20 50 70

38 38 28 30 0

Demand 40 20 60 30 50



Maximum profit
=20 Rs.40+30Rs.33+20*Rs.44+10*Rs.30+20*Rs.38+50*Rs.28+50*Rs.0
=Rs.800+Rs.990+Rs.880+Rs.300+Rs.760+Rs.1,400
=Rs.5,130

Ans. 21
The given information can be tabulated in following transportation problem:
Project
Auditor 1 2 3 Time available
(Hours)
(Rs.) (Rs.) (Rs.)
1 1,200 1,500 1,900 160
2 1,400 1,300 1,200 160
3 1,600 1,400 1,500 160
Time
Required 130 140 160
(Hours)



The given problem is an unbalanced transportation problem. Introducing a dummy project to
balance it, we get





357
Project
Auditor 1 2 3 Dummy Time
available
(Hours)
1 1,200 1,500 1,900 0 160
2 1,400 1,300 1,200 0 160
3 1,600 1,400 1,500 0 160
Time
Required
(hrs.)
130 140 160 50 480

The objective here is to maximize total billing amount of the auditors. For achieving this
objective, let us convert this maximization problem into a minimization problem by
subtracting all the elements of the above payoff matrix from the highest payoff i.e. Rs. 1900.
Project
Auditor 1 2 3 Dummy Time available
1 700 400 0 1900 160
2 500 600 700 1900 160
3 300 500 400 1900 160
Time
required
(Hrs)
130 140 160 50 480


Now, let us apply Volgels Approximation Method to the above matrix for finding the initial
feasible solution.


Project (Figure of payoffs in Rs. 00s)
Auditor 1 2 3 Dummy Time
Available
Difference

1

7

4
160
0

19

160/0

4/-/-/-

2

5
110
6

7
50
19

160/50/0

1/1/13/13

3
130
3
30
5

4

19

160/30/0

1/2/14/-
Time 130/0 140/110/0 160/0 50/0
Required

Difference 2/2/-/- 1/1/1/1 4/-/- 0/0/0

The initial solution is given below. It can be seen that it is a degenerate solution since the
number of allocation is 5. In order to apply optimality test, the total number of allocations
should be 6 (= m + n -1). To make the initial solution a non-degenerate, we introduce a very
small quantity in the least cost independent cell which is cell of Auditor 3, Project 3.








358
Project
Auditor 1 2 3 Dummy Time Available

1

7

4
160
0

19

160

2

5
110
6

7
50
19

160

3
130
3
30
5
e
4

19

160
Time
Required
130 140 160 50
Introduce u
is
and v
js
such that
ij
= C
ij
(u
i
+v
j
) (for I, = 1 to 3; j = 1,2,3, dummy). To
determine the values of u
is
and v
js
we assume that u
3
= 0, values of other variables i.e. u
is
,
v
js
Project
and are calculated as follows:
Auditor 1 2 3 Dummy U
j

s
U
1

=-4
U
2

=1
U
3
=0
1 8
7
3
4
160
0
5
19
2 1
5
110
6
2
7
50
19
3 130
3
30
5
e
4
1
19

V
j
s v
1
=3 v
2
= 5 v
3
=4 v
4
Since all for non basic cells are positive, therefore the initial solution obtained above is
optimal. The allocation of projects to auditors and their billing amount is given below: Here
an auditor may be involved in more one project as apparent from the following solutions.
=18

Auditor Project Billing amount (Rs.)
1 3 160xRs. 1900 = 3,04,000
2 2 110xRs. 1300 = 1,43,000
3 1 130xRs. 1600 = 2,08,000
3 2 30xRs. 1400 = 42,000
Total billing = 6,97,000

Hence, the maximum total billing during the next month will be Rs. 6,97,000

359
Assi gnment

Ans. 1:
I II III IV
1 16 52 34 22
2 26 56 8 52
3 76 38 36 30
4 38 52 48 20
Step 1:
Subtract the smallest element of each row from every element of the corresponding row
I II III IV
1 0 36 18 6
2 18 48 0 44
3 46 8 6 0
4 18 32 28 0
Step 2: Subtract the smallest element of each column from every element in that column
I II III IV
1 0 28 18 6
2 18 40 0 44
3 46 0 6 0
4 18 24 28 0
Step 3: Drew minimum number of horizontal and vertical lines to cover all the zeros
I II III IV
1 0 28 18 6
2 18 40 0 44
3 46 0 6 0
4 18 24 28 0
The optimal assignment is
1 I = 16
2 III = 8
3 II = 38
4 IV =

20
Minimum time taken = 82 hours
82 hours


Ans. 2:
(a) Consider the following assignment problem:
Division

N E W S
360

A 14 20 11 19

B 12 10 15 9
Marketing Executives
C 16 19 18 15

D 17 13 15 14

Step 1
Select the minimum element of first row and subtract it from all the elements of the row. On
repeating the step with all the rows of the above matrix, we get the following

Division
N E W S

A 3 9 0 8

B 3 1 6 0
Marketing Executives
C 1 4 3 0

D 4 0 2 1
Step 2

Select the minimum element of first column and subtract it from all the elements of the
column. On repeating this step with all the columns of the above matrix; we get the following
Division

N E W S

A 2 9 0 8
B 2 1 6 0
Marketing Executives
C 0 4 3 0
D 3 0 2 1
Step 3
On drawing the minimum number of lines in the above matrix, so as to cover at the zeros,
we get the following matrix. Division

N E W S

A 2 9 0 8
B 2 1 6 0
Marketing Executives
C 0 4 3 0

D 3 0 2 1
Since the minimum number of lines drawn under the step is equal to number of marketing
executives or number of divisions, therefore we go over to the final step for determining the
required optimal solution.

Step 4
For determining the optimal solution scan each row in turn for a single uncovered zero in it,
encircle it and pass a line in its column.
361
Division

N E W S

A 2 9 0 8
B 2 1 6 0
Marketing Executives
C 0 4 3 0

D 3 0 2 1

The optimal assignment obtained in this case is as under:

Marketing Division Cost
Executive Rs.
A W 11
B S 09
C N 16
D E
Total minimum cost
13

49

Using the information that the factory works effectively 7 hours (=420 minutes) a day and the time required by each
operator for producing each of the products, we obtain the following production and profit matrices:
Ans. 5:
Production Matrix (units) Profit Matrix (in Rs.)
Operator Product Operator Product
A B C D A B C D
P 70 42 30 35 P 210 84 120 35
Q 60 84 140 105 Q 180 168 560 105
R 70 60 42 42 R 210 120 168 42
S 21 42 28 28 S 63 84 112 28
In order to apply the assignment algorithm for minimizing losses, let us first convert this profit matrix to a loss matrix by
subtracting all the elements of the given matrix from its highest element which is equal to Rs.560. The matrix so obtained
is given below:
Operator Product
A B C D
P 350 476 440 525
Q 380 392 0 455
R 350 440 392 518
S 497 476 448 532
Now apply the assignment algorithm to the above loss matrix. Subtracting the minimum element of each row from all
elements of that row, we get the following matrix:


362
Operator Product
A B C D
P 0 126 90 175
Q 380 392 0 455
R 0 90 42 168
S 49 28 0 84
Now subtract the minimum element of each column from the elements of that column to get the following matrix:
Operator Product
A B C D
P 0 98 90 91
Q 380 364 0 371
R 0 62 42 84
S 49 0 0 0
Draw the minimum number of lines to cover all zeros. The minimum number of lines to cover all zeros is three which is
less than the order of the square matrix (i.e.4) thus the above matrix will not give the optimal solution. Subtract the
minimum uncovered element (=62) from all uncovered elements and add it to the elements lying on the intersection of two
lines, we get the following matrix:
Operator Product
A B C D
P 0 36 90 29
Q 380 302 0 309
R 0 0 42 22
S 111 0 62 0
The minimum number of lines which cover all zeros is 4 which is equal to the order of the matrix, hence, the above matrix
will give the optimal solution. Specific assignments in this case are as below:
Operator Product
A B C D
P 0 36 90 29
Q 380 302 0 309
R 0 0 42 22
S 111 0 62 0

Operator Product Profit (Rs.)
P A 210
Q C 560
R B 120
S D 28
Total Profit (Rs.) 918

Ans. 8:
363
(i )

4 12 16 8
20 28 32 24
36 44 48 40
52 60 64 56

Subtracting minimum element each row.


0 8 12 4

0 8 12 4

0 8 12 4

0 8 12 4

Subtracting minimum element each column,


0 0 0 0

0 0 0 0

0 0 0 0

0 0 0 0

Minimum no. of lines to cover all zeros = 4 = order of matrix. Hence optional
assignment is possible.

Minimum cost = 4 + 28 + 48 + 56 = 136.

= AR
1
+ BR
2
+ CR
3
+ DR
4

Since all are zeros, there are 24 solutions to this assignment problem.

Viz. A B C D
R
1
R
2
R
3
R
4

R
2
R
3
R
4
R
1

R
3
R
4
R
1
R
2

R
4
R
1
R
2
R
3

R
1
R
3
R
4
R
2 etc.
A can be assigned in 4 ways, B in 3 ways for each of As 4 ways.

(i i ) SP VC = 100 Rs.

A B C D
R
1 96 88 84 92
R
2 80 72 68 76
R
3 64 56 52 60
R
4 48 40 36 44
Subtracting the highest term

0 8 12 4
16 24 28 20
32 40 44 36
48 56 60 52
Subtracting minimum term of each row.

364
0 8 12 4
0 8 12 4
0 8 12 4
0 8 12 4
Which is the same as the earlier matrix

Maximum contribution = Rs. (96 + 72 + 52 + 44) = Rs. 264.
Al ternative Solution:

Maximisation of contribution is same as minimizing cost. Hence, same assignments as in
(i) will be the optional solution.

Maximum Contribution Rs. (400 136) = Rs. 264

(i ii ) (a) The relative cost of assigning person i to region r does not change by addition or subtraction of a
constant from either a row, or column or all elements of the matrix.
(b) Minimising cost is the same as maximizing contribution. Hence, the assignment solution will be
the same, applying point (i) above.
(c) Many zeros represent many feasible least cost assignment. Here, all zeros mean maximum
permutation of a 4 4 matrix, viz. 4 3 2 1 = 24 solutions are possible.

Reducing minimum from each column element (figure in 000s)
Ans. 9:
Step 1 Step 2
R R
1
R
2
R
3

4
R R
1
R
2
R
3

C
4

1
1
1 C 0
1
0
C
2
0 0 C
2
0 0
C 0
3
0 C 0
3
0
C
4
2 1 C
4
1 0
Number of lines to connect all zeros nos. is 4 which is optional.
Alternatively you may also reduce the minimum from each row.
Step 1 Step 2
R R
1
R
2
R
3

4
R R
1
R
2
R
3

C
4

0
1
1 C 0
1
1
C
2
0 0 C
2
0 0
C 1
3
0 C 0
3
0
C
4
0 1 C
4
0 0

Number of lines to connect all zeros nos. is 4 which is optional.
All diagonal elements are zeros and are chosen. The minimum cost is Rs.15,000 C
1
R
1
4,000; C
2
R
2

4,000; C
3
R
3
2,000; C
4
R
4

5,000; (Total) = 15,000.
Let us first formulate the preference ranking assignment problem.
Ans.10:

MANAGERS

365
Room No. M
1
M
2
M
3
M
4
M
5

301 4 2 1
302 1 1 5 1 2
303 2 1 4
304
305 3 4 2
3 2 3 3 3
We have to find an assignment so that total preference ranking is minimum. In a cell (-) indicates
that no assignment is to be made in that particular cell. Let us assign a very large ranking value M to
all such cells.
Step 1 : From each row, subtract the minimum element of that row, from all the elements of that row
to get the following matrix.
MANAGERS

Room No M
1
M
2
M
3
M
4
M
5

301 M 3 1 M 0
302 0 0 4 0 1
303 1 M 0 3 M
304 1 0 1 1 1
305 M 1 2 0 M
Draw the minimum number of lines in the above table to cover all zeros. In this case the number of
such lines is five, so the above matrix will give the optimal solution. The assignment is made as below:

MANAGERS
Rooms No.

M
1

M
2

M
3

M
4

M
5

301 M 3 1 M 0
302
0 0 4 0 1
303

1 M 0 3 M
304

1 0 1 1 l
305 M 1 2 0 M
Thus, the assignment is
M
1
302, M
2
304, M
3
303, M
4
305, M
5
301

and the total minimum ranking = 1 + 2 + 1 + 2 + 1 = 7

Ans. 11:
Dummy machine (M5) is inserted to make it a balanced cost matrix and assume i ts installation cost to be
zero. Cost of install at cell M3 (J) and M2 (L) is very high marked as .


J K L M N
M1

M2

M3

M4

M5 (Dummy)
18

24



28

0
22

18

22

16

0
30



28

24

0
20

20

22

14

0
22

18

14

16

0
Step 1

Subtract the minimum element of each row from each element of that row


J K L M N
366
M1
M2
M3
0
6

4
0
8
12

14
2
2
8
4
0
0


M4 14 2 10 0 2
M5 (Dummy) 0 0 0 0 0

Step 2

Subtract the minimum element of each column from each element of that column


J K L M N
M1
M2
M3
M4
M5 (Dummy)
0
6

14
0
4
0
8
2
0
12

14
10
0
2
2
8
0
0
4
0
0
2
0

Step 3

Draw lines to connect the zeros as under:


J K L M N
M1 0 4 12 2 4
M2 6 0 2 0
M3 8 14 8 0
M4 14 2 10 0 2
M5 (Dummy) 0 0 0 0 0


There are five lines which are equal to the order of the matrix. Hence the solution is
optimal. We may proceed to make the assignment as under:


J K L M N
M1


M2


M3


M4


M5 (Dummy)

0

6


e


14


0
4


0

8


2


0
12


e


14


10


0
2


2


8


0

0
4


0


0

2


0

The following is the assignment which keeps the total cost at minimum:

Machines Location Costs Rs.
M1 J 18
M2 K 18
367
M3 N 14
M4 M 14
M5 (Dummy) L 0
Total

64


Since the Executive Director of the 5 star hotel is interested in maximizing the revenue of the hotel, therefore,
the objective of the given problem is to identify the preferences of marriage parties about halls so that hotel
management could maximize its profit. To solve this problem first convert it to a minimization problem by
subtracting all the elements of the given matrix from its highest element which is equal to Rs. 10,000. The
matrix so obtained which is known as loss matrix is given below:
Ans. 12:

Loss matrix/Hall
Marriage party 1 2 3 4

A 0 1000 M M
B 2000 0 2000 5000
C 3000 0 4000 2000
D 0 2000 M M

Now apply the assignment algorithm to the above loss matrix. Subtracting the minimum element of each
column from all elements of that column, we get the following matrix.

Loss matrix/Hall
Marriage party 1 2 3 4

A 0 1000 M M
B 2000 0 0 3000
C 3000 0 2000 0
D 0 2000 M M


The minimum number of lines to cover all zeros is 3 which is less than the order of the square matrix (i.e. 4),
the above matrix will not give the optimal solution. Subtracting the minimum uncovered element (= 1000) from
all uncovered elements and add it to the elements lying on the intersection of two lines, we get the following
matrix

Marriage party 1 2 3 4

A 0 0 M M
B 3000 0 0 3000
C 4000 0 2000 0
D 0 1000 M M


Since the minimum number of lines to cover all zeros is 4 which is equal to the order of the matrix, the
above matrix will give the optimal solution which is given below:


Marriage party 1 2 3 4

A 0 0 M M
B 3000 0 0 3000
C 4000 0 2000 0
D 0 1000 M M

368

and the optimal schedule is :

Revenue (Rs.)
Marriage party A Hall 2 9,000
B Hall 3 8,000
C Hall 4 8,000
D Hall 1 10,000
Total 35,000

The following matrix gives the cost incurred if the typist (i = A, B, C, D, E) executes the job (j = P, Q, R, S, T).
Ans. 14:
Job
Typist P Q R S T
A 85 75 65 125 75
B 90 78 66 132 78
C 75 66 57 114 69
D 80 72 60 120 72
E 76 64 56 112 68
Subtracting the minimum element of each row from all its elements in turn, the above matrix reduces to
Job
Typist P Q R S T
A 20 10 0 60 10
B 24 12 0 66 12
C 18 9 0 57 12
D 20 12 0 60 12
E 20 8 0 56 12
Now subtract the minimum element of each from all its elements in turn, and draw minimum number of lines horizontal or
vertical so as to cover all zeros . All zeros can be covered by four lines as given below:
2 2 0 4 0
6 4 0 10 2
0 1 0 1 2
2 4 0 4 2
2 0 0 0 2
Since there are only 4 lines (<5) to cover all zeros, optimal assignments cannot be made. The minimum uncovered
element is 2.
We subtract the value 2 from all uncovered elements. Add this value to al junction values and leave the other elements
undisturbed. The revised matrix to obtained is given below:
2 2 2 4 0
4 2 0 8 0
0 1 2 1 2
0 2 0 2 0
2 0 2 0 2
369
Since the minimum no. of lines required to cover al the zeros is only 4(<5), optimal assignment cannot be made at this
stage also.
The minimum uncovered element is 1, repeating the usual process again, we get the following matrix.
2 1 2 8 0
4 1 0 7 0
0 0 2 0 2
0 1 0 1 0
3 0 3 0 3
Since the minimum number of lines to cover all zeros is equal to 5, is this matrix will give optimal solution? The optimal
assignment is made in the matrix below:
Typist P Q R S T
A 2 1 2 3 0
B 4 1 0 7 0
C 0 0 2 0 2
D 0 1 0 1 0
E 3 0 0 0 3
Cost ( Rs.)
Thus typist A is given job 75 T :
Thus typist B is given job 66 R :
Thus typist C is given job 66 Q :
Thus typist D is given job 80 P :
Thus typist E is given job S 112
Total Rs.399
Note: In case the above solution is not unique. Alternate solution also exists.

Ans. 17:
(a) Sum of the proportion = (8 + 7 + 5 + 4) = 24

Assuming Rs. 1,000 as one unit, the effective matrix is as follows:

Effecti ve Matri x
Managers
East West North South
Z

(8/24) 240 = 80 (8/24) 192 = 64 (8/24) 144 = 48 (8/24) 20 = 40
N
(7/24) 240 = 70 (7/24) 192 = 56 (7/24) 144 = 42 (7/24) 120 = 35
O
(5/24) 240 = 50 (5/24) 192 = 40 (5/24) 144 = 30 (5/24) 120 = 25
P
(4/24) 240 = 40 (4/24) 192 = 32 (4/24) 144 = 24 (4/24) 120 = 20

Convert the maximization problem to minimization problem

The resultant loss matrix is as follows:

Loss Matri x

Managers East West North South
M 0 16 32 40
370
N 10 24 38 45
O 30 40 50 55
P 40 48 56 60


Row operati on

Managers East West North South
M 0 16 32 40
N 0 14 28 35
O 0 10 20 25
P 0 8 16 20

Col umn operati on

Managers East West North South
M 0 8 16 20
N 0 6 12 15
O 0 2 4 5
P 0 0 0 0

Managers

East

West

North

South
M 0 6 14 18
N 0 4 10 13
O 0 0 2 3
P 2 0 0 0

Managers

East

West

North

South
M 0 2 10 14
N 0 0 6 9
O 4 0 2 3
P 6 0 0 0

Managers

East

West

North

South
M 0 2 8 12
N 0 0 4 7
O 4 0 0 1
P 8 2 0 0

Assi gnment Sal es
Rs.

M East
N West
O North
P South
371

1,86,000


Ans. 20

The initial matrix relating to nurse-patient combination is as under:

Nurse Patients
W X Y
K 10 10 30
L 30 10 20
M 20 30 20
Deducting the lowest element of each row from the other elements of the same row, we get
the following matrix:
0 0 20
20 0 10
0 10 0
We deduct the lowest element of each column from the other elements of the same column.
Since there is zero in each column, the same matrix will be returned.

Draw lines to connect zeros as under:

0 0 20
20 0 10
0 10 0
There are three lines as required by the order of matrix of three.

Hence the solution is optimal.

Allocation of patients to nurses as under to minimize the cost

0 0
20 0
0 10
K W 10 400 400

L X 10 400 400

M Y 20 800

800
Total minimum cost

1600
(i ii ) With the introduction of a new patient and a new nurse, the original matrix of nurse-patient
combinations will stand revised as under:

Nurse

Patients


W

X

Y Z
K 10

10

30 40
L 30

10

20 40
M 20

30

20 40
N 50

50

50 50
372


9



Deducting the lowest element of each row from the other element of the same
row, we get the following matrix:

0 0 20 30
20 0 10 30
0 10 0 20
0 0 0 0

Deduct the lowest element of each column from the other elements of the
same column. Since there is zero in each column, the same matrix will be returned.
Draw lines to connect zeros as under:

0 0 20 30
20 0 10 30
0 10 0 20
0 0 0 0
There are four lines as required by the order of matrix of four

Hence the solution is optimal.

Proceed to allocate the patients to nurses as under to minimize the cost.

0 0 20
20 0 10
0 10 0
0 0 0

K W 10 400 400
L X 10 400 400
M Y 20 800 800
N Z 50 2000 2000
Total minimum cost

3600
(i v) The cost of new nurse per hour is Rs. 50 in respect of any patient and the cost of the existing
nurses for attending to the new patient is Rs. 40 per hour. Both these rates are greater than
the values of other elements of existing nurse-




patient combination matrix. Thus the new nurse row and new patient column
will have a higher value than the element of the existing matrix. Hence the new nurse can
be allocated to the new patient without having to redo the assignment exercise.
Hence we need not to a fresh assignment. N will be assigned to patient Z at 50/ hr is Rs.
2000/ week.
This will be the extra minimum cost to the hospital i.e. 2000 + 1600 = 3600.

373
Ans. 12
PERT & CPM
(i) The required network is given below:

(ii) Critical path : B,E,F =7+7+6 = 20 days

Ans. 13
The network is constructed as given in figure below:

(i) The T
E
s and T
L
Event No.:
s for various events computed on the network are as follows:
1 2 3 4 5 6 7 8 9 10
T 0
E
4 1 5 7 11 15 17 18 25
T 0
L
12 1 13 7 17 15 17 18 25
(ii) Activity floats are computed using the following formula:
374
Float = T
L
(Head event) T
E
Activity
(Tail event) Duration
Duration T
E
T (Tail Event)
L
Float (Head Event)
1-2 4 0 12 8
1-3 1 0 1 0
2-4 1 4 13 8
3-4 1 1 13 11
3-5 6 1 7 0
4-9 5 5 18 8
5-6 4 7 16 5
5-7 8 7 15 0
6-8 1 11 17 5
7-8 2 15 17 0
8-9 1 17 18 0
8-10 8 17 25 0
9-10 7 18 25 0
Critical path is given by all those activities which have zero floats. Along the zero float activities, there are two such
critical paths:
(i) 1 3 5 7 8 9 10
(ii) 1 3 5 7 8 10
The project duration is 25 weeks.

Ans. 16(i)

A D F = 16+ 10+12 = 38
B E F = 20+ 6+ 12= 38
(ii) A-C E- F = 16+8 +6 +12 = 42 Critical Path

(iii) Total float and free float for each activity

375
(iii) Total float and free float for each activity

Activity Normal Earliest Time Latest Time Float Free
Time start finish start finish total
Days
A 16 0 16 0 16 0 0
B 20 0 20 4 24 4 4
C 8 16 24 16 24 0 0
D 10 16 26 20 30 4 4
E 6 24 30 24 30 0 0
F 12 30 42 30 42 0 0

Ans. 18
(a)
20 20
4
Z

= = 0; Probability = 0.50
(b)
18 20
4
Z

= = 0.50; Probability = 0.31
(c)
24 20
4
Z

= = 1; Probability = 0.84

Ans. 19
Tcp = 60 S.D. = 9 = 3.
60 + 3 2.3 = 67 weeks (Answer)

Ans. 21
The required network is drawn below:

376
(i) From the above network, it can be noted that the critical path is 1 2 4 6 8.
(ii) Expected cost of construction of the plant = (5 + 3 + 4 + 9 + 2 + 12 + 20 + 7 + 14 + 4) millions of Rs.
= Rs.80 million
(iii) Expected time required to build the plant = 4 + 6 + 9 + 1 = 20 months.
(iv) It is given that the time required for one activity is independent of the time and cost of any other activity and variations
are expected to follow normal distribution, the S.D.
Hence, the variance of the expected time is determined by summing the variance of critical activities and is = 1 + 2 + 5
+ 1 = 9.
Standard Deviation of the expected time = 9 = 3 months.

Ans. 24 The earliest expected completion time, latest allowable completion time and slack time for each event is:-
Event
2
te Earliest Earliest Latest Latest Slack
(I j) start finish start finish
1 2 2 4 0 04 0 4 0
1 3 3 5 0 5 16 21 16
2 4 01 20 4 24 4 24 0
2 5 10 20 4 24 14 34 10
3 4 2 3 5 08 21 24 16
3 6 4 8 5 13 24 32 19
4 5 4 10 24 34 24 34 0
4 6 2 6 24 30 26 32 2
5 7 1 8 34 42 34 42 0
6 7 8 10 30 40 32 42 2

-
The critical path is 1 2 4 5 7 = 42
Variance of project time

2
= 2 + 1 + 4 + 1 = 8
Therefore,
2
= 8 and scheduled time T
5

= 38
Z =
8
42 38
=
8
4
= - 1.41
From table on normal curve, the area of Z = 1.41 is given as 0.4207
Therefore the probability of completion of the project by the scheduled time = (0.5 0.4207) =
7.93%
377

Ans. 25

Calculation of expected time and variance of each activity:

Activity Optimistic Most likely Pessimistic Expected Variance

Days Days Days Duration

12 4 10 16 10 4
13 3 6 9 6 1
14 4 7 16 8 4
25 5 5 5 5 0
35 8 11 32 14 16
46 4 10 16 10 4
56 2 5 8 5 1

The network diagram is as under:


According to probability values given in the question probability is 11.9% To obtain 95% confidence level:

Critical Path: 1-3 3-5 5-6
Duration (days) 6 14 5 = 25 days
Standard deviation: 1 + 16 + 1 = 18
18 = 4.24

Probability that the project will be completed five days earlier:
Z =
24 . 4
25 20
= 1.18
According to probability values given in the question probability is 11.9%
To obtain 95% confidence level:

1.65=
24 . 4
25 X

X 25 = 6.996
X = 32 days
378

Ans. 26:

i. Activity t
o
t
m
t
p
2
6
0
t
p
t
2

|
|
.
|

\
|

= Expected Variance
----------------------------------------------- duration
(in weeks) t
e
= (t
0
+ 4t
m
+ t
p

) / 6


1-2 3 6 15 7 4
1-3 2 5 14 6 4
1-4 6 12 30 14 16
2-5 2 5 8 5 1
2-6 5 11 17 11 4
3-6 3 6 15 7 4
4-7 3 9 27 11 16



Critical Path 1-2-6-7
Expected project duration = (7+11+18)=36 week
(iv)
Probability of project completion in 38 weeks
o = 4 + 4 +16 = 24
o = 24 = 4.90
o
38 - 36 2s
Z = -------- = --------- = 0.41
4.9 4.9
Value of Z = 0.41 in Z tables is 0.1591
P(Z) = 0.5+ 0.1591
Probability of project completion in 38 weeks is 66%

379
(v) Project duration (say x weeks) with 95% chances of completion:
x - 36
1.65 = ---------
4.9
or 8.085 = x - 36
or x = 44 weeks

Ans. 27:
The earliest and latest expected time for each event is calculated by considering the expected time of each activity as
shown in the table below:
Activity
(i j)
t t
0
t
m
t
p
e
= (t
0
+ 4t
m
+ t
p
2
6
0
t
p
t
2

|
|
.
|

\
|

=
) /
6

1-2 2 2 14 4 4
1-3 2 8 14 8 4
1-4 4 4 16 6 4
2-5 2 2 2 2 0
3-5 4 10 28 12 16
4-6 4 10 16 10 4
5-6 6 12 30 14 16

(a) The project network is drawn below:

(i) Critical Path is : 1 3- 5 6
(ii) The expected duration and variance of each activity is shown in the table above.
The expected project length is the sum of the duration of critical activities. Hence,
Expected project Length = 8 + 12 + 14 = 34 months
380
(iii) Variance of the project length is the sum of the variances of critical activities.
Variance of project length = = 4 + 16 + 16 = 36 months
Therefore, Standard Deviation = = 36 = 6
(iv) Probability that the project will be completed at lest 8 months earlier than the expected time of 34 months is
given by
Prob.
(

6
34 8) (34
e

e
T
s
T
Z = Prob.[Z - 1.33]
But Z = -1.33 from the normal distribution table is 0.0918.
Students may pl ease note that the val ues for the Prob. For a Z val ue correspond tot e shaded area as shown i n the
di agram bel ow:

Thus, the probability of completing the project within 26 months is 9.18%.
(v) If the project due date is 38 months, then the probability of not meeting the due date is given by
Prob.
(

>
6
34) (38
e

e
T
s
T
Z = Prob.[Z > 0.67]
But Z = 0.67 from the normal distribution is 0.2514.
Thus, the probability of not meeting the due date is 25.14%.

Ans. 28:
The required network is drawn below:
381

The expected time marked in the above network diagram for various activities is calculated in the table below:
Activity Time (in weeks) Expected
time (weeks)
t
e
= (t
0
+ 4t
m

+ t
p
Variance
) / 6
2
6
0
t
p
t
2

|
|
.
|

\
|

=
Optimistic
(t
o
Most likely
(t )
m
Pessimistic
(t )
p
)
1-2 3 3 3 3 0
2-3 3 6 9 6 1
2-4 2 4 6 4 4/9
3-5 4 6 8 6 4/9
4-6 4 6 8 6 4/9
5-6 0 0 0 0 0
5-7 3 4 5 4 1/9
6-7 2 5 8 5 1
(i) Variance of each of the activities has been calculated in the last column of the above table.
(ii) Critical path is given by 1 2 3 5 6 7 and the expected project length is 20 weeks.
(iii) Variance of the critical path = = 0 + 1 + 4/9 + 0 + 1 = 22/9 = 2.444
Mean = x = 20 weeks

To calculate the probability of completing the project in 23 weeks, we will first calculate the normal Z as
below:

Z =

x D
=
444 . 2
20 23
= 1.92

P (x < 23) = P (z < 1.92) = 0.9726 (from the normal table)

Thus, the probability that the project will be completed in 23 weeks is 97.26%.

Ans. 29:
382
The network for the given problem is drawn below:

1
2
9
4 6
8
17
17.83
17.83
19
17.67
3 7
22.83
5
20
1
6
.
6
7



In the table below, we have calculated the expected duration and variance of each activity.

Activity Time Expected
duration
{(a+4m+b)6}
Variance
{(b-a)6}
2

Optimistic
a
Most Likely
m
Pessimistic
b
1-2
2-3
2-4
2-8
3-4
3-5
4-6
5-7
5-9
6-7
6-8
7-9
14
14
13
16
-
15
13
-
14
-
-
16
17
18
15
19
-
18
17
-
18
-
-
20
25
21
18
28
-
27
21
-
20
-
-
41
17.83
17.83
15.17
20
-
19
17
-
17.67
-
-
22.83
16.67
3.36
1.36



4





17.36

20.08

383
8-9 14 16 22

Variance paths are:
1-2-3-5-7-9 77.49
1-2-3-5-9 72.33
1-2-3-4-6-7-9 75.49
1-2-3-4-6-8-9 69.33
1-2-8-9 54.5
1-2-4-6-8-9 66.67
1-2-4-6-7-9 72.83

Hence the critical path is 1-2-3-5-7-9 with duration of 77-49 days or 78 days approximately.

Variances of various activities on critical path have been calculated in the last column of the above
table.

Hence standard deviation of critical path = 26.08 = 5.12

Now we want to find out that within how many days the project should be completed so as to
provide 95% probability of break even.

Z
0.95

= 1.65
Hence, 1.65 = {(D-77.49)5.12}
Or, D = 1.65 5.12+77.49 = 85.94 or 86 days

The fixed cost of the project is Rs. 8 lakhs and the variable cost is Rs. 9,000 per day.

Thus, amount to bid = Rs. 8 lakhs+ Rs. 9,00086
= Rs. 8 lakhs + Rs. 7,74,000
= Rs. 15,74,000

Ans. 34:
(a) Critical Paths:

All are critical paths:

384
(i) 1 2 5 6 2 + 8 + 5 = 15
(ii) 1 3 5 6 3 + 7 + 5 = 15
(iii) 1 4 5 6 4 + 6 + 5 = 15
(iv) 1 3 4 5 6 3 + 1 + 6 + 5 = 15
(i) Choose 5 6, common path; Crash by 1 day
(ii) Choose: 1 2, 1 3, 1 4

Or

(iii) Choose: 1 2, 3 5, 4 5

Or

(iv) Choose: 2 - 5 , 3 5, 4 5 Or

(v) Choose: 1 3, 1 4, 2 - 5
Ans. 35:
(i ) Assuming that the duration of activity 3 5 is 4 weeks.
The various critical paths are:
1-2-5-8-9 15 weeks
1-3-4-7-8-9 15 weeks
1-3-4-6-7-8-9 15 weeks
1-3-5-8-9 15 weeks
(ii) Note: Since the duration for activity 3-5 is not specified it is open for you to assume the duration.
Depending upon the duration assume three possibilities emerge.
1. If the duration assumed is more than 4 weeks then that path (13, 35, 58, 89) alone will be critical. In
that case you can choose any of the activity in the critical path.
2. If the duration assumed is exactly 4 weeks then it will be one of the 4 critical paths and the various
possibilities are given below.
3. If the duration assumed is less than 4 weeks then the solution should be based on 3 of the critical
paths namely 12,589, 1346789 and 134789. This has 16 combinations.
Reduce in the following ways, the project duration is. Since all the paths are critical, reduction is
possible by combining activities. The activities can be independent, common to few paths and
common to all the paths. The various categories are as follows:
1. Common to all the paths. 8-9
2. Independent : Combination 1. 1-2,3-5,4-6 and 4-7.
Combination 2. 2-5,3-5,4-6 and 4-7.
Combination 3. 1-2,3-5,4-7, 6-7.
Combination 4. 2-5,3-5,4-7, 6-7.
3. Activities common to two of the paths.
Combination 1. 1-2,1-3.
Combination 2. 1-3,2-5.
Combination 3. 3-4,5-8.
Combination 4. 5-8,7-8.
385
4. Activities common to two of the paths and two independent activities.
Combination 1. 1-2,3-4,3-5.
Combination 2. 1-2,3-5,7-8.
Combination 3. 2-5,3-4,3-5.
Combination 4. 2-5,3-5,7-8.
Combination 5. 4-6,4-7,5-8.
Combination 6. 4-7,5-8,6-7.
(Any three of the above combi nati on.)

Ans. 36:
(i) Project network based on the given activities is as under :


(ii) A review of the above network clearly shows that there are four paths 1 4 5; 1 2 5 ; 1 2 3 5;&
1 3 5 of duration 10 days; 11 days; 13 days and 4 days respectively. The longest path of 13 days
viz,. 1 2 3 5 is the critical path of the drawn network.

(iii) The optimum duration of a project is that duration of the project for which the total cost (direct & indirect)
will be minimum. The cost corresponding to optimal duration is known as resultant cost of the project.
To determine optimum duration and resultant cost of the project based on the given activities we
proceed as follows:

Activity Normal Crash Normal Crash Cost slope
Time Time Cost Cost per day
(days) (days) Rs. Rs Rs.

1 2 4 3 1,500 2,000 500

1 3 2 2 1,000 1,000 --

1 4 5 4 1,875 2,250 375

386
2 3 7 5 1,000 1,500 250

2 5 7 6 2,000 2,500 500

3 5 2 1 1,250 1,625 375

4 5 5 4 1,500
Total direct cost 10,125
2,125 625

The normal total cost (direct & Indirect) of completing the project in 13 days is :

Normal direct cost : (Rs) 10,125

Indirect cost 6,500
13 days x Rs. 500
______
Total normal cost : (Rs)

16,625
To determine the optimum duration and resultant cost we crash activities on the critical path by properly
selecting them as under :

Activities 1 2 2 3 3 5

No. of available crash days 1 2 1

Cost slope per day (Rs) 500 250 375

Indirect cost per day (Rs) 500 500 500

Saving in cash -- 250 125

Ranking -- 1 2

The above ranking clearly shows that we should select the activity 2 3 and crash it for one day, as it
results in maximum saving of Rs. 250 per day.

Let us crash 2 3 by 2 days.
Rs.
Normal direct cost 10,125

Cost slope (2 days x Rs. 250) 500

Indirect cost (11 days x Rs. 500) 5,500
______
Total cost

16,125
After crashing the activity 2 3 we are left with the following paths as under :

1 2 2 3 3 5 of 11 days duration

1 2 2 5 of 11 days duration

1 4 4 5 of 10 days duration
387

1 3 3 5 of 4 days duration

1 2 is a common activity in the first two paths with cost slope of Rs. 500/- per day. There is no profit or
loss in crashing this actively. Hence crash it by one by.

Rs.
Normal direct cost 10,125

Total cost slope (Rs. 500 + 1 day x Rs. 500) 1,000

Indirect cost (10 days x Rs. 500) 5,000
______
Total cost

16,125
Now we have the following four paths are as under :

1 2 2 3 3 5 of 10 days duration

1 2 2 5 of 10 days duration

1 4 4 5 of 10 days duration

1 3 3 5 of 4 days duration

To reduce the duration of project further, we are required to select the activities on all the three paths.
These activities may be 3 5, 2 5, and 1 4. if all of these activities are crash by even 1 day each, then
the total increase in cost would be (Rs. 375 + Rs. 500 + Rs. 375) or Rs. 1,250/- for saving Rs. 500. At this
stage, we stop the process of crashing.

Hence optimal project duration

10 days
Resultant project cost/optimal cost : (Rs)

16,125
Ans. 38:
(i) The required network is given below:

The various paths in the network are:
388
1 2 4 5 with project duration = 16 days
1 4 5 with project duration = 17 days
1 3 4 5 with project duration = 20 days
The critical path is 1 3 4 5. The normal length of the project is 20 days and minimum project length is 12
days.
(ii) Since the present schedule consumers more time than the minimum project length, the duration can be reduced by
crashing some of the activities. Also, since the project duration is controlled by the activities lying on the critical path,
the duration of some of the activities lying on critical path can be reduced. It is given that overhead cost is Rs.60 per
day.
Step I: First, the crashing cost of activity (3, 4) being minimum, the duration of this activity can be compressed from
10 days to 9 days. The total cost for 19 days schedule
= Rs.15 + Rs.19 60 = Rs.1,155
Step II: Since the critical path remains unchanged, the duration of activity (3, 4) can be further reduced from 9 days
to 8 days resulting in an additional cost of Rs.15 so that total cost for 18 days schedule = Rs.30 + Rs.60 18 = Rs.30
+ Rs.1,080 = Rs.1,110.
Step III: Continue this procedure till the minimum project length schedule. The calculations are given below:
Normal
Project
length (days)
Job crashed Crashing Cost (Rs.) Overhead
cost @
Rs.60 / day
Total
Cost.
(Rs.)
20 -- -- 2060 1,200
19 34 1 15 = 15 1960 1,155
18 34 2 15 = 30 1860 1,110
17 34 3 15 = 45 1760 1,065
16 45 315+140 = 85 1660 1,045
15 34, 14 415+140+130= 130 1560 1,030
14 13, 14, 24 130+130+125+110=195 1560 1,035
13 13, 14, 24 195+125+130+110=260 1360 1,040
12 13, 14, 12 260+25+30+20=335 1260 1,055
(iii) Since the total cost starts increasing from 14 days duration onwards, the minimum total cost of Rs.1,030 for the
optimum project duration of 15 days occurs for optimum duration of each job as given below:
Job: (1,2) (1,3) (1,4) (2,4) (3,4) (4,5)
Optimum: 9 8 14 5 6 1

Duration (day)
389

Path 1 2 4 5 = 9 + 5 + 1= 15 days
Path 1 4 5 = 14 + 1 = 15 days
Path 1 3 4 5 = 8 + 6 + 1 = 15 days.
Hence, the optimum duration of the project is 15 days.

Ans. 39 :
(a) (i) Net work diagram
Critical Path is 1-2-5-6-7-8 = 32 weeks
Associated Cost = 4220 + 3250 = 5820


(i i ) Total floats

Activity Duration
weeks
Early start Latest
start
Early
finish
Latest
finish
Total
float
390
The critical path activities are 1-2 2-5 5-6 6-7 7-8
Slope 100 45 45 70 200

1-2
2-3
2-4
2-5
3-5
4-5
5-6
6-7
6-8
7-8

3
3
7
9
5
0
6
4
13
10

0
3
3
3
6
10
12
18
18
22
0
4
5
3
7
12
12
18
19
22
3
6
10
12
11
10
18
22
31
32
3
7
12
12
12
12
18
22
32
32
0
1
2
0
1
2
0
0
1
0





(i ii ) Calculati on of crashi ng

Activity Nt Nc Ct Cc Slop =
(Cc-Nc) / (Nt-Ct)
1-2
2-3
2-4
2-5
3-5
4-5
3
3
7
9
5
0
300
30
420
720
250
0
2
3
5
7
4
0
400
30
580
810
300
0
100
0
80
45
50
0
5-6 6 320 4 410 45
6-7
6-8
7-8
4
13
10

400
780
1000
3
10
9
470
900
1200
70
40
200









Two activities cost slope cost is minimum (2-5 and 5-6) but activity 5-6 is common
and critical, it also continuing so reduce by 2 weeks, then reduce activity 2 -5 by one week.

Activity From-to Project durations Cost
I 5-6 6-4 weeks 32-2 = 30 4220 + (245) + (3050) = 5810
II 2-5 9-8 30-1 = 29 4220+90+(145)+(2950) = 5805
After this reduction now two paths are critical 1-2-3-5-6-7 = 28 and 1-2-5-6-7 = 28

So 1-2 3-5 6-7
391

2-5

Slope cost 100 50+45=95 70
As cost per week for every alternative is greater than Rs.50 (overhead cost p er week). Therefore, any
reduction in the duration of project will increase the cost of project completion. Therefore, time for
projects is 29 weeks, minimum cost is Rs.5805.

Answer 40:
The network is given below:


(i) The critical path of the project is ACEG or 1-2-3-4-6-7 with normal duration of 25 days.
The minimum duration of the project is 18 days.

(ii) The cost slope for various activities is given below:

Activity Normal
Duration
Crash
duration
Normal cost
(Rs.)
Crash cost
(Rs.)
Cost slope (Rs.)
A (1-2)


B (2-4)

C (2-3)

D (2-5)


E (4-6)


F (5-6)

G (6-7)

7


4

5

6


7


5

6

5


2

5

4


4


2

4

500


400

500

800


700


800

800

900


600

500

1,000


1,000


1,400

1,600
200
5 7
500 900
=


100
2 4
400 600
=


N.A.
100
4 6
800 000 , 1
=


100
4 7
700 000 , 1
=


200
4 5
800 400 , 1
=



400
4 6
800 600 , 1
=


392
Total 4,500

Step 1: Various paths of the network are given below:
1-2-3-4-6-7 With duration = 25 days
1-2-4-6-7 With duration = 24 days
1-2-3-5-6-7 With duration = 23 days
1-2-5-6-7 With duration = 24 days

In order to determine the cost of completing the project in 21 days, let us crash that activity on the
critical path, which has minimum cost slope. It can be seen that the minimum cost slope of
Rs.100 corresponds to activity E (4-6) and it lies on the critical path. Hence, we crash activity E (4
6) by 1 day at an additional cost of Rs. 100.

Step- 2: Various paths now are:
1-2-3-4-6-7 With duration = 24 days
1-2-4-6-7 With duration = 23 days
1-2-4-6-8 With duration = 23 days
1-2-4-6-9 With duration = 24 days

An examination of the above four paths clearly points out that there are two critical paths namely
1-2-3-4-6-7 and 1-2-5-6-7, each with duration = 24 days. To reduce the project duration by three
days more, there are following possible combination of activities.

1. Crash activities 4-6 on the path 1-2-3-4-6-7 and 5-6 on the path 1-2-5-6-7 by one day each at
an addition cost of Rs. 100 +Rs. 200 = Rs. 300.

2. Crash activities 4-6 on path 1-2-3-4-6-7 and 2-5 on path 1-2-5-6-7 by one day each at an
additional cost of Rs. 100 +Rs. 100 = Rs. 200

3. Crash activity 1-2 by one day at an additional cost of Rs. 200.

It can be observed that the additional cost of reducing the project duration by one day in
combination 2 as well as combination 3 is Rs. 200. Hence any of these two can be selected for
crashing. However, since crashing activity 1-2 by 1 day reduces the duration of all the paths by1
day, we will crash it by I day. The project duration becomes = 23 days at an additional cost = Rs.
200.

Step 3: Crash activity 1-2 by 1 day further, it would reduce the project duration to 22 days at an
additional cost = Rs. 200.
Step 4: Activity 1-2 can not be crashed further. So, we now select the combination 2 stated above
for crashing. Crash activities 4-6 and 2-5 by one day each at an additional cost of Rs. 100 +Rs.
100 = Rs. 200.

Hence, in order to complete the project in 21 days, an additional cost of Rs. 100 +Rs. 200 +Rs.
200 +Rs. 200 = Rs. 700 will be incurred.
The normal cot of completing the project in 25 days =Rs. 4,500.
Hence, the percentage increase in cost to complete the project in 21 days
393
=
500 , 4 .
700 .
Rs
Rs
100 = 15.5%.

Answer 42
The requires network based on the given activities and duration is drawn below :

The critical path of the network is 1-3-4-5-6 i.e. B-E-G-H. The duration of the project is 14 weeks.

E = 4
L = 6

A C E = 14
4 D E = 9 3 L = 14
3 L = 9

3
E = 0 B H
L = 0 E 2 G
2
7 F
E = 11
2 L = 11
E = 7
L = 7

The time scale diagram for various activities along the resource accumulation table showing the
number of workers required on each day are drawn on next page.

C(2)
3 7

A(4) D(4)

4 3 2


B(2) E(6) G(3) H(4)


7 2 2 3

F(3)

2 2

Crew size
1
2
6
5
3
4
2
1 3 4 5 6
394

1 2 3 4 5 6 7 8 9 10 11 12 13 14
6 6 6 6 8
-2
8
-2
8
-2
9 9 3 3 4
+2
4
+2
4
+2
6 6 6 6 6 6 6 9 9 3 3 6 6 6

It can be seen that the demand on the resources is not even. On the 8
th
and 9
th
week, the demand
of workers is as high as 9 whereas on the 10
th
and 11
th
As can be seen from the above network diagram, activity C has a float of 7 weeks and activity
week, it is only three. If 9 workers are to be
hired for the entire project duration of 14 weeks, then during most of the days they will be idle. We
will attempt to re-schedule our activities in such away so as to utilize the workers in a fairly uniform
manner.
F has a float of 2 weeks. The maximum demand on the resources occurs during 5
th
week to 7
th

week. (i.e. 8 workers) and during 8
th
to 9
th
week (i.e. 9 workers). We will shift activity C by seven
weeks so that it starts on 12
th
week instead of 5ht week. This reduces the demand of the workers
from 8 to 6 during 5
th
to 7
th

weeks. The modified resource requirements are shown in the last row
of the above table.
Activity F has a float of two weeks. It is shifted by two weeks so that it starts on 10
th
week instead
of 9
th
Crew size
workers required earlier. The modified resource accumulation table is given:

1 2 3 4 5 6 7 8 9 10 11 12 13 14
6 6 6 6 8

8

8

9
-3
9
-3
3
+3
3
+3
4

4

4

6 6 6 6 6 6 6 9 9 6 6 6 6 6

It is evident from the last row of the above table that there is a uniform demand of 6 workers
throughout the duration of the project.

Ans. 46:
The network diagram is drawn below:

E= 4
L = 4


4 8

E = 0 4
L = 0 6
6 E = 14
L = 14
E = 8
3 L = 8

2
5
4
1
3
395
4


E = 3
L = 4


The critical path is 1-2-4-5. The total floats of all the activities are calculated below:





1-2 4 0
Activity Duration Total float
1-3 3 1
1-4 6 2
2-4 4 0
2-5 8 2
3-4 4 1
3-5 4 7
4-5 6 0


(b) The resource allocation table is given below:

Starting day 1 4
st
5
th
9
th
10
th
13
th
18
th
21
th

st

Equipment X job done
No. of men required day
completed
(1,2)
30
4
(1,2)
30
4
(2,4)
30
8
(4,5)
30
18
(4,5)
30
18

Equipment Y Job done
No. of men required
Day completed
(1,3)
20
3
(3,4)
20
12
(3,4)
20
12
(3,5)
20
21
(3,5)
20
21
Equipment Z Job done
No. of men required
Day completed
(1,4)
20
9
(1,4)
20
9
(1,4)
20
9
(2,5)
20
17
(2,5)
20
17

Total no. of men 50 50 50 40 40 50 50 20

Explanation:
This is basically a problem of resource-leveling whereby the main constraint would be on the
resources. It the maximum demand on any resource is not to exceed a certain limit, the activities
will have to be rescheduled so that the total demand on the resources at any time will be within the
limit and consequent the project duration time is exceeded. The criterion to be followed in such a
case is to delay the job with a large float. In this way we tend to absorb the float and cutdown the
demand on the resource. If two or more jobs are competing
Ans. 47:
396

Paths Duration
1-2-5-7-8 7+16+9+8 = 40
1-2-4-7-8 7+12+19+8 = 46
1-4-7-8 6+19+8 = 33
1-3-4-7-8 8+6+19+8 =41
1-3-6-7-8 8+24+7+8 =47
1-3-6-8 8+24+4 = 36

Critical path = 1-3-6-7-8 = 47 days

397
Ans. 6
Si mul ati on
The numbers 00-99 are allocated in proportion to the probabilities associated with each event as given below:
Daily Demand Probability Cumulative
Probability
Random Numbers
Allocated
0 0.01 0.01 00-00
10 0.20 0.21 01-20
20 0.15 0.36 21-35
30 0.50 0.86 36-85
40 0.12 0.98 8697
50 0.02 1.00 98-99
Let us simulate the demand for the next 10 days using the given random numbers in order to find out the stock
position if the owner of the bakery decides to make 30 breads every day. We will also estimate the daily average
demand for the bread on the basis of simulated data.
Day Random Number Simulated Demand Stock if 30 breads
are prepared every
day
1 48 30 0
2 78 30 0
3 19 10 20
4 51 30 20
5 56 30 20
6 77 30 20
7 15 10 40
8 14 10 60
9 68 30 60
10 9 10 80
Total 220
Daily average demand of the basis of simulated data = 22

Ans. 7:
The random numbers are established as in Table below:

Production probability cumulative Random number
Per day probability
196 0.05 0.05 00-04
197 0.09 0.14 05-13
198 0.12 0.26 14-25
199 0.14 0.40 26-39
200 0.20 0.60 40-59
201 0.15 0.75 60-74
202 0.11 0.86 75-85
203 0.08 0.94 86-93
398
204 0.06 1.00 94-99
Based on the 15 random numbers given we simulate the production per day as above in table 2
below.

Random No. Estimated No. of mopeds waiting No. of empty
Production spaces in the lorry
Per day
Opening current current Total
Balance excess short waiting
Production production

1 82 202 -- 2 -- 2 ---
2 89 203 2 3 --- 5 ---
3 78 202 5 2 --- 7 ---
4 24 198 7 -- 2 5 ---
5 53 200 5 --- -- 5 --
6 61 201 5 1 --- 6 ---
7 18 198 6 --- 2 4 ---
8 45 200 4 --- -- 4 ---
9 04 196 4 --- 4 0 ---
10 23 198 0 --- 2 0 2
11 50 200 0 -- -- -- ---
12 77 202 0 2 -- 2 ---
13 27 199 2 --- 1 1 ---
14 54 200 1 -- -- 1 ---
15 10 197 1 --- 3 _--
Total
__2
42

__4
Average number of mopeds waiting =
15
42
= 2.80
Average number of empty spaces in lorry =
15
4
= 0.266
Ans. 8:
If the numbers 00-99 are allocated in proportion to the probabilities associated with each
category of work, then various kinds of dental work can be sampled, using random number
table :-

Type Probability Random Numbers

Filling 0.40 00-39
Crown 0.15 40-54
Cleaning 0.15 55-69
Extraction 0.10 70-79
Checkup 0.20 80-99

Using the given random numbers, a work sheet can now be completed as follows :-

FUTURE EVENTS

PATIENT SCHEDULED ARRIVAL RN CATEGORY SERVICE TIME
1 8.00 40 Crown 60 minutes
2 8.30 82 Checkup 15 minutes
3 9.00 11 Filling 45 minutes
399
4 9.30 34 Filling 45 minutes
5 10.00 25 Filling 45 minutes
6 10.30 66 Cleaning 15 minutes
7 11.00 17 Filling 45 minutes
8 11.30 79 Extraction 45 minutes

Now, let us simulate the dentists clinic for four hours starting at 8.00 A.M.

STATUS

Time Event Number of the patient Patients
being served (time to go) waiting

8.0
1st
patient arrives 1
st
8.30 2
(60) -
nd
arrives 1
st
(30) 2
9.00 1
nd

st
3
departs
rd
arrives 2
nd
(15) 3
9.15 2
rd

nd
departs 3
rd
9.30 4
(45) -
th
arrives 3
rd
(30) 4
10.00 3
th

rd
5
departs
th
arrives 4
th
(45) 5
10.30 6
th

th
arrives 4
th
(15) 5
th
& 6
10.45 4
th

th
departs 5
th
(45) 6
11.00 7
th

th
arrives 5
th
(30) 6
th
& 7
11.30 5
th

th
8
departs
th
arrives 6
th
(15) 7
th
& 8
11.45 6
th

th
departs 7
th
(45) 8
12.00 End 7
th

th
(30) 8
12.30 - 8
th

th

(45) -
The dentist was not idle during the entire simulated period :-
The waiting times for the patients were as follows :-

Patient Arrival Service Starts Waiting (Minutes)

1 8.00 8.00 0
2 8.30 9.00 30
3 9.00 9.15 15
4 9.30 10.00 30
5 10.00 10.45 45
6 10.30 11.30 60
7 11.00 11.45 45
8 11.30 12.30 60
Total

285
The average waiting time of a patient was
285
15
= 35.625 minutes.

Ans. 9: Random allocation tables are as under:
Time Arrival Arrivals Random Time Service Service Random
(Mts) (Proba.) cumulative No. (Mts) (Proba.) Cumulative No.

Probability allocated Probability allocated
400
1 0.05 0.05 00-04 1 0.10 0.10 00-09
2 0.20 0.25 05-24 2 0.20 0.30 10-29
3 0.35 0.60 25-59 3 0.40 0.70 30-69
4 0.25 0.85 60-84 4 0.20 0.90 70-89
5 0.10 0.95 85-94 5 0.10 1.00 90-99
6 0.05 1.00 95-99

Simulation of ten trails:

R. No. Arrival Mts. Time Start R. No. Time Mts. Finish
Time
Waiting Time

Clerk Passanger
60 4 9.04 9.04 09 1 9.05 4

16 2 9.06 9.06 12 2 9.08 1

08 2 9.08 9.08 18 2 9.10

36 3 9.11 9.11 65 3 9.14 1

38 3 9.14 9.14 25 2 9.16

07 2 9.16 9.16 11 2 9.18

08 2 9.18 9.18 79 4 9.22

59 3 9.21 9.22 61 3 9.25 1
53 3 9.24 9.25 77 4 9.29

1
03 1 9.25 9.29 10 2 9.31 _ 4

Total

6 6
In half an hour trial, the clerk was idle for 6 minutes and the passengers had to wait for 6 minutes.

Ans. 10:
From the frequency distribution of arrivals and service times, probabilities and cumulat ive probabilities are
first worked out as shown in the following table:

Time
between
arrivals


Frequency


Probability

Cum.
Prob.

Service
Time


Frequency


Prob.

Cum.
Prob.
1

2

3

4

5

6
5

20

35

25

10

5
0.05

0.20

0.35

0.25

0.10

0.05
0.05

0.25

0.60

0.85

0.95

1.00
1

2

3

4

5

6
1

2

4

2

1

0
0.10

0.20

0.40

0.20

0.10

0.00
0.10

0.30

0.70

0.90

1.00

1.00
Total 100

10

The random numbers to various intervals have been allotted in the following table:

Time between
arrivals
Probability Random
numbers
allotted
Service Time Probability Random
numbers
allotted
401
1
2
3
4
5
6
0.05
0.20
0.35
0.25
0.10
0.05
00-04
05-24
25-59
60-84
85-94
95-99
1
2
3
4
5
6
0.10
0.20
0.40
0.20
0.10
0.00
00-09
10-29
30-69
70-89
90-99
-

Simulation Work Sheet

Random
Number
Time till
next
arrival
Arrival
Time
a.m.
Service
begins
a.m.
Random
number
Service
time
Service
Ends
a.m.
Clerk
Waiting
time
Customer
waiting
Time
Time
spend by
customer
in system
Length
of
waiting
line
64 4 11.04 11.04 30 3 11.07 04 - 3 -
04 1 11.05 11.07 75 4 11.11 - 2 6 1
02 1 11.06 11.11 38 3 11.14 - 5 8 2
70 4 11.10 11.14 24 2 11.16 - 4 6 2
03 1 11.11 11.16 57 3 11.19 - 5 8 2
60 4 11.15 11.19 09 1 11.20 - 4 5 2
16 2 11.17 11.20 12 2 11.22 - 3 5 2
18 2 11.19 11.22 18 2 11.24 - 3 5 2
36 3 11.22 11.24 65 3 11.27 - 2 5 1
38 3 11.25 11.27 25 2 11.29 - 2 4 1
07 2 11.27 11.29 11 2 11.31 - 2 4 1
08 2 11.29 11.31 79 4 11.35 - 2 6 1
59 3 11.32 11.35 61 3 11.38 - 3 6 1
53 3 11.35 11.38 77 4 11.42 - 3 7 1
01 1 11.36 11.42 10 2 11.44 - 6 8 2
62 4 11.40 11.44 16 2 11.46 - 4 6 2
36 3 11.43 11.46 55 3 11.49 - 3 6 2
27 3 11.46 11.49 52 3 11.52 - 3 6 1
97 6 11.52 11.52 59 3 11.55 - - 3 -
86 5 11.57 11.57 63 3 12.00 2 - 3 -
20 57 54 6 56 26

Average queue length =
Number of customers in waiting line
Number of arrivals
=
26
1.3
20
=
Average waiting time per customer =
56
2.8
20
= minutes
Average service time =
54
2.7
20
= minutes

Ans. 11:
Cumulative frequency distribution for Ramu is derived below. Also fitted against it are the eight given random
numbers. In parentheses are shown the serial numbers of random numbers.
402
10 4 01 (2) 00 (7) 03 (8)
20 10
30 20 14 (1)
40 40
50 80 44 (4) 61 (5)
60 91 82 (6)
70 96 95 (3)
80 100
Thus the eight times are: 30, 10, 70, 50, 60, 10 and 10 respectively.
Like wise we can derive eight times for Raju also.
Col-1 Col-2 Col-3 (2 Col-2)
10 4 8
20 9 18
30 15 30 25 (4)
40 22 44 36 (1) 34 (8) 41 (6)
50 32 64 55 (3) 56 (7)
60 40 80 76 (2)
70 46 92
80 50 100 97 (5)
(Note that cumulative frequency has been multiplied by 2 in column 3 so that all the given random numbers are
utilized).
Thus, Rajus times are: 40, 60, 50, 30, 80 40, 50 and 40 seconds respectively.
Ramus and Rajus times are shown below to observe for waiting time, if any.
1 2 3 4
Ramu Cum. Times Raju Initial Rajus cumulative time with 30 seconds
included
30 30 40 70
10 40 60 130
70 110 50 180
50 160 30 210
50 210 80 290
60 270 40 330
10 280 70 400
10 290 40 440
Since col. 4 is consistently greater than Co.2, no subsequent waiting is involved.


Ans. 12: The numbers 00-99 are allocated in proportion to the probabilities associated with each
event. If it rained on the previous day, the rain distribution & the random no allocation are given
below:

403
Event Probability Cumulative Random
Probability numbers
Assigned
No rain 0.50 0.50 00-49
1 cm rain 0.25 0.75 50-74
2 cm rain 0.15 0.90 75-89
3 cm rain 0.05 0.95 90-94
4 cm rain 0.03 0.98 95-97
5 cm rain 0.02 1.00 98-99

Table 1 Rain on previous day
Similarly, if it did not rain the previous day, the necessary distribution and the random number
allocation is given below:
Event Probability Cumulative Random
Probability numbers
Assigned
No rain 0.75 0.75 00-74
1 cm rain 0.15 0.90 75-89
2 cm rain 0.06 0.96 90-95
3 0.04 1.00 96-99

Table 2- No rain on previous day
Let us now simulate the rain fall for 10 days using the given random numbers. For the first day it is
assumed that it had not rained the day before:

Day Random Numbers Event
1 67 No rain (from table 2)
2 63 No rain (from table 2)
3 39 No rain (from table 2)
4 55 No rain (from table 2)
5 29 No rain (from table 2)
6 78 1 cm rain (from table 2)
7 70 1 cm rain (from table 1)
8 06 No rain (from table 1)
9 78 1 cm rain (from table 2)
10 76 2 cm rain (from table 1)

Hence, during the simulated period, it did not rain on 6 days out of 10 days. The total rain fall during
the period was 5 cm.

Ans.13:
The probabilities of occurrence of A, B and C defects are 0.15, 0.20 and 0.10 respectively. So, tile
numbers 00-99 are allocated in proportion to the probabilities associated with each of the three
defects

Exists Random Exists? Random Exists? Random
Defect-A Defect-B Defect-C
Numbers numbers numbers
Assigned assigned
Yes 00-14 yes 00-19 yes 00-09
assigned
No 15-99 No 20-99 no 10-99
404
Let us now simulate the output of the assembly line for 10 items using the given random numbers in
order to determine the number of items without any defect, the number of items scrapped and the
total mi
Item RN for RN for RN for whether Rework
Remarks
nutes of rework time required:
No. defect A defect B defect C any defect time (in

1 48 47 82 none -- --
Exists minutes)
2 555 36 95 none -- --
3 91 57 18 none -- --
4 40 04 96 B 15 --
5 93 79 20 None -- --
6 01 55 84 A -- Scrap
7 83 10 56 B 15 ---
8 63 13 11 B 15 ---
9 47 57 52 None -- --

During the simulated period, 5 out of the ten items had no defects, one item was scrapped and 90
minutes of total rework time was required by 3 items.
10 52 09 03 B,C 15+30 =45 --

Answer 14:
The question is not happily worded, if we go by the language of the question, the following solution can be worked
out:
First of all, random numbers 00-99 are allocated in proportion to the probabilities associated with demand as given
below:
Demand Probability Cum. Probability Random Nos.
0 0.05 0.05 00-04
1 0.10 0.15 05-14
2 0.30 0.45 15-44
3 0.45 0.90 45-89
4 0.10 1.00 90-99
Based on the ten random numbers given, we simulate the demand per day in the table given below.
It is given that stock n hand = 8 and stock on order = 6 (expected next day). Let us now consider both the options
stated in the question.
Opti on A: Order 5 Books, when the inventory at the beginning of the day plus orders outstanding is less than 8
books:
Day Random
No.
Sales
Demand
Op.
Stock in
hand
Qty.
Order
Qty.
Recd. At
end of
the day
Total
Qty. on
order
Closing
Stock
1 89 3 8 - - 6 5
2 34 2 5 - 6 - 9
3 78 3 9 - - - 6
4 63 3 6 5 - 5 3
405
5 61 3 3 - - 5 0
6 81 3 0 0
7 39 2
8 16 2
9 13 1
10 73 3
Now on day 6, there is stock out position since 5 units will be received at the end of the day and demand occurring
during the day can not be met. Hence, it will into be possible to proceed further and we will have to leave the answer
at this stage.
Random
No.
Sales
Demand
Opening
Stock in
hand
Qty.
Order
Qty.
Recd. At
end of
the day
Total
Qty. on
order
Closing
Stock
1 89 3 8 -- -- 6 5
2 34 2 5 -- 6 -- 9
3 78 3 9 -- -- -- 6
4 63 3 6 8 -- 8 3
5 61 3 3 -- -- 8 0
6 81 3 0 -- 8 --
7 39 2
8 16 2
9 13 1
10 73 3
Now on day 6, there is stock out position since 8 units will be received at the end of the day and demand occurring
during the day can not be met. Hence, it is not possible to proceed further and we may leave the answer at this
stage.
Alternatively, if we assume that the demand occurring during the day can be met out of stock received at the end of
the day, the solution will be as follows:
Stock in hand = 8 and stock on order = 6 (expected next day)
Random
No.
Sales
Demand
Opening
Stock in
hand
Qty.
Order
Qty.
Recd. At
end of
the day
Total
Qty. on
order
Closing
Stock
1 89 3 8 -- -- 6 5
2 34 2 5 -- 6 -- 9
3 78 3 9 -- -- -- 6
4 63 3 6 5 -- 5 3
5 61 3 3 -- -- 5 0
6 81 3 0 5 5 5 2
7 39 2 2 5 -- 10 0
8 16 2 0 -- 5 5 3
9 13 1 3 -- 5 -- 7
10 73 3 7 5 -- 5 4
406
Carrying Cost = 39 0.50 = Rs.19.50
Ordering Cost = 4 10 = Rs.40.00
Total Cost = Rs.59.50
Opti on B: Order 8 Books, when the inventory at the beginning of the day plus orders outstanding is less than 8
books:
Random
No.
Sales
Demand
Opening
Stock in
hand
Qty.
Order
Qty.
Recd. At
end of
the day
Total
Qty. on
order
Closing
Stock
1 89 3 8 -- -- 6 5
2 34 2 5 -- 6 -- 9
3 78 3 9 -- -- -- 6
4 63 3 6 8 -- 8 3
5 61 3 3 -- -- 8 0
6 81 3 0 -- 8 -- 5
7 39 2 5 8 -- 8 3
8 16 2 3 -- -- 8 1
9 13 1 1 -- 8 -- 8
10 73 3 8 -- -- -- 5
Carrying Cost = 45 0.50 = Rs.22.50
Ordering Cost = 2 10 = Rs.20.00
Total Cost = Rs.42.50
Since Option B has lower cost, Manager should order 8 books.
Ans.15
Demand (Tons) Probability Cumulative Probability Random Nos. Allocated
1 0.15 0.15 00-14
2 0.30 0.45 15-44
3 0.45 0.90 45-89
4 0.10 1.00 90-99
Option-I
RN Demand Opening
Stock
Receipts Closing
Stock
Op.Stock
on Order
Order Cl.Stock on
Order
88 3 8 - 5 - - 6
41 2 5 6 9 - - -
67 3 9 - 6 - 5 5
63 3 6 - 3 5 - 5
48 3 3 - 0 5 5 10
74 3 0 5 2 5 5 10
27 2 2 - 0 10 - 10
16 2 0 5 3 5 - 5
11 1 3 5 7 - 5 5
64 3 7 - 4 5 - 5
49 3 4 - 1 5 5 10
21 2 1 5 5 4 - 5
407
44
(Rs.)
No of order placed 5
Ordering cost (5x1000)
Closing Stock 44
Carrying cost (44x50)
Total

5,000

2,200
7,200

Option-II
RN Demand Opening
Stock
Receipts Closing
Stock
Op.Stock
on Order
Order Cl.Stock on
Order
88 3 8 - 5 - - 6
41 2 5 6 9 - - -
67 3 9 - 6 - 8 8
63 3 6 - 3 8 - 8
48 3 3 - 0 8 - 8
74 3 0 8 5 - 8 8
27 2 5 - 3 8 - 8
16 2 3 - 1 8 - 8
11 1 1 8 8 - - -
64 3 8 - 5 - 8 8
49 3 5 - 2 8 - 8
21 2 2 - 0 8
47
- 8


(Rs.)
No of orders 3 Ordering cost 3 x 1000
Closing stock 47 Carrying cost 47x50
Total
3,000
2,350
5,350

Analysis: Since the cost of inventory is less in Option II, it is suggested to implement.

Ans. 16
(i ) Allocation of random numbers
Demand Probability Cumulative probability Allocated RN
0<300 0.18 0.18 0017
300 < 600 0.32 0.50 1849
600 < 900 0.25 0.75 5074
900 < 1200 0.15 0.90 7589
1200 <1500 0.06 0.96 9095
1500 < 1800 0.04 1.00 9699
(i i ) Simulation: twelve months sales, monthly and annual profit/loss
Month RN Demand Sold Return Profit
on
sales
(Rs.)
Loss on
return
(Rs.)
Net
(Rs.)
Loss on
lost
units
408
1 27 450 450 300 3375 12000 2175
2 15 150 150 600 1125 2400 -1275
3 56 750 750 -- 5625 -- 5625
4 17 150 150 600 1125 2400 -1275
5 98 1650 750 -- 5625 --- 5625 900
6 71 750 750 -- 5625 -- 5625
7 51 750 750 -- 5625 -- 2175
8 32 450 450 300 3375 1200 5625
9 62 750 750 -- 5625 -- 5625 300
10 83 1050 750 -- 5625 -- 5625 900
11 96 1650 750 -- 5625 -- 5625
12 69 750 750 -- 5625 5625
54000 7200 46800 2100

(i i i ) Loss on lost sale 21007.5 = Rs15750.

Ans. 17
The demand and suppl y patterns yi el d the fol l owi ng probabi l i ty di stri buti on. The numbers 00-99 are
al l ocated i n proporti on to the probabi l i ti es associ ated wi th each event.
Availability
(Kg.)
Prob. Cum.
Prob.
Random
Numbers
allocated
Demand
(Kg)
Prob. Cum.
Prob.
Random
number
allocated
10 0.08 0.08 00-07 10 0.10 0.10 00-09
20 0.10 0.18 08-17 20 0.22 0.32 10-31
30 0.38 0.56 18-55 30 0.40 0.72 32-71
40 0.30 0.86 56-85 40 0.20 0.92 72-91
50 0.14 1.00 86-99 50 0.08 1.00 92-99
Let us simulate the supply and demand for the next six days using the given random numbers in order to find the
profit if the cost of the commodity is Rs.20 per kg, the selling price is Rs.30 per kg, loss on any unsatisfied demand
is Rs.8 per kg and unsold commodities at the end of the day have no saleable value.
Day Random
no.
Supply
availability
Random
no.
Demand Buying
cost
Rs.
Selling
cost
Rs.
Loss for
unsatisfied
demand
Profit
1 31 30 18 20 600 600 -- --
2 63 40 84 40 800 1200 -- 400
3 15 20 32 40 400 600 160 40
4 07 10 32 30 200 300 160 -60
5 43 30 75 40 600 900 80 220
6 81 40 27 20 800 600 -- -200
During the simulated period of six days, the net profit of the retailer is
= (400 + 40 + 220) (60 + 200)
409
= 660 260
= Rs.400

Ans. 19:
Random No. Coding Table - Receipts
Amount (Rs. In crores) Probability Cum. Probability Random No. Interval
30 0.20 0.20 00-19
42 0.40 0.60 20-59
36 0.25 0.85 60-84
99 0.15 1.00 85-99

Random No. Coding Table - Payments
Amount (Rs. In crores) Probability Cum. Probability Random No. Interval
33 0.15 0.15 00-14
60 0.20 0.35 15-34
39 0.40 0.75 35-74
57 0.25 1.00 75-99

Simulation Table
Week Op.Balance Receipts Payments Cl.Balance
Random
No.
Amount
(in crores)
Random
No.
Amount
(in crores)

1 15 17 30 78 57 -12
2 -12 43 42 16 60 -30
3 -30 74 36 35 39 -33
4 -33 31 42 23 60 -51
5 -51 72 36 44 39 -54
6 -54 46 42 92 57 -69
7 -69 51 42 58 39 -66
8 -66 68 36 8 33 -63
9 -63 93 99 58 39 -3
10 -3 54 42 78 57 -18
11 -18 96 99 54 39 42
12 42 9 30 77 57 15

(i) Probability is = 10 12 = 0.83

(ii) Total Shortfall is Rs. 399 crores. Therefore average shortfall is 399 12 = Rs. 33.25 crores
Alternatively, average shortfall is 399 10 = Rs. 39.90 crores

(iii) There will be a shortfall in 5 months i.e. 4,5,6,7,8. therefore the probability is 5 12 = 0.42

410
Ans. 6 (i) Actual learning curve rate is 80%.
Learning Curve

Time taken to produce the first machine = 600 hours

Average time taken to produce two machines = 600 80% hours

= 480 hours.
Cumulative time taken to produce two machines = 480 2 hours
= 960 hours.

Time taken to produce the second machine = (960 600)hours

= 360 hours.

(ii) Actual learning curve rate is 90%.

Time taken to produce the first machine = 600 hours

Average time taken to produce two machines = 600 90% hours

= 540 hours.
Cumulative time taken to produce two machines = 540 2 hours
= 1080 hours.

Time taken to produce the second machine = (1080 600) hours

= 480 hours.

The time taken to produce the second machine is lower at 80% learning rate and
hence 80% learning rate shows faster learning rate.


Ans. 10: (i) Rs/u

1
st
unit Avg/u after 4
th
at
Variable Cost
Labour
Target Contribution
Price to be quoted
2000
1000
2000
810
1500
4310 (Rs./u)
(i i ) No, the company cannot quote this price for varying products because the learning
curve Ratio does not apply to non-repeated jobs. Each product will carry a different
price according to its direct labour hours.

Ans. 13:

5,000 units 20,000 units

Material 1,50,000

6,00,000
Direct Labour 1,00,000 2,56,000
Refer to W Note i
Variable Overhead 50,000 2,00,000

Total Variable Cost 3,00,000

10,56,000
Fixed Cost 1,50,000 1,50,000

Total Cost 4,50,000

12,06,000
Total cost / unit 90 60.3
Sales 100 5,000 5,00,000 5,00,000
15,000 x(assumed selling price) 15,000 x
(Total Sales less Total Cost) = Profit 50,000 15,000 x 7,06,000

Or minimum selling price = 50.4(refer to Working Note ii)

Worki ng Note: I

411
Units Hours
5,000 5,000
10,000
10,000 1 .8 = 8,000 hours
20,000 20,000 1 .8 .8 = 12,800 hours
Worki ng Note: II

15,000 x 7,06,000 > 50,000

15,000 x > 7,56,000

or x > 50.4

Al ternati ve Sol uti on:

Total cost / unit of capacity 20,000 = 60.3

Weighted average selling price > 80.4

i.e.
5,000 100 + 15,000 x
> 60.3

20,000

= 5,00,000 + 15,000 x > 60.3 20,000

= 15,000 x > 12,06,000 5,00,000

Or

15,000 x > 7,06,000

x > 47.06

Minimum price to cover production Cost = 47.06

Minimum price to cover same amount of profit = 50.40 (refer to W orking Note 1)

Worki ng Note 1
( 47.06 + 50.04) 15,000 units

= Rs. 50,000

Ans. 14: Units Average/ hrs/u.

1 2,000

2 1,600

4 1,280

8 1,024

Material Cost / u = 10,000

Variable cost = 2,000

Variable Cost = 12,000

Opti on I

If both the orders came together, learning rate 80% applies and 8 units can be
made, with average time of 1,024 hours per unit.
Cost to PQ:

Variable cost excl. labour = Rs.12,000

Labour cost 1,024 hrs 4 Rs./hr = Rs. 4,096
412

Y X
Selling Price Rs.17,200 Rs.14,000
Variable Cost (excl. labour) Rs.12,000 Rs.12,000
Labour cost:
1280 4 Rs.5,120


= Rs.16,096

In this case,

Y X
Selling Price p. u. Rs.17,200 Rs.16,500 (under option I)
Variable Cost p. u. Rs.16,096 Rs.16,096
Contribution p. u. Rs.1,104 Rs.404
No. of units 4 4
Contribution (Rs.) 4416 1616 6032
Opti on II

If X Ltd supplies its labour. 80% learning curve will apply to 4 units each of PQ & X.
Hence: hrs/ u = 1280









1280 1 . Rs.1280

Total Variable Cost Rs.17,120 Rs.13,280
Contribution Rs.80 Rs.720
Units 4 4
Contribution (Rs.) 320 2,880 3,200
PQ should not take labour from X Ltd. It should choose option I.

Ans. 16: Working notes :

(1) By the theory of learning curve
YX = KX5 (i)
Here X is the cumulative number of units or lots produced, Y is the
cumulative average unit time of those X units. K is the average time of the
first unit or lot, s is the improvement exponent or the learning coefficient or
the index of learning.

Taking log on both sides of relation (i) we have
Log YX = log K + s log X (ii)

(2) Time required for 30 units order (when the time required for the first unit
is 40 hours)

Log 40 + (-0.322) log 30 = 1.6021 + (- 0.322) (1.4771) = 1.6021
0.4756
= 1.1264
Anti log of 1.1264 = 13.38
Hence hours required
Per unit = 13.38

Total time required for 30 units = 30 units x 13.38 hours = 401.40

413
(3) Time required for 50 units order (When the time required for first unit is
40 hours)

log 40 + (-0.322) log 50 = 1.6021 + (-0.322) 1.6990
= 1.055
Anti log of 1.055 = 11.35
Hence hours required per unit 11.35 hours
Total time required for 50 units = 11.35 x 50 units = 567.5 hours

(4) Fixed overhead recovery rate per labour hour

Total labour hours 2,000
10 men x 25 days x 8 hours
Less : 25% downtime (in hours) 500
_____
Total effective hours 1,500
Total fixed overheads per month (Rs.) 7,500
Fixed overhead recovery rate per labour hour (Rs) 5
(Rs. 7,500/1,500 hours)

(i) Computation of cost per unit of the first order of 30 units
Rs.
Direct material cost 1,800.00
(30 units x Rs. 60)
Direct labour cost 2,408.40
(401.4 hours x Rs. 6)
Variable overheads 401.40
(401.40 hours x Re 1)
Fixed overheads
(401.4 hours x Rs. 5)
2,007.00
Total cost of 30 units
Cost per unit 220.56
6,616.80
(Rs. 6,616.80/30 units)

(ii) Cost per unit, when a repeat order for 20 units is also placed.
Rs.
Direct material cost 1,200.00
(20 units x Rs. 60)
Direct labour 996.60
(567.5 hours 401.40 hours) x Rs. 6
Variable overheads 166.10
(1.66.1 hours x Re 1)
Fixed overheads 830.50
(166.1 hours x Rs. 5)
________
Total cost of 20 additional units

3,193.20
Cost per unit 159.66
(Rs. 3,193.20/20 units)

Price to be quoted to yield a profit of 25% on selling price
414

If selling price is Rs. 100 then profit is Rs. 25 and cost is Rs. 75

Hence selling price per unit = 100
75
x 159.66

= Rs. 212.88

Ans. 18 (i ) Pri ce per uni t for fi rst order of 100 uni ts
Rs Rs
Direct material 500.00
Direct labour Dept A 20 Hrs @ 10 = 200
Dept B 40 Hrs @ 15 = 600
800.00
Variable Overhead 20% of Rs 800 160.00
Fixed Overhead Dept A 20 Hrs @ 8 = 160
Dept B 40 Hrs @ 5 = 200
360.00
Total cost 1,820.00
Profit 25% 455.00
Selling price per
unit
2,275.00

(i i ) Pri ce per uni t for second order of 60 uni ts
Learning will be applicable only in department B.
Cumulative output becomes 100 units + 60 units = 160 units i.e 1.6 times for which
learning is 86.1 % from the tables.
Therefore Total Hrs for 160 units = 160 units 40 .861 = 5,510.4 Hrs
Therefore Hrs for 60 units = Hrs for 160 units less Hrs for 100 units
Or 5510.4 less 40 100 = 1510.4 Hrs
Therefore Hrs per unit =
60
4 . 1510
= 25.17
Cal cul ati on of sel l i ng pri ce per uni t
Rs
Direct materials 500.00
Direct labour Dept A 20 Hrs @ 10 = 200
Dept B 25.17 Hrs @ 15 = 377.55
577.55
Variable Overhead 20% of 577.55 115.51
Fixed Overhead Dept A 20 Hrs @8= 160
Dept B 25.17 Hrs @5=125.85
285.85
Total cost 1,478.91
Profit 25% 369.73
Selling price per unit 1,848.64
(i i i ) Pri ce per uni t for thi rd order of 40 uni ts
Cumulative output becomes 100 + 60 + 40 = 200 units i.e. 2 times for which learning is
80% from the table
415
Total Hrs for 200 units = 200 40 .80 = 6,400 Hrs
Hrs for 40 units = Hrs for 200 units less Hrs for 160 units
Or 6,400 less 5510.4 = 889.6 Hrs
Therefore Hrs per unit =
40
6 . 889
= 22.24
Cal cul ati on of sel l i ng pri ce per uni t
Rs
Direct materials 500.00
Direct labour Dept A 20 Hrs @ 10 = 200.00
Dept B 22.24 @ 15 = 333.60
533.60
Variable Overhead 20% of 533.60 106.72
Fixed Overhead Dept A 20 Hrs @ 8 = 160
Dept B 22.24 Hrs @ 5 = 111.20
271.20
Total cost 1,411.52
Profit 25% 352.88
Selling price per unit 1,764.40


416

You might also like